paramedic final part 2

Réussis tes devoirs et examens dès maintenant avec Quizwiz!

15. The odor of bitter almonds on a patient's breath should make you suspicious for exposure to: A) cyanide. B) arsenic. C) phosphorus. D) turpentine.

A

11. What is the cardiac output for an 80-kg man who has a heart rate of 80 beats/min and a stroke volume of 60 mL? A) 4,800 mL B) 5,200 mL C) 6,000 mL D) 6,400 mL

A) 4,800 mL

54. Newborn hypoglycemia is defined as a blood glucose level lower than: A) 45 mg/dL. B) 50 mg/dL. C) 55 mg/dL. D) 60 mg/dL.

A) 45 mg/dL

31. Traumatic injuries to the aorta are MOST commonly the result of: A) shearing forces. B) rear-end collisions. C) penetrating trauma. D) motorcycle crashes.

A) shearing forces.

36. A patient with quiet tachypnea is MOST likely experiencing: A) shock. B) asthma. C) alkalosis. D) airway swelling.

A) shock.

47. The chief clinical manifestation of Addisonian crisis is: A) shock. B) dehydration. C) lower back pain. D) an elevated temperature.

A) shock.

91. Hemodynamically stable children with a wide QRS complex tachycardia: A) should receive amiodarone. B) respond well to adenosine. C) are likely experiencing supraventricular tachycardia. D) will respond to vagal maneuvers.

A) should receive amiodarone.

hich of the following will be of MOST benefit in helping the paramedic predict the type of injuries that a patient experienced? A) Index of suspicion B) Past medical history C) Age of the patient D) Mechanism of injury

Mechanism of injury

Which of the following mechanisms of injury poses the LEAST threat for significant injury? A) Rear-end collision with restrained driver B) Death of an occupant in the same vehicle C) Motorcycle crash at greater than 20 mph D) Vehicular intrusion of greater than 12 inches

Rear-end collision with restrained driver

Signs and symptoms of compartment syndrome include all of the following, EXCEPT: A) pressure. B) erythema. C) parasthesia. D) passive stretch pain

erythema

When managing a patient who is entrapped by a crushing object, it is MOST important to: A) assess perfusion and sensory and motor functions every 5 minutes. B) make every effort to treat the patient before removing the crushing object. C) infuse 2 L of lactated Ringer's solution to combat hyperkalemia. D) give sodium bicarbonate immediately after removing the crushing object.

make every effort to treat the patient before removing the crushing object

The "platinum 10 minutes" refers to the: A) maximum amount of time to extricate a patient. B) maximum time spent at a scene for a trauma patient. C) amount of time before decompensated shock occurs. D) amount of time taken to perform a rapid assessment.

maximum time spent at a scene for a trauma patient

Which of the following is NOT a benefit of a properly worn seat belt? A) Minimal risk of whiplash injuries after a rear-end collision B) Prevention of partial or complete ejection from the vehicle C) Prevention of occupants from violently contacting each other D) Distribution of deceleration energy over a greater surface area

minimal risk of whiplash injuries after a rear-end collision

Which of the following patients is at HIGHEST risk for a pressure injury? A) An obese patient B) A bedridden patient C) A hypertensive patient D) A patient with diabetes

a bedridden patient

Icteric sclera are: A) white. B) cloudy. C) yellow. D) bloodshot.

C

The greatest amount of kinetic energy would be created if a ____-pound driver struck a tree while traveling at ____ mph. A) 140, 50 B) 160, 30 C) 150, 40 D) 170, 30

140, 50

The MOST significant fall occurs from a height greater than: A) 10 feet. B) 15 feet. C) 1.5 times the patient's height. D) 2 times the patient's height.

15 feet

Which of the following is NOT an inner ear structure? A) Cochlea B) Oval window C) Eustachian tube D) Semicircular canal

C

. Injury to the temporal lobe on the left side would MOST likely cause: A) abnormal speech. B) visual disturbances. C) sleep abnormalities. D) lack of coordination.

A

. The crista galli: A) is the anatomic point of attachment of the meninges. B) is divided into the anterior, middle, and posterior fossae. C) is a horizontal bone perforated with numerous foramina. D) allows the passage of the olfactory nerves from the nasal cavity.

A

1. A poison is a substance that is: A) toxic by nature, no matter how it enters the body or in what quantities it is taken. B) damaging to the tissues and cells, especially if injected or taken in large quantities. C) legal or illegal, and has the potential of causing permanent damage if it is ingested. D) capable of making a person ill, at a minimum, and has a great chance of causing death.

A

102. You are caring for an alcoholic patient who has been abstinent for about 2 days. The patient is confused, restless, and tells you that he sees snakes crawling on the walls. His blood pressure is 76/52 mm Hg, pulse rate is 140 beats/min and weak, and respirations are 24 breaths/min with adequate depth. In addition to administering oxygen, you should: A) treat his hypotension with crystalloid fluid boluses. B) administer 6 mg of adenosine to slow his heart rate. C) sedate him with 5 mg of Valium and transport at once. D) provide emotional support only and transport immediately.

A

106. During your assessment of a 33-year-old woman who you suspect is under the influence of a drug, the patient tells you that she was "listening to the painting on the wall" before you arrived. Her pulse rate and blood pressure are both elevated. This clinical presentation is MOST consistent with the use of: A) LSD. B) PCP. C) marijuana. D) methamphetamine.

A

107. A 29-year-old woman was found unresponsive by her husband. When you arrive at the scene and begin your assessment, you note that the patient's respirations are slow and shallow, her pulse is slow and weak, and her pupils are markedly dilated. Your partner begins assisting the patient's ventilations as you assess her blood pressure, which is 70/48 mm Hg. The patient's husband hands you an empty bottle of phenobarbital, which was filled the day before, and tells you that his wife takes the medication for seizures. After establishing vascular access, you should: A) administer crystalloid fluid boluses to improve her blood pressure. B) give her up to 10 mg of naloxone to reverse the effects of the drug. C) instruct your partner to hyperventilate the patient at 24 breaths/min. D) begin a dopamine infusion at 10 µg/kg/min and titrate as needed.

A

111. A woman drives her husband to your EMS station after he was exposed to a large amount of pesticide. Your assessment reveals that he is responsive to pain only, is hypoventilating, is markedly bradycardic, and is incontinent of urine and feces. The cardiac monitor reveals marked sinus bradycardia. As your partner assists the patient's ventilations, you should: A) establish vascular access and begin administering atropine sulfate. B) administer 1 to 2 mg of pralidoxime IM and transport immediately. C) obtain a 12-lead ECG tracing to detect signs of myocardial injury. D) start an IV line and give sodium bicarbonate to alkalinize his urine.

A

112. You are dispatched to a residence for a 61-year-old woman with flu-like symptoms. Upon your arrival, the patient greets you at the door. She complains of a headache and nausea, and tells you that she has vomited twice. Her husband, who is lying on the couch in the living room, began experiencing the same symptoms at about the same time. You should: A) remove both patients from the residence at once. B) immediately open all of the windows in the house. C) carefully assess the residence for any unusual findings. D) suspect that both patients have been exposed to cyanide.

A

________ radiation is very penetrating and easily passes through the body and solid materials. A) Alpha B) Beta C) Gamma D) Ionizing

C) Gamma

20. A middle-aged woman who has been taking 2 mg of clonazepam each day for 6 months finds that she now requires 4 mg each day to achieve the same effect. This is an example of: A) tolerance. B) drug abuse. C) habituation. D) physical dependence.

A

21. Bleeding from a scalp laceration with an underlying skull deformity: A) may contribute to hypovolemia in adults. B) commonly causes severe shock in adults. C) should be controlled with firm direct pressure. D) is limited due to the scalp's minimal vasculature.

A

23. Which of the following statements regarding alcoholism is correct? A) A person who consumes alcohol is considered to be physically dependent if abrupt cessation of drinking causes withdrawal symptoms. B) Patients with alcoholism typically do not become psychologically dependent on alcohol until they have been drinking for many years. C) Delirium tremens occur any time a person suddenly stops drinking excessive amounts of alcohol, regardless of whether or not he or she is addicted. D) Increased blood pressure and hallucinations are common physical manifestations when a short-term alcoholic slowly tapers his or her consumption.

A

25. What type of skull fracture is MOST common following high-energy direct trauma to a small surface area of the head with a blunt object? A) Depressed fracture B) Open fracture C) Basilar fracture D) Nondisplaced fracture

A

28. Secondary brain injuries include all of the following, EXCEPT: A) axonal injury. B) cerebral edema. C) cerebral ischemia. D) intracranial hemorrhage.

A

29. Patients with delirium tremens often experience: A) hallucinations. B) AV heart blocks. C) hypothermia. D) acute hypertension.

A

3. The bioavailability and excretion rate of a toxin are influenced by the: A) amount of toxin and the relative speed at which it is metabolized. B) type of toxin and the condition of the patient's underlying health. C) route by which the toxin entered the body and the age of the patient. D) the presence of other substances in the body at the time of exposure.

A

34. The clinical presentation of a stimulant abuser includes: A) excitement, hypertension, tachycardia, and dilated pupils. B) somnolence, hypotension, bradycardia, and a staggering gait. C) hypotension, tachycardia, constricted pupils, and hypothermia. D) an irregular pulse, hyperpyrexia, hypotension, and bradycardia.

A

35. Crack is a combination of: A) cocaine, baking soda, and water. B) marijuana, heroin, and baking soda. C) heroin, cocaine, and distilled water. D) ecstasy, marijuana, and alcohol.

A

39. Unlike the effects of cocaine, the effects of methamphetamine: A) last much longer. B) often result in paranoia. C) can be reversed with naloxone. D) predispose the patient to violence.

A

41. Which of the following roles would the code team leader MOST likely perform? A) Interpreting the ECG B) Establishing IV access C) Managing the airway D) Chest compressions

A

43. A 100-pound violent young woman who requires four large paramedics to subdue and contain her is MOST likely under the influence of: A) PCP. B) LSD. C) ketamine. D) mescaline.

A

45. Fluid-refractory hypotension following a barbiturate overdose is treated MOST effectively with: A) dopamine. B) naloxone. C) atropine. D) calcium.

A

47. Pupils that are slow (sluggish) to react to light: A) are a sign of cerebral hypoxia. B) indicate impending brain herniation. C) suggest significant intracranial pressure. D) indicate compression of an oculomotor nerve.

A

49. Which of the following drugs is a narcotic? A) Meperidine B) Flumazenil C) Secobarbital D) Lorazepam

A

54. Which of the following cardiac medications has a small therapeutic window and the GREATEST propensity to reach toxic levels? A) Digoxin B) Vasotec C) Cardizem D) Lisinopril

A

56. Signs and symptoms of organophosphate poisoning include: A) vomiting. B) tachycardia. C) constipation. D) pupillary dilation.

A

61. The MOST important prehospital treatment intervention for a patient with carbon monoxide poisoning is: A) high-flow oxygen. B) establishing vascular access. C) cardiac rhythm monitoring. D) monitoring pulse oximetry.

A

62. With hyperbaric oxygen therapy, carbon monoxide is typically eliminated from the body within: A) 15 to 20 minutes. B) 60 to 90 minutes. C) 90 to 120 minutes. D) 120 to 240 minutes.

A

67. In adult patients, oral ingestion of a caustic substance: A) is usually intentional. B) causes immediate death. C) contraindicates intubation. D) requires activated charcoal.

A

68. Unlike dermal exposure to a strong acid, dermal exposure to a strong alkali: A) requires longer irrigation with water because alkalis are less water soluble. B) should not be treated by irrigation with water as this will worsen the burn. C) should be neutralized on the skin by applying lemon juice or dilute vinegar. D) generally causes less damage to the skin because alkalis are water soluble.

A

71. Vagal tone remains intact following a spine injury because: A) the vagus nerve originates outside the medulla and regulates the heart via the carotid arteries. B) parasympathetic nerve fibers are largely under the control of the involuntary nervous system. C) the nerve fibers of the parasympathetic nervous system do not originate from the brainstem or spinal cord. D) the thoracolumbar system provides parasympathetic stimulation to the periphery via alpha and beta receptors.

A

74. A patient who ingested a significant amount of ethylene glycol 6 hours ago would MOST likely present with: A) slurred speech and ataxia. B) hypertension and tachycardia. C) an ethanol odor on the breath. D) flank pain and absent urination.

A

75. Spray paints and lacquer thinner contain __________, and typically cause __________________ when they are inhaled recreationally. A) toluene, hallucinations and mania B) carbon tetrachloride, CNS depression C) methylene chloride, pulmonary edema D) benzene, drunken behavior and dizziness

A

77. Which of the following medications is NOT a tricyclic antidepressant? A) Prozac B) Aventyl C) Sinequan D) Tofranil

A

83. Which of the following statements regarding SSRIs is correct? A) SSRIs have fewer anticholinergic and cardiac effects than tricyclics. B) Bradycardia with AV heart block is a hallmark sign of SSRI toxicity. C) The most popular SSRIs include Pamelor, Zonalon, and Norpramin. D) They are the least preferred antidepressant because they are cardiotoxic.

A

88. A patient who ingested a significant quantity of acetaminophen less than 24 hours ago would MOST likely present with: A) malaise, nausea, and a loss of appetite. B) signs of renal failure and severe vomiting. C) pain in the right upper abdominal quadrant. D) flushed skin, high fever, and abdominal pain.

A

9. Unlike an opioid, an opiate: A) is a natural product derived from opium. B) produces a distinctly different toxidrome. C) is not reversed by naloxone administration. D) is a synthetic, non-opium-derived narcotic.

A

90. Once in the body, approximately 90% of inorganic lead accumulates in: A) bone. B) the liver. C) the spleen. D) white blood cells.

A

91. Hematologic manifestations of lead poisoning include: A) anemia. B) leukopenia. C) coagulopathy. D) polycythemia.

A

96. The toxic chemical in castor beans is: A) ricin. B) cyanide. C) lantadene A. D) caladium oxalate.

A

A 26-year-old female presents with a fever, rash, and joint pain. Which of the following should you suspect? A) Lupus B) Anaphylaxis C) Scleroderma D) Allergic reaction

A

A 31-year-old man presents with diffuse hives, intense itching, and watery eyes that began acutely about an hour ago. He is conscious and alert, is breathing without difficulty, and tells you that he does not have any allergies or significant medical problems. His blood pressure is 126/76 mm Hg, pulse is 110 beats/min and strong, and respirations are 16 breaths/min and unlabored. The MOST appropriate drug, dose, and route for this patient are: A) diphendydramine, 25 to 50 mg, IM. B) epinephrine, 0.3 mg 1:1,000, IV. C) albuterol, 2.5 mg, via nebulizer. D) epinephrine, 0.01 mg/kg 1:1,000, SQ.

A

A 55-year-old female complains of severe vertigo, tinnitus, nausea, and a sense of fullness in her right ear. You should: A) administer an antiemetic and monitor her airway in case she vomits. B) carefully inspect her ear with an otoscope and administer furosemide. C) administer diphenhydramine to reduce any swelling in her inner ear. D) keep her supine, administer crystalloid fluid boluses, and inspect her ear.

A

A 6-year-old, 40-pound child remains in ventricular fibrillation after an initial defibrillation and 2 minutes of CPR. Vascular access has not been obtained. Your next action should be to: A) defibrillate with 70 joules. B) insert an advanced airway. C) check for a carotid pulse. D) insert an IO catheter.

A

A 70-year-old homeless man presents with a rash to his hands, wrists, and ankles. He denies any known allergies and states that the rash itches severely at night. His vital signs are stable, and he is breathing without difficulty. You should: A) transport him to the hospital and thoroughly wash your hands after patient care has been completed. B) establish vascular access in case he begins to experience signs and symptoms of a severe allergic reaction. C) be highly suspicious that he has body lice and use a high-level disinfectant when cleaning the ambulance. D) administer 25 mg of diphenhydramine IM and transport him to an appropriate medical facility.

A

A 90-year old nursing home resident presents with confusion and a cough after several days of complaining of being weak. She is bedridden, has numerous medial conditions, and takes a variety of medications. Your assessment reveals that her temperature is 99 F, her skin is cool and dry, her BP is 118/66 mm Hg, and her heart rate is 68 beats/min and regular. She is not experiencing obvious respiratory distress and her oxygen saturation is 93% on room air. this p/t is most likely experiencing a. pneumonia b. heart failure c. hypoglycemia d. a pulmonary embolism

A

A confused, bedridden resident in a skilled nursing facility who tells you that he or she cooks dinner for the other residents each night: A) is confabulating. B) is experiencing delirium. C) has a disorder of affect. D) is experiencing echolalia.

A

A distorted sense of smell in which a person perceives unpleasant odors when the odors do not exist is called: A) dysosmia. B) hyperosmia. C) presbyosmia. D) anosmia.

A

A known intravenous drug abuser presents with anorexia, body aches, a low-grade fever, and scleral icterus. She is very quiet and is not willing to share her medical history information with you. Which of the following additional clinical signs would reinforce your suspicion regarding the cause of this patient's condition? A) Jaundiced skin B) Blood-tinged sputum C) Swollen lymph glands D) Red or purple skin lesions

A

A man who was using an arc welder without eye protection presents with bilateral eye pain and diminished vision. He is conscious and alert, has a patent airway, and has stable vital signs. Treatment should include: A) covering his eyes with sterile, moist dressings; applying cool compresses lightly over his eyes; and placing him in a supine position. B) immediately irrigating his eyes with sterile saline or water; covering both eyes with dry, sterile dressings; and allowing him to sit up. C) placing him in a lateral recumbent position, keeping his eyelids closed with tape, and applying chemical warm compresses to his eyes. D) mixing baking powder with sterile water or saline and irrigating his eyes to prevent further damage caused by the ultraviolet light.

A

A patient who is taking Aventyl: A) may have nonspecific T-wave changes on the ECG. B) is usually hospitalized for debilitating, chronic depression. C) can safely take an MAOI drug at the same time. D) responds better than he or she will respond to treatment with a tricyclic antidepressant.

A

A patient with a loss of peripheral vision MOST likely has damage to the: A) retina. B) pupil. C) cornea. D) lens.

A

A person is exposed to the mumps virus, is asymptomatic for 16 days, and then becomes ill. The 16-day period is called the: A) incubation period. B) resistance period. C) virulent period. D) communicable period.

A

A psychotic episode MOST often leads to violence because of: A) exaggerated fear or paranoia. B) overreaction of the paramedic. C) the influence of an illicit drug. D) the patient's family's reaction.

A

A spinal cord concussion is: A) caused by a short-duration shock or pressure wave within the cord. B) a condition that results in a permanent loss of neurologic function. C) characterized by temporary dysfunction that lasts for up to 1 week. D) the result of direct trauma and is associated with spinal cord edema.

A

A state of delusion in which a person is out of touch with reality is MOST appropriately termed: A) psychosis. B) derealization. C) schizophrenia. D) tangential thinking.

A

A yeast infection that causes white patches in the mouth or on the tongue is called: A) thrush. B) gingivitis. C) leukoplakia. D) halitosis.

A

According to the Centers for Disease Control and Prevention, all children should be immunized against all of the following diseases, EXCEPT: A) smallpox. B) hepatitis B. C) seasonal influenza. D) Haemophilus influenzae type b.

A

According to the Ryan White Comprehensive AIDS Resources Emergency Act, the medical facility must: A) release the source patient's status to the designated infection control officer. B) hold the source patient's laboratory results for 24 hours before releasing them. C) avoid releasing the source patient's status to anyone due to HIPAA regulations. D) release the source patient's HIV and hepatitis B status to the exposed employee.

A

An agitated man with an antisocial personality: A) will not think twice about hurting you. B) is at highest risk for suicidal behavior. C) may be demanding and dictate your care. D) has a morbid fear of crowded environments.

A

Patients with a corneal abrasion may present with all of the following, EXCEPT: A) hyphema. B) severe pain. C) photophobia. D) excess tearing.

A

An unresponsive patient with signs of anaphylactic shock: A) indicates cerebral hypoxia and hypercarbia. B) requires immediate antihistamine therapy. C) is often unable to be intubated successfully. D) needs hyperventilation with a bag-mask device.

A

Anterior-posterior placement of the defibrillation pads should be used if the patient is younger than ___ year(s) of age or less than ___ kg.

A

Antibiotic therapy following exposure to meningitis is NOT appropriate for individuals who are: A) taking birth control pills. B) older than 45 years of age. C) asymptomatic after 24 hours. D) severely immunocompromised.

A

Aqueous humor: A) is contained in the anterior chamber of the eye. B) cannot be replenished by the body if it is lost. C) is contained in the posterior chamber of the eye. D) is found between the iris and the lens of the eye.

A

As you are administering 40 units of vasopressin to an adult woman in ventricular fibrillation, your team members continue CPR. After 2 minutes, you reassess her and determine that she is still in ventricular fibrillation. You should: A) instruct your team to continue CPR as the defibrillator is charging. B) administer 300 mg of amiodarone while CPR remains uninterrupted. C) deliver a monophasic defibrillation with 200 joules and resume CPR. D) advise your team to stop CPR as you prepare to deliver another shock.

A

Assessment of patients with collagen vascular diseases should focus on: A) ruling out life threats. B) high-dose analgesia. C) crystalloid fluid boluses. D) high-flow oxygen therapy.

A

At the present time, the ratio of caregivers to older people is: a. low b.high c. adequate d. increasing consistently

A

At what age do the sutures of the skull solidify and the fontanelles close? A) 18 months B) 2 years C) 36 months D) 4 years

A

Chlamydia trachomatis is caused by a _________ and is treated with an _________. A) bacterium, antibiotic B) virus, antiviral drug C) fungus, antifungal drug D) virus, immunoglobulin

A

Common clinical findings associated with a subdural hematoma include all of the following, EXCEPT: A) rapidly increasing intracranial pressure. B) an underlying skull fracture. C) a fluctuating level of consciousness. D) unilateral hemiparesis or slurred speech

A

Delirium is most accurately defined as: a. an acute alteration in mentation that indicates an underlying condition b. any alteration in cognitive function that may or may not be reversible c. a pattern of disorganized thinking that progresses over several weeks d. an altered mental status that is caused by structural damage to the brain

A

Patients with epiglottitis present with: A) dysphagia. B) wheezing. C) dentalgia. D) hyperosmia.

A

Detecting disorders of perception is often difficult because: A) patients are often hesitant to answer direct questions regarding hallucinations or illusions. B) patients experiencing a behavioral emergency are generally agitated and uncooperative. C) the paramedic's line of questioning is often too difficult or impossible for the patient to understand. D) it is often difficult for the paramedic to determine the patient's baseline level of mentation.

A

Early signs and symptoms of increased intracranial pressure include: A) headache and vomiting. B) hypertension and bradycardia. C) widening of the pulse pressure. D) arm flexion and leg extension.

A

Epinephrine is primarily administered during cardiac arrest because its ________effects cause ________. A) vasoconstrictive, enhanced coronary blood flow B) antidysrhythmic, decreased cardiac irritability C) beta-2 agonistic, dilation of the bronchioles D) vasodilatory, a reduction in cardiac afterload

A

Following a traumatic injury, a 19-year-old woman presents with confusion, tachycardia, and significant hypotension. Her skin is cool, clammy, and pale. Further assessment reveals abdominal rigidity and deformity with severe pain over her thoracic vertebrae. In addition to administering high-flow oxygen and immobilizing her spine, you should: A) start at least one large-bore IV line and give crystalloid boluses as needed to maintain adequate perfusion. B) conclude that she is in neurogenic shock, start an IV line of normal saline, and initiate a dopamine infusion. C) start at least one large-bore IV of normal saline and administer a narcotic analgesic to treat her severe pain. D) perform a focused history and physical exam, start an IV of normal saline, and administer a corticosteroid.

A

Following exposure to the hepatitis B virus, a person may remain asymptomatic for up to: A) 200 days. B) 250 days. C) 300 days. D) an entire year.

A

Herniation of an intervertebral disc occurs when: A) stress on the vertebral column forces a disc into the spinal canal. B) the spinal column sustains flexion or extension of more than 50%. C) the spinous process fractures and puts pressure on one or more discs. D) significant blunt trauma displaces the disc away from the spinal cord.

A

Hyperventilation of the brain-injured patient: A) shunts oxygen away from the brain and may result in decreased cerebral perfusion pressure. B) has clearly demonstrated decreased mortality and morbidity in patients with a severe head injury. C) causes cerebral vasodilation with increased intracranial pressure and should be avoided. D) is only appropriate if the patient is unresponsive and has bilaterally dilated and sluggishly reactive pupils.

A

Hypotension that is associated with neurogenic shock is the result of: A) loss of alpha receptor stimulation. B) concomitant internal hemorrhage. C) increased peripheral vascular tone. D) profound peripheral vasoconstriction.

A

In addition to high-quality CPR, which of the following interventions has clearly made a positive and measurable difference in survival from sudden cardiac arrest? A) Defibrillation B) IV fluid boluses C) Epinephrine therapy D) Tracheal intubation

A

In contrast to cellular immunity, humeral immunity: A) involves the use of antibodies dissolved in the blood plasma to fight off invading organisms. B) is the result of the body's production of leukocytes called T cells that attack and destroy invaders. C) is an acquired form of immunity that involves desensitization through the use of immunizations. D) protects the body against foreign substances by antibodies that are located exclusively in the lymph nodes.

A

In contrast to the oculomotor nerve, the optic nerve: A) is the second cranial nerve and provides the sense of vision. B) carries parasympathetic nerve fibers that constrict the pupil. C) is the third cranial nerve and regulates movement of the eyes. D) innervates the muscles that cause motion of the upper eyelids.

A

In developing countries, there is a strong association between the hepatitis E virus and: A) inadequate hygiene. B) sexual intercourse. C) infection with HIV. D) blood transfusions.

A

In the absence of IV or IO access, the ____ route is the preferred route for the administration of epinephrine to a patient in anaphylactic shock. A) IM B) ET C) SQ D) intradermal

A

In which of the following situations would spinal motion restriction precautions likely NOT be necessary? A) Syncopal episode in which the patient was already seated or supine B) Unrestrained occupant of moderate- to high-speed motor vehicle crash C) Isolated head injury without gross signs or symptoms of a spinal injury D) Vehicular damage with compartmental intrusion of greater than 12 inches

A

Law enforcement personnel request your assistance in caring for a violent patient. When you arrive at the scene, the patient, a 48-year-old man, is yelling obscenities and is threatening to kill anyone who comes near him. Despite your attempts to calm him verbally, he continues his threatening behavior. It is MOST important for you to: A) ensure that you have a route for rapid egress and visually scan the patient for potential weapons. B) utilize at least four people to physically restrain the patient so you can administer haloperidol. C) leave the scene immediately and allow law enforcement personnel to take control of the situation. D) ask law enforcement to immobilize the patient with a TASER so you can assess his blood glucose level.

A

Many EMS personnel may experience a lack of confidence in their ability to deal with emotional disturbances for all of the following reasons, EXCEPT: A) the uncontrollable fear of personal injury. B) the fact that they are action-oriented people. C) they like to see tangible results of their actions. D) their training only addresses such issues topically.

A

Many paramedics are reluctant to broach the subject of suicide with a depressed patient for fear that: A) they might put ideas into the patient's head. B) the patient may become violent and homicidal. C) the patient will be offended by such a question. D) the patient will become even more depressed.

A

Mastication is defined as: A) chewing. B) swallowing. C) digesting. D) teeth clenching.

A

Metabolic causes of abnormal behavior include: A) diabetic ketoacidosis. B) vitamin deficiencies. C) thyroid hyperfunction. D) amphetamine toxicity.

A

Mononucleosis is caused by the ___________________ and grows in the ________________. A) Epstein-Barr virus, epithelium of the oropharynx B) pneumococcal bacterium, inner lining of the lungs C) streptococcus bacterium, epithelial cells of the trachea D) cytomegalovirus, nasopharyngeal mucous membranes

A

Patients who alternate between mania and depression: A) are referred to as bipolar. B) present with a flat affect. C) have multiple personalities. D) have a history of schizophrenia.

A

Patients with glaucoma often complain of: A) a blind spot toward the center of vision. B) a sudden painless complete loss of vision. C) specks or floaters in the field of vision. D) intense burning or itching around the eyes.

A

People who are overly concerned with their physical health and appearance MOST likely have a(n): A) somatoform disorder. B) dissociative disorder. C) personality disorder. D) compulsive disorder.

A

Personal protective equipment: A) serves as a secondary protective barrier beyond what your body provides. B) is a standardized set of equipment that is used with every patient contact. C) is the most effective means of preventing the spread of an infectious disease. D) is required by the CDC when a paramedic draws blood or gives an injection.

A

Prehospital treatment of the patient with a traumatic brain injury must focus primarily on: A) maintaining cerebral perfusion pressure. B) hyperventilating the patient at 20 breaths/min. C) maintaining a systolic blood pressure of at least 120 mm Hg. D) taking measures to decrease intracranial pressure.

A

Retinal detachment is especially common in: A) boxing. B) football. C) diabetics. D) water sports.

A

Retinal injuries that are caused by exposure to extremely bright light: A) are generally not painful but may result in permanent damage. B) cause immediate intense pain and result in permanent blindness. C) are painless and almost always heal without permanent damage. D) cause severe pain but heal quickly and without permanent damage.

A

Rubella is characterized by: A) a low-grade fever. B) visual disturbances. C) abdominal discomfort. D) an isolated facial rash.

A

Secondary syphilitic infection is characterized by all of the following, EXCEPT: A) petechiae. B) a skin rash. C) patchy hair loss. D) swollen lymph glands.

A

Signs and symptoms of retinal detachment include: A) flashing lights, specks, or floaters in the field of vision. B) double vision and partial or complete loss of peripheral vision. C) immediate pain and total loss of vision following blunt eye trauma. D) paralysis of upward gaze and greater than 50% loss of central vision.

A

Spinal cord injury without radiographic abnormalities (SCIWORA. can occur in children because: A) their vertebrae lie flatter on top of each other. B) they have excessive mobility of C1 and C2. C) unlike adults, their vertebrae are more curved. D) their spinal cord is more compressed than an adult's.

A

The main disadvantage of using a scoop stretcher to transfer a patient to a long backboard is: A) inability to conduct a visual exam of the back for injuries. B) patient discomfort due the cold metal frame of the scoop. C) unnecessary patient movement and the risk for further harm. D) inability to palpate the spinal vertebrae for gross deformities.

A

The only area of the spine that allows for significant rotation is: A) C1-C2. B) C2-C4. C) C6-C7. D) T1-T2.

A

Systemic lupus erythematosus is a disease caused by: A) a multisystem autoimmune disorder. B) a marked deficiency of neutrophils. C) primary immune system failure. D) excessive IgE antibody production.

A

Systemic signs of a dental abscess include: A) fever and chills. B) inflammation. C) swelling and pain. D) mandibular pain.

A

The MOST ominous respiratory sign in a patient with anaphylactic shock is: A) diminished lung sounds. B) loud expiratory wheezing. C) diffuse coarse crackles. D) labored tachypnea.

A

The MOST significant complication associated with prolonged immobilization of a patient on a long backboard is: A) pressure lesion development. B) compression of the vena cava. C) increased intracranial pressure. D) patient discomfort and frustration

A

The ONLY indication for removing contact lenses in the prehospital setting is: A) chemical eye burns. B) acute conjunctivitis. C) cardiopulmonary arrest. D) a foreign body in the eye.

A

The __________ is the largest component of the central nervous system and contains billions of neurons that serve a variety of functions. A) brain B) medulla C) cerebellum D) spinal cord

A

The ___________ nerve provides motor function to the muscles of the tongue. A) hypoglossal B) trigeminal C) accessory D) glossopharyngeal

A

The avian flu: A) is caused by a virus that occurs naturally in the bird population. B) is typically contracted by people who cook and eat infected chickens. C) has been linked directly to the hantavirus found in the feces of rodents. D) is preventable if antiviral drugs are given within 48 hours after exposure.

A

The clinical presentation of mononucleosis includes: A) fever, swollen lymph glands, and an enlarged spleen. B) vomiting, a fever greater than 102°F, and shaking chills. C) hemoptysis, low-grade fever, and up to 10% weight loss. D) nasal drainage, a dry cough, and right upper quadrant pain.

A

The communicable period for HIV: A) is largely unknown. B) ranges from 7 to 10 days. C) begins at the onset of infection. D) is decreased with antiretroviral therapy.

A

The dura mater: A) folds in to form the tentorium, a structure that separates the cerebral hemispheres from the cerebellum and brainstem. B) is the middle meningeal layer and is comprised of a delicate transparent membrane that is damaged easily by trauma. C) anatomically separates the cerebellum and the brainstem and contains vasculature that resembles a spider web. D) is the inner meningeal layer and is comprised of a thin, translucent, highly vascular membrane that adheres firmly directly to the surface of the brain.

A

The primary target of infection with the human immunodeficiency virus is the: A) immune system. B) lymphatic system. C) pulmonary system. D) central nervous system.

A

The primary treatment for hypotension secondary to anaphylaxis is: A) epinephrine. B) diphenhydramine. C) isotonic crystalloid. D) a dopamine infusion.

A

The social assessment component of the GEMS diamond includes: a. determining if activities of daily living are being provided by another person. b. considering that what appears to be a medical problem may have a trauma component. c. assessing the presence of functional smoke detectors in the patient's home. d. looking for multiple prescription medications that are used to treat the same condition.

A

The spine: A) is the major structural component of the axial skeleton. B) is comprised of irregular bones that are all fused together. C) consists of 23 bones articulating to form the spinal column. D) provides support and strength for the appendicular skeleton.

A

The third dose of the three-series hepatitis B vaccine is given: A) 6 months after the first dose. B) 4 weeks after the second dose. C) 12 months after the initial dose. D) within 2 to 3 months of the second dose.

A

The upper thoracic spinal nerves: A) innervate the muscles of the chest that help in breathing and coughing. B) regulate the parasympathetic nervous system and inhibit inspiration. C) are the only pair of spinal nerves that do not truly exit the spinal cord. D) provide abdominal muscle control and contain sympathetic nerve fibers.

A

Therapeutic doses of certain drugs may reach toxic levels in older people due to deterioration of the: a. liver b. spleen c. gallbladder d. intestinal tract

A

Venlafaxine is the generic name for: A) Effexor. B) Lexapro. C) Librium. D) Neurontin.

A

Vertigo or loss of balance following an ear infection or upper respiratory infection is MOST consistent with: A) labyrinthitis. B) Meniere disease. C) otitis externa. D) impacted cerumen.

A

When a person is vaccinated against a disease: A) the body develops antibodies in response to the vaccine and produces an immune response before the disease can enter the body and cause damage. B) the immune system does not produce any antibodies against that particular disease unless the person is directly or indirectly exposed to it. C) a secondary response occurs, as antibodies are produced and the vaccinated person experiences a milder form of the disease against which he or she has been vaccinated. D) smaller titers of the disease are injected into the person, which results in the production of antibodies that cause histamine release and a mild allergic reaction.

A

When assessing a patient with a behavioral emergency, the MOST important assessment tool you have is: A) your mind. B) a penlight. C) a glucometer. D) cardiac monitoring.

A

When assessing an unresponsive older person's airway it is most important to remember that: a. delated gastric emptying increased the risk of aspiration b/ age-related tooth loss may cause obstruction of the airway c. a marked reduction in saliva cases dryness of the oral mucosa d. the head should not be extended due to weakened cervical vertebrae

A

When confronted with a feared object or situation, the phobic person: A) experiences intolerable anxiety and autonomic symptoms. B) truly believes that his or her fear is completely reasonable. C) is usually able to mitigate his or her fear with redirection. D) typically becomes catatonic and unable to communicate.

A

When extreme anxiety occurs in conjunction with restlessness, a patient: A) becomes agitated. B) is not distractible. C) often becomes suicidal. D) presents with a flat affect.

A

When immobilizing a patient to a long backboard, you should take standard precautions and then: A) ensure that the patient's head is stabilized manually. B) assess distal pulse, sensory, and motor functions. C) log roll the patient as a unit and assess his or her back. D) apply the appropriately sized cervical collar to the patient.

A

When painful memories are repressed, __________ is impaired. A) recall B) retention C) recognition D) registration

A

When performing a cranial nerve assessment of a patient with a suspected spinal injury, you note that the patient's pupil is constricted and the upper eyelid droops. This indicates an injury to: A) C3. B) C4. C) C5. D) C6

A

When practicing a "code," whether in the prehospital or in-hospital setting, the primary focus should be on: A) teamwork and minimal interruptions in CPR. B) the use of technology during a cardiac arrest. C) timely defibrillation and early tracheal intubation. D) independent performance and drug administration.

A

When using an impedance threshold device during cardiac arrest, it is important to: A) deliver each ventilation over a period of 1 second. B) increase the ventilation rate by 6 to 8 breaths/min. C) allow partial chest recoil in between compressions. D) hyperinflate the lungs to improve coronary perfusion.

A

Which of the following clinical signs or symptoms are MOST indicative of upper airway swelling in a patient experiencing a severe allergic reaction? A) Hoarseness and stridor B) Crackles and wheezing C) Facial edema and coughing D) Chest tightness and dyspnea

A

Which of the following conditions has been associated with death following exposure to a TASER device? A) Excited delirium B) Rhabdomyolysis C) Spinal cord injury D) Electrocution injury

A

Which of the following conditions that can cause an airway obstruction is unique to patients with an injury to the upper cervical spine? A) Retropharyngeal hematoma B) Blood or secretions in the mouth C) Oropharyngeal occlusion by the tongue D) Improperly inserted oropharyngeal airway

A

Which of the following conditions would be the LEAST likely to produce psychotic symptoms? A) Hyperglycemia B) Closed head injury C) Low cardiac output D) Temporal lobe seizures

A

Which of the following immunoglobulins is thought to stimulate antibody-producing cells to make antibodies? A) IgD B) IgE C) IgG D) IgM

A

Which of the following is NOT a general guideline to follow when caring for a patient with a psychiatric problem? A) Be indirect when possible. B) Provide honest reassurance. C) Remain confident in your abilities. D) Maintain a nonjudgmental attitude.

A

Which of the following is NOT included in the Centers for Disease Control and Prevention's list of recommended immunizations and tests for health care providers? A) Annual HIV testing B) Hepatitis B vaccine C) TB testing D) Measles, mumps, and rubella

A

Which of the following statements about panic disorder is correct? A) Panic disorder is characterized by sudden, unexpected fear and dread. B) Most initial panic attacks occur without a known precipitating stressor. C) Men are two thirds more likely to experience panic disorder than women. D) Panic disorder is a random disorder that usually does not run in the family.

A

Which of the following statements regarding a closed head injury is correct? A) In a closed head injury, the dura mater remains intact. B) Diffuse brain injury occurs with all open head injuries. C) Closed head injuries are less common than open head injuries. D) Intracranial pressure is usually minimal in a closed head injury.

A

Which of the following statements regarding anisocoria is correct? A) Anisocoria is a condition in which the pupils are unequal. B) Anisocoria is a normal finding in the majority of the population. C) Physiologic anisocoria indicates significant intracranial pressure. D) Anisocoria is a condition characterized by bilateral pupillary dilation.

A

Which of the following statements regarding central cord syndrome is correct? A) The patient typically presents with greater loss of function in the upper extremities than in the lower extremities. B) Central cord syndrome is almost always associated with a vertebral fracture and has an overall poor prognosis. C) Patients with cervical spondylosis or stenosis are at a lower risk for central cord syndrome following an injury. D) Central cord syndrome typically causes complete paralysis of the lower extremities and decreased proprioception.

A

Which of the following statements regarding meningitis is correct? A) Most epidemic outbreaks involve meningococcal meningitis. B) Neisseria meningitidis is the least common type of meningitis. C) The viral form of meningitis is a highly communicable disease. D) Meningitis is an acute viral inflammation of the cerebral meninges.

A

Which of the following statements regarding the brain is correct? A) The brain uses 45 to 50 L/min of oxygen. B) The brain occupies approximately 60% of the cranium. C) The brain metabolizes minimal amounts of glucose. D) The brain has the ability to store oxygen and glucose.

A

Which of the following statements regarding the nasal septum is correct? A) It may be slightly deviated to one side or the other. B) The nasal septum is comprised mainly of cartilage. C) Inflammation of the nasal septum is common during infection. D) The nasal septum separates the oropharynx and nasopharynx.

A

Which of the following statements regarding the use of vasopressin during cardiac arrest is correct? A) Vasopressin may be substituted for the second dose of epinephrine. B) If the patient is in asystole, vasopressin can be given every 5 minutes. C) If used during cardiac arrest, vasopressin is given in a dose of 20 units. D) Vasopressin has been shown to be clinically superior to epinephrine.

A

Which of the following structures receives light impulses and converts them to nerve signals that are conducted to the brain by the optic nerve and interpreted as vision? A) Retina B) Cornea C) Pupil D) Lens

A

Which of the following words would MOST likely de-escalate the symptoms of a panic attack to a more manageable level? A) Safe B) Care C) Help D) Relax

A

While en route to the scene of a patient in cardiac arrest, the emergency medical dispatcher advises you that she has the caller on the phone, but the caller refuses to do CPR on the patient. With an estimated time of arrival at the scene of 5 minutes, you should: A) ask the dispatcher to inform the man to do chest compressions only. B) recall that the caller has a legal and moral duty to act in this situation. C) have the dispatcher reassure the caller that he will not contract a disease. D) advise the dispatcher to tell the caller that the patient will die without CPR.

A

You and your partner are off duty and are playing golf. Suddenly, you see an elderly man grab his chest and collapse to the ground. You should: A) tell your partner to call 9-1-1 as you proceed to assess the man. B) both proceed to the man and begin two-rescuer CPR if needed. C) perform 2 minutes of CPR if needed and then call an ambulance. D) tell the man's golf buddy to perform a precordial thump at once.

A

You are assessing a 34-year-old woman who appears to be depressed. You were summoned to see the patient by her husband, who tells you that she won't talk to him. During your interview of the patient, which of the following questions or comments would MOST likely yield a response from the patient? A) "You appear to be very sad." B) "Do you want to hurt anyone?" C) "Are you under psychiatric care?" D) "Are you mad at your husband?"

A

You are assessing a patient who sustained blunt trauma to the center of his back. He is conscious, but is unable to feel or move his lower extremities. His blood pressure is 80/50 mm Hg, pulse is 40 beats/min and weak, and respirations are 24 breaths/min and shallow. If IV fluids do not adequately improve perfusion, you should: A) give 0.5 mg of atropine and consider a dopamine infusion. B) administer 1 mg of epinephrine 1:10,000 via rapid IV push. C) apply the pneumatic antishock garment and inflate all sections. D) administer a corticosteroid to reduce spinal cord inflammation.

A

You are called to a residence for a 74-year-old woman who fainted. During your assessment, the patient's son tells you that he was standing next to his mother when she fainted and that he caught her before she fell to the floor. He further tells you that his mother has angina and kidney disease. The patient is now conscious but confused. Her blood pressure is 80/50mm Hg, pulse is 110 beats/min and weak, and respirations are 22 breaths/min and regular. As your partner administers supplemental oxygen to the patient, you should: a. assess for the presence of a nitroglycerin patch on the patient and remove it if she is wearing one. b. quickly establish vascular access and administer a 20-mL/kg bolus of an isotonic crystalloid. c. obtain a detailed list of all of the patient's medical problems and the medications that she is taking. d. perform a rapid head-to-toe assessment to determine if she has gross injuries or any hidden conditions.

A

You are dispatched to a skilled nursing care facility for a 74-year-old male resident who is ill. During your assessment, you note that the patient has his head cocked to the side and is unable to move it. The charge nurse tells you that the patient was placed on Seroquel 2 days ago. Based on this patient's clinical presentation and medication history, you should: A) administer 25 to 50 mg of diphenhydramine. B) suspect that the patient has catatonic schizophrenia. C) establish vascular access and administer naloxone. D) expect to find that his blood sugar level is elevated.

A

fall-related fractures in the elderly occur MOST often to the __________, usually in p/t with underlying ___________ a. hip, osteoporosis b. humerus, arthritis c. elbow, osteoporosis d. pelvis, osteoarthritis

A

You are transporting a young female who intentionally ingested a large quantity of her prescribed Pamelor. She is conscious, but drowsy, and complains of a dry mouth and blurred vision. The cardiac monitor reveals sinus tachycardia at 120 beats/min. You are administering high-flow oxygen and have established a patent IV line. With regard to her ECG rhythm, you should be especially alert for: A) QRS widening. B) AV heart block. C) QT interval narrowing. D) a prolonged PR interval.

A

You transported a patient with flu-like symptoms to the hospital 4 days ago. Your designated infection control officer advises you that the patient was diagnosed with the avian flu. If you documented an exposure to this patient, you will MOST likely be: A) offered an antiviral medication. B) referred to an infectious disease physician. C) restricted from duty for a 2-week period. D) mandated to get a regular flu vaccination.

A

You would MOST likely have to place several blankets or pillows under a patient's upper back prior to immobilization if he or she has: A) kyphosis. B) osteoarthritis. C) spondylosis. D) osteoporosis.

A

Your partner returns with the AED as you are performing CPR on a 5-year-old child. As he opens the AED, he tells you that there are no pediatric pads, only adult pads. You should: A) instruct him to apply the adult pads as you continue one-rescuer CPR. B) tell him to resume one-rescuer CPR as you try to locate pediatric pads. C) use the adult AED pads, but only provide a total of two defibrillations. D) continue two-rescuer CPR until a manual defibrillator is available.

A

aging brings a widespread decrease in bone mass, especially a. in postmenopausal women b. in men over 50 years of age c. if the person falls freq. d. in the presence of hypertension

A

an elderly person is more likely to sustain serious injury following trauma due to a. brittle demineralized bone b. ineffective vasoconstriction c. chronic renal hypertrophy d. decreased respiratory function

A

decreased elasticity of the lungs and calcification of the costochodral cartilage results in: a. an increase in residual lung volume b. a significant increase in vital capacity c.a decrease in airway size and resistance a decrease in the total amount of air in the lungs

A

elder p/ts experience a slower physiologic response to hypoxemia and hypercarbia b/c of: a. decreased sensitivity to changes in arterial blood content b.a chronically elevated partial pressure of carbon dioxide c. an increased PaO2 due to a natural increase in respirations d. failure of the baroreceptors to detect blood pressure changes

A

hypertensive emergencies in the geriatric population a. require a controlled decline in blood pressure that often cannot be achieved in the prehospital setting b. can cause a ruptured cerebral or aortic aneurysm and should be treated in the field with antihypertensives c. are relatively uncommon owing to increased elasticity of the blood vessels which facilitates vasodilation d. are typically treated in the prehospital setting with beta blockers

A

hypotension and orthostatic vital sign changes would most likely occur in elderly people who take a. diuretic b. beta blockers c. antidepressants d. ACE inhibitors

A

impaired proprioception predisposes an elderly person to : a. falls b. skin tears. c. depression d. hearing loss

A

physiologic age-related decreases in skin elasticity are the results of: a. decreased collagen and elastin production b. chronic use of corticosteroid medications c. slower replenishment of epidermal cells d. frequent fungal or viral skin infections

A

the risk of a pulmonary embolus increases with age b/c of a. increased immobility b. chronic use of aspirin c. frequent lung infections d. drug-induced bradycardia

A

the son of a 76 year old woman called 911 b/c his mother is ill. the p/t presents with mild confusion poor skin turgor and tachycardia. she is incontinent of urine and asks you for a glass of water her son tells you that she had not been feeling well for the past several weeks but would not allow him to take her to the doctor . which of the following assessment parameters will most likely reinforce your suspicion regarding the underlying cause of this patient's condition a. blood glucose b. cardiac rhythm c. orthostatic vital signs d. Cincinnati stroke scale

A

type 2 osteoporosis tends to progress more rapidly in a. postmenopausal women b. the lower extremity bones c. males over 60 years of age d. p/ts with excess calcium

A

which of the following is the best example of inadvertent medication noncompliance? a. a person take multiple doses of the same medication b/c of an inability to distinguish the flavors b. the person does not fill a prescription because of a lack of money or insufficient insurance coverage c. a person decreases the dosage of one or more prescribed medications to make them last longer d. a p/t discontinues a medication b/c he or she feels better and deems the medication unnecessary

A

115. The incidence of sudden infant death syndrome peaks between the ages of: A) 2 and 4 months. B) 3 and 6 months. C) 4 and 8 months. D) 8 and 12 months.

A) 2 and 4 months.

9. According to the Apgar score, a newborn with a heart rate of 80 beats/min and slow, irregular breathing should receive a combined score of: A) 2. B) 3. C) 4. D) 5.

A) 2.

73. The approximate total blood volume of a 60-pound child is: A) 2.2 L. B) 2.9 L. C) 3.4 L. D) 3.8 L.

A) 2.2 L.

170. A 6-year-old child has burns to his head, face, neck, and anterior chest. What percentage of his body surface area has been burned? A) 21% B) 27% C) 30% D) 36%

A) 21%

5. During normal menstruation, approximately ____ to ____ mL of blood is discharged from the vagina. A) 25, 65 B) 50, 75 C) 65, 80 D) 75, 100

A) 25, 65

77. Adequately perfused kidneys produce at least ___ to ___ mL of urine per hour. A) 30, 50 B) 50, 70 C) 70, 100 D) 100, 110

A) 30, 50

56. You have given an 800-mL normal saline bolus to a patient in shock. How much of this fluid will remain in the intravascular space after 20 minutes? A) 320 mL B) 380 mL C) 480 mL D) 600 mL

A) 320 mL

44. The blood pressure of an infant or child can be maintained with blood loss of up to: A) 35% to 40%. B) 40% to 45% C) 45% to 50% D) 50% to 55%

A) 35% to 40%.

You and your partner are transferring a severely burned patient from a community hospital to a burn specialty center. The patient, a 110-pound woman, has partial- and full-thickness burns that cover approximately 55% of her body. She has two large-bore IV lines in place, is intubated, and is on a cardiac monitor. According to the Parkland formula, how much normal saline should she receive in 30 minutes? A) 340 mL B) 355 mL C) 370 mL D) 395 mL

A) 340 mL

4. Which of the following types of maltreatment is perhaps the MOST common? A) Neglect B) Physical abuse C) Sexual abuse D) Abandonment

A) Neglect

12. When attempting to resuscitate a patient in cardiac arrest, which of the following questions would you pose if you suspect that the patient is hyperkalemic? A) "Does this patient undergo dialysis?" B) "Is this patient a known diabetic?" C) "Does this patient take blood thinners?" D) "Has this patient had vomiting or diarrhea?"

A) "Does this patient undergo dialysis?"

36. How much naloxone should you give to a 6.5-pound newborn with respiratory depression secondary to maternal narcotic administration? A) 0.3 mg B) 0.4 mg C) 0.5 mg D) 0.6 mg

A) 0.3 mg

83. Average blood loss during the third stage of labor is approximately: A) 150 mL. B) 250 mL. C) 400 mL. D) 500 mL.

A) 150 mL.

42. A 25-year-old woman presents with acute abdominal pain and vaginal bleeding. She tells you that she has soaked eight high-absorbency tampons in the past 2 hours. Approximately how much blood has she lost externally? A) 160 mL B) 200 mL C) 240 mL D) 300 mL

A) 160 mL

According to the rule of nines, an adult man with partial- and full-thickness burns to his head, face, and anterior chest has burns to ____% of his total body surface area. A) 18 B) 27 C) 36 D) 45

A) 18

56. What dose and concentration of glucose would be MOST appropriate for a 6-pound hypoglycemic newborn? A) 5.5 mL of 10% dextrose (D10) B) 6 mL of 25% dextrose (D25) C) 6.5 mL of 10% dextrose (D10) D) 7.5 mL of 25% dextrose (D25)

A) 5.5 mL of 10% dextrose (D10)

163. A 4-year-old girl presents with a fever of 103.2°F. The child's mother states that the fever came on suddenly and was not preceded by any symptoms. The child is conscious and alert with unlabored tachypnea, tachycardia, and a blood pressure that is consistent with her age. Prehospital treatment for this child includes all of the following, EXCEPT: A) 81 mg of aspirin. B) free-flow oxygen. C) 250 mg of acetaminophen. D) simple cooling measures.

A) 81 mg of aspirin.

134. Which of the following statements regarding burns in the pediatric patient is correct? A) A child's larger skin surface-to-body mass ratio increases his or her susceptibility to heat and fluid loss. B) A burn that is characterized by clear demarcation lines is generally suggestive of an unintentional burn. C) Unlike adults, the rule of palm is an inaccurate tool to determine the extent of burns in pediatric patients. D) A child with burns to both lower extremities has burns to approximately 36% of his or her body surface area.

A) A child's larger skin surface-to-body mass ratio increases his or her susceptibility to heat and fluid loss.

99. A 20-year-old female presents with severe lower abdominal pain. She does not believe that she is pregnant, but states that her breasts have been unusually tender and that she has been nauseated. What is the pathophysiology of this patient's suspected condition? A) A fertilized ovum is implanted somewhere other than the uterus. B) The placenta has prematurely detached from the uterine wall. C) The normal flow of bile from the liver has been interrupted. D) High levels of estrogen are affecting the gastrointestinal system.

A) A fertilized ovum is implanted somewhere other than the uterus.

79. Which of the following statements regarding paralysis is correct? A) A paralyzed patient has lost the ability to voluntarily move a body part. B) Paralysis always entails the loss of both sensory and motor functions. C) Injuries to the thoracic or lumbar spine generally result in quadriplegia. D) Most patients who are paralyzed have normal sensation or hyperesthesia.

A) A paralyzed patient has lost the ability to voluntarily move a body part.

44. Which of the following statements regarding Addison disease is MOST correct? A) Addison disease most often occurs when the immune system creates antibodies that attack and destroy the adrenal cortex. B) Addison disease is also known as secondary adrenal insufficiency and is most often the result of a viral infection. C) The signs and symptoms of Addison disease are a direct result of the overproduction of cortisol and aldosterone. D) Hallmark signs of Addison disease are hypertension and fluid retention that result from excess sodium reabsorption.

A) Addison disease most often occurs when the immune system creates antibodies that attack and destroy the adrenal cortex.

9. Which of the following statements regarding anatomic dead space is correct? A) Anatomic dead space is about 1 mL per pound of body weight. B) Air in the dead space participates in pulmonary gas exchange. C) The amount of dead space increases as tidal volume increases. D) If tidal volume is 500 mL, 200 mL remains in the dead space.

A) Anatomic dead space is about 1 mL per pound of body weight.

Which of the following statements regarding anisocoria is correct? A) Anisocoria is a condition in which the pupils are unequal. B) Anisocoria is a normal finding in the majority of the population C) Physiologic anisocoria indicates significant intracranial pressure. D) Anisocoria is a condition characterized by bilateral pupillary dilation.

A) Anisocoria is a condition in which the pupils are unequal.

27. Which of the following is a normal response of the body to hypoperfusion? A) Antidiuretic hormone is released by the pituitary gland. B) Vasodilation causes increased blood flow to the kidneys. C) Sodium and water are excreted to maintain the blood's pH. D) Peripheral vasoconstriction preserves blood flow to the skin.

A) Antidiuretic hormone is released by the pituitary gland.

41. Which of the following is a proper technique when accessing an implantable venous access device? A) Aspirate 5 mL of blood and then block the flow in the line with the crimping device. B) Stabilize the implantable device and insert the needle at a 45-degree angle to the skin. C) Flush the device with 20 mL of normal saline as soon as you are able to aspirate blood. D) Remove the syringe from the needle and then block the flow in the line with the crimping device.

A) Aspirate 5 mL of blood and then block the flow in the line with the crimping device.

24. ___________ respirations are characterized by a grossly irregular pattern of breathing that may be accompanied by lengthy periods of apnea. A) Biot B) Agonal C) Eupneic D) Cheyne-Stokes

A) Biot

169. You receive a call to a residence for a 6-year-old girl with a decreased level of consciousness. The child has hydrocephalus following surgery to remove a brain tumor and has a ventricular shunt in place. The child's level of consciousness is markedly decreased from its baseline, and the child's caregiver tells you that she thinks the shunt is obstructed. Which of the following sets of vital signs is MOST indicative of shunt obstruction and increased intracranial pressure? A) Blood pressure 140/92 mm Hg; pulse 58 beats/min; respirations 8 breaths/min B) Blood pressure 106/66 mm Hg; pulse 80 beats/min; respirations 14 breaths/min C) Blood pressure 90/50 mm Hg; pulse 110 beats/min; respirations 10 breaths/min D) Blood pressure 130/68 mm Hg; pulse 70 beats/min; respirations 28 breaths/min

A) Blood pressure 140/92 mm Hg; pulse 58 beats/min; respirations 8 breaths/min

78. Which of the following statements regarding a breech presentation is correct? A) Breech presentations are more common with premature births. B) A breech birth is characterized by a vertex position of the baby. C) Breech presentations are frequently accompanied by a nuchal cord. D) Most breech presentations are associated with a legs-first delivery.

A) Breech presentations are more common with premature births.

Which of the following statements regarding carbon monoxide (CO) poisoning is correct? A) Never rule out CO poisoning because of the absence of cherry red skin. B) The most common symptom of CO poisoning is chest pressure. C) CO results in systemic hypoxia by disintegrating red blood cells. D) Hyperbaric therapy is beneficial only if CO levels are above 40%.

A) Never rule out CO poisoning because of the absence of cherry red skin.

37. Which of the following is the MOST accurate definition of multiple-organ dysfunction syndrome? A) Combined failure of two or more organs or organ systems that were initially unharmed by the acute disorder or injury that caused the patient's initial illness B) Sequential failure of two or more organs or organ systems caused by an acute injury or illness affecting any part of the patient's central nervous system C) Acute and predictable failure of the kidneys, liver, lungs, and heart that resulted from any disorder or injury that directly affected these organs D) Progressive failure of two or more organs or organ systems that were directly affected by the acute disorder or injury that caused the patient's initial illness

A) Combined failure of two or more organs or organ systems that were initially unharmed by the acute disorder or injury that caused the patient's initial illness

81. Which of the following volume expanders has been shown to interfere with platelet function and cause clotting problems? A) Dextran B) Hespan C) Plasmanate D) Lactated Ringer's

A) Dextran

48. Which of the following medical conditions would MOST likely cause changes in sensation in a patient with an extremity injury? A) Diabetes B) Hypertension C) Renal insufficiency D) Rheumatoid arthritis

A) Diabetes

3. Which of the following events is a critical part of fetal transition? A) Diversion of blood flow to the fetus's lungs B) An acute increase in intrapulmonary pressure C) Fetal lung expansion within 5 minutes after birth D) Blood flow diversion across the ductus arteriosis

A) Diversion of blood flow to the fetus's lungs

Which of the following statements regarding sodium metal chemical burns is correct? A) Do not flush with water as doing so may produce heat and cause an explosion. B) Sodium metal burns should be covered with oil after irrigating with water. C) They react violently with oil and should only be flushed with sterile water. D) Applying baking soda to the wound effectively neutralizes sodium metals.

A) Do not flush with water as doing so may produce heat and cause an explosion.

43. Any normotensive patient with a sternal fracture should receive: A) ECG monitoring. B) IV fluid boluses. C) ventilation assistance. D) antiarrhythmic drugs.

A) ECG monitoring.

3. Which of the following structures is NOT part of the axial skeleton? A) Femoral shaft B) Vertebral column C) Ribs and sternum D) Basilar skull and face

A) Femoral shaft

55. A woman has been pregnant three times, has had two spontaneous abortions, and has carried one baby to term. You should document her obstetric history as: A) G3A2P1. B) G1A2P3. C) G5A2P1. D) G1A2P5.

A) G3A2P1.

A small, button-shaped device with a rubber septum that can be punctured with a dialysis needle is called a(n): A) HemaSite. B) Thomas shunt. C) Scribner shunt. D) internal shunt.

A) HemaSite.

A 59-year-old woman with chronic renal failure presents with an acute onset of dyspnea while undergoing a hemodialysis treatment. She is conscious but in obvious respiratory distress. Further assessment reveals perioral cyanosis and a blood pressure of 96/56 mm Hg. Based on this patient's medical history and clinical presentation, which of the following interventions is likely NOT indicated? A) IV crystalloid fluid boluses B) Left lateral recumbent position C) Rapid transport to the hospital D) Ventilation assistance as needed

A) IV crystalloid fluid boluses

8. Which of the following statements regarding the Apgar score is correct? A) If resuscitation is necessary, the Apgar score is completed to determine the result of the resuscitation. B) The Apgar score is determined on the basis of the newborn's condition at 2 and 10 minutes after birth. C) If resuscitation is needed, it should commence immediately after you obtain the 1-minute Apgar score. D) A newborn with a heart rate of greater than 80 beats/min would be assigned a score of 2 on the Apgar score.

A) If resuscitation is necessary, the Apgar score is completed to determine the result of the resuscitation.

18. Which of the following statements regarding the amniotic sac and fluid is correct? A) In the latter stages of pregnancy, the fetus swallows amniotic fluid and passes wastes out into the fluid. B) The volume of amniotic fluid reaches about 500 mL by the end of pregnancy and nourishes the fetus. C) Amniotic fluid serves no real physiologic purpose and the fetus could easily survive in utero without it. D) The amniotic sac is composed of a tough, fibrous membrane that generally does not rupture until birth.

A) In the latter stages of pregnancy, the fetus swallows amniotic fluid and passes wastes out into the fluid.

38. Which of the following interventions would the paramedic LEAST likely perform on a patient with diabetic ketoacidosis? A) Insulin administration B) Endotracheal intubation C) Sodium bicarbonate administration D) Infusion of 1 to 2 L of normal saline

A) Insulin administration

27. Which of the following would NOT cause hypoglycemia in the patient with type 1 diabetes? A) Insulin underdose B) Too much insulin C) Too little food D) Strenuous exertion

A) Insulin underdose

50. Which of the following would you MOST likely observe in a newborn with hemolytic disease? A) Jaundice B) Polycythemia C) Hot flushed skin D) Splenomegaly

A) Jaundice

22. Which of the following would be considered a primary cause of obesity? A) Lack of exercise B) Hormonal changes C) Genetic predisposition D) Low basal metabolic rate

A) Lack of exercise

55. If a colorimetric ETCO2 detector turns purple during the exhalation phase through an ET tube, approximately how much carbon dioxide is being exhaled? A) Less than 0.5% B) Between 1% and 2% C) Between 2% and 5% D) More than 5%

A) Less than 0.5%

6. Which of the following is NOT a mediastinal structure? A) Lung B) Trachea C) Esophagus D) Mainstem bronchi

A) Lung

21. Which of the following statements regarding uterine fundus measurement is correct? A) Measurement of the fundus in centimeters corresponds to the number of gestational weeks. B) If the fundus is longer than expected, it could indicate uterine growth problems. C) A shorter than expected uterine fundus measurement is most suggestive of a breech position. D) If the fundus measures 36 cm, the woman is between 32 and 34 weeks pregnant.

A) Measurement of the fundus in centimeters corresponds to the number of gestational weeks.

25. What size and type of laryngoscope blade is recommended for use in a full-term newborn? A) No. 1, straight B) No. 2, straight C) No. 1, curved D) No. 2, curved

A) No. 1, straight

33. Which of the following statements regarding a nondisplaced fracture is correct? A) Nondisplaced fractures are generally caused by low-energy trauma and are typically not associated with deformity. B) Nondisplaced fractures occur when a massive compressive force is applied to the bone, causing it to become wedged into another bone. C) In a nondisplaced fracture, muscles pull the distal fracture fragment alongside the proximal one, causing them to overlap. D) Nondisplaced fractures are caused by low-energy trauma and occur when the ends of the fracture move from their normal positions.

A) Nondisplaced fractures are generally caused by low-energy trauma and are typically not associated with deformity.

Which of the following statements regarding partial-thickness burns is correct? A) Partial-thickness burns are usually extremely painful for the patient. B) Partial-thickness burns are difficult to distinguish from superficial burns in the field. C) The majority of partial-thickness burns are caused by an open flame. D) Partial-thickness burns typically heal spontaneously without scarring.

A) Partial-thickness burns are usually extremely painful for the patient.

Which of the following chemicals causes a painless burn and can result in significant damage before it is identified? A) Phenol B) Sulfur mustard C) Sulfuric acid D) Potassium hydroxide

A) Phenol

40. Which of the following conditions would LEAST likely present with an acute onset of respiratory distress? A) Pneumonia B) Anaphylaxis C) Pneumothorax D) Pulmonary embolism

A) Pneumonia

25. Which of the following conditions is characterized by a lack of progesterone and increased androgen levels, and can lead to gestational diabetes and cardiac problems? A) Polycystic ovaries B) Ectopic pregnancy C) Corpus luteum cyst D) Tubo-ovarian abscess

A) Polycystic ovaries

56. Which of the following is NOT an intervention the paramedic can perform to help reduce the risk of long-term disability following a musculoskeletal injury? A) Prehospital fracture reduction B) Prevention of gross contamination C) Pain reduction with cold and analgesia D) Transport to an appropriate medical facility

A) Prehospital fracture reduction

60. Which of the following clinical presentations is MOST consistent with thyrotoxicosis? A) Severe tachycardia, fever, nausea and vomiting, and confusion B) Profound bradycardia, hypothermia, and respiratory depression C) Apathy, hypoglycemia, abdominal pain, and an irregular pulse D) Obesity, cool skin, severe hypotension, and respiratory distress

A) Severe tachycardia, fever, nausea and vomiting, and confusion

What type of thermal burn is MOST commonly associated with inhalation injury? A) Steam burns B) Flame burns C) Scald burns D) Arc burns

A) Steam burns

29. Which of the following interventions is especially important when caring for a patient with a tracheostomy tube? A) Suctioning B) Mask ventilation C) Hyperventilation D) Head positioning

A) Suctioning

23. What type of chest injury is characterized by air accumulation in the pleural space when a perforation in the lung parenchyma acts as a one-way valve? A) Tension pneumothorax B) Simple pneumothorax C) Massive hemothorax D) Spontaneous pneumothorax

A) Tension pneumothorax

Which of the following statements regarding the nasal septum is correct? A) The nasal septum may be slightly deviated to one side or the other. B) The nasal septum is comprised mainly of cartilage. C) Inflammation of the nasal septum is common during infection. D) The nasal septum separates the oropharynx and nasopharynx

A) The nasal septum may be slightly deviated to one side or the other.

42. Which of the following depicts a positive orthostatic tilt test? A) The patient becomes dizzy upon standing B) Systolic blood pressure drops 10 mm Hg upon standing C) Pulse increases from 80 to 90 upon standing D) The patient begins to vomit upon sitting up

A) The patient becomes dizzy upon standing

Which of the following statements regarding the rule of palms is correct? A) The patient's palm, excluding the fingers, represents 1% of his or her total body surface area. B) The rule of palms is not an accurate estimator of total body surface area burned in pediatric patients. C) The patient's palm, including the fingers, represents 1% of his or her total body surface area. D) The rule of palms is most accurate when a patient has experienced burns to less than 20% of his or her total body surface area

A) The patient's palm, excluding the fingers, represents 1% of his or her total body surface area.

10. Which of the following statements regarding Mongolian spots is correct? A) They resemble bruises and are found most commonly on the back and buttocks. B) They are abnormal bruising patterns on the back and are an indicator of abuse. C) They indicate abnormal bleeding and are associated with a high mortality rate. D) They are bruiselike patterns that most often appear in severely premature infants.

A) They resemble bruises and are found most commonly on the back and buttocks.

45. What type of seizure is MOST common in premature infants? A) Tonic seizure B) Subtle seizure C) Myoclonic seizure D) Focal clonic seizure

A) Tonic seizure

74. Which of the following congenital defects results in an undersized or absent right ventricle? A) Tricuspid atresia B) Pulmonary stenosis C) Tetralogy of Fallot D) Atrial septal defect

A) Tricuspid atresia

14. Which of the following statements regarding endometritis is correct? A) Untreated endometritis may result in septic shock. B) Endometritis is defined as an enlargement of the uterus. C) Endometritis is most commonly caused by an intrauterine device. D) Endometritis results when endometrial tissue grows outside the uterus.

A) Untreated endometritis may result in septic shock.

62. Which of the following statements regarding abdominal trauma during pregnancy is correct? A) Use of a lap belt increases the risk of uterine injury. B) Deceleration injuries often result in placenta previa. C) Uterine trauma is common during the first trimester. D) The pubic bone protects the bladder in late pregnancy.

A) Use of a lap belt increases the risk of uterine injury.

2. Which of the following is an example of a peri-arrest condition? A) Ventricular tachycardia and a systolic blood pressure of 60 mm Hg B) Sinus tachycardia secondary to fever and a mild infection C) Ventricular fibrillation that has persisted for 6 minutes D) Bradycardia in the absence of chest pain or other symptoms

A) Ventricular tachycardia and a systolic blood pressure of 60 mm Hg

46. __________ breath sounds are the MOST commonly heard breath sounds and have a much more obvious inspiratory component. A) Vesicular B) Bronchial C) Tracheal D) Bronchovesicular

A) Vesicular

66. Which of the following is the MOST accurate definition of distributive shock? A) Widespread dilation of the resistance and capacitance vessels B) Decreased perfusion due to sympathetic nervous system failure C) Sustained constriction of the small venules and small arterioles D) Shunting of blood from the periphery to the body's vital organs

A) Widespread dilation of the resistance and capacitance vessels

86. Multiple sclerosis is: A) a chronic central nervous system disease caused by destruction of the myelin and nerve axons within the brain and spinal cord. B) most often secondary to a diffuse axonal brain injury and causes neuromuscular disability due to stretching or tearing of the axons. C) a progressive disease in 90% of patients who have it, and is characterized by unrelenting pain, weakness, and visual impairment. D) chronic in most cases, and is the result of degenerative changes in the muscle that results in muscle atrophy and decreased bone density.

A) a chronic central nervous system disease caused by destruction of the myelin and nerve axons within the brain and spinal cord.

If a knife is impaled in the neck: A) a cricothyrotomy may be required to establish a patent airway. B) it should be removed in case the airway becomes compromised. C) you should stabilize the object in place, regardless of its location. D) it should be shortened to facilitate proper airway management.

A) a cricothyrotomy may be required to establish a patent airway.

61. Common clinical findings in patients with obstructive lung disease include all of the following, EXCEPT: A) a decreased expiratory phase. B) pursed-lip breathing. C) abdominal muscle use. D) chronic air trapping in the lungs.

A) a decreased expiratory phase.

40. Common signs of impending respiratory failure in infants and children include: A) a falling oxygen saturation despite high-flow oxygen administration. B) abdominal breathing and a pulse rate less than 120 beats per minute. C) marked agitation and tachycardia with ectopic ventricular complexes. D) tachypnea and hyperpnea with nasal flaring and prominent retractions.

A) a falling oxygen saturation despite high-flow oxygen administration.

16. A flail chest is characterized by: A) a free-floating segment of fractured ribs. B) bulging of fractured ribs during inspiration. C) excessive negative intrathoracic pressure. D) drawing in of fractured ribs during expiration.

A) a free-floating segment of fractured ribs.

32. Hyperglycemia is characterized by: A) a gradual onset and warm, dry skin. B) shallow respirations and bradycardia. C) a rapid onset and cool, clammy skin. D) a blood glucose level above 110 mg/dL.

A) a gradual onset and warm, dry skin.

37. A subluxation occurs when: A) a joint is partially dislocated. B) a dislocation spontaneously reduces. C) a fracture occurs through both cortices. D) dislocated bones are locked in position.

A) a joint is partially dislocated.

19. Parasympathetic nervous system stimulation results in: A) a negative dromotropic effect.. B) increased myocardial contractility. C) a decreased inotropic effect. D) decreased gastrointestinal motility.

A) a negative dromotropic effect..

38. A shift of heart tones and severe respiratory distress despite positive-pressure ventilations is indicative of: A) a pneumothorax. B) a diaphragmatic hernia. C) Pierre Robin sequence. D) a pericardial tamponade.

A) a pneumothorax

67. Characteristic physical features of Down syndrome include: A) a protruding tongue. B) an excessively long neck. C) bulging of the nose and face. D) downward slanting eyes.

A) a protruding tongue.

Acute renal failure is MOST accurately defined as: A) a sudden decrease in filtration through the glomeruli. B) irreversible damage to the nephrons and renal tubules. C) an acute drop in urine output to less than 750 mL/day. D) sudden damage to the renal parenchyma due to sepsis.

A) a sudden decrease in filtration through the glomeruli.

24. A ruptured ovarian cyst typically presents with: A) a sudden onset of abdominal pain that can be related to the menstrual cycle. B) chronic waxing and waning abdominal cramping between menstrual periods. C) dull or aching epigastric pain that radiates to the shoulders or lower back. D) a sudden onset of upper abdominal pain and nausea without vomiting.

A) a sudden onset of abdominal pain that can be related to the menstrual cycle.

71. Signs of compensated shock in the infant or child include all of the following, EXCEPT: A) abnormal mentation. B) tachycardia and pallor. C) prolonged capillary refill. D) decreased peripheral perfusion.

A) abnormal mentation.

66. You are transporting a newborn who requires ongoing ventilatory support and chest compressions for severe bradycardia. Your estimated time of arrival at the hospital is 45 minutes. Air medical transport was unavailable due to severe weather in the vicinity. A peripheral IV line has been established in the antecubital vein and you are in the process of attempting intubation. Approximately 10 seconds into your intubation attempt, the newborn's heart rate suddenly drops more. You should: A) abort the intubation attempt and continue ventilations. B) continue the intubation attempt and administer atropine. C) administer 0.1 to 0.3 mL/kg of epinephrine rapid IV push. D) ensure that chest compressions are of adequate rate and depth.

A) abort the intubation attempt and continue ventilations.

51. If a patient's hemoglobin level is 8 g/dL due to hemorrhage and all of the hemoglobin molecules are attached to oxygen, the patient's oxygen saturation would MOST likely read: A) above 95%. B) between 90% and 95% C) between 85% and 90%. D) significantly lower than 85%.

A) above 95%.

110. Beta blocker ingestion in small children would MOST likely cause: A) acute hypoglycemia. B) agitation or irritability. C) marked hypertension. D) ventricular fibrillation.

A) acute hypoglycemia.

94. A 25-year-old woman was involved in a motor vehicle accident in which she struck the rear end of another car at a low speed. When you arrive at the scene and exit the ambulance, you immediately hear the patient screaming, "My baby, my baby!" After calming the patient down, she tells you that she is 10 weeks pregnant and that she is afraid that the car accident injured her child. She confirms that she was properly restrained at the time of impact. She is conscious and alert, denies abdominal pain, and has stable vital signs. She refuses spinal motion restriction precautions but does consent to EMS transport. You should: A) administer supplemental oxygen, establish a large-bore IV line and set the rate to keep the vein open, provide emotional support, and transport her to an appropriate hospital. B) be concerned that the traumatic injury may have caused an abruptio placenta, administer high-flow oxygen, establish a large-bore IV, and transport her to a trauma center. C) reassure her that her baby was not injured, offer her oxygen via nasal cannula, defer IV therapy, and transport her to the closest hospital with continuous emotional support en route. D) provide emotional support, administer diazepam to prevent her from becoming frantic, administer oxygen via nasal cannula at 6 L/min, and transport her to a minor emergency clinic.

A) administer supplemental oxygen, establish a large-bore IV line and set the rate to keep the vein open, provide emotional support, and transport her to an appropriate hospital.

143. A 7-year-old conscious boy presents with marked respiratory distress. Your assessment reveals the presence of intercostal and supraclavicular retractions and nasal flaring. His oxygen saturation is 93% on room air, and his heart rate is rapid. The MOST appropriate initial treatment for this child involves: A) administering high-flow oxygen as tolerated, auscultating his lung sounds, and being prepared to assist his ventilations. B) conducting a focused history and physical exam and allowing him to breathe room air to see if his oxygen saturation falls. C) recognizing that the child is in respiratory failure and making immediate preparations to perform endotracheal intubation. D) assisting his ventilations with a bag-mask device and determining if his tachycardia is ventricular or supraventricular in origin.

A) administering high-flow oxygen as tolerated, auscultating his lung sounds, and being prepared to assist his ventilations.

4. Molecules that bind to a cell's receptor and trigger a response, resulting in some kind of action or biologic effect, are called: A) agonists. B) mediators. C) antagonists. D) neurotransmitters.

A) agonists.

20. Pneumothorax is MOST accurately defined as: A) air or gas within the pleural cavity. B) perforation of a lung by a broken rib. C) injury to the visceral or parietal pleura. D) partial or complete collapse of a lung.

A) air or gas within the pleural cavity.

Significant blunt injuries to the larynx or trachea pose an IMMEDIATE risk of: A) airway compromise. B) hypovolemic shock. C) mediastinal inflammation. D) aspiration of gastric contents.

A) airway compromise.

14. If the amount of pulmonary surfactant is decreased: A) alveolar surface tension increases. B) diffuse alveolar hyperinflation occurs. C) alveoli are able to expand more easily. D) pulmonary gas exchange is enhanced.

A) alveolar surface tension increases.

25. A pathologic fracture occurs when: A) an occult medical condition causes abnormal bone weakness. B) greater-than-usual forces are required to fracture a large bone. C) a particular mechanism of injury cannot be identified readily. D) underdeveloped bones sustain a low-impact traumatic injury.

A) an occult medical condition causes abnormal bone weakness.

23. A woman who does not take prenatal vitamins during the course of her pregnancy is at greatest risk for: A) anemia. B) leukopenia. C) post-term labor. D) abnormal bleeding.

A) anemia.

2. A patient who is experiencing an allergic reaction states that his tongue "feels thick" and speaks at a low volume. You should immediately evaluate for: A) angioedema. B) hypotension. C) a gag reflex. D) adventitious breath sounds.

A) angioedema.

96. A 29-year-old woman is experiencing a severe asthma attack. Her husband reports that she was admitted to an intensive care unit about 6 months ago, and had a breathing tube in place. Prior to your arrival, the patient took 3 puffs of her rescue inhaler without effect. She is anxious and restless, is tachypneic, and has audible wheezing. You should: A) apply a CPAP unit, transport immediately, and attempt to establish vascular access en route to the hospital. B) begin assisting her ventilations with a bag-mask device and 100% oxygen and prepare to intubate her trachea. C) start an IV of normal saline, administer methylprednisolone via IV push, and transport as soon as possible. D) attempt to slow her breathing with respiratory coaching, administer a nebulized bronchodilator, and transport.

A) apply a CPAP unit, transport immediately, and attempt to establish vascular access en route to the hospital.

4. The eleventh and twelfth ribs are known as the floating ribs because they: A) are not attached anteriorly to the sternum. B) are not connected to any bony structures. C) do not have a posterior point of attachment. D) are attached to the sternum only by cartilage.

A) are not attached anteriorly to the sternum.

Flash burns: A) are usually relatively minor compared with the potential for trauma from whatever caused the flash. B) are caused by prolonged exposure to intense heat, usually resulting in burns that extend deep into the dermis. C) are a common source of burn injury and are most often the result of hot liquids, such as radiator fluid. D) are generally confined to a very small area of the body, but cause extensive damage to the dermis.

A) are usually relatively minor compared with the potential for trauma from whatever caused the flash.

58. You respond to a dialysis center for a patient with shortness of breath. When you arrive, you find the patient, an older male, still receiving dialysis. He is conscious and alert and is experiencing mild respiratory distress. As your partner administers oxygen, you should: A) ask the dialysis technician how long the patient has been on the dialysis machine. B) start an IV in the arm opposite the AV shunt and set the flow rate to 25 mL/hr. C) instruct the dialysis technician to remove the patient from the dialysis machine. D) administer a beta-2 agonist medication and contact medical control for guidance.

A) ask the dialysis technician how long the patient has been on the dialysis machine.

48. Uterine rupture MOST commonly occurs: A) during active labor. B) in primiparous women. C) during the third trimester. D) after the placenta delivers.

A) during active labor.

48. You are caring for a 44-year-old woman who was sexually assaulted by several men. She is conscious but very quiet. Your partner, a female paramedic, quickly examines her and finds no immediate life-threatening injuries. The patient tells you that all she wants to do is go home and take a shower. After multiple attempts to convince the patient to consent to transport, you are unsuccessful. Your MOST appropriate action should be to: A) ask the patient if there is a friend you can call with whom she can stay. B) advise her that she cannot take a shower because her body is evidence. C) explain the seriousness of the incident and have her sign a refusal form. D) allow her to take a shower, but insist upon EMS transport to the hospital.

A) ask the patient if there is a friend you can call with whom she can stay.

90. When assessing a patient who is disabled from a previous brain injury, you should: A) ask the patient's family what is normal for him or her. B) be prepared to hyperventilate the patient if apnea occurs. C) consider asymmetric pupils to be the result of the past injury. D) recall that most disabled patients are unable to communicate with you.

A) ask the patient's family what is normal for him or her.

The two MOST common causes of death from an electrical injury are: A) asphyxia and cardiopulmonary arrest. B) full-thickness burns and respiratory arrest. C) nervous system damage and massive sepsis. D) myoglobinuria and diaphragmatic paralysis

A) asphyxia and cardiopulmonary arrest.

37. When conducting the scene size-up of a call involving a gynecologic emergency, it is MOST important to: A) assess for danger because any scene should be considered volatile. B) quickly ascertain if the patient's problem is medical or trauma in nature. C) take standard precautions because many of these calls involve a lot of blood. D) quickly assess the need for additional resources and summon them early.

A) assess for danger because any scene should be considered volatile.

101. A 36-year-old man with a history of asthma presents with severe respiratory distress. You attempt to administer a nebulized beta-2 agonist, but his poor respiratory effort is inhibiting effective drug delivery via the nebulizer and his mental status is deteriorating. You should: A) assist his ventilations and establish vascular access. B) start an IV of normal saline and administer a steroid. C) apply high-flow oxygen via a nonrebreathing mask. D) assist him with a metered-dose inhaler bronchodilator.

A) assist his ventilations and establish vascular access.

149. You are providing high-flow oxygen to a 3-year-old boy with severe respiratory distress. When you reassess him, you note that he is pale and his respiratory rate has decreased from 30 breaths/min to 12 breaths/min. You should: A) assist his ventilations with a bag-mask device. B) secure his airway with an ET tube. C) begin treatment with a beta-2 agonist medication. D) auscultate his lung sounds and reassess his SpO2.

A) assist his ventilations with a bag-mask device.

50. When performing a needle decompression of the chest, you should insert the needle: A) at a 90-degree angle and listen for the release of air. B) on the side of the chest that has audible breath sounds. C) at a 45-degree angle until you hear a sudden release of air. D) on the inferior rib border to avoid vasculature and nerves.

A) at a 90-degree angle and listen for the release of air.

20. Skeletal muscles that are not used or exercised tend to: A) atrophy. B) develop necrosis. C) hypertrophy. D) increase in size.

A) atrophy.

74. When troubleshooting a hearing aid that is not working, you should: A) avoid attempting to clean the device. B) soak the device in hydrogen peroxide. C) ensure the device is set to telephone mode. D) carefully clean the device with an alcohol prep.

A) avoid attempting to clean the device.

9. When assessing a 5-year-old child, you should: A) be able to conduct a head-to-toe exam. B) ask simple yes or no questions if possible. C) generally use a toe-to-head exam approach. D) first ask a parent where the child is hurting.

A) be able to conduct a head-to-toe exam

44. When caring for a woman with an inevitable or incomplete abortion, you should be MOST concerned with: A) bleeding and shock. B) severe maternal infection. C) maternal emotional trauma. D) the risk of airway compromise.

A) bleeding and shock.

Initial signs or symptoms of end-stage renal disease include: A) bone pain. B) uremic frost. C) pruritus. D) hallucinations.

A) bone pain.

62. Reactive airway disease is characterized by: A) bronchospasm, edema, and mucus production. B) chronic bronchoconstriction of varying severity. C) acute, reversible swelling of the laryngeal muscles. D) excessive mucus production and a chronic cough.

A) bronchospasm, edema, and mucus production.

17. A ____________ is a padlike sac or cavity located within the connective tissue, usually in proximity to a joint. A) bursa B) ligament C) joint capsule D) synovial membrane

A) bursa

Specific treatment for a hydrofluoric acid burn is: A) calcium chloride. B) sodium bicarbonate. C) magnesium sulfate. D) viscous lidocaine gel.

A) calcium chloride.

56. Women who have had a cesarean section: A) can have a normal vaginal delivery. B) most likely delivered two or more babies. C) usually have a vertical scar on the abdomen. D) are precluded from having a vaginal delivery.

A) can have a normal vaginal delivery.

31. If a newborn requires epinephrine and peripheral venous access is unsuccessful, you should: A) cannulate the umbilical vein. B) insert an IO catheter. C) perform intubation immediately. D) inject the drug directly into a vein.

A) cannulate the umbilical vein

You are caring for a man with a chemical burn to both eyes. The patient, who has contact lenses in place, is in severe pain and tells you that he can't see. Proper care for this patient includes: A) carefully removing his contact lenses, flushing both eyes for at least 20 minutes, and transporting with continuous eye irrigation. B) leaving his contact lenses in place to avoid further injury and transporting at once with irrigation of both eyes performed en route. C) removing his contact lenses, covering both eyes with moist, sterile dressings, administering a narcotic analgesic, and transporting. D) asking the patient to remove his contact lenses, irrigating both eyes for no more than 10 minutes, covering both eyes with sterile dressings, and transporting.

A) carefully removing his contact lenses, flushing both eyes for at least 20 minutes, and transporting with continuous eye irrigation.

20. Hyperpnea and tachypnea: A) cause an increase in minute ventilation. B) are caused by decreased diaphragmatic function. C) are characterized by shallow chest wall movement. D) result from decreased negative-pressure ventilation.

A) cause an increase in minute ventilation.

The ONLY indication for removing contact lenses in the prehospital setting is: A) chemical eye burns. B) acute conjunctivitis. C) cardiopulmonary arrest. D) a foreign body in the eye.

A) chemical eye burns.

67. Because stimulation of the parasympathetic nervous system and bradycardia can occur during intubation of a child, you should: A) closely monitor the child's cardiac rhythm. B) premedicate with 0.04 mg/kg of atropine. C) limit your intubation attempt to 10 seconds. D) use a curved blade instead of a straight blade.

A) closely monitor the child's cardiac rhythm.

30. When performing chest compressions on a newborn, you should: A) compress the chest one third the anteroposterior depth of the chest. B) use the two-finger compression technique if two rescuers are present. C) reassess the newborn's heart rate after every 60 seconds of compressions. D) deliver 120 compressions and 40 ventilations during any 60-second period.

A) compress the chest one third the anteroposterior depth of the chest.

71. An untreated patent ductus arteriosus may cause subsequent development of: A) congestive heart failure. B) ventricular septal defect. C) pulmonary stenosis. D) a patent foramen ovale.

A) congestive heart failure

Diuril would MOST likely be prescribed to a patient with: A) congestive heart failure. B) persistent hypokalemia. C) a decreased cardiac output. D) vasoconstriction-induced hypertension.

A) congestive heart failure.

7. The _________ cartilage forms a complete ring and maintains the trachea in an open position. A) cricoid B) thyroid C) arytenoid D) laryngeal

A) cricoid

11. You are assessing a 6-year-old Asian child who presents with a fever. During your assessment, you note red, flat, rounded lesions on the child's torso. This finding is MOST indicative of: A) cupping. B) coining. C) physical abuse. D) phytophotodermatitis.

A) cupping.

28. At term, displacement of the diaphragm by the uterus causes a(n): A) decrease in expiratory reserve volume. B) increase in functional reserve capacity. C) decrease in inspiratory reserve volume. D) marked increase in residual volume.

A) decrease in expiratory reserve volume.

64. The risk of aspiration in the pregnant woman is increased significantly because: A) decreased digestion causes a delay in gastric emptying. B) pregnancy hormones often cause nausea and vomiting. C) increased tidal volume causes air to enter the stomach. D) the gastric lining is extremely irritable during pregnancy.

A) decreased digestion causes a delay in gastric emptying.

The LEAST significant complication associated with damage to the skin following a burn injury is: A) decreased melanin granules. B) disturbances in fluid balance. C) difficulty with thermoregulation. D) susceptibility to bacterial invasion.

A) decreased melanin granules.

24. As air accumulates in the pleural space, the FIRST thing to occur is: A) decreased pulmonary function. B) contralateral tracheal deviation. C) compression of the great vessels. D) marked decrease in venous return.

A) decreased pulmonary function.

72. Sensorineural hearing loss is caused by: A) decreased sound uptake through tiny hairs within the cochlea. B) an inability of sound to travel from the outer ear to the inner ear. C) destruction of the acoustic nerve from the use of certain drugs. D) failure of the brainstem to transmit messages via the acoustic nerve.

A) decreased sound uptake through tiny hairs within the cochlea.

32. The MOST reliable sign of a fracture is: A) deformity. B) ecchymosis. C) localized pain. D) severe swelling.

A) deformity.

Clinical manifestations of chronic renal failure include all of the following, EXCEPT: A) dehydration. B) hyperkalemia. C) hypotension. D) QT prolongation.

A) dehydration.

The MOST important aspect of assessing a patient with a genitourinary emergency is to: A) detect and treat life-threatening conditions. B) perform an immediate secondary assessment. C) rapidly determine the cause of the emergency. D) determine if the patient requires narcotic analgesia.

A) detect and treat life-threatening conditions.

Assessment of a patient who may have been exposed to radiation begins by: A) determining if the scene is safe to enter. B) thoroughly decontaminating the patient. C) quickly moving the patient to a safe area. D) evaluating airway, breathing, and circulation.

A) determining if the scene is safe to enter.

31. Upon arriving at the residence of a 27-year-old man who has a tracheostomy tube and is being mechanically ventilated, you note that he is breathing shallowly, is cyanotic, and is diaphoretic. You should: A) disconnect the patient from the mechanical ventilator and begin bag-mask ventilations. B) immediately check the settings on the mechanical ventilator to ensure that it is working properly. C) remove the ventilator tubing from the tracheostomy tube and suction the tube for 10 to 15 seconds. D) assess his oxygen saturation level and auscultate his breath sounds to determine if he is moving adequate air.

A) disconnect the patient from the mechanical ventilator and begin bag-mask ventilations.

16. A terminal illness is MOST accurately defined as a(n): A) disease process that is expected to cause death within 6 months, verified by a physician. B) disease that will ultimately cause death due to a lack of effective medical treatment. C) disease that is fatal in greater than 50% of a given population, even with timely treatment. D) disease process that will ultimately require ongoing treatment in order to prevent death.

A) disease process that is expected to cause death within 6 months, verified by a physician.

73. An increase in the number of EMS calls for patients with chronic respiratory problems MOST commonly occurs: A) during sudden weather changes. B) during an influenza outbreak. C) when the relative humidity is low. D) when people travel during a holiday.

A) during sudden weather changes.

87. In pregnancy, magnesium sulfate is used principally for: A) eclamptic seizures. B) tocolytic therapy. C) ventricular dysrhythmias. D) hyperemesis gravidarum.

A) eclamptic seizures.

32. Common signs and symptoms of preeclampsia include: A) edema, hypertension, and headache. B) weight loss, blurred vision, and diarrhea. C) ketones in the urine and rapid weight gain. D) facial swelling, dysuria, and chest pain.

A) edema, hypertension, and headache.

91. A critical step when using a CPAP unit to treat a patient with severe respiratory distress is: A) ensuring an adequate mask seal with minimal leakage. B) holding the mask to the noncompliant patient's face. C) starting with CPAP levels above 10 to 15 cm of water. D) setting the oxygen flow rate to at least 6 L/min.

A) ensuring an adequate mask seal with minimal leakage

71. The negative target-organ effects of anaphylactic shock are reversed with: A) epinephrine. B) methylprednisolone. C) diphenhydramine. D) low-dose dopamine.

A) epinephrine.

16. Polycythemia is a condition in which: A) excess red blood cells are produced in response to chronic hypoxia. B) an abundance of red blood cells causes severe thinning of the blood. C) fewer red blood cells are produced, resulting in decreased oxygenation. D) increased platelet production causes the blood to become abnormally thick.

A) excess red blood cells are produced in response to chronic hypoxia.

68. The classic presentation of chronic bronchitis is: A) excessive mucus production and a chronic or recurrent productive cough. B) a thin male with pursed-lip breathing and a history of heavy cigarette smoking. C) a dry, hacking cough and a barrel chest due to chronic pulmonary air trapping. D) expiratory wheezing and jugular venous distention due to pulmonary hypertension.

A) excessive mucus production and a chronic or recurrent productive cough.

126. In contrast to adults, young children are more prone to liver and spleen injuries because the organs: A) extend well below the rib cage. B) are both highly vascular. C) are more mobile and less supported. D) are relatively smaller and less protected.

A) extend well below the rib cage.

82. Pickwickian syndrome is a condition in which respiratory compromise results from: A) extreme obesity. B) pulmonary edema. C) cervical spine injury. D) diaphragmatic rupture.

A) extreme obesity.

155. You are transporting an unresponsive intubated 4-year-old child. An IO catheter is in place, and you are ventilating the child at an age-appropriate rate. Suddenly, the child becomes cyanotic and experiences a significant drop in her heart rate and oxygen saturation, and loss of a capnographic waveform. You attempt to auscultate her lung sounds but are unable to hear over the drone of the engine. You should: A) extubate immediately and ventilate with a bag-mask device. B) increase your ventilation rate and reassess the child's condition. C) administer 0.02 mg/kg of atropine via rapid IO push and reassess. D) look for vapor mist in the ET tube and attach a colorimetric device.

A) extubate immediately and ventilate with a bag-mask device.

44. A subtle seizure in the newborn is characterized by: A) eye deviation. B) repetitive jerking. C) flexion of the arms. D) tonic limb extension.

A) eye deviation

9. Hip fractures are actually fractures of the: A) femoral neck. B) pelvic girdle. C) femoral head. D) ischial tuberosity.

A) femoral neck.

10. All of the following processes take place in the uterus, EXCEPT: A) fertilization. B) implantation. C) the act of labor. D) fetal development.

A) fertilization.

144. Upon arriving at the scene of a 4-year-old girl who is ill, you assess her and note that she is tachypneic and tachycardic. Her skin is warm and moist, and there are no signs of increased work of breathing. The child's mother denies any vomiting or diarrhea. This child's tachycardia and tachypnea are MOST likely the result of: A) fever and anxiety. B) early hypoxemia. C) a cardiac problem. D) moderate dehydration.

A) fever and anxiety.

A renal calculus that has become lodged in a lower ureter would likely produce all of the following signs and symptoms, EXCEPT: A) fever. B) hematuria. C) urinary urgency. D) painful urination.

A) fever.

Phosphorus is found in _____________ and burns when exposed to _____________. A) fireworks, air B) oven cleaner, water C) drain cleaner, air D) battery acid, water

A) fireworks, air

Signs and symptoms of retinal detachment include: A) flashing lights, specks, or floaters in the field of vision. B) double vision and partial or complete loss of peripheral vision. C) immediate pain and total loss of vision following blunt eye trauma. D) paralysis of upward gaze and greater than 50% loss of central vision.

A) flashing lights, specks, or floaters in the field of vision.

103. In children, complex partial seizures would MOST likely manifest with: A) focal motor jerking with loss of consciousness. B) generalized tonic-clonic movement of all extremities. C) focal motor jerking without loss of consciousness. D) a brief loss of attention without abnormal body movement.

A) focal motor jerking with loss of consciousness.

62. You are transporting a conscious and alert man who experienced an isolated blunt injury to the right anterolateral chest. His vital signs are stable, but he is dyspneic and his breath sounds are diminished over the apex of his right lung. In addition to administering high-flow oxygen, the MOST critical intervention for this patient involves: A) frequently reassessing him for signs of clinical deterioration. B) performing a needle thoracentesis to release intrapleural tension. C) positioning him on his right side to facilitate effective breathing. D) administering a 500-mL normal saline bolus to maintain perfusion.

A) frequently reassessing him for signs of clinical deterioration.

The structural and functional unit of the kidney is the: A) nephron. B) medulla. C) renal cortex. D) podocyte.

A) nephron.

62. During your assessment of a 30-year-old woman in active labor, she admits to being a chronic heroin abuser and states that she last "shot up" about 6 hours ago. After the baby delivers, you will MOST likely need to: A) give positive-pressure ventilations. B) administer 0.1 mg/kg of naloxone. C) suction meconium from its airway. D) administer free-flow oxygen by mask.

A) give positive-pressure ventilations

1. The endocrine system comprises a network of ___________ that produce and secrete chemical messengers called ____________. A) glands, hormones B) nodes, catecholamines C) vessels, leukotrienes D) synapses, neurotransmitters

A) glands, hormones

77. The fetal side of the placenta should normally be: A) gray and shiny with a smooth texture. B) pale and dull with a rough texture. C) dark maroon with a smooth texture. D) dark maroon with a rough texture.

A) gray and shiny with a smooth texture.

Common signs and symptoms of a lower urinary tract infection include all of the following, EXCEPT: A) gross hematuria. B) painful urination. C) increased urinary frequency. D) localized pain in the pelvis.

A) gross hematuria.

54. You should NOT apply a pneumatic splint on a patient if he or she: A) has an open fracture in which bone ends are exposed. B) is experiencing severe pain despite narcotic analgesia. C) has a closed fracture involving the lower leg or forearm. D) experienced a fracture or dislocation involving a major joint.

A) has an open fracture in which bone ends are exposed.

Compared to beta radiation particles, alpha radiation particles: A) have minimal penetrating energy. B) easily pass through solid materials. C) are able to travel much farther in air. D) are not dangerous if they are ingested.

A) have minimal penetrating energy.

80. Shock in the trauma patient should be considered _____________ until proved otherwise. A) hemorrhagic B) distributive C) obstructive D) neurogenic

A) hemorrhagic

38. Cholestasis occurs when: A) hormones slow or block the normal flow of bile from the liver. B) progesterone and estrogen block the production of bile in the liver. C) pressure on the gallbladder prevents normal contraction and relaxation. D) hormone levels drop after pregnancy, resulting in gallbladder disease.

A) hormones slow or block the normal flow of bile from the liver.

35. Common clinical findings associated with a traumatic asphyxia include all of the following, EXCEPT: A) hyphema. B) exopthalmos. C) facial cyanosis. D) tongue swelling.

A) hyphema.

Motor function to the muscles of the tongue is provided by the ___________ nerve. A) hypoglossal B) trigeminal C) mandibular D) glossopharyngeal

A) hypoglossal

50. Swelling and inflammation associated with musculoskeletal injuries are reduced: A) if cold packs are applied during the acute stage of the injury. B) if an appropriate dose of a narcotic analgesic is administered. C) when heat therapy is used within 48 to 72 hours after the injury. D) if a vasodilator drug is given within 12 hours following the injury.

A) if cold packs are applied during the acute stage of the injury.

90. CPAP in the emergency setting is used to treat patients with certain obstructive airway diseases by: A) improving patency of the lower airway through the use of positive-end expiratory pressure. B) maintaining stability of the posterior pharynx, thereby preventing upper airway obstruction. C) increasing the rate and depth of ventilation, thus improving minute volume and mitigating hypoxia. D) delivering one pressure during the inspiratory phase and a different pressure during the expiratory phase.

A) improving patency of the lower airway through the use of positive-end expiratory pressure.

44. A patient with a small simple pneumothorax would MOST likely present with diminished breath sounds: A) in the apices of the affected lung if he or she is sitting upright. B) after more than 50% of the affected lung has been collapsed. C) in the posterior bases of the affected lung if he or she is sitting. D) on the contralateral side as the mediastinum begins to shift.

A) in the apices of the affected lung if he or she is sitting upright.

54. Medications used to prevent an asthma attack include: A) inhaled steroids. B) beta-2 agonists. C) inhaled albuterol. D) oral ibuprofen.

A) inhaled steroids.

32. The lesions associated with genital herpes: A) initially appear as small red bumps. B) present as moderately sized blisters. C) are isolated to the external genitalia. D) typically cause scarring after they heal.

A) initially appear as small red bumps.

52. When urine becomes evident in the tubing during insertion of an indwelling urinary catheter in a female, you should: A) insert the catheter another 1 to 3 inches. B) pull back on the catheter about 2 inches. C) carefully secure the catheter to the leg. D) inflate the balloon with the prefilled syringe.

A) insert the catheter another 1 to 3 inches.

61. A 30-year-old man felt a snap in his chest when he abruptly twisted his torso. He is conscious and alert, and complains of severe pain during inhalation. Your assessment reveals palpable tenderness over the fifth and sixth ribs on the left side. His vital signs are stable and he denies other injuries. In addition to administering supplemental oxygen, the MOST appropriate treatment for this patient involves: A) instructing the patient to hold a pillow against his chest, considering IV analgesics, and transporting to the hospital. B) stabilizing the injured area by circumferentially wrapping the chest with 3-inch tape and transporting him to a local hospital. C) encouraging the patient to take deeper breaths to maintain adequate minute volume and transporting him to the hospital. D) sedating the patient with midazolam or diazepam, assisting ventilations with a bag-mask device, and transporting at once.

A) instructing the patient to hold a pillow against his chest, considering IV analgesics, and transporting to the hospital.

Nasotracheal intubation of a patient with upper airway burns: A) is a complicated procedure and should be avoided. B) should be performed if the patient has mild stridor. C) is indicated if the patient is unconscious and apneic. D) is generally well tolerated in patients who are awake.

A) is a complicated procedure and should be avoided.

38. A dislocation is considered an urgent injury because of its potential to cause: A) neurovascular compromise. B) significant internal bleeding. C) severe hemodynamic instability. D) proximal sensory and motor loss.

A) neurovascular compromise.

A fracture of all midfacial bones, separating the entire midface from the cranium: A) is commonly associated with facial elongation and dental malocclusion. B) should be stabilized by placing bulky dressings across the fractured area. C) is almost always accompanied by multiple severe fractures of the mandible. D) is referred to as a Le Fort I fracture and most commonly results from a fall

A) is commonly associated with facial elongation and dental malocclusion.

21. In contrast to negative-pressure ventilation, positive-pressure ventilation: A) is the forcing of air into the lungs. B) occurs when the diaphragm descends. C) is provided with a nonrebreathing mask. D) can only be provided to intubated patients.

A) is the forcing of air into the lungs.

21. A conscious child who is in the sniffing position: A) is trying to align the axes of the airway to improve ventilation. B) is clearly experiencing a lower airway obstruction. C) will refuse to lie down and leans forward on outstretched arms. D) assumes a physical position that optimizes accessory muscle use.

A) is trying to align the axes of the airway to improve ventilation.

84. First-degree heart block in children: A) is typically asymptomatic and does not require special treatment. B) should be suspected when a randomly dropped QRS is observed. C) should be treated with cardiac pacing, even if the child is stable. D) does not respond to atropine and should be treated with dopamine.

A) is typically asymptomatic and does not require special treatment.

37. Increased intravenous pressure commonly manifests as: A) jugular venous distention. B) a widened pulse pressure. C) bounding peripheral pulses. D) a pulsating abdominal mass.

A) jugular venous distention.

30. When suctioning and cleaning the tracheostomy of a ventilator-dependent patient, it is MOST important to: A) keep the patient well oxygenated. B) have a new tube readily available. C) suction for no longer than 5 seconds. D) soak the inner cannula in sterile water.

A) keep the patient well oxygenated.

104. In contrast to a complex febrile seizure, a simple febrile seizure: A) lasts less than 15 minutes and occurs in children without underlying neurologic abnormalities. B) is focal in nature and tends to occur in children with a baseline developmental abnormality. C) is not associated with tonic-clonic body movement and occurs in children older than 6 years of age. D) is of short duration and occurs when the child's body temperature gradually rises above 102.5°F.

A) lasts less than 15 minutes and occurs in children without underlying neurologic abnormalities.

8. The FIRST step in examining a toddler in stable condition is to: A) let the child sit on a parent's lap. B) place yourself at the child's level. C) quickly examine any painful areas. D) allow the child to hold a favorite toy.

A) let the child sit on a parent's lap

38. Unlike a midline venous catheter, a peripherally inserted central catheter can be used for: A) long-term drug therapy. B) the delivery of antibiotics. C) chemotherapy in cancer patients. D) narcotic analgesia administration.

A) long-term drug therapy.

50. When administering IV fluid boluses to an elderly patient in shock, it is especially important to monitor his or her: A) lung sounds. B) mental status. C) pulse rate. D) blood pressure.

A) lung sounds.

56. Management of a diaphragmatic injury focuses on: A) maintaining adequate oxygenation and ventilation, and rapid transport. B) inserting a nasogastric tube to decompress the gastrointestinal organs. C) applying the pneumatic antishock garment to stabilize the diaphragm. D) intubation and hyperventilation with 100% supplemental oxygen.

A) maintaining adequate oxygenation and ventilation, and rapid transport.

84. A hyperventilating patient: A) may be acidotic and is trying to decrease his or her pH level. B) is most effectively treated by administering a sedative drug. C) should rebreathe his or her carbon dioxide to effect resolution. D) presents with tachypnea and marked use of accessory muscles.

A) may be acidotic and is trying to decrease his or her pH level.

27. A normal respiratory rate in a child: A) may be observed if the child has been breathing rapidly with increased work of breathing and is becoming fatigued. B) generally ranges between 15 and 20 breaths per minute and is influenced easily by factors such as excitement, fear, or fever. C) cannot be established accurately because a toddler's respirations generally are grossly irregular and extremely difficult to count. D) is a sign of impending respiratory failure if it is observed in conjunction with a room air oxygen saturation reading of less than 96%.

A) may be observed if the child has been breathing rapidly with increased work of breathing and is becoming fatigued.

15. In contrast to endometritis, endometriosis: A) may present without abdominal pain. B) is an inflammation of the uterine lining. C) generally causes light menstrual periods. D) is often the result of gynecologic surgery.

A) may present without abdominal pain.

19. Risk factors for pelvic inflammatory disease include all of the following, EXCEPT: A) monogamy. B) an intrauterine device. C) heterosexual sex with multiple partners. D) 20- to 24-year-old age group.

A) monogamy.

22. In contrast to adults, retractions in children are: A) more evident in the intercostal area. B) less commonly seen below the sternum. C) usually less prominent above the clavicles. D) evident in the sternocleidomastoid muscles.

A) more evident in the intercostal area.

20. Common causes of respiratory distress in the newborn include: A) mucous obstruction of the nose. B) unrecognized metabolic alkalosis. C) persistent pulmonary hypotension. D) maternal use of a narcotic analgesic

A) mucous obstruction of the nose

61. The MOST common cause of cardiogenic shock is: A) myocardial infarction. B) ventricular aneurysm. C) a sudden dysrhythmia. D) papillary muscle rupture.

A) myocardial infarction.

59. A 68-year-old obese woman presents with a markedly decreased level of consciousness. She was found in bed by her husband. Your primary assessment reveals that her respirations are slow and shallow, her pulse is slow and weak, and her skin is cold and dry. According to the patient's husband, she has had a recent infection, but he cannot remember what the doctor called it. You should be MOST suspicious that this patient is experiencing: A) myxedema coma. B) Addisonian crisis. C) diabetic ketoacidosis. D) acute hypothyroidism.

A) myxedema coma.

75. Intrapulmonary shunting occurs when: A) nonfunctional alveoli inhibit pulmonary gas exchange. B) the volume of anatomic dead space suddenly increases. C) hyperinflated alveoli retain high levels of carbon dioxide. D) resistance to airflow increases due to bronchoconstriction.

A) nonfunctional alveoli inhibit pulmonary gas exchange.

83. Cerebral palsy is a: A) nonprogressive, bilateral neuromuscular disorder in which voluntary muscles are poorly controlled. B) degenerative, unilateral neuromuscular disorder in which control of autonomic functions is impaired. C) chronic dysfunction of the endocrine system that primarily targets the respiratory and digestive systems. D) progressive weakening of the body's voluntary muscles that leaves the patient confined to a wheelchair.

A) nonprogressive, bilateral neuromuscular disorder in which voluntary muscles are poorly controlled.

2. During the menstrual cycle: A) numerous follicles begin the process of maturation, but only one ultimately matures and releases an ovum. B) all of the follicles that are released begin to mature and then ultimately die in a process called atresia. C) the anterior pituitary gland releases luteinizing hormone, which facilitates the process of oocyte maturation. D) the glands of the endometrium decrease in size and secrete the materials on which the egg will implant and grow.

A) numerous follicles begin the process of maturation, but only one ultimately matures and releases an ovum.

50. When fever is suspected in the newborn, you should: A) observe for the presence of a rash. B) assist ventilations with a bag-mask device. C) administer acetaminophen or ibuprofen. D) quickly lower the newborn's body temperature.

A) observe for the presence of a rash

7. Paramedic intuition is BEST described as: A) pattern recognition and matching based on past experience. B) the quick formulation of a field diagnosis based on assessment. C) the field diagnosis arrived at based on the general impression. D) instinct or a "gut feeling" based on past reading or studying.

A) pattern recognition and matching based on past experience.

53. You are dispatched to the residence of a 60-year-old woman who was found unresponsive by her husband. As you are assessing the patient, her husband tells you that she is a diabetic and has recently experienced several "small strokes." In addition to properly managing her airway, you should: A) perform a field glucose test to rule out hypoglycemia. B) start an IV line and give her 50 mL of 50% dextrose. C) avoid giving her glucose because of her small strokes. D) start an IV line and give her a 20-mL/kg fluid bolus.

A) perform a field glucose test to rule out hypoglycemia.

160. You and your partner are caring for a child with stable supraventricular tachycardia that was refractory to initial treatment. As your partner is preparing to establish vascular access, the child's level of consciousness decreases markedly. You reassess the child and note that his femoral pulse is rapid and weak. You should: A) perform immediate synchronized cardioversion and reassess. B) begin chest compressions as your partner establishes the IV line. C) preoxygenate the child and then perform endotracheal intubation. D) establish vascular access and administer 0.1 mg/kg of adenosine.

A) perform immediate synchronized cardioversion and reassess.

150. Several cycles of chest compressions have failed to remove a foreign body airway obstruction in an unresponsive infant. Your next action should be to: A) perform laryngoscopy and try to visualize the foreign body. B) continue chest compressions and perform a cricothyrotomy. C) open the infant's airway and sweep the infant's mouth with your finger. D) perform back slaps and chest thrusts and then look in the mouth.

A) perform laryngoscopy and try to visualize the foreign body.

85. The MOST immediate treatment to prevent placental hypoperfusion in a pregnant woman who is lying on her back is to: A) place the woman in a left lateral recumbent position. B) keep the woman supine and elevate her legs 12 inches. C) manually displace the gravid uterus to the right side. D) administer 1 to 2 L of isotonic crystalloid solution.

A) place the woman in a left lateral recumbent position.

76. Bedridden patients with excessive pulmonary secretions are MOST prone to developing: A) pneumonia. B) bronchospasm. C) a pneumothorax. D) a pulmonary embolism.

A) pneumonia.

39. Patients with hyperosmolar hyperglycemic nonketotic coma: A) present with severe dehydration and neurologic deficits. B) experience more severe acidosis than patients with diabetic ketoacidosis. C) typically require prehospital sodium bicarbonate therapy. D) most commonly have a history of type 1 diabetes mellitus.

A) present with severe dehydration and neurologic deficits.

45. After ensuring your own safety, your next priority when caring for a patient with an extremity injury is to: A) prevent further injury. B) assess neurovascular function. C) splint the injured extremity. D) administer an analgesic.

A) prevent further injury.

The micturition reflex: A) produces the urge to void. B) collapses the walls of the bladder. C) is regulated by one of the cranial nerves. D) causes relaxation of the urinary sphincter.

A) produces the urge to void.

31. Common signs and symptoms of infection with the cytomegalovirus include: A) prolonged high fever. B) lesions on the genitalia. C) enlargement of the liver. D) severe nausea and diarrhea.

A) prolonged high fever.

A 20-year-old male presents with an acute onset of severe testicular pain. He denies any trauma to the genital region. He is conscious and alert, his blood pressure is 144/84 mm Hg, his heart rate is 120 beats/min, and his respirations are 24 breaths/min with adequate depth. The MOST important aspect in the care of this patient involves: A) prompt transport. B) narcotic analgesia. C) IV fluid therapy. D) high-flow oxygen.

A) prompt transport.

45. The purpose of the wafer that is included in an ostomy kit is to: A) protect the skin from irritation. B) seal the ostomy bag to the skin. C) maintain sterility of the ostomy bag. D) cover the stoma until the bag is attached.

A) protect the skin from irritation.

72. A woman is in the second stage of labor when: A) she feels a strong urge to move her bowels. B) the cervix is fully effaced and partially dilated. C) a gush of amniotic fluid pours from the vagina. D) contractions occur in 5- to 10-minute intervals

A) she feels a strong urge to move her bowels.

44. A 30-year-old woman complains of an "achy" pain to both lower abdominal quadrants, which she states is made worse by walking. She further tells you that she recently finished her menstrual period. She has a fever of 101.9°F. Her blood pressure is 122/62 mm Hg, pulse rate is 84 beats/min and strong, and respirations are 14 breaths/min and unlabored. After gathering the rest of her medical history, you should: A) provide emotional support, make her as comfortable as possible, and safely transport her to an appropriate hospital. B) advise her that she can probably go to the hospital via personal vehicle since she is not showing signs of shock. C) visually inspect her vagina for bleeding or discharge, start an IV line and set it to keep the vein open, and transport. D) establish vascular access and give her a 250-mL normal saline bolus, consider analgesia for her pain, and transport.

A) provide emotional support, make her as comfortable as possible, and safely transport her to an appropriate hospital.

15. If the parent or caregiver of a sick or injured child is emotionally distraught: A) provide support, but remember that your first priority is the child. B) you should firmly tell him or her that the situation is under control. C) he or she should follow the ambulance in his or her personal vehicle. D) the parent or caregiver should be removed from the scene immediately.

A) provide support, but remember that your first priority is the child.

60. Common signs and symptoms of an acute pulmonary embolism include all of the following, EXCEPT: A) pulmonary edema. B) pleuritic chest pain. C) right-sided heart failure. D) tachycardia and tachypnea.

A) pulmonary edema.

00. Meningococcal meningitis with sepsis is typically characterized by a(n): A) purpuric rash. B) insidious onset. C) low-grade fever. D) persistent cough.

A) purpuric rash.

32. During anaerobic metabolism, the precapillary sphincters __________ in response to __________. A) relax, lactic acid buildup B) constrict, capillary engorgement C) relax, systemic vasoconstriction D) constrict, decreased carbon dioxide

A) relax, lactic acid buildup

You are transporting a conscious but confused 29-year-old man after he was electrocuted. The patient is on high-flow oxygen, has an IV line of normal saline in place, is on a cardiac monitor, and has his spine fully immobilized. During transport, it is especially important for you to: A) remain alert for lethal cardiac dysrhythmias and be prepared to defibrillate. B) administer at least 2 L of normal saline solution to prevent renal failure. C) thoroughly assess and clean the entry and exit wounds to prevent an infection. D) reassess his vital signs every 15 minutes and treat any fractures or dislocations.

A) remain alert for lethal cardiac dysrhythmias and be prepared to defibrillate.

63. You are performing an interfacility transfer of a patient who received a cardiac catheterization and has a sheath in his femoral artery. During transport, it is important that the patient: A) remain in a supine position with his legs straight. B) be placed onto his left side with his head elevated. C) receive IV fluid boluses to keep the sheath patent. D) remain in a sitting position to prevent an embolism.

A) remain in a supine position with his legs straight.

You respond to an industrial plant for a 42-year-old man with a chemical burn. Upon arrival at the scene, you find the patient to be ambulatory. He tells you that he was moving some bags of dry lime when one of the bags broke and spilled lime all over him. After donning the appropriate personal protective equipment, you should: A) remove his clothing, brush as much of the lime off of him as possible, and flush the affected areas with copious amounts of water. B) avoid brushing any of the lime from his skin, as doing so may cause additional injury, and flush his entire body with water for 30 minutes. C) remove his clothing, carefully brush the lime away from his skin, but avoid flushing with water, as doing so will likely increase burn severity. D) remove all of his clothing, apply baking powder to neutralize the lime, and begin flushing his body with copious amounts of sterile saline.

A) remove his clothing, brush as much of the lime off of him as possible, and flush the affected areas with copious amounts of water.

When triggered by changes in the blood pressure, the juxtaglomerular cells release: A) renin. B) aldosterone. C) angiotensin I. D) angiotensinogen.

A) renin.

39. A pregnant woman with cephalopelvic disproportion: A) requires a cesarean section to prevent fetal distress. B) should be given a muscle relaxant prior to delivery. C) delivers her baby vaginally in about 75% of cases. D) generally does not experience problems during delivery.

A) requires a cesarean section to prevent fetal distress.

A patient with a loss of peripheral vision MOST likely has damage to the: A) retina. B) pupil. C) cornea. D) lens.

A) retina.

30. A patient with orthopnea: A) seeks a sitting position when short of breath. B) is awakened from sleep with severe dyspnea. C) prefers to lie flat in order to facilitate breathing. D) generally has a slow, shallow respiratory pattern.

A) seeks a sitting position when short of breath.

33. Phrenic nerve stimulators function by: A) sending electrical impulses to the diaphragm, causing it to contract and passively relax. B) keeping the upper airway patent in patients who experience frequent occurrences of sleep apnea. C) sending electrical impulses to the respiratory centers in the brain that stimulate inhalation. D) sending electrical impulses to the intercostal muscles, causing them to contract and expand the thorax.

A) sending electrical impulses to the diaphragm, causing it to contract and passively relax.

26. Baroreceptors function by: A) sensing decreased blood flow and activating the vasomotor center. B) dilating the blood vessels when systemic vascular resistance is low. C) measuring subtle shifts in arterial oxygen and carbon dioxide levels. D) constricting the blood vessels when systemic vascular resistance is high.

A) sensing decreased blood flow and activating the vasomotor center.

76. Distributive shock in children is MOST often the result of: A) sepsis. B) spinal injury. C) heart failure. D) anaphylaxis.

A) sepsis.

14. The MOST common etiology for bradycardia in a newborn is: A) severe hypoxia. B) untreated acidosis. C) occult hypovolemia. D) increased vagal tone

A) severe hypoxia.

82. Bradydysrhythmias in children MOST often occur secondary to: A) severe hypoxia. B) drug ingestion. C) AV heart block. D) cardiac irritability.

A) severe hypoxia.

65. A 19-year-old man experienced direct trauma to his left elbow. Your assessment reveals gross deformity and ecchymosis. His arm is pink and warm, and he has a strong radial pulse. Your transport time to the hospital will be delayed. You should: A) splint the elbow in the position found and reassess distal circulation. B) carefully straighten the arm to facilitate placement of a vacuum splint. C) apply a sling and swathe to immobilize the injury and then apply heat. D) administer fentanyl for pain relief and then carefully straighten the arm.

A) splint the elbow in the position found and reassess distal circulation.

14. Cardiac output is dependent upon: A) stroke volume and heart rate. B) blood pressure and heart rate. C) afterload and peripheral resistance. D) peripheral resistance and heart rate.

A) stroke volume and heart rate.

49. The preferred site for performing a needle thoracentesis is: A) superior to the third rib into the intercostal space at the midclavicular line. B) just below the second rib into the intercostal space at the midaxillary line. C) just above to the fifth rib into the intercostal space at the midaxillary line. D) inferior to the third rib into the intercostal space at the midclavicular line.

A) superior to the third rib into the intercostal space at the midclavicular line.

24. Bag-mask ventilation of the obese patient would MOST likely be ineffective when the patient is: A) supine. B) apneic. C) semisitting. D) in reverse Trendelenburg.

A) supine.

Most cases of chronic renal failure are caused by: A) systemic diseases such as diabetes. B) nephron deterioration due to aging. C) frequent lower urinary tract infections. D) nephron destruction due to medications.

A) systemic diseases such as diabetes.

133. Signs of pain in an infant would MOST likely include: A) tachycardia and inconsolability. B) a heart rate that is not variable. C) diaphoresis and dilated pupils. D) labored tachypnea and pallor.

A) tachycardia and inconsolability.

29. Early hypoxia in a child would MOST likely present with: A) tachycardia. B) bradypnea. C) mottled skin. D) bradycardia.

A) tachycardia.

All of the following conditions may cause urinary retention, EXCEPT: A) testicular torsion. B) nerve damage. C) urinary tract infections. D) benign prostatic hypertrophy.

A) testicular torsion.

21. Type 1 diabetes that is secondary to an autoimmune disorder occurs when: A) the body builds up antibodies that destroy the islets of Langerhans. B) insufficient white blood cells predispose the pancreas to infection. C) glucagon and insulin are destroyed by phagocytic white blood cells. D) the patient experiences an allergic reaction to his or her own glucose.

A) the body builds up antibodies that destroy the islets of Langerhans.

35. The decision to transport an acutely ill child immediately or remain at the scene to perform additional interventions is LEAST dependent on: A) the child's age and fear level. B) transport time to the hospital. C) expected benefits of treatment. D) your EMS system's regulations.

A) the child's age and fear level.

128. The general area of a child's body that sustains initial trauma after being struck by an automobile depends MAINLY on: A) the child's height and the height of the bumper upon impact. B) the travel speed of the vehicle and the weight of the child. C) whether the child turns away from or toward the vehicle. D) whether the vehicle ran over the child following impact.

A) the child's height and the height of the bumper upon impact.

60. Nasopharyngeal airways are rarely used in children younger than 1 year of age because: A) the diameter of their nares is small and easily obstructed by secretions. B) most nasopharyngeal airways are too large and result in an obstruction. C) nasopharyngeal stimulation commonly results in a tachycardic response. D) unlike older children, small children often have a more active gag reflex.

A) the diameter of their nares is small and easily obstructed by secretions.

15. By the end of the third week after conception, all of the following processes occur, EXCEPT: A) the embryo is officially referred to as the fetus. B) the embryonic process of forming specialized body systems. C) the appearance of rudimentary functions of various body systems. D) the woman has missed her menstrual period and suspects pregnancy.

A) the embryo is officially referred to as the fetus.

35. Paradoxical respiratory movement is characterized by: A) the epigastrium and thorax moving in opposite directions. B) bulging of the intercostal muscles during deep inhalation. C) pulling upward of the suprasternal notch during inhalation. D) a marked decrease in movement in one of the hemithoraces.

A) the epigastrium and thorax moving in opposite directions.

3. A child's vocal cords can be difficult to visualize during intubation because: A) the epiglottis is floppy and U-shaped. B) the cords themselves are more posterior. C) a sniffing position is difficult to achieve. D) the area of the cricoid cartilage is narrow

A) the epiglottis is floppy and U-shaped

78. Dysarthria is: A) the inability to make speech sounds correctly. B) a speech disorder that primarily affects adults. C) the loss of ability to communicate in speech or writing. D) caused by damage to the language center of the brain.

A) the inability to make speech sounds correctly.

Supraglottic damage following a burn is MOST often caused by: A) the inhalation of superheated gases. B) exposure to carbon monoxide or cyanide. C) the inhalation of hot particulate steam. D) direct flame exposure to the oropharynx

A) the inhalation of superheated gases.

The progression of a chemical burn is MOSTLY dependent on: A) the length of time the corrosive chemical remains on the skin. B) the surface area of the body exposed to a corrosive chemical. C) whether the corrosive substance is a strong acid or alkali. D) the patient's general health and the thickness of his or her skin.

A) the length of time the corrosive chemical remains on the skin.

69. Cervical effacement occurs when: A) the less muscular lower part of the uterus is pulled upward over the presenting part of the baby. B) the opening of the cervix stretches enough to accommodate passage of the entire baby's body. C) the uterine wall thickens during the wavelike contractions that cause progressive cervical dilation. D) the cervix becomes fully dilated and the baby exits the uterus and enters the birth canal.

A) the less muscular lower part of the uterus is pulled upward over the presenting part of the baby.

15. The release of glucagon into the bloodstream stimulates: A) the liver to convert glycogen to glucose. B) the liver to take in and store more glucose. C) the cells to uptake sugar from the bloodstream. D) the vessels to constrict, thus increasing blood pressure.

A) the liver to convert glycogen to glucose.

45. During third trimester vaginal bleeding: A) the mother may lose as much as 40% of her blood volume before significant signs of hypovolemia occur. B) signs of shock in the mother typically appear after she has lost approximately 500 to 1000 mL of blood. C) most patients stabilize after the vagina has been packed carefully with several sterile trauma dressings. D) you should focus your efforts on keeping the fetus oxygenated, as he or she is the one at highest risk for death.

A) the mother may lose as much as 40% of her blood volume before significant signs of hypovolemia occur.

17. Isolated rib fractures may result in inadequate ventilation because: A) the patient often purposely limits chest wall movement. B) most rib fractures cause paradoxical chest wall movement. C) the pain associated with the fracture causes hyperventilation. D) preferential use of the intercostal muscles reduces tidal volume.

A) the patient often purposely limits chest wall movement.

The outer zone of an entrance or exit wound caused by a contact electrical burn is: A) the red zone of coagulation necrosis. B) simply caused by local inflammation. C) a charred area of full-thickness burn. D) characterized by cold, gray, dry tissue.

A) the red zone of coagulation necrosis

9. When the body's metabolic rate decreases: A) the thyroid gland secretes thyroxine. B) oxygen demand increases accordingly. C) the kidneys excrete more sodium and water. D) thyroid-stimulating hormone secretion is reduced.

A) the thyroid gland secretes thyroxine.

120. You should be MOST suspicious for child abuse when caring for an injured 4-year-old child if: A) there was an unusual delay in calling 9-1-1. B) the child presents with bruises to both shins. C) the caregiver demands that you treat the child. D) you can smell alcohol on the caregiver's breath.

A) there was an unusual delay in calling 9-1-1.

While standing by at the scene of a structural fire, it is MOST important to remember that: A) toxic gases are often present, even after the fire is out. B) the lead paramedic determines where you should stage. C) most fabric materials release cyanide when they burn. D) you may need to provide rehabilitation for fire fighters.

A) toxic gases are often present, even after the fire is out.

51. The goal in treating a child with epiglottitis is to: A) transport him or her to the hospital with a maintainable airway. B) administer corticosteroids to reduce edema in the upper airway. C) intubate him or her before the epiglottis blocks the upper airway. D) administer oxygen by nonrebreathing mask and transport at once.

A) transport him or her to the hospital with a maintainable airway.

10. Mittelschmerz is defined as: A) unilateral abdominal pain and cramping that occur during the ovulatory process. B) bilateral abdominal pain and vaginal bleeding caused by excess hormone release. C) abdominal cramping, vaginal bleeding, and a headache lasting more than a week. D) severe abdominal cramping and a vascular headache that are caused by ovulation.

A) unilateral abdominal pain and cramping that occur during the ovulatory process.

59. When inserting an oropharyngeal airway in a child, you should: A) use a tongue blade to depress the tongue. B) open the mouth with the tongue-jaw lift. C) hyperextend the head to facilitate insertion. D) suction the oropharynx for 15 seconds first.

A) use a tongue blade to depress the tongue.

93. You are in the process of assisting a woman in labor. She is 38 weeks pregnant with her second child, and her contractions are 3 minutes apart and are strong and regular. Her first baby was delivered by cesarean section. As you prepare for imminent delivery, the woman tells you that her contractions have decreased significantly and that she is very dizzy. Assessment of her vaginal area reveals moderate vaginal bleeding. You quickly assess her and note that she is diaphoretic, tachycardic, and hypotensive. You should be MOST suspicious for: A) uterine rupture. B) placenta previa. C) abruptio placenta. D) threatened abortion.

A) uterine rupture.

30. Commotio cordis is a phenomenon in which: A) ventricular fibrillation is induced following blunt trauma to the chest during the heart's repolarization period. B) excessive pressure within the pericardial sac impairs cardiac contractility as well as venous return to the heart. C) penetrating thoracic trauma perforates the atria or ventricles, causing acute rupture and massive hemorrhage. D) myocardial tissue at the cellular level is damaged by blunt or penetrating trauma, resulting in cardiac arrest.

A) ventricular fibrillation is induced following blunt trauma to the chest during the heart's repolarization period.

77. Early distributive shock in children is characterized by: A) warm, flushed skin. B) weak peripheral pulses. C) pallor and diaphoresis. D) gross neurologic deficits.

A) warm, flushed skin.

34. When a child who is too young to verbalize is in significant pain: A) your ability to assess accurately for physiologic abnormalities is impaired. B) narcotic analgesic drugs should be avoided unless transport will be delayed. C) benzodiazepine drugs are preferred over opiates to minimize central nervous system depression. D) pain scales using facial expressions are a valuable tool to assess pain severity.

A) your ability to assess accurately for physiologic abnormalities is impaired.

A flattened appearance to the face and loss of sensation over the cheek following blunt facial trauma is MOST indicative of a(n): A) zygomatic fracture. B) orbital skull fracture. C) Le Fort I fracture. D) temporomandibular joint dislocation.

A) zygomatic fracture.

76. According to the Los Angeles Prehospital Stroke Screen, the possibility of a stroke is increased if: A) the patient is younger than 45 years of age. B) the blood glucose is between 60 and 400 mg/dL. C) at baseline, the patient is normally bedridden. D) a bilateral facial droop is present during the exam.

Ans: B Page: 1064

5. Red blood cell production is stimulated by erythropoietin, which is secreted by the: A) liver. B) spleen. C) kidneys. D) bone marrow.

Ans: C Page: 1245

10. Which of the following is NOT a major component of the hematologic system? A) Liver B) Spleen C) Pancreas D) Bone marrow

Ans: C Page: 1246

15. In which of the following situations would a transfusion reaction MOST likely occur? A) A person with type AB blood receives type O blood B) A person with type O blood receives type AB blood C) A person with type A-negative blood receives type O blood D) A person with type B-positive blood receives type B-positive blood

Ans: B Page: 1248, 1258

58. A 19-year-old man presents with a decreased level of consciousness. According to his girlfriend, he has no known medical problems and takes no medications. Initial treatment for this patient involves: A) administering oxygen via nonrebreathing mask. B) ensuring airway patency and adequate breathing. C) evaluating his cardiac rhythm with the ECG monitor. D) performing a rapid check of his blood glucose level.

Ans: B Page: 1065-1066

47. Icteric sclera and acholic stools are clinical manifestations of: A) colitis. B) cirrhosis. C) pancreatitis. D) diverticulitis.

Ans: B Page: 1148

31. An incarcerated hernia is one that: A) causes immediate bowel necrosis. B) cannot be reduced and becomes trapped. C) spontaneously returns to its normal location. D) protrudes through an incision from a recent surgery.

Ans: B Page: 1150

5. The cone-shaped section of bone located at the base of each temporal bone is called the: A) lamboid suture. B) mastoid process. C) cribriform plate. D) ethmoid bone.

B

Causes of rhinitis may include all of the following, EXCEPT: A) chemical irritants. B) high humidity. C) antihypertensives. D) cold temperature.

B

29. Pyloric stenosis is the most common cause of: A) Mallory-Weiss syndrome. B) infantile intestinal obstruction. C) gastrointestinal bleeding in adults. D) mesenteric ischemia.

Ans: B Page: 1152

77. Which of the following causes of altered mental status is NOT an acute process? A) Uremia B) Acidosis C) Psychosis D) Epilepsy

Ans: A Page: 1066

48. An idiopathic seizure is one in which: A) the cause is not known. B) the entire brain is affected. C) a part of the brain is affected. D) a postictal phase is not present.

Ans: A Page: 1067

1. Regulatory functions of blood include: A) carrying nutrients throughout the body. B) transporting hormones to their target organs. C) transporting oxygen from the lungs to the tissues. D) carrying antibodies that protect against foreign organisms.

Ans: B Page: 1245

25. Hyperventilating a patient who has increased intracranial pressure (ICP) will: A) dilate the cerebral vasculature and cause further increases in ICP. B) constrict the cerebral vasculature and decrease cerebral perfusion. C) increase the carbon dioxide levels in the brain through vasodilation. D) decrease ICP and maintain adequate cerebral perfusion.

Ans: B Page: 1061

6. The hematocrit is the: A) ratio of red blood cells to platelets. B) overall proportion of red blood cells in the blood. C) percentage of blood accounted for by the plasma. D) percentage of hemoglobin found within red blood cells.

Ans: B Page: 1245

7. You would expect a person's hematocrit to be low if he or she: A) is dehydrated. B) is hemorrhaging. C) has polycythemia. D) is losing plasma.

Ans: B Page: 1245-1246

4. What portion of the brainstem regulates respiratory rate and depth? A) Pons B) Medulla C) Midbrain D) Basal nuclei

Ans: A Page: 1041

10. When a neuron generates an electrical impulse, it first sends the signal: A) along its axon to the axon terminal. B) to the cell's nucleus via a dendrite. C) via neurotransmitters at the synapse. D) to the cell's dendrites simultaneously.

Ans: A Page: 1042

11. Myelin functions by: A) allowing the neuron to send its signal consistently along the axon without losing its electricity. B) insulating the neuron, thus decreasing the speed of electrical conduction between two neurons. C) covering the neurons in the body that do not require rapid conduction of an electrical impulse. D) briefly delaying the conduction of an electrical impulse between the synaptic cleft and dendrite.

Ans: A Page: 1042-1043

28. All of the following cranial nerves are responsible for airway control, EXCEPT the: A) abducens. B) trigeminal. C) hypoglossal. D) glossopharyngeal.

Ans: A Page: 1044

34. Common causes of trismus in an unresponsive patient include all of the following, EXCEPT: A) opiate toxicity. B) cerebral hypoxia. C) a head injury. D) an active seizure.

Ans: A Page: 1044

35. A patient with a Glasgow Coma Scale score of 8 or less: A) may require definitive airway control. B) has moderate neurologic dysfunction. C) is often able to localize his or her pain. D) is likely experiencing brain herniation.

Ans: A Page: 1051

72. Which of the following cranial nerves are assessed when you ask a patient to follow your finger as you move it in an "H" shape? A) Trochlear, oculomotor, abducens B) Facial, accessory, glossopharyngeal C) Accessory, olfactory, hypoglossal D) Optic, trigeminal, glossopharyngeal

Ans: A Page: 1054

43. __________ is a term used to describe changes in a person's ability to perform coordinated motions, such as walking. A) Ataxia B) Myoclonus C) Bradykinesia D) Decussation

Ans: A Page: 1055

54. You respond to the residence of a known heroin abuser. The patient, a 30-year-old man, is unconscious and unresponsive. He is hypoventilating, bradycardic, and hypotensive. Administration of 10 mg of naloxone has had no effect, and your transport time to the hospital is approximately 30 minutes. You should: A) check his blood glucose level as your partner prepares to intubate. B) transport at once and administer additional naloxone while en route. C) administer 25 g of 50% dextrose and reassess his level of consciousness. D) hyperventilate him with high-flow oxygen to minimize tissue hypoxia.

Ans: A Page: 1056-1057

17. When neurons are deprived of oxygen and glucose: A) they convert to anaerobic metabolism. B) they produce alkalotic waste products. C) spontaneous intracerebral bleeding occurs. D) the process of neuronal regeneration begins.

Ans: A Page: 1058

23. Which of the following would have the MOST negative effect on the outcome of a patient with an intracranial hemorrhage? A) Hypotension B) Tachycardia C) Hypertension D) Slow rise in intracranial pressure

Ans: A Page: 1060

53. A 39-year-old woman presents with signs and symptoms of an acute hemorrhagic stroke. She is responsive to deep painful stimuli only and has bilaterally dilated and sluggishly reactive pupils. Her respirations are slow and irregular, blood pressure is 80/50 mm Hg, and pulse is 40 and bounding. Initial treatment for this patient involves: A) positive-pressure ventilatory support. B) rapid infusion of a crystalloid solution. C) high-flow oxygen via nonrebreathing mask. D) immediate transcutaneous cardiac pacing.

Ans: A Page: 1061

59. You are dispatched to a local pharmacy, where a 24-year-old woman experienced an apparent seizure. During your assessment, you note that the patient is conscious but combative. The patient's supervisor states that she has a history of seizures and takes Tegretol. The patient's blood pressure is 146/90 mm Hg, pulse rate is 110 beats/min and regular, and respirations are 24 breaths/min with adequate depth. The MOST appropriate treatment for this patient includes: A) administering oxygen as tolerated, establishing an IV line, padding the rails of the ambulance cot, and transporting without lights and siren. B) establishing vascular access, administering diazepam or lorazepam to reduce her combativeness, and transporting to the closest hospital. C) giving her high-flow oxygen, inserting an IO catheter in her proximal tibia, transporting, and observing for further seizure activity. D) administering oxygen, restraining her to protect her from further injury, placing a bite block in her mouth in case she seizes again, and transporting.

Ans: A Page: 1067-1069

60. You arrive at the scene shortly after a 7-year-old girl experienced a seizure. According to the child's mother, she was sitting at the dinner table and then suddenly stopped speaking and started blinking her eyes very rapidly. The episode lasted less than 1 minute, after which the child's condition rapidly improved. This clinical presentation is consistent with a(n) _____________ seizure. A) absence B) tonic-clonic C) simple partial D) complex partial

Ans: A Page: 1068

40. The involuntary, rhythmic movement of the eyes is called: A) nystagmus. B) hyperopia. C) anisocoria. D) dysphagia.

Ans: A Page: 1073

16. The MOST common sign of an infectious disease is the presence of: A) a fever. B) seizures. C) a headache. D) tachycardia.

Ans: A Page: 1077

65. A 56-year-old diabetic woman presents with numbness and tingling to both of her hands that have gradually intensified over the past few weeks. Your assessment reveals stable vital signs, adequate breathing, and a blood glucose level of 190 mg/dL. The cardiac monitor reveals a normal sinus rhythm. The patient tells you that she has eaten but has not taken her insulin yet. You should: A) provide supportive care and safely transport her to the hospital. B) assist her with her prescribed insulin to lower her blood glucose. C) advise her to take ibuprofen and to follow up with her physician. D) conclude that she has peripheral neuropathy and give her fentanyl.

Ans: A Page: 1079

2. The portal vein transports venous blood from the gastrointestinal tract directly to the: A) liver. B) colon. C) spleen. D) gallbladder.

Ans: A Page: 1126

5. Drinking alcohol with a fatty meal: A) delays gastric emptying. B) slows duodenal absorption. C) increases gastric absorption. D) rapidly increases blood alcohol levels.

Ans: A Page: 1127

9. Which of the following is NOT a function of the liver? A) Storage of bile B) Drug detoxification C) Storage of vitamins and minerals D) Completion of red and white blood cell breakdown

Ans: A Page: 1127

11. If water is not reabsorbed in the colon: A) diarrhea occurs. B) constipation occurs. C) the appendix may rupture. D) waste material is solidified.

Ans: A Page: 1128

32. A patient with an advanced bowel obstruction would MOST likely present with: A) a feculent breath odor. B) black, tarry stools. C) bright red hematemesis. D) deep, rapid respirations.

Ans: A Page: 1130

38. If a patient complains of left lower quadrant abdominal pain, you should FIRST palpate the: A) right upper quadrant. B) right lower quadrant. C) left lower quadrant. D) left upper quadrant.

Ans: A Page: 1133

51. A 38-year-old man presents with an acute onset of severe right upper quadrant abdominal pain, pain to his right shoulder, and nausea. He is conscious and alert, but is restless from the pain. His blood pressure is 150/86 mm Hg, pulse rate is 120 beats/min and strong, and respirations are 22 breaths/min and regular. In addition to administering supplemental oxygen, you should: A) start an IV with normal saline and set it to keep the vein open, place him in a position of comfort, and administer 4 mg of morphine and 12.5 mg of promethazine. B) encourage him to remain supine to relieve his pain, monitor his oxygen saturation level, and administer 10 to 20 mg of Nubain via the IM route. C) establish vascular access with a large-bore catheter, administer a 500-mL normal saline bolus, and avoid analgesics due to the potential for intra-abdominal bleeding. D) perform a comprehensive abdominal exam, start at least one large-bore IV line, infuse normal saline at 125 mL/hr, and consider giving him an antiemetic medication.

Ans: A Page: 1135-1136

45. A sudden onset of discomfort in the throat, severe dysphagia, and vomiting bright red blood are MOST indicative of: A) ruptured esophageal varices. B) gastroesophageal reflux disease. C) a malignancy in the esophagus. D) hemorrhage from a peptic ulcer.

Ans: A Page: 1139-1140

26. An obese 52-year-old woman who presents with severe pain in the right upper quadrant of her abdomen and pain in her right shoulder is MOST likely experiencing: A) cholecystitis. B) acute hepatitis. C) Crohn disease. D) Mallory-Weiss syndrome.

Ans: A Page: 1143

27. The MOST likely cause of Crohn disease is: A) autoantibody destruction of the intestinal wall. B) precancerous lesions in the lower intestinal tract. C) chronically high triglyceride levels in the blood. D) infection of the gastrointestinal tract with Helicobacter pylori.

Ans: A Page: 1146

19. Common signs and symptoms of acute gastroenteritis include all of the following, EXCEPT: A) fever. B) chest pain. C) severe diarrhea. D) abdominal pain.

Ans: A Page: 1147

39. Hepatic encephalopathy is a condition in which: A) liver disease causes reduced brain function. B) the liver is acutely inflamed from a virus. C) acute hypoglycemia results from liver failure. D) alcohol progressively destroys the liver tissue.

Ans: A Page: 1149

2. The volume of plasma in an adult with a total blood volume of 6.5 liters is approximately: A) 3.6 liters. B) 4.1 liters. C) 4.5 liters. D) 5.1 liters.

Ans: A Page: 1245

13. The primary component of the immune system is the: A) leukocyte. B) neutrophil. C) eosinophil. D) lymphocyte.

Ans: A Page: 1245, 1247

8. Approximately one third of the body's platelets: A) are stored in the spleen. B) are produced by the liver. C) circulate in the bloodstream. D) have a life span of 72 hours.

Ans: A Page: 1246

31. You are dispatched to a residence for a 29-year-old woman with difficulty breathing. The patient was recently diagnosed with leukemia and has just completed her first round of chemotherapy and radiation therapy. Upon arriving at the scene, you find the patient sitting in a recliner. Her level of consciousness is markedly diminished and her breathing is shallow. Her husband, obviously frightened, pleads with you to do something. You should: A) move her to the floor and open her airway. B) assess her carotid pulse for rate and regularity. C) assist her ventilations while she is in a sitting position. D) administer high-flow oxygen via nonrebreathing mask.

Ans: A Page: 1249-1250

20. In contrast to chronic leukemia, acute leukemia occurs when: A) bone marrow is replaced with abnormal lymphoblasts. B) abnormal lymphoid cells accumulate in the bone marrow. C) the patient experiences easy bruising and frequent bleeding. D) mutated fibroblasts rapidly accumulate in the circulatory system.

Ans: A Page: 1254

19. Which of the following is NOT typically associated with leukemia? A) Absence of fever B) Frequent bleeding C) Chronic infections D) Bruising from minor trauma

Ans: A Page: 1254-1255

25. Abdominal pain that is associated with polycythemia is usually associated with: A) an enlarged spleen. B) liver engorgement. C) acute pancreatitis. D) gastrointestinal bleeding.

Ans: A Page: 1256

26. During the second stage of disseminated intravascular coagulopathy: A) decreased clotting factors cause uncontrolled hemorrhage. B) free thrombin and fibrin deposits increase in the bloodstream. C) the coagulation and fibrinolytic systems become overwhelmed. D) large quantities of platelets aggregate and cause the blood to clot.

Ans: A Page: 1256

5. Emotions such as rage and anger are generated in the: A) thalamus. B) limbic system. C) hypothalamus. D) diencephalon.

Ans: B Page: 1041

6. Among other functions, the medulla oblongata: A) directly regulates body temperature. B) controls blood pressure and heart rate. C) communicates with the pituitary gland. D) sends messages to move skeletal muscles.

Ans: B Page: 1041

26. Management for a patient with a neurologic emergency begins by: A) ensuring that the patient's airway remains patent. B) taking deliberate actions to ensure personal safety. C) determining the degree of neurologic impairment. D) protecting the patient from further injury or harm.

Ans: B Page: 1043-1044

51. An elderly man who is a resident of a skilled nursing facility is found unresponsive by a staff nurse. When you and your partner arrive, you assess the patient and note that his respirations are slow and shallow; his heart rate is slow, weak, and irregular; and his skin is cool and clammy. You should: A) obtain a 12-lead ECG tracing and assess his pupils. B) assist his ventilations and assess his oxygen saturation. C) assess his blood glucose level and give high-flow oxygen. D) apply oxygen via nonrebreathing mask and apply the ECG.

Ans: B Page: 1044-1045

36. A patient with a unilateral eyelid droop when smiling: A) has a condition called miosis. B) may be experiencing Bell palsy. C) likely has an intracerebral hemorrhage. D) is experiencing transient cerebral ischemia.

Ans: B Page: 1048

74. Pupils that differ in size by less than 1 mm: A) indicate increased intracranial pressure. B) may be a normal variant in some patients. C) are a sign of methamphetamine abuse. D) are a sign of optic nerve compression.

Ans: B Page: 1052-1053

37. A patient who is experiencing agnosia: A) has neuronal dysfunction between the temporal and frontal lobe. B) may call a knife a fork when he or she is asked to name the object. C) is probably experiencing an ischemic stroke in the cerebellar area. D) cannot use a pencil secondary to dysfunction of the occipital lobe.

Ans: B Page: 1053

41. Because the facial nerve does not decussate, a stroke to the right cerebral hemisphere would MOST likely cause: A) left-sided weakness and a left-sided facial droop. B) left-sided weakness and a right-sided facial droop. C) right-sided weakness and a left-sided facial droop. D) right-sided weakness and a right-sided facial droop.

Ans: B Page: 1054-1055

31. What type of tremor occurs when a body part is placed in a particular position and required to maintain that position for a long period of time? A) Rest B) Postural C) Intention D) Jacksonian

Ans: B Page: 1055

45. When performing the arm drift test on a patient with a suspected stroke, a positive finding is characterized by: A) one arm drifting downward and turning away from the body when the patient's eyes are closed. B) one arm drifting downward and turning toward the body when the patient's eyes are closed. C) one arm drifting downward and turning away from the body when the patient's eyes are open. D) both arms moving downward more than 3 inches in 30 seconds when the patient's eyes are open.

Ans: B Page: 1055

47. A rhythmic contraction and relaxation of muscle groups that is commonly observed during a seizure is called __________ activity. A) tonic B) clonic C) hypertonic D) myoclonic

Ans: B Page: 1056

18. All of the following are examples of acute cerebrovascular emergencies, EXCEPT: A) embolic blockage of a cerebral artery. B) accumulation of atherosclerotic plaque. C) rupture of a cerebral arterial aneurysm. D) stroke secondary to thrombus rupture.

Ans: B Page: 1058-1060

21. What is the cerebral perfusion pressure of a patient with a mean arterial pressure of 80 mm Hg and an intracranial pressure of 5 mm Hg? A) 60 mm Hg B) 75 mm Hg C) 90 mm Hg D) 95 mm Hg

Ans: B Page: 1060

56. An elderly man presents with slurred speech, confusion, and unilateral facial asymmetry. When asked to squeeze your hands, the strength in his left hand is markedly less than the strength in his right hand. The patient's wife tells you that her husband has type 2 diabetes and hypertension. On the basis of your clinical findings, you should: A) consider him a candidate for fibrinolytic therapy if his symptoms began less than 12 hours ago. B) rule out hypoglycemia by assessing his blood sugar, but suspect a right-sided ischemic stroke. C) start an IV and administer crystalloid fluid boluses if his systolic blood pressure is less than 110 mm Hg. D) suspect that he is experiencing a hemorrhagic stroke, begin immediate transport, and start an IV en route.

Ans: B Page: 1060-1063

12. Production of clotting factors is a function of the: A) kidney. B) liver. C) spleen. D) bone marrow.

Ans: B Page: 1246

62. A 33-year-old woman had an apparent syncopal episode. According to her husband, she complained of dizziness shortly before the episode. He further states that he caught her before she fell to the ground. Upon your arrival, the patient is conscious but confused and is sitting in a chair. Her blood pressure is 90/60 mm Hg, pulse rate is 110 beats/min and weak, and respirations are 22 breaths/min and regular. Her blood glucose level is 74 mg/dL. The MOST likely cause of her syncopal episode is: A) a seizure. B) dehydration. C) a vasovagal response. D) acute hypoglycemia.

Ans: B Page: 1069-1070

79. In contrast to syncope, a seizure: A) is less commonly observed in older patients. B) can occur when the patient is in any position. C) most often occurs when the patient is standing. D) is characterized by a quick return of orientation.

Ans: B Page: 1069-1070

12. A neoplasm is MOST accurately defined as a: A) normal cell. B) new growth. C) damaged cell. D) cancerous tumor.

Ans: B Page: 1071

13. In contrast to a benign neoplasm, a malignant neoplasm: A) is relatively easy to remove surgically. B) can metastasize to other parts of the body. C) results in death in the vast majority of cases. D) remains within a capsule and grows minimally.

Ans: B Page: 1071-1072

64. A 51-year-old man complains of chronic headaches that have worsened progressively over the past 3 months. Today, he called 9-1-1 because his headache is severe and he is nauseated. His vital signs are stable and he is breathing adequately. The patient denies any medical problems and states that he has been taking acetaminophen for the headaches. You should be MOST suspicious for: A) acute sinusitis. B) an intracranial neoplasm. C) acute hemorrhagic stroke. D) a ruptured cerebral artery.

Ans: B Page: 1071-1072

68. Increased intracranial pressure hydrocephalus occurs when: A) the movement of CSF in the brain exceeds the production of CSF. B) the movement of CSF decreases, but the production of CSF does not. C) CSF production increases, which causes increased pressure in the skull. D) total CSF volume decreases, which leads to excessive cerebral edema.

Ans: B Page: 1080

69. Which of the following conditions is a neural tube defect in which part of the nervous system remains outside the body? A) Multiple sclerosis B) Spina bifida C) Poliomyelitis D) Cerebral palsy

Ans: B Page: 1080-1081

3. Peristalsis is defined as: A) the churning of food. B) rhythmic contraction. C) turbulent blood flow. D) passive regurgitation.

Ans: B Page: 1126

4. The ___________ controls the amount of food that moves back up the esophagus. A) duodenum B) cardiac sphincter C) gastric mucosa D) pyloric sphincter

Ans: B Page: 1126

8. The conversion of glycogen to glucose occurs in the: A) blood. B) liver. C) spleen. D) pancreas.

Ans: B Page: 1127

13. Filtering of blood and recycling of dead red blood cells are functions of the: A) liver. B) spleen. C) pancreas. D) kidneys.

Ans: B Page: 1129

49. You are caring for a middle-aged man with severe abdominal pain and dark, tarry stools. He is conscious but very restless. His blood pressure is 78/52 mm Hg, pulse rate is 130 beats/min and weak, and respirations are 24 breaths/min and shallow. Further assessment reveals that his skin is cool and clammy and his radial pulses are weakly present. You should: A) apply a nasal cannula in case he vomits, start at least one large-bore IV line, and administer up to 3 liters of normal saline solution. B) administer high-flow oxygen, start two large-bore IV lines, and administer 20-mL/kg normal saline boluses until his radial pulses strengthen. C) apply supplemental oxygen, establish vascular access, and give isotonic crystalloid boluses until his systolic BP is at least 110 mm Hg. D) administer high-flow oxygen, start a large-bore IV line, administer a 20-mL/kg normal saline bolus, and give 1 µg/kg of fentanyl for pain.

Ans: B Page: 1130, 1137

36. A protuberant abdomen would MOST likely be encountered in a patient with: A) an abdominal evisceration. B) peritoneal fluid accumulation. C) decreased abdominal volume. D) an acutely inflamed appendix.

Ans: B Page: 1132

41. Irritation or injury to abdominal tissue, causing activation of peripheral nerve tracts, would MOST likely result in ___________ pain. A) visceral B) somatic C) rebound D) parietal

Ans: B Page: 1134

43. A positive Murphy sign is characterized by: A) slow, shallow breathing in an attempt to reduce the severe pain associated with cholecystitis. B) a sudden stop in inspiration due to sharp pain when pressure is applied to the right upper quadrant. C) ecchymosis to the flank area, indicative of free blood in the retroperitoneal compartment. D) periumbilical ecchymosis and is a late, but highly suggestive sign of blood in the peritoneum.

Ans: B Page: 1135

25. Common signs of bleeding in the upper gastrointestinal tract include all of the following, EXCEPT: A) melena. B) hematochezia. C) hematemesis. D) dark, tarry stools.

Ans: B Page: 1139-1141

22. Dunphy sign is observed when: A) right upper quadrant pain is referred to the right shoulder. B) coughing causes severe right lower quadrant pain. C) the patient's abdominal pain radiates to the shoulder. D) pressure on the abdominal wall is suddenly released.

Ans: B Page: 1143

53. You are dispatched to a residence for a 33-year-old woman with excruciating pain to the right lower quadrant of her abdomen. She is conscious and alert and tells you that she has been experiencing pain to the same area for the past few days. Her blood pressure is 110/66 mm Hg, pulse rate is 118 beats/min and strong, and respirations are 22 breaths/min with adequate depth. Her skin is warm and moist. The MOST appropriate treatment for this patient involves: A) high-flow oxygen, two large-bore IV lines, a 20-mL/kg normal saline bolus, and rapid transport to an appropriate medical facility. B) oxygen via nonrebreathing mask, an IV of isotonic crystalloid set to keep the vein open, morphine or fentanyl, and prompt transport. C) ventilation assistance with a bag-mask device, immediate transport, and establishment of vascular access en route to the closest hospital. D) supplemental oxygen via nasal cannula, 25 mg of Phenergan IM, an IV of normal saline set to keep the vein open, and prompt transport.

Ans: B Page: 1143

20. Which of the following conditions would be the LEAST likely to result in peritonitis? A) Stomach rupture B) Splenic laceration C) Acute appendicitis D) Bowel perforation

Ans: B Page: 1147

30. During your primary assessment of a patient with a hematologic disorder, your priority should be to: A) perform a rapid head-to-toe exam to look for spontaneous hemorrhage. B) note any signs and symptoms that may be immediately life threatening. C) specifically inquire if the patient complains of dyspnea or chest pressure. D) apply a cardiac monitor to detect the presence of lethal cardiac dysrhythmias.

Ans: B Page: 1249-1250

35. A 23-year-old woman with sickle cell disease presents with severe joint pain and a fever of 102.5°F. She is conscious and alert, and tells you that her symptoms began yesterday and suddenly worsened today. Her blood pressure is 118/76 mm Hg, pulse rate is 120 beats/min and regular, and respirations are 24 breaths/min with adequate depth. After applying high-flow oxygen and initiating transport, you should: A) obtain a 12-lead ECG tracing, start a large-bore IV line, administer a 20-mL/kg normal saline bolus, and reassess her blood pressure. B) monitor her oxygen saturation and cardiac rhythm, start an IV line with normal saline, administer 1 µg/kg of fentanyl, and reassess her vital signs. C) establish vascular access, administer 2 to 5 mg of midazolam, monitor her cardiac rhythm, and notify the receiving facility as soon as possible. D) ensure that she is in a comfortable position, cover her with a blanket, start an IV line at a keep-vein-open rate, and monitor her throughout transport.

Ans: B Page: 1252-1253

16. Anemia would result from all of the following conditions, EXCEPT: A) acute blood loss. B) an increase in iron. C) chronic hemorrhage. D) erythrocyte hemolysis.

Ans: B Page: 1253

22. Indolent non-Hodgkin lymphoma: A) is the most aggressive form of the disease. B) may never spread from the lymphoid system. C) affects multiple organs over a long period of time. D) most frequently occurs in men over 50 years of age.

Ans: B Page: 1255

24. Common signs and symptoms of lymphoma include: A) an increased appetite. B) night sweats and chills. C) weight gain and nausea. D) irritability and a skin rash.

Ans: B Page: 1255

29. As multiple myeloma progresses, the patient would MOST likely experience: A) frequent nosebleeds. B) spontaneous fractures. C) acute intracranial bleeding. D) atrophy of large muscle groups.

Ans: B Page: 1257

34. You receive a call to a residence for a patient who is "sick." Upon arriving at the scene, you find the patient, a 39-year-old woman, lying on the couch with a wet washcloth on her forehead. She is conscious and alert, and tells you that she has had several episodes of diarrhea and noticed bright red blood in her stool. Her pulse rate is rapid and weak, her skin is cool and clammy, and her blood pressure is 98/58 mm Hg. Her medical history is significant for hemophilia, for which she is receiving factor VIII therapy. As you pull out a nonrebreathing mask, she tells you that her husband will be home in a few hours and that he will take her to the hospital. You should: A) notify her husband, explain the situation to him, and advise him that you will be transporting his wife to the hospital. B) advise her that her condition dictates immediate transport to the hospital and that delaying transport could result in death. C) tell her that her condition is critical and that you are legally required to transport her to the closest appropriate medical facility. D) ask her to sign a refusal of EMS transport form, have a neighbor sit with her until her husband gets home, and then return to service.

Ans: B Page: 1257

2. Components of the diencephalon include the: A) pons and medulla. B) brainstem and midbrain. C) thalamus and hypothalamus. D) cerebellum and cerebral cortex.

Ans: C Page: 1040

8. A synapse is: A) a cluster of sensory nerve cells. B) the body's main neurotransmitter. C) a slight gap between nerve cells. D) the point where a nerve cell terminates.

Ans: C Page: 1042

29. Patients with trismus often require: A) premedication with lidocaine before being intubated. B) hyperventilation to reduce severe intracranial pressure. C) sedation or chemical paralysis to facilitate airway control. D) immediate tracheal intubation before the airway swells shut.

Ans: C Page: 1044, 1057

30. In contrast to patients in shock, patients with increased intracranial pressure MOST often experience: A) systolic hypotension. B) an increased diastolic blood pressure. C) a widened pulse pressure. D) tachycardia and tachypnea.

Ans: C Page: 1045

50. Applying pressure to the supraorbital foramen of a patient with a neurologic insult is intended to: A) stimulate the vagus nerve. B) determine if the spinal cord is intact. C) elicit a response to painful stimuli. D) assess for an orbital skull fracture.

Ans: C Page: 1049-1050

39. Which of the following is an example of receptive aphasia? A) A patient with slurred speech is able to tell you his or her name. B) You hand a patient a pencil and he or she tries to cut paper with it. C) You ask a patient who the president is and he or she says, "January." D) A patient responds with "no" when asked if he or she has hypertension.

Ans: C Page: 1053

73. Which of the following cranial nerves regulates movement of the head and shoulders? A) Hypoglossal B) Abducens C) Accessory D) Trigeminal

Ans: C Page: 1054

20. As intracranial pressure rises: A) the heart rate acutely increases. B) the brain becomes hypocarbic. C) cerebral herniation may occur. D) mean arterial pressure decreases.

Ans: C Page: 1060

24. For any patient with an increase in intracranial pressure, the paramedic must: A) avoid administering IV fluid boluses. B) give glucose to prevent hypoglycemia. C) maintain an adequate blood pressure. D) take measures to lower blood pressure.

Ans: C Page: 1061

14. Prehospital treatment for a patient with a suspected stroke may include all of the following, EXCEPT: A) 30° elevation of the head. B) diazepam or lorazepam. C) up to 325 mg of aspirin. D) crystalloid fluid boluses.

Ans: C Page: 1061-1064

21. In contrast to a patient with peritonitis, a patient with hepatitis would MOST likely initially experience: A) hematochezia. B) diffuse abdominal pain. C) right upper quadrant pain. D) referred pain to the left shoulder.

Ans: C Page: 1142

30. Intestinal adhesions would MOST likely result in: A) diarrhea. B) gastroenteritis. C) bowel obstruction. D) ulcerative colitis.

Ans: C Page: 1149-1150

3. Formed elements of the blood account for approximately ___% of the total blood volume. A) 25 B) 35 C) 45 D) 55

Ans: C Page: 1245

57. You are dispatched to a residence for a middle-aged woman with generalized weakness of approximately 18 hours' duration. Your primary assessment reveals right-sided hemiparesis, a left-sided facial droop, and bilaterally equal and reactive pupils. Further assessment reveals that her blood glucose level is 70 mg/dL. En route to the hospital, you note increased movement of her right arm. She is receiving oxygen via nasal cannula and has a patent IV line in place. Which of the following statements regarding this scenario is correct? A) You should administer nitroglycerin or labetalol if her systolic blood pressure is greater than 150 mm Hg. B) The patient will likely receive fibrinolytic therapy in the emergency department if no contraindications exist. C) Although the patient is likely experiencing a TIA, you should treat her as though she is experiencing a stroke. D) The patient's signs and symptoms are likely the result of her blood sugar and will resolve with dextrose.

Ans: C Page: 1064-1065

78. During a generalized tonic/clonic seizure, the patient is rigid and his back is arched. This represents the _________ phase of the seizure. A) tonic B) clonic C) hypertonic D) postictal

Ans: C Page: 1067

63. You are dispatched to a residence for a 44-year-old woman with a severe headache. You arrive to find the patient lying on her sofa with a wet washcloth on her forehead. She tells you that she has a history of migraine headaches and that this is one of her "typical" headaches. She also complains of nausea and photophobia. Her blood pressure is 170/94 mm Hg, pulse rate is 120 beats/min and regular, and respirations are 22 breaths/min with adequate depth. The MOST appropriate treatment for this patient includes: A) carefully assessing her pupils, administering oxygen, starting an IV and giving her morphine, and transporting. B) placing her in a Fowler position, administering oxygen, and transporting her to the hospital for definitive care. C) oxygen as tolerated, starting an IV line and administering ondansetron, and transporting without lights or siren. D) administering high-flow oxygen, establishing vascular access, and administering nitroglycerin to lower her blood pressure.

Ans: C Page: 1070-1071

44. Which of the following disease processes is characterized by an abnormal gait in which the patient places his or her feet very close together and shuffles while walking? A) Cerebral palsy B) Multiple sclerosis C) Parkinson disease D) Alzheimer disease

Ans: C Page: 1075

49. A patient experiences severe, shock-like or stabbing pain to one side of the face. This is consistent with: A) an acoustic neuroma. B) hemifacial spasm. C) trigeminal neuralgia. D) glossopharyngeal neuralgia.

Ans: C Page: 1076

66. A 29-year-old man, who was recently prescribed an antipsychotic medication, presents with an acute onset of bizarre contortions of the face. Treatment should include: A) diazepam, 5 mg. B) promethazine, 25 mg. C) diphenhydramine, 25 mg. D) ondansetron, 4 mg.

Ans: C Page: 1077

70. Common signs and symptoms of a cerebral abscess include: A) hemiplegia, low-grade fever, hyperactivity, and a diffuse stabbing headache. B) paralysis below the waist, high fever, a diffuse headache, and slurred speech. C) high-grade fever, persistent localized headache, confusion, and focal impairment. D) an absence of fever, severe localized headache, and vomiting without nausea.

Ans: C Page: 1079

1. Which of the following behaviors does NOT place a person at risk for gastrointestinal disease? A) Stress B) Smoking C) High-fiber diet D) Salicylate ingestion

Ans: C Page: 1125

7. Bile is stored in the _________ and is released into the __________, where it helps to _______. A) liver, gallbladder, break down fats B) duodenum, gallbladder, digest food C) gallbladder, duodenum, emulsify fats D) liver, gallbladder, metabolize proteins

Ans: C Page: 1127

12. The exocrine function of the pancreas produces: A) insulin. B) glucagon. C) enzymes. D) somatostatin.

Ans: C Page: 1127, 1129

14. Pain over the suprapubic region is MOST suggestive of injury to the: A) urethra. B) ureters. C) bladder. D) iliac arteries.

Ans: C Page: 1129

52. A 62-year-old man presents with an acute onset of bright red vomiting. According to his wife, he ingests excessive amounts of alcohol each day. As you are assessing the patient, you note that his level of consciousness has decreased markedly. His mouth is full of blood, and his skin is pale and moist. You should: A) assist his ventilations for 2 to 3 minutes, insert a Combitube, suction his airway for up to 15 seconds, start two large-bore IV lines with normal saline, and administer a 1-L fluid bolus. B) perform immediate tracheal intubation, insert a nasogastric tube, establish IV or IO access, and administer 10- to 20-mL/kg normal saline or lactated Ringer's boluses to maintain a systolic blood pressure of at least 90 mm Hg. C) turn him on his side and suction his oropharynx, intubate his trachea if the oral bleeding continues, establish at least one large-bore IV with normal saline, and administer enough fluids to maintain adequate perfusion. D) suction his mouth for up to 20 seconds, insert a nasopharyngeal airway, ventilate him at a rate of 20 breaths/min, consider endotracheal intubation, start a large-bore IV line, and run the IV wide open until signs of improvement are noted.

Ans: C Page: 1130, 1139-1140

35. During your examination of a patient's abdomen, you note the presence of striae. This finding is MOST indicative of: A) pregnancy within the previous 24 to 36 months. B) abdominal surgery within the previous 12 to 24 months. C) a change in the size of the abdomen over a short period of time. D) a significant decrease in weight over a long period of time.

Ans: C Page: 1132

40. In contrast to somatic pain, visceral pain: A) is well localized. B) indicates peritonitis. C) is difficult to localize. D) increases with movement.

Ans: C Page: 1134

34. A patient with orthostatic vital sign changes: A) often requires atropine. B) is severely hypotensive. C) may faint upon standing. D) has intra-abdominal bleeding.

Ans: C Page: 1135

23. Esophageal varices are a direct result of: A) esophageal erosion. B) alcohol consumption. C) portal hypertension. D) cirrhosis of the liver.

Ans: C Page: 1139

46. The presentation of Mallory-Weiss syndrome is linked to _________ and is caused by _________. A) eating spicy foods, erosion of the lining of the gastrointestinal tract B) spastic coughing, rupture of esophageal veins due to portal hypertension C) severe vomiting, a tear at the junction between the esophagus and stomach D) blunt trauma, rupture of hollow organs with resultant peritoneal inflammation

Ans: C Page: 1140

9. A patient with thrombocytopenia: A) is severely anemic. B) has blood that clots rapidly. C) has a decreased platelet count. D) is at high risk for a pulmonary embolism.

Ans: C Page: 1246, 1253

14. If an individual has A-positive blood: A) he or she can receive type AB blood. B) his or her blood contains no ABO antigens. C) his or her blood contains the Rh antigen. D) his or her blood contains anti-A antibodies.

Ans: C Page: 1248

28. Which of the following statements regarding sickle cell disease is correct? A) Sickle cell disease is an acquired blood disorder that exclusively affects the African American population. B) When the defective HbA gene is inherited from both parents, the patient will not develop the sickle cell trait. C) Patients with sickle cell disease become hypoxic because their misshapen red blood cells are poor carriers of oxygen. D) Patients with sickle cell disease are at a lower risk for thrombotic diseases because their blood contains fewer platelets.

Ans: C Page: 1252

18. Which of the following statements regarding leukemia is correct? A) Leukemia may cause leukocytosis, a low white blood cell count. B) A hallmark of leukemia is an excess production of platelets. C) Leukemia is the result of abnormal white blood cell development. D) Most patients with leukemia have a normal white blood cell count.

Ans: C Page: 1254-1255

23. A painless, progressive enlargement of the lymphoid glands that most commonly affects the spleen and lymph nodes is called: A) sarcoma. B) adenocarcinoma. C) Hodgkin lymphoma. D) non-Hodgkin lymphoma.

Ans: C Page: 1255

33. You are transferring a 60-year-old man with COPD from a community hospital to a metropolitan hospital. The patient is mildly dyspneic, but is otherwise stable. He is receiving oxygen via nasal cannula, has an IV line of normal saline in place, and has an ECG applied. When reading his chart, you note that he has polycythemia, a history of a prior stroke, and hypertension. The patient tells you that he feels a fluttering sensation in his chest. You glance at the cardiac monitor and see a narrow complex tachycardia at a rate of 160 beats/min. The patient's blood pressure is 116/70 mm Hg and he remains conscious and alert. You should: A) carefully auscultate his lung sounds and then administer a 500-mL bolus of normal saline solution. B) administer 5 mg of midazolam, perform synchronized cardioversion with 50 joules, and reassess his cardiac rhythm. C) ensure adequate oxygenation and ventilation, administer 6 mg of adenosine, and reassess his cardiac rhythm. D) place the patient supine, elevate his legs 6 to 12 inches, and administer 150 mg of amiodarone over 10 minutes.

Ans: C Page: 1256

1. The peripheral nervous system is responsible for: A) memory, understanding, and thought processes. B) feeling and autonomic sensory and motor functions. C) thought, perception, and voluntary motor functions. D) sending messages to and receiving messages from the brain.

Ans: D Page: 1038

3. A person's level of consciousness is regulated by the: A) diencephalon. B) cerebral cortex. C) occipital lobe of the brain. D) reticular activating system.

Ans: D Page: 1040-1041

7. A loss of balance and equilibrium suggests injury to the: A) midbrain. B) cerebrum. C) thalamus. D) cerebellum.

Ans: D Page: 1041-1042

9. Chemicals that relay electrically conducted signals from one neuron to another are called: A) hormones. B) endorphins. C) catecholamines. D) neurotransmitters.

Ans: D Page: 1042

52. You are assessing an unresponsive 66-year-old man with a history of two prior strokes. According to the patient's wife, he complained of a severe headache and then passed out. His respiratory effort is poor, blood pressure is elevated markedly, and pulse is slow and bounding. The glucometer reads "error." You should: A) give high-flow oxygen, apply the cardiac monitor, start an IV line with normal saline, and administer 0.5 mg of atropine to treat his bradycardia. B) hyperventilate with a bag-mask at 30 breaths/min, begin transcutaneous pacing, establish vascular access, and administer 25 g of 50% dextrose. C) intubate after 30 seconds of preoxygenation, ventilate at a rate of 8 breaths/min, start an IV with normal saline, administer 50 g of 50% dextrose, and reassess. D) assist ventilations, apply the cardiac monitor, start an IV line with normal saline, administer 12.5 g of 50% dextrose, and reassess his level of consciousness.

Ans: D Page: 1044-1046, 1057

32. Apneustic breathing is characterized by: A) an irregular respiratory pattern with unpredictable periods of apnea. B) a gradual increase and decrease in breathing with periods of apnea. C) extreme tachypnea and hyperpnea with a fruity or acetone breath odor. D) a prolonged inspiratory phase with a shortened expiratory phase and bradypnea.

Ans: D Page: 1045

27. Decerebrate posturing: A) is characterized by abnormal flexion of the arms and extension of the lower extremities. B) is considered less severe than decorticate posturing, because it indicates that the brainstem is intact. C) is an early clinical finding, because it occurs with minimal increases in intracranial pressure. D) is a more severe finding than decorticate posturing, as it indicates damage in or near the brainstem.

Ans: D Page: 1050

33. An adult who opens her eyes in response to verbal stimuli, is disoriented, and pushes your hand away when you palpate a painful area has a Glasgow Coma Scale score of: A) 9. B) 10. C) 11. D) 12.

Ans: D Page: 1051

15. Common reality is defined as: A) one person's perception of his or her surroundings. B) a perceived thought that is not based on reality. C) minimal shared reality between two individuals. D) sensory stimulation that can be confirmed by others.

Ans: D Page: 1051-1052

38. A patient who is unable to use a common object is said to be: A) agnosic. B) aphasic. C) atonic. D) apraxic.

Ans: D Page: 1053

42. A staggering gait is MOST suggestive of damage to the: A) medulla. B) cerebrum. C) brainstem. D) cerebellum.

Ans: D Page: 1055

46. Tremors that increase as the patient's hand gets closer to an object that he or she is trying to grab are called _________ tremors and are MOST common in patients with _________. A) rest, Parkinson disease B) postural, Alzheimer disease C) clonic, cerebral palsy D) intention, multiple sclerosis

Ans: D Page: 1055, 1073

19. The MOST immediate and significant complication associated with a hemorrhagic stroke is: A) acute hypovolemic shock. B) hypertension and bradycardia. C) mean arterial pressure increase. D) increased intracranial pressure.

Ans: D Page: 1060

22. Which of the following would MOST likely cause a sustained increase in intracranial pressure? A) Bearing down B) Frequent coughing C) Projectile vomiting D) Intracranial tumor

Ans: D Page: 1060

75. A high level of oxygen to the brain of a patient with a hemorrhagic stroke and increased intracranial pressure: A) dilates the blood vessels and can cause brain herniation. B) lowers intracranial pressure and oxygenates the brain. C) increases intracranial pressure, but oxygenates the brain. D) causes vasoconstriction and can impair brain perfusion.

Ans: D Page: 1061

55. A 59-year-old woman presents with acute onset of confusion, left-sided hemiparesis, and a right-sided facial droop. Her airway is patent and she is breathing adequately. Her blood pressure is 150/100 mm Hg and her pulse is 70 beats/min. The cardiac monitor displays atrial fibrillation with a variable rate of 60 to 90 beats/min. When obtaining the patient's medical history from her husband, the MOST important question to ask him is: A) "Does your wife have a history of diabetes?" B) "Is your wife allergic to aspirin or contrast dye?" C) "When did your wife last see her physician?" D) "When did you first notice your wife's symptoms?"

Ans: D Page: 1064

61. A woman brings her 18-year-old son to your EMS station. The patient is actively seizing and, according to the mother, has been seizing for the past 10 minutes. She states that her son has a history of seizures and takes Depakote. The patient is cyanotic, is breathing erratically, and has generalized muscle twitching to all extremities. You should: A) place a bite block between his molars to prevent oral trauma, administer high-flow oxygen, and give lorazepam IM. B) protect him from further injury by restraining him, attempt orotracheal intubation, establish an IV, and give Valium. C) hyperventilate him with a bag-mask to eliminate excess carbon dioxide, establish vascular access, and give lorazepam. D) open his airway and begin assisting his ventilations, establish an IV or IO line, and administer 5 to 10 mg of diazepam.

Ans: D Page: 1069

71. When a patient with Parkinson disease is asked to turn, he or she takes small steps until the turn is complete. This is called: A) ataxia. B) decussation. C) dystonia. D) bradykinesia.

Ans: D Page: 1075

80. Spasmodic torticollis is characterized by: A) sustained and painful contraction of the eyelids. B) involuntary unilateral myoclonic contractions of the face. C) severe stabbing pain, usually to one side of the face. D) neck muscle contraction, which twists the head to one side.

Ans: D Page: 1076-1077

67. A patient with suspected meningitis involuntarily flexes her knees when her head is flexed toward her chest. This is called: A) Kernig sign. B) Grey Turner sign. C) Cullen sign. D) Brudzinski sign.

Ans: D Page: 1078

10. Most of the digestive process occurs in the: A) liver. B) pancreas. C) jejunum. D) small intestine.

Ans: D Page: 1127

6. The pancreas, liver, and gallbladder connect to the digestive system at the: A) ilium. B) secum. C) jejunum. D) duodenum.

Ans: D Page: 1127

15. When forming your general impression of a patient with gastrointestinal distress, which of the following observations would provide the MOST information regarding what happened? A) Resting blood pressure B) Skin condition and temperature C) The patient's facial expression D) Body posture or position

Ans: D Page: 1130

16. Patients with an extremely advanced bowel obstruction may have feculent breath, which is the odor of: A) blood. B) chyme. C) acetone. D) stool.

Ans: D Page: 1130

44. Blood that has been digested by stomach acids manifests as: A) hematochezia. B) bright red vomitus. C) gray-colored stool. D) coffee-ground emesis.

Ans: D Page: 1131

37. When auscultating bowel sounds, borborygmi is characterized by: A) absent sounds after 2 minutes of auscultation. B) quiet sounds, occurring at less than 1/sec. C) soft gurgles or clicks occurring at 5-30/min. D) loud gurgles occurring greater than 30/min.

Ans: D Page: 1133

33. Which of the following assessment findings or techniques would prove MOST useful when determining the extent of internal volume loss? A) Resting blood pressure B) Temperature of the skin C) Duration of the illness D) Orthostatic vital signs

Ans: D Page: 1135

50. A 52-year-old man complains of vomiting and diarrhea for the past day. He is conscious and alert, has a blood pressure of 130/70 mm Hg, has a heart rate of 120 beats/min and strong, and has respirations of 20 breaths/min and regular. His past medical history is significant for anemia and congestive heart failure. Prior to administering isotonic crystalloid solutions to this patient, it is MOST important to: A) check his blood sugar. B) apply a pulse oximeter. C) obtain a 12-lead ECG. D) auscultate his lung sounds.

Ans: D Page: 1135, 1137

48. A 52-year-old man complains of severe abdominal pain but denies nausea or vomiting. He is conscious and alert, has a blood pressure of 130/70 mm Hg, a heart rate of 120 beats/min and strong, and respirations of 20 breaths/min and regular. In addition to providing supplemental oxygen, you should start an IV line and give: A) ondansetron, 4 mg. B) a 20-mL/kg fluid bolus. C) diphenhydramine, 25 mg. D) nalbuphine, 10 mg.

Ans: D Page: 1135-1137

17. Hypotension during an episode of dehydration indicates that the body is: A) decompensating due to an excessive loss of blood as well as water and key electrolytes. B) shifting water from inside the cells to the interstitial space in an attempt to maintain perfusion. C) compensating by releasing epinephrine and norepinephrine from the sympathetic nervous system. D) no longer capable of effectively pulling fluid from the interstitial space and the cellular area.

Ans: D Page: 1138

18. When diarrhea contains more water than it does fecal material: A) the body loses an equal amount of electrolytes and water. B) the concentration of electrolytes that are lost increases significantly. C) the body's key electrolytes are spared and are not passed in the diarrhea. D) smaller amounts of electrolytes are lost compared to the amount of lost water.

Ans: D Page: 1138

24. Chronic use of nonsteroidal anti-inflammatory drugs would MOST likely result in: A) diverticulitis. B) esophageal varices. C) acute gastroenteritis. D) peptic ulcer disease.

Ans: D Page: 1140

42. A patient with pancreatitis would MOST likely present with pain that: A) radiates around to the right side of the back and angle of the scapula. B) is severe and radiates from the flank to the groin and external genitalia. C) is localized to the left upper quadrant and referred pain to the left shoulder. D) goes straight through to the back in the midline of the lower thoracic area.

Ans: D Page: 1144

28. Which of the following statements regarding Grey Turner sign is correct? A) Grey Turner sign is characterized by bruising around the umbilicus. B) The presence of Grey Turner sign should make you suspicious for hepatitis. C) Grey Turner sign is the cessation of inspiration during abdominal palpation. D) Grey Turner sign is characterized by flank bruising and indicates retroperitoneal hemorrhage.

Ans: D Page: 1145

4. Most of the blood's formed elements are: A) platelets. B) leukocytes. C) electrolytes. D) erythrocytes.

Ans: D Page: 1245

11. The primary site for cell production in the human body is the: A) kidney. B) spleen. C) liver. D) bone marrow.

Ans: D Page: 1246

17. Anemia resulting from an autoimmune disorder occurs when: A) the body's red blood cells destroy certain white blood cells. B) a patient receives blood of a type different than his or her own. C) hemoglobin becomes desaturated due to a massive infection. D) red blood cells are destroyed by the body's own antibodies.

Ans: D Page: 1253

21. A patient who presents with petechiae is MOST likely: A) anemic. B) leukopenic. C) polycythemic. D) thrombocytopenic.

Ans: D Page: 1253

32. You are transporting a 55-year-old woman to the hospital. The patient has lymphoma and complains of generalized, severe pain. She is receiving supplemental oxygen and has an IV line of normal saline established. Her blood pressure is 110/64 mm Hg, pulse rate is 104 beats/min and regular, and respirations are 22 breaths/min and adequate. You should: A) ensure that she is in a comfortable position and administer a 250-mL saline bolus. B) provide emotional support, but refrain from analgesia as this may cause hypotension. C) give her 2 to 5 mg of a benzodiazepine sedative and provide constant reassurance. D) administer narcotic analgesia as needed while closely monitoring her blood pressure.

Ans: D Page: 1255-1256

27. Patients with type A hemophilia: A) have a low platelet count. B) require infusions of factor IX. C) bleed due to thrombocytopenia. D) have a deficiency of factor VIII.

Ans: D Page: 1257

The three MOST significant indicators of anaphylactic shock are: A) hives, chest tightness, and restlessness. B) dyspnea, hypotension, and tachycardia. C) pruritus, peripheral swelling, and urticaria. D) dizziness, flushed skin, and abdominal pain.

B

. Displacement of bony fragments into the anterior portion of the spinal cord results in: A) central cord syndrome. B) anterior cord syndrome. C) Brown-Séquard syndrome. D) complete spinal cord injury.

B

. The brain connects to the spinal cord through a large opening at the base of the skull called the: A) cribriform plate. B) foramen magnum. C) occipital condyle. D) palatine bone.

B

. The cerebellum is located in the ____________ part of the brain and is responsible for _______________. A) anteromedial, voluntary motor functions B) inferoposterior, posture and equilibrium C) anterolateral, short- and long-term memory D) inferolateral, involuntary motor functions

B

10. You would NOT expect a person using methamphetamine to present with: A) insomnia. B) bradypnea. C) restlessness. D) hypertension.

B

100. If the mechanism of injury indicates that your patient may have sustained a spinal cord injury: A) contact medical control to determine if spinal immobilization is needed. B) assume that a spine injury exists, regardless of the neurologic findings. C) apply a cervical collar and transport the patient in a position of comfort. D) fully immobilize the spine only if gross neurologic deficits are present

B

101. A 45-year-old man is found unresponsive in an alley. During your assessment, you note that he is tachycardic and breathing rapidly. He has an obvious odor of alcohol on his breath. Your MOST immediate concern should be to: A) obtain a blood glucose reading. B) take actions to prevent aspiration. C) determine the etiology of his tachycardia. D) start an IV line and administer naloxone.

B

105. You are transporting a patient who is under the influence of methamphetamine. The patient, who is clearly anxious, has a blood pressure of 160/90 mm Hg, a pulse rate of 140 beats/min, and a respiratory rate of 24 breaths/min. The patient suddenly becomes violent and begins thrashing around, trying to get off the stretcher. After asking your partner to stop the ambulance to assist you with the patient, you should: A) assess his blood glucose level. B) administer IM haloperidol. C) start an IV line and give him morphine. D) administer a beta blocker and reassess.

B

110. An injured patient's head should be secured to the long backboard only after: A) you have placed padding under the shoulders. B) his or her torso has been secured adequately. C) both of the legs are secured to the board properly. D) a vest-style immobilization device has been applied.

B

110. You have administered a total of 10 mg of Narcan to an unresponsive 30-year-old man whom you believe has overdosed on a narcotic. However, the patient remains unresponsive, is hypoventilating, and is bradycardic. Your transport time to the closest appropriate hospital is 40 minutes. You should: A) insert a nasogastric tube to decompress his stomach, administer another 2 mg of Narcan, and transport. B) continue assisted ventilation for 2 to 3 minutes, insert an advanced airway device, and transport immediately. C) insert a laryngeal mask airway, transport at once, and begin an epinephrine infusion en route to the hospital. D) insert an oropharyngeal airway, continue bag-mask ventilations at a rate of 20 breaths/min, and transport.

B

114. A 22-year-old man experienced an acid chemical burn to his left forearm. He complains of intense pain and tingling in his fingers. He is conscious and alert, and denies any other symptoms. You should: A) cover the burn and transport at once. B) begin immediate irrigation with water. C) apply a light coat of baking soda to the burn. D) administer oxygen via nonrebreathing mask.

B

115. If a trauma patient cannot be assessed properly in his or her vehicle, you should: A) apply a rigid cervical collar, perform a rapid assessment only, and then remove the patient with the two-person lift technique. B) maintain manual stabilization of the head, apply a cervical collar, and move the patient from the vehicle onto a long backboard. C) apply a vest-type extrication device and then rapidly remove him or her from the vehicle using at least three people. D) grasp the patient behind the shoulders, cradle his or her head in your arms to protect the spine, and rapidly extricate him or her from the car.

B

116. A known alcoholic man is found unresponsive by a law enforcement officer. An empty container of antifreeze was found near him. Your assessment reveals that his respirations are deep and rapid, his pulse rate is rapid and weak, and his pupils are dilated and sluggishly reactive. As your partner administers high-flow oxygen to the patient, you should: A) start an IV line and give 1 mEq/mg of sodium bicarbonate. B) assess his blood glucose level and apply a cardiac monitor. C) start an IV line and begin administering a saline fluid bolus. D) give him 100 mg of thiamine IM and assess his blood pressure.

B

12. Any sympathomimetic drug will cause: A) ataxia. B) tachycardia. C) hallucinations. D) hypothermia.

B

13. Which of the following drugs is classified as an anticholinergic? A) Diazinon B) Atropine C) Thiopental D) Phenylephrine

B

15. The respiratory centers of the brainstem lie within the: A) basal ganglia. B) pons and medulla. C) diencephalon. D) midbrain.

B

18. Drug abuse is MOST accurately defined as: A) the habitual use of illicit drugs for the purpose of inducing a euphoric feeling. B) any use of drugs that causes physical, psychological, or legal harm to the user. C) the use of legal medications that is not in accordance with a physician's order. D) inadvertent misuse of a licit or illicit drug that causes physical harm to the user.

B

20. Most head injuries are the result of: A) assault or abuse. B) motor vehicle crashes. C) sports-related incidents. D) falls from a significant height.

B

21. A person who compulsively uses a drug, despite the fact that he or she knows the drug will cause physical or psychological harm, is: A) tolerant. B) addicted. C) dependent. D) an abuser.

B

25. Patients with alcoholism are prone to subdural hematomas and gastrointestinal bleeding because: A) they fall more frequently than sober people. B) their blood-clotting mechanisms are impaired. C) they are at higher risk for violent assault. D) alcohol causes significant immunocompromise.

B

26 Which of the following signs of a basilar skull fracture would MOST likely be observed in the prehospital setting? A) Battle's sign B) cerebrospinal fluid drainage from the ear C) Ecchymosis around the eyes D) Bruising over the mastoid process

B

27. Open fractures of the cranial vault: A) cause death due to increased intracranial pressure. B) are associated with a high risk of bacterial meningitis. C) typically cause lethal atrial cardiac dysrhythmias. D) are uncommonly associated with multisystem trauma.

B

27. Which of the following is the MOST immediate danger to an unresponsive patient with acute alcohol intoxication? A) Acute hypovolemia B) Aspiration of vomitus C) Profound bradycardia D) Ventricular dysrhythmias

B

32. Which of the following questions often yields the LEAST reliable answer when questioning a patient who intentionally exposed himself or herself to a toxic substance? A) Have you vomited? B) Why did you take the substance? C) When did you take the substance? D) How much of the substance did you take?

B

36. Which of the following statements regarding a cerebral concussion is correct? A) A cerebral concussion is a moderate focal brain injury. B) Concussions are usually not associated with structural brain injury. C) Prolonged loss of consciousness is common with a cerebral concussion. D) A concussion results in permanent dysfunction of the cerebral cortex.

B

37. A person who is "speedballing" is: A) highly addicted to methamphetamine, cocaine, and marijuana and mixes all three drugs together to achieve various levels of euphoria. B) using cocaine in combination with heroin, by injecting them either underneath the skin or directly into a vein, in order to regulate the high. C) packaging cocaine in small plastic bags and swallowing them for the purpose of transporting the cocaine from one location to another location. D) using heroin to withdraw or detoxify himself or herself from cocaine by gradually increasing the amounts of heroin taken while decreasing the amounts of cocaine used.

B

40. Appropriate prehospital treatment for a patient who has overdosed on a stimulant and is excessively tachycardic and violent includes all of the following, EXCEPT: A) IM haloperidol. B) beta-adrenergic antagonists. C) benzodiazepines if seizures occur. D) fluid boluses if hypotension occurs.

B

42. LSD is classified as a: A) psychedelic. B) hallucinogen. C) sympatholytic. D) sedative/hypnotic.

B

47. Which of the following is a sign of severe barbiturate withdrawal? A) Diaphoresis B) Hallucinations C) Nausea and vomiting D) Abdominal cramping

B

48. If you are treating a patient with a suspected benzodiazepine overdose and find that the patient is hypotensive, bradycardic, and comatose: A) avoid administering flumazenil and transport the patient immediately. B) you should consider concomitant overdose with another CNS depressant. C) it is likely that the patient is also under the influence of methamphetamine. D) you should rapidly administer 2 mg of naloxone via the IV, IO, or IM route.

B

5. When poisoning occurs because of a toxic environment: A) the patient typically does not present with symptoms for hours. B) you are more likely to encounter more than one patient at the scene. C) you should limit your exposure to the environment to less than 5 minutes. D) exposure continues, even after the patient is removed from the environment.

B

50. A male patient with a closed head injury opens his eyes in response to pain, makes incomprehensible sounds, and responds to pain with flexion of his arms. His Glasgow Coma Scale score is ___, and the MOST appropriate treatment for him involves: A) 6; intubation, hyperventilation at a rate of 20 breaths/min, two large-bore IV lines running wide open, keeping him warm, and maintaining his oxygen saturation at greater than 90%. B) 7; intubation, ventilations performed at a rate of 10 breaths/min, IV fluids as needed to maintain a systolic blood pressure of at least 90 mm Hg, and maintaining his oxygen saturation at greater than 95%. C) 8; ventilation assistance with a bag-mask device, a 2-L bolus of normal saline or lactated Ringer's solution, elevating his head 12 inches, and hyperventilating him if his heart rate falls below 60 beats/min. D) 9; high-flow oxygen via nonrebreathing mask, IV fluids as needed to maintain a systolic blood pressure of at least 100 mm Hg, monitoring his oxygen saturation, and administering 1.5 mg/kg of lidocaine to decrease intracranial pressure.

B

53. Which of the following cardiac rhythm disturbances MOST commonly results from inadvertent overdose of a prescribed cardiac medication? A) Atrial flutter B) Bradycardia C) Tachycardia D) Atrial fibrillation

B

55. The anterior weight-bearing structure of the vertebra is the: A) pedicle. B) vertebral body. C) lamina. D) spinous process

B

57. The toxicity of carbon monoxide arises primarily from: A) its destructive properties on ferric ions. B) its affinity for hemoglobin in red blood cells. C) the fact that carbon monoxide destroys hemoglobin molecules. D) its ability to markedly decrease the metabolic rate.

B

59. Although extremely painful, injuries to the ________ spine are typically neurologically insignificant. A) lumbar B) coccygeal C) thoracic D) cervical

B

59. The LEAST common sign or symptom of carbon monoxide toxicity is: A) nausea and vomiting. B) cherry red skin color. C) pallor or cyanosis. D) roaring sensation in the ears.

B

6. From an anatomic and physiologic perspective, inhaled toxins: A) generally provide a large window of opportunity for treatment. B) quickly reach the alveoli and rapidly gain access to the circulatory system. C) typically take between 15 and 20 minutes to exert a systemic effect. D) often take several hours before clinical signs and symptoms manifest.

B

64. Upon arriving at the scene of an incident involving a chlorine gas spill, you should: A) begin triaging all patients. B) park the ambulance upwind. C) remove all patients from the scene. D) don a protective breathing apparatus.

B

65. The __________ is a continuation of the central nervous system and exits the skull through the _______. A) vagus nerve, spinal cord B) spinal cord, foramen magnum C) brainstem, vertebral foramen D) medulla, cauda equina

B

69. If administered in conjunction with nitrates, sildenafil would MOST likely cause: A) hypertensive crisis. B) severe hypotension. C) coronary vasospasm. D) ventricular tachycardia.

B

76. Tricyclic antidepressant medications: A) are the first-line therapy for the treatment of depression. B) may produce toxic effects with even minimal dosing errors. C) generally require high doses to achieve a therapeutic effect. D) are very safe because they have a large therapeutic window.

B

79. The MOST common cause of death following a tricyclic antidepressant overdose is: A) respiratory failure. B) a cardiac dysrhythmia. C) intracerebral hemorrhage. D) pulmonary aspiration.

B

80. A major issue for people taking MAOIs is that: A) they are associated with a high rate of suicidal behavior. B) they have a high potential for lethal drug interactions. C) high doses are required to achieve a therapeutic effect. D) they must be used in conjunction with other antidepressants.

B

85. Inadvertent lithium toxicity would MOST likely occur in a patient who is taking: A) a tricyclic antidepressant. B) NSAIDs. C) SSRIs. D) any medication used to control blood pressure.

B

85. Spinal shock is a condition that: A) generally affects the sensory nerves but spares the motor nerves. B) is usually temporary and results from swelling of the spinal cord. C) typically manifests within 24 to 36 hours following a spinal injury. D) results in permanent neurologic deficits in the majority of patients.

B

86. Severe salicylate toxicity produces: A) bradypnea. B) metabolic acidosis. C) increased pH levels. D) respiratory acidosis.

B

90. Modification of your physical examination of a patient with a suspected spinal cord injury following a two-car motor vehicle crash is based on all of the following factors, EXCEPT: A) the mechanism of injury. B) injuries to patients in the other vehicle. C) the patient's level of consciousness. D) reliability of the patient as a historian

B

92. Which of the following is NOT a common sign of lead poisoning? A) Irritability B) Hypotension C) Hypertension D) Constipation

B

96. Prior to immobilizing an anxious patient with a suspected spinal injury on a backboard, it is important to: A) ask the patient to pull against your hands with his or her feet. B) make note of any neurologic deficits or gross injuries up to that point. C) administer the appropriate dose of a sedative to facilitate patient compliance. D) apply a cervical collar to free up the medic that is holding the patient's head.

B

97. The foxglove plant contains ______________ and can result in ______________ when it is ingested. A) lantadene A, renal failure B) cardiac glycosides, dysrhythmias C) solanine, severe gastroenteritis D) caladium oxalate crystals, bradycardia

B

A "code" runs MOST efficiently when: A) the AED replaces manual defibrillation. B) a designated team leader is assigned. C) tracheal intubation is performed early. D) at least five paramedics are participating

B

A 21-year-old woman was thrown from a horse and landed on her head. Upon arrival at the scene, you find the patient lying supine. She is conscious and has her head turned to the side. As you attempt to move her head to a neutral in-line position, she screams in pain. You should: A) gently continue to move her head until it is in a neutral in-line position. B) maintain her head in the position found and continue with your assessment. C) perform a quick neurologic exam prior to attempting further movement of her head. D) administer an analgesic to relieve the pain so you can move her head to a neutral position.

B

A 49-year-old woman presents with a severe headache, a temperature of 103.2°F, and photosensitivity. Her blood pressure is 140/76 mm Hg, pulse rate is 120 beats/min and strong, and respiratory rate is 22 breaths/min and regular. While caring for this patient, it is MOST important to: A) apply a cardiac monitor and assess her tachycardia. B) protect yourself from any nasopharyngeal secretions. C) treat her as though she is experiencing viral meningitis. D) attempt to assist her ventilations with a bag-mask device.

B

A 71-year old man with a history of emphysema, coronary artery disease, and hypertension presents with increased shortness of breath and fatigue. His oxygen saturation is 80% on home oxygen at 2 L/min via nasal cannula. His blood pressure is 140/76 mm Hg, pulse rate is 104 beats/min, and respirations are 28 breaths/min and labored. His medications include an albuterol inhaler, prednisone, hydrochlorothiazide, and lisinopril. Auscultation of his lungs reveals scattered wheezing in all fields. Which of the following prehospital interventions is NOT indicated for this patient? a. Nebulized ipratropium b. Subcutaneous epinephrine c. High-flow oxygen therapy d. Continuous positive airway pressure

B

A common sign of the measles is: A) petechiae. B) a blotchy red rash. C) severe diarrhea. D) a purpuric rash.

B

A construction worker was hammering when he experienced a sudden severe pain to his right eye. Assessment of his eye reveals that it is irritated and you can see a small sliver of metal imbedded in the globe. He is also wearing contact lenses. You should: A) remove his contact lenses and irrigate his eye with copious amounts of water. B) leave his contact lenses in place and cover both eyes with a sterile dressing. C) attempt to remove the foreign object only if he has any visual disturbances. D) remove his contact lenses and administer a narcotic analgesic for pain.

B

A key element of the "G" in the GEMS diamond includes: a. checking the physical condition of the patient's home. b. recalling that elderly patients often present atypically. c. assessing the elderly patient's activities of daily living. d. obtaining a complete medical history.

B

A paramedic would MOST likely be infected with TB if he or she: A) was close to a coughing patient who had a positive TB skin test. B) performed mouth-to-mouth on a patient with active untreated TB. C) was exposed to blood-stained vomitus of a patient with active TB. D) received a needlestick from a person suspected of having active TB.

B

A patient remains in pulseless ventricular tachycardia despite two shocks, a dose of epinephrine, high-quality CPR, and 300 mg of amiodarone. Following the next shock, you should resume CPR and then: A) cardiovert with 100 biphasic joules. B) administer 150 mg of amiodarone. C) double the dose of the epinephrine. D) administer 1.5 mg/kg of lidocaine.

B

A psychotic person may have thought insertions, which are defined as: A) a gross distortion of your comments into what he or she believes to be true. B) the belief that thoughts are being thrust into his or her mind by another person. C) the fear that his or her thoughts are being broadcast aloud and heard by others. D) strange or pressured speech because of unusual words the patient has invented.

B

A small percentage of clinically depressed patients: A) complain of persistent anorexia. B) may report an increased appetite. C) experience guilt and self-reproach. D) have pervasive thoughts of suicide.

B

A subdural hematoma is classified as acute if clinical signs and symptoms develop: A) immediately following the injury. B) within 24 hours following the injury. C) within 36 hours following the injury. D) within 48 hours following the injury

B

A young female presents with a headache, severe aching around her right ear, and difficulty chewing. Assessment and treatment for her should focus on: A) anticipating airway compromise. B) analgesia as needed and transport. C) placing her in a semi-Fowler position. D) avoiding the use of a nasopharyngeal airway.

B

A) is a specific, grossly observable brain injury that can easily be diagnosed with a computer tomography scan of the head. B) involves stretching, shearing, or tearing of the extension of the neuron that conducts electrical impulses away from the cell body. C) results in severe stretching or tearing of the portion of the nerve cell that receives sensory messages from the rest of the body. D) is generally associated with better neurologic outcomes than a cerebral concussion because permanent brain damage does not occur.

B

After an employee who believes he or she has been exposed to an infectious disease notifies the designated infection control officer (DICO), the DICO should: A) execute the postexposure plan within 24 to 36 hours. B) determine whether an actual exposure occurred. C) immediately refer the employee to a designated physician. D) obtain the patient's consent to have his or her blood drawn.

B

After approximately 6 minutes of attempted resuscitation, your patient experiences a return of spontaneous circulation. He remains unresponsive and apneic, and the cardiac monitor reveals sinus tachycardia. In addition to continuing ventilations, what should be done next? A) Amiodarone infusion B) Hypothermia treatment C) Dopamine infusion D) Crystalloid fluid bolus

B

After determining that an unresponsive adult patient is not breathing, you should: A) give two rescue breaths that make the chest visibly rise. B) assess for a carotid pulse for no longer than 10 seconds. C) reposition the patient's airway and reassess for breathing. D) perform a finger sweep of the patient's mouth to remove any debris.

B

After obtaining an elderly patient's chief complaint, gathering additional information about the history of present illness would MOST likely be complicated because: a. the patient may ask you to repeat your questions. b. chronic problems may affect the acute problem. c. most elderly patients take numerous medications. d. the aging process causes difficulty in understanding.

B

Aging is a linear process, which means that A. the older get the faster our bodies decline in function b. the rate at which we lose functions does not increase with age c. older people exhibit the cumulative results of a longer aging process d. older adults experience decreased functions faster than younger adults

B

Almost all disoriented behavior: A) has an underlying physical etiology that may be life threatening. B) represents a person's effort to adapt to internal or external stress. C) can be identified and treated definitively in the prehospital setting. D) is the result of an acute behavioral crisis that requires intervention.

B

An antigen is MOST accurately defined as a: A) chemical the immune system produces to destroy an allergen. B) substance that causes the immune system to produce antibodies. C) chemical mediator that deactivates foreign substances in the body. D) harmless substance that the body does not recognize as being foreign.

B

An unresponsive, apneic, and pulseless man presents with a regular rhythm on the cardiac monitor. In addition to information regarding the events that led to his arrest, which of the following assessment findings would cause you to suspect that cardiac tamponade is the underlying cause of his condition? A) Bilaterally absent breath sounds and severe pallor B) No pulse with CPR and jugular venous distention C) Unilaterally absent breath sounds and mottled skin D) Profoundly cyanotic skin and collapsed jugular veins

B

Autoregulation is defined as: A) reflex bradycardia that occurs secondary to systemic hypertension. B) an increase in mean arterial pressure to maintain cerebral blood flow. C) the forcing of cerebrospinal fluid into the spinal cord as intracranial pressure increases. D) a decrease in cerebral perfusion pressure that reduces intracranial pressure.

B

Cardiac monitoring is recommended when caring for a patient with an eye-related emergency because: A) eye drops or medication can cause a marked elevation in the heart rate. B) ocular pressure can stimulate the vagus nerve and cause bradycardia. C) vitreous humor loss can result in a variety of ventricular dysrhythmias. D) AV heart blocks commonly occur when vitreous humor is lost.

B

Cataracts are caused by clouding of the: A) iris. B) cornea. C) retina. D) sclera.

B

Cervical spondylosis is a degenerative change in the cervical spine that causes a. destruction of the intervertebral discs and vertebral fractures b. narrowing of the spinal canal and pressure on the spinal cord c. lateral curvature of the cervical spine with cord impingement d. fracture of the odontoid process of the 2nd cervical vertebra

B

Chronic subdural hematomas are MOST commonly seen in patients who: A) are less than 2 years of age. B) have alcoholism. C) are prone to hypoglycemia. D) have high cholesterol.

B

Common causes of a ruptured tympanic membrane include all of the following, EXCEPT: A) diving injuries. B) otitis externa. C) foreign bodies. D) blast injuries.

B

Common signs and symptoms of TB include all of the following, EXCEPT: A) hemoptysis. B) photophobia. C) weight loss. D) a persistent cough

B

Death following a head injury is MOST often the result of: A) an epidural hematoma. B) trauma to the brain. C) airway compromise. D) spinal cord transection.

B

Diphenhydramine (Benadryl) is used to treat allergic reactions because it: A) binds to H2 receptors and blocks histamine release. B) blocks the histamine effects at the H1 receptor sites. C) destroys histamines and blocks their further release. D) reverses the vasodilatory and bronchoconstrictive effects.

B

Disorders of thinking include all of the following, EXCEPT: A) phobias. B) anxiety. C) delusions. D) obsessions.

B

Early clinical manifestations of an allergic reaction include all of the following, EXCEPT: A) pruritus. B) stridor. C) urticaria. D) edema.

B

Flexion injuries to the spine would MOST likely result from: A) rapid acceleration forces. B) rapid deceleration forces. C) a rear-end motor vehicle crash. D) a direct blow to the frontal lobe.

B

Following a spinal injury, a patient presents with abdominal breathing and use of the accessory muscles in the neck. This suggests injury at or above: A) C1-C2. B) C3-C4. C) T1-T4. D) T2-T5.

B

Full recoil of the chest in between compressions enhances blood return to the heart by which of the following mechanisms? A) Increased residual lung volume B) Negative intrathoracic pressure C) Decreased myocardial preload D) Coronary artery vasoconstriction

B

Generalized anxiety disorder is characterized by: A) symptoms for at least 1 month. B) persistent and unproductive worrying. C) feelings of grandeur and inattentiveness. D) unreasonable fear of a particular situation.

B

Headaches, nausea and vomiting, narrowing vision fields, and a "graying" in the field of vision are consistent with: A) glaucoma. B) papilledema. C) corneal abrasion. D) anterior uveitis.

B

Heat gain or loss in response to environment changes is delayed in elderly people for all of the following reasons, EXCEPT a. impaired circulation b. chronic hyperthyroidism c. atherosclerotic vessels d. decreased sweat production

B

Hyphema is defined as: A) severe ecchymosis to the orbital region. B) blood in the anterior chamber of the eye. C) marked swelling of the globe of the eye. D) double vision following blunt eye trauma.

B

If a paramedic is exposed to lice: A) he or she should immediately report to the designated infection control officer for prophylactic antiviral treatment. B) permethrin cream treatment may be prescribed and restrictions from patient care may be indicated until the paramedic is free of lice. C) infection is unlikely because paramedics have strong immune systems due to exposure to patients with various diseases. D) a specially made shampoo that contains a combination of a steroid and an antibiotic should be used within 6 hours after exposure.

B

If a patient with a behavioral emergency misperceives reality, you should: A) disagree and correct the patient's perception. B) avoid arguing about the patient's misperception. C) conclude that he or she is suffering from schizophrenia. D) agree with the patient to facilitate his or her cooperation.

B

If the source patient's blood tests positive for HIV: A) the exposed individual will receive a one-time injection of immune globulin and will be tested for HIV in 2 weeks. B) the blood will be assessed for viral load and the exposed individual may be offered a 4-week trial of antiretroviral therapy. C) the most rapid method for determining if the exposed individual was infected is by assessing his or her lymphocyte count. D) federal law requires that the exposed individual be placed on antiretroviral therapy and not be allowed to work in a health care setting.

B

If you are exposed to the hepatitis B virus and have a positive titer on file: A) the source patient will be retested. B) no follow-up treatment is required. C) the vaccination series should be repeated. D) you will be offered HBV immune globulin.

B

If you do not receive a flu vaccine and are exposed to a person with the flu: A) symptoms of the flu, if you contract it, will become evident within 12 to 24 hours after the exposure. B) antiviral drugs may be given within 48 hours after the exposure to reduce the severity of the flu if you contract it. C) an injection of immune globulin given within 12 hours after the exposure will prevent you from contracting the flu. D) a nasal spray that contains the live flu virus in an attenuated form will be administered to you within 48 hours.

B

Impedance threshold devices, such as the ResQPOD, function by: A) dilating the coronary arteries, which reduces cardiac afterload. B) creating a vacuum in the chest, which increases blood flow to the heart. C) facilitating vasoconstriction, which maintains coronary perfusion pressure. D) increasing intrathoracic pressure, which reduces blood return to the heart.

B

In addition to children, the hepatitis A vaccine is recommended for: A) any health care worker who functions in an actual patient care setting. B) emergency response team members traveling outside the United States. C) all Federal Emergency Management Agency response team members. D) all health care workers when an outbreak of hepatitis A is documented.

B

In contrast to dementia, delirium: A) generally results from conditions such as Alzheimer disease. B) is characterized by acute global impairment of cognitive function. C) is an irreversible process caused by premature cerebral deterioration. D) is a chronic process that causes deficits in memory and abstract thinking.

B

In contrast to secondary spinal cord injury, primary spinal cord injury occurs: A) from progressive swelling. B) at the moment of impact. C) from penetrating mechanisms. D) within 24 hours of the injury.

B

In hypochondriasis, patients have significant anxiety or fear that they: A) will attempt to hurt others. B) may have a serious disease. C) are being undermedicated. D) will go to sleep and not awake.

B

In males, infection with Chlamydia trachomatis would MOST likely lead to: A) hepatitis. B) prostatitis. C) bladder cancer. D) testicular torsion.

B

In older adults, shingles arises when the _________ virus resides in the ganglion of a nerve. A) rubella B) varicella C) rubeola D) herpes

B

In the context of a communicable disease, a ___________ is a place where organisms may live and multiply. A) host B) reservoir C) carrier D) contaminant

B

In which of the following situations is the Morgan lens NOT appropriate to use? A) Eye burn from a strong alkali B) An object impaled in the eye C) Eye burn from a strong acid D) Any foreign body in the eye

B

Klonopin and BuSpar: A) are both benzodiazepines. B) are anxiolytic medications. C) are used to control seizures. D) block the reuptake of serotonin.

B

Manic patients may report that their thoughts are racing and they may rapidly skip from one topic to another. This is called: A) confabulation. B) tangential thinking. C) circumstantial thinking. D) thought broadcasting.

B

Most age-related visual disturbances are the result of: a. diabetic retinopathy b. cataracts or glaucoma c. macular degeneration d. retinal artery occlusion

B

Most patients infected with the hepatitis C virus are unaware that they acquired the infection because: A) a blood test to detect the virus does not exist. B) they do not develop phase 2 signs and symptoms. C) the incubation period ranges from 15 to 20 years. D) hepatitis C does not produce any signs or symptoms.

B

Movement of both of the eyes in unison is called: A) dysconjugate gaze. B) sympathetic eye movement. C) extraocular movement. D) physiologic anisocoria.

B

Patients with systemic lupus erythematosus: A) are routinely treated with high-dose antibiotic therapy. B) often take medications that suppress their immune system. C) have increases in their red blood cell and platelet counts. D) are typically males between the ages of 50 and 75 years.

B

Physiologic effects of histamine include all of the following, EXCEPT: A) systemic vasodilation. B) increased cardiac contractility. C) severe bronchoconstriction. D) increased vascular permeability.

B

Polypharmacy is most accurately defined as: A. a harmful interaction when several drugs are taken together b. the prescribing of multiple drugs to treat multiple conditions c. unnecessarily prescribing numerous drugs to prevent a disease d. the unintentional ingestion of multiple doses of the same doses

B

Postexposure prophylaxis is available for all of the following diseases, EXCEPT: A) HIV. B) hepatitis C. C) syphilis. D) gonorrhea.

B

Repetitive actions that are carried out to relieve the anxiety of obsessive thoughts are called: A) perseverations. B) compulsions. C) stereotyped movements. D) confabulations.

B

Signs and symptoms of anticholinergic medication toxicity include: a. hypotension, palpitations and respiratory distress b. urinary difficulty constipation and hypertension c. epistaxis abdominal pain and blood in the urine d. hypoglycemia, depression and poor muscle movement

B

The BEST example of an impulse control disorder is: A) stealing. B) pathologic gambling. C) uncontrollable worrying. D) obsessing over one's health.

B

The BEST way to ensure that no one is harmed during an EMS call is to: A) ensure that all EMS providers wear body armor. B) assess the potential for violence on every EMS call. C) stand to the side of the patient's door when knocking. D) routinely ask the police to respond to the scene with you.

B

The MOST disastrous consequence of a severe traumatic brain injury is: A) an increase in intracranial pressure. B) an increase in mean arterial pressure. C) severe hypertension and bradycardia. D) a decrease in cerebral perfusion pressure.

B

The MOST effective method for decreasing morbidity and mortality associated with spinal cord injury is: A) rapid transportation to a trauma center. B) public education and prevention strategies. C) minimizing scene time to 10 minutes or less. D) routine use of spinal motion restriction precautions.

B

The MOST important aspect in the care of a patient who is at risk for suicide is: A) identifying the patient's plan. B) never leaving the patient alone. C) acknowledging the patient's feelings. D) eliciting the help of family members.

B

The _______, also called the eyeball, is a spherical structure measuring about 1 inch in diameter. A) iris B) globe C) retina D) cornea

B

The __________ are cone-shaped fossae that enclose and protect the eyes. A) zygomas B) orbits C) condyles D) crista galli

B

The chemical mediators that initiate and maintain the immune response are: A) heparin and T cells. B) basophils and mast cells. C) macrophages and cytokines. D) eosinophils and neutrophils.

B

The correct dose, concentration, and route of epinephrine for a 40-pound child with an allergic reaction and no signs of cardiovascular collapse are: A) 0.12 mg 1:1,000 IM. B) 0.18 mg 1:1,000 SQ. C) 0.21 mg 1:10,000 IV. D) 0.25 mg 1:1,000 IV.

B

The longer the time between exposure to a substance: A) the greater the chance of massive IgE antibody production. B) the less likely a severe anaphylactic reaction will occur. C) the greater the chance that severe anaphylaxis will occur. D) the less likely that any kind of allergic reaction will occur.

B

The phrenic nerve arises from the _________ plexus and innervates the _______. A) sacral, lower limbs B) cervical, diaphragm C) lumbar, abdominal wall D) brachial, upper extremities

B

_________ forms the principal mass of the tooth and is much denser and stronger than bone. A) Pulp B) Dentin C) Enamel D) Gingiva

B

Transmission of gonorrhea occurs when contact is made with: A) the infected person's blood through an area where the skin is not intact. B) pus-containing fluid from the mucous membranes of the infected person. C) any portion of the infected person's genitalia during sexual intercourse. D) all of the infected person's bodily fluids, with or without sexual contact.

B

Transmission of mumps occurs by direct contact with the ________ of an infected person. A) tears B) saliva C) sweat D) blood

B

Treatment for Meniere disease includes: A) tubes placed in the ears. B) diuretics and antiemetics. C) hearing aids and analgesics. D) hydration and antipyretics.

B

Unlike bacteria, viruses: A) can only multiply outside a host. B) die when exposed to the environment. C) can only be contracted by direct contact. D) are larger and reproduce outside the cell.

B

Upon arriving at the scene of a motor vehicle crash, you find the driver of the car still seated in her two-door vehicle. The passenger side of the vehicle has sustained severe damage and is inaccessible. The driver is conscious and alert and complains only of lower back pain. The backseat passenger, a young child who was unrestrained, is bleeding from the head and appears to be unconscious. You should: A) ask the driver to step out of the vehicle so you can access the backseat passenger. B) rapidly extricate the driver so you can gain quick access to the child in the backseat. C) carefully assess the driver for occult injuries before removing her from the vehicle. D) apply a vest-type extrication device to the driver and quickly remove her from the car.

B

What spinal cord injury is characterized by motor loss on the same side as the injury, but below the lesion? A) Central cord syndrome B) Brown-Séquard syndrome C) Anterior cord syndrome D) Posterior cord syndrome

B

What type of intracranial hemorrhage would MOST likely be caused by a penetrating head injury? A) Subdural hematoma B) Intracerebral hematoma C) Epidural hematoma D) Subarachnoid hemorrhage

B

When applying a vest-type extrication device or short backboard to a seated patient, his or her head should be secured to the device: A) after you apply a cervical collar. B) only after the torso is fastened securely. C) before you secure his or her torso. D) after he or she is moved to a long backboard.

B

When assigning a Glasgow Coma Scale (GCS) score to a patient who has limb paralysis due to a spinal cord injury, you should: A) score the patient as having no motor response. B) ask the patient to blink or move a facial muscle. C) exclude the motor response portion of the GCS. D) use another method for assessing neurologic function.

B

When moving an injured patient from the ground onto a long backboard, it is preferred that you: A) slide the patient onto the backboard. B) use the four-person log roll technique. C) log roll the patient away from you. D) use a scoop stretcher to move the patient.

B

When obtaining the SAMPLE history of a person suspected of having an infectious disease, you should specifically inquire about: A) the patient's HIV status. B) a history of recent travel. C) any prior hospitalizations. D) the last sexual encounter.

B

When washing your hands after a call, you should: A) wash your hands for at least 10 seconds. B) use an antimicrobial, alcohol-based foam or gel. C) wash with cold water and let your hands air dry. D) scrub your hands vigorously with an antibacterial gel.

B

Which cranial nerve provides taste sensation to the posterior portions of the tongue and carries parasympathetic fibers to the salivary glands? A) Vagus B) Glossopharyngeal C) Hypoglossal D) Vestibulocochlear

B

Which of the following assessment findings is MOST suggestive of an organic cause of abnormal behavior? A) Multiple tattoos B) Unusual breath odor C) Skin that is cool and dry D) Auditory hallucinations

B

Which of the following drugs is an MAOI? A) Elavil B) Parnate C) Sinequan D) Pamelor

B

Which of the following examples provides the BEST description of indirect contact with a microorganism? A) Becoming infected with West Nile virus from a mosquito bite B) Touching a bloody stretcher railing with an open wound on your hand C) Inhaling infected droplets from a person after he or she sneezes or coughs D) Making brief physical contact with a person who has an infectious disease

B

Which of the following general statements regarding anaphylactic shock is correct? A) In order to provide appropriate treatment, you must first determine what caused the allergic reaction. B) In the presence of anaphylaxis, intervention takes precedence over identifying the offending antigen. C) Anaphylactic shock would most likely occur following initial exposure to an offending antigen. D) Most patients who carry a prescribed EpiPen are not completely aware of what substances they are allergic to.

B

Which of the following interventions is emphasized the MOST in the 2010 guidelines for emergency cardiac care? A) Defibrillation B) Chest compressions C) Artificial ventilation D) Tracheal intubation

B

17. The mean arterial pressure must be greater than or equal to ____ mm Hg to ensure that the brain, coronary arteries, and kidneys remained perfused. A) 50 B) 60 C) 70 D) 80

B) 60

Which of the following scenarios is MOST indicative of posttraumatic stress disorder? A) An advanced EMT student who has a morbid fear of needles experiences a syncopal episode during IV skills practice during class B) A paramedic whose son was struck and killed by a car becomes acutely anxious upon arriving at the scene of a motor-vehicle-versus-pedestrian incident C) An elderly woman who is afraid of losing her independence becomes agitated when being told that she is being admitted to a skilled nursing care facility D) An experienced paramedic who had a nightmare regarding a case of sudden infant death syndrome cries at the scene of a cardiac arrest involving an infant

B

Which of the following should occur when integrating a mechanical CPR device into your resuscitation attempt of an adult patient? A) Compression depth should be at least 1½ inches after the device is applied. B) Compressions should be continuous after an advanced airway device is inserted. C) The compression rate should be set to deliver at least 80 compressions per minute. D) One breath should be given every 6 to 8 seconds until an advanced airway is inserted.

B

Which of the following statements regarding a person's intelligence is MOST correct? A) All individuals possess the same basic level of intelligence. B) A person with mental retardation has a disorder of intelligence. C) Patients with Down syndrome typically lack basic intelligence. D) Intelligence is generally a reflection of a person's past education.

B

Which of the following statements regarding conjunctivitis is correct? A) Allergic conjunctivitis is extremely contagious. B) Conjunctivitis usually spreads to the opposite eye. C) Viral and bacterial conjunctivitis are not contagious. D) Lower respiratory infections often cause conjunctivitis.

B

Which of the following statements regarding epiglottitis is correct? A) It is caused by the Staphylococcus bacterium. B) It now occurs more often in adults. C) It is much less severe than croup. D) It affects children under 2 years of age.

B

Which of the following statements regarding leukotrienes is correct? A) In contrast to histamine, leukotrienes are less potent chemicals and do not cause vasodilation. B) Leukotrienes compound the physiologic effects of histamine and cause additional bronchoconstriction. C) Leukotriene release stimulates the release of histamine, which increases the severity of the allergic response. D) Leukotrienes attempt to mitigate the negative effects of histamine by causing coronary vasodilation.

B

Which of the following statements regarding vitreous humor is correct? A) Vitreous humor is a jelly-like substance that is replenished if lost. B) Vitreous humor fills the posterior chamber and maintains the shape of the globe. C) Vitreous humor is a clear watery fluid that fills the anterior chamber. D) The most significant risk caused by a loss of vitreous humor is infection.

B

You and your partner arrive at the scene of an unresponsive middle-aged man. Your primary assessment reveals that he is apneic and pulseless. Which of the following interventions will provide the BEST chance of survival for this patient? A) Five minutes of CPR prior to analyzing his cardiac rhythm B) CPR at the appropriate rate and with minimal interruptions C) Prompt insertion of an advanced airway to prevent aspiration D) Immediate defibrillation for presumed ventricular fibrillation

B

You are assessing a man who was boxing with his friend. He complains of seeing flashing lights and "floaters" in his field of vision. The MOST important intervention is: A) high-flow oxygen. B) immediate transport. C) irrigation of his eyes. D) covering both of his eyes

B

You are assessing a middle-aged woman who is acutely disoriented. According to her husband, she has bipolar disorder and, to the best of his knowledge, has been compliant with her medication. Her blood pressure is 106/66 mm Hg, pulse rate is 100 beats/min and strong, and respirations are 14 breaths/min and regular. During your care of this patient, it is MOST important to: A) establish vascular access and administer naloxone. B) assess her oxygen saturation and blood glucose level. C) attempt to assist her breathing with a bag-mask device. D) ask her simple questions that can be answered yes or no.

B

You are caring for a 40-year-old man in obvious anaphylactic shock after being stung by a scorpion. The patient is responsive to pain only, has poor respiratory effort, and is hypotensive and tachycardic. Which of the following represents the MOST appropriate treatment sequence for this patient? A) Immediate intubation, 0.5 mg epinephrine 1:1,000 SQ, two large-bore IV lines with normal saline, a 250-mL normal saline bolus, and 25 mg of Benadryl IM B) Assisted ventilation, intubation if necessary, at least one large-bore IV with normal saline, 1 mg epinephrine 1:10,000 IV, and up to 50 mg of Benadryl IV or IM C) High-flow oxygen via nonrebreathing mask, 0.1 to 0.5 mg epinephrine 1:1,000 IM, two large-bore IV lines with normal saline, and 20-mL/kg boluses of normal saline D) Assisted ventilation, Combitube insertion, 25 to 50 mg of Benadryl IM followed immediately by 0.3 mg of epinephrine 1:1,000 SQ, and a large-bore IV with normal saline

B

You are dispatched to a residence for a patient who has overdosed. You immediately recognize the address as that of a patient with a known psychiatric condition. Law enforcement is at the scene and has ensured its safety. When you arrive and enter the residence, you find the patient, a man in his mid-20s, sitting on the floor. He is conscious and appears anxious. Which of the following questions should you ask the patient FIRST? A) "How are you feeling right now?" B) "What medication did you take?" C) "Did you attempt to kill yourself?" D) "How long ago did this happen?"

B

You are dispatched to a senior citizen's center where an elderly woman fainted. When you arrive, you find the patient sitting in a chair. An employee of the center tells you that he caught the patient before she fell to the ground. Your primary assessment reveals that the patient is conscious and alert and is breathing adequately. You should: A) forego spinal immobilization and transport only. B) obtain vital signs and assess her blood glucose level. C) apply oxygen at 15 L/min via nonrebreathing mask. D) perform a rapid head-to-toe assessment to detect injuries.

B

You are transporting a 22-year-old man whose violent behavior required physical restraint. A law enforcement officer is in the back of the ambulance with you. The patient tells you that he is sorry for his behavior and did not mean to take it out on you and your partner. You should: A) loosen the restraints if the patient promises to cooperate with you. B) continue to talk to the patient and monitor his peripheral circulation. C) administer droperidol IM and then remove the restraints. D) tell the patient that his behavior is inexcusable and keep him restrained.

B

You have administered the appropriate dose of epinephrine to a patient with a severe allergic reaction. Reassessment reveals that the patient's condition has improved markedly. The patient, who has a history of coronary artery disease, is receiving high-flow oxygen and is on a cardiac monitor. You should next: A) start an epinephrine infusion at 4 µg/min, administer 25 mg of Benadryl IV or IM, and begin transport. B) transport at once, monitor airway and breathing en route, and administer up to 50 mg of Benadryl IV or IM. C) administer a half dose of epinephrine, begin transport, and give the patient 125 mg of methylprednisolone en route. D) transport immediately, monitor the patient's blood pressure en route, and give IV fluid boluses if symptoms recur.

B

You have an impedance threshold device (ITD) attached to the endotracheal tube as you ventilate an apneic and pulseless patient. Following defibrillation and 2 additional minutes of CPR, it is determined that return of spontaneous circulation has occurred. However, the patient is still apneic. You should: A) leave the ITD attached to the endotracheal tube and continue to ventilate. B) remove the ITD and continue ventilations at a rate of 10 to 12 breaths/min. C) leave the ITD attached, but increase your ventilation rate to 15 breaths/min. D) hyperventilate the patient to eliminate excess carbon dioxide from the blood.

B

You have intubated an unresponsive, apneic patient with a suspected spinal injury. After confirming proper ET tube placement and securing the tube, you should: A) request medical control authorization to give Solu-Medrol. B) ventilate at 10 to 12 breaths/min and monitor end-tidal CO2. C) maintain an end-tidal CO2 reading of greater than 45 mm Hg. D) provide mild hyperventilation in case a head injury is present.

B

You have treated the same patient several times for a severe allergic reaction. While educating him about the prevention of future reactions, you should advise him to: A) wear an identification bracelet. B) avoid the substance he is allergic to. C) carry at least two EpiPen injectors. D) call 9-1-1 as soon as he is exposed.

B

You receive a call for a 33-year-old man with difficulty breathing. Upon arrival, you begin to assess the patient, who tells you that he is HIV-positive. During the primary assessment, you should: A) immediately place a nonrebreathing mask on the patient. B) identify and correct immediately life-threatening conditions. C) inquire about any antiretroviral medications he is taking. D) apply two pairs of gloves in case you encounter any gross bleeding.

B

You receive a call for an "unresponsive person." Law enforcement arrives at the scene before you and advises that the scene is secure. When you arrive, you find the patient, a young man, lying supine on the floor of his poorly kept apartment. He is unresponsive and is breathing with a marked reduction in tidal volume. One of the police officers recognizes the patient as a known intravenous drug abuser. During your care of this patient, you should: A) apply a nonrebreathing mask and set the flow rate at 15 L/min. B) avoid removing his wallet or any other possessions from his pockets. C) defer vascular access until the patient is in the emergency department. D) establish an IV line of normal saline and administer 2 mg of flumazenil.

B

Your assessment of a 68-year-old man reveals an organized cardiac rhythm at a rate of 80 beats per minute and an absent carotid pulse. Treatment for this patient may include all of the following, EXCEPT: A) epinephrine. B) cardiac pacing. C) vasopressin. D) IV fluid boluses.

B

Your team is attempting resuscitation of a man in cardiac arrest. One of the team members intubates the patient and confirms proper placement of the endotracheal tube. Your MOST appropriate next action should be to: A) presume that the patient is in severe metabolic acidosis and hyperventilate him for at least 2 to 3 minutes. B) instruct the person ventilating to provide one breath every 6 to 8 seconds while chest compressions are continuous. C) administer 2.5 mg of epinephrine 1:10,000 via the endotracheal tube without interrupting chest compressions. D) defibrillate the patient with the maximum energy setting, reassess for a pulse, and continue CPR if he remains pulseless.

B

____ antibodies respond in allergic reactions and are located _______. A) IgD, in the lymph and blood B) IgE, on the mast and basophil cells C) IgG, in the blood, lymph, and intestines D) IgA, in tears, saliva, blood, and lymph

B

a decreased number of pacemaker cells in the sinoatrial node would most likely contribute to a decline in cardiac output secondary to: a. heart block b. bradycardia c. tachycardia d. atrial fibrillation

B

an 82 year old man presents with confusion that has worsening progressively over the past 2 weeks. b/c of his confusion and the fact that he is hearing impaired, you obtained the majority of your medical history information form his daughter. she tells you that her father has high blood pressure but refuses to take medication for it. she also tells you that she thinks he fell about 3 weeks ago, although he denies falling. the p/t's blood pressure is 168/98 mm Hg, pulse rate is 60 beats/ min and occasionally irregular and respiration are 22 breaths/min and regular this p/t most likely a. has an acute onset of dementia b. is experiencing a subdural hematoma c. had a stroke secondary to atrial fibrillation d. hsa normal age-related physiologic changes

B

considering the physiologic changes that occur with age, which of the following interventions would pose the GREATEST potential for further harm when caring for an elderly p/t with severe burn that is complicated by a spinal injury a. intubation b. fluid replacement c. thermal management d. spinal immobilization

B

due to the process of gained an elderly person commonly experiences a. increases thirst b. moderate hyperglycemia c. increased insulin secretion d. high fever with a minor illness

B

elderly _________ are at highest risk for suicide and most often use __________ as their suicide method of choice a. women, pills b. men, firearms c. women, knives d. men, hanging

B

the leading cause of death from infection in americans older than 65 yrs of age is: a. nephritis b. pneumonia c. encephalitis d. meningitis

B

when elderly p/t's are prescribed medications, the risk for drug toxicity increases b/c of a. an increase in the glomerular filtration rate b. age-related changes in hepatic enzyme systems c. the presence of chronic conditions, such as diabetes d. an increase in the production of antidiuretic hormone

B

which of the following conditions would most likely impair pulmonary function by limiting lung volume and maximal inspiratory pressure? a. asthma b. kyphosis c. spondylosis d. osteoporosis

B

which of the following disease processes would most likely present with signs and symptoms that mimic those seen normal aging? a. type 2 diabetes b. hypothyroidism c. thyrotoxicosis d. hyperglycemia

B

which of the following is not a predisposing risk factor for trauma in the elderly? A. slower reflexes b. decreased body water c. equilibrium disorders d. overall decrease in agility

B

which of the following statements regarding Alzheimer disease is NOT correct a. Alzheimer disease is likely not a part of the normal process of aging b. age alone is thought to be underlying cause of Alzheimer disease c. the symptoms of Alzheimer disease are generally obvious at its onset d. common signs of Alzheimer disease included paranoia and delusions

B

which of the following statements regarding aging is correct? a. although aging causes various changes physiologic functions generally remain intact b. health care providers may attribute genuine disease symptoms to age, resulting in inadequate care c. all of the tissues and organs in the human body undergo the effects of aging at the same rate d. health care providers typically recognize the normal changes of aging and tend not to render unnecessary care

B

6. Most children begin to develop stranger anxiety between ___ and ___ months of age. A) 3, 6 B) 6, 12 C) 12, 18 D) 18, 24

B) 6, 12

15. Which of the following statements would be appropriate when documenting a case of suspected abuse? A) "The patient's injury is not consistent with an accident." B) "The injury was reported to have occurred at 1420 hours." C) "The abdominal exam of the patient was within normal limits." D) "There was a gross discrepancy between the caregivers' stories."

B) "The injury was reported to have occurred at 1420 hours."

17. The pediatric assessment triangle will help answer all of the following questions, EXCEPT: A) "Is the child sick or not sick?" B) "Will the child cooperate during my exam?" C) "Does the child require emergency treatment?" D) "What is the most likely physiologic abnormality?"

B) "Will the child cooperate during my exam?"

47. Appropriate treatment for a conscious child with anaphylaxis includes: A) 0.5 mg/kg of diphenhydramine IV. B) 0.01 mg/kg epinephrine 1:1,000 IM. C) pharmacologically assisted intubation. D) a dopamine infusion to increase the blood pressure

B) 0.01 mg/kg epinephrine 1:1,000 IM.

117. Fever in infants younger than 2 months of age is defined as a body temperature that is ____°F or greater. A) 99.2 B) 100.4 C) 101.2 D) 102.0

B) 100.4

40. Abortion is defined as expulsion of the fetus, from any cause, before the ____ week of pregnancy. A) 18th B) 20th C) 24th D) 28th

B) 20th

35. If hypovolemia is suspected or confirmed, you should administer ____ mL of normal saline to a 6-pound newborn over a period of ____. A) 18, 10 to 20 minutes B) 27, 5 to 10 minutes C) 33, 10 to 20 minutes D) 55, 5 to 10 minutes

B) 27, 5 to 10 minutes

55. A trauma patient has lost an estimated 1..5 L of blood through external bleeding and is hypotensive.. How much normal saline or lactated Ringer's solution should he receive? A) 1.5 L B) 3 to 4.5 L C) At least 5 L D) 250 to 500 mL

B) 3 to 4.5 L

70. Your assessment of a depressed 7-pound newborn reveals tachypnea, pallor, weak peripheral pulses, a heart rate of 120 beats/min, and a blood glucose level of 58 mg/dL. Which of the following interventions will MOST likely cause improvement in this newborn's condition? A) 0.3 mg of naloxone B) 32 mL of normal saline C) 6.5 mL of 10% dextrose D) 0.06 mg of epinephrine

B) 32 mL of normal saline

5. A newborn born between ___ and ___ weeks of gestation is described as term. A) 36, 38 B) 38, 42 C) 40, 42 D) 42, 44

B) 38, 42

88. When administering a nebulized bronchodilator, the oxygen flow rate should be set to at least ____ liters per minute. A) 4 B) 6 C) 8 D) 10

B) 6

29. Chest compressions are indicated in the newborn if its heart rate remains less than ____ beats/min despite effective positive-pressure ventilations for ____ seconds. A) 80, 30 B) 60, 30 C) 60, 60 D) 80, 60

B) 60, 30

59. Approximately ___% of all penetrating abdominal wounds to the mother result in fetal injury. A) 50 B) 70 C) 85 D) 90

B) 70

75. If you can feel a pulse over the femoral artery of an adult, but are unable to feel a pulse over the radial artery, his or her systolic blood pressure is MOST likely between ___ and ___ mm Hg. A) 60, 70 B) 70, 80 C) 80, 90 D) 90, 100

B) 70, 80

25. If a woman's prepregnancy resting heart rate is 70 beats per minute, you would expect her heart rate to range between ____ and ____ at term. A) 75, 85 B) 85, 90 C) 90, 100 D) 100, 110

B) 85, 90

65. Which of the following blood pressures is MOST consistent with a pericardial tamponade? A) 80/50 mm Hg B) 90/70 mm Hg C) 100/60 mm Hg D) 110/80 mm Hg

B) 90/70 mm Hg

85. What is the pathophysiology of cystic fibrosis? A) Acquisition of a virus or bacterium that takes residence inside the parenchyma of the lungs, resulting in the production of thin but copious secretions B) A defective gene that makes it difficult for chloride to move through the cells, which causes unusually high sodium loss and abnormally thick mucus secretions C) An underproduction of pulmonary surfactant, which causes chronic respiratory distress, thick pulmonary secretions, and severe intrapulmonary shunting D) Delayed lung growth and development that is typically caused by a variety of congenital cardiovascular defects, most notably a ventriculoseptal defect

B) A defective gene that makes it difficult for chloride to move through the cells, which causes unusually high sodium loss and abnormally thick mucus secretions

68. Which of the following statements regarding patients with emotional or mental impairment is correct? A) Repeating the patient's statements often causes him or her to become agitated. B) A person's mental status can influence his or her physical well-being, and vice versa. C) Most patients with a mental impairment must be spoken to in a firm, direct tone of voice. D) Agitated speech is common in this patient population and generally does not suggest a potential for violence.

B) A person's mental status can influence his or her physical well-being, and vice versa.

35. Which of the following is NOT characteristic of the secondary stage of syphilis? A) The presence of a skin rash B) A single or multiple chancres C) Mucous membrane lesions D) Fever and swollen lymph glands

B) A single or multiple chancres

48. Which of the following would MOST likely cause Cushing syndrome? A) Abrupt termination of steroids such as prednisone and hydrocortisone B) Administration of large amounts of methylprednisolone to an asthma patient C) An acute decrease in cortisol secretion secondary to an infection or malignancy D) Underuse of corticosteroids for illnesses such as rheumatoid arthritis and asthma

B) Administration of large amounts of methylprednisolone to an asthma patient

79. Your primary assessment of a multisystems trauma patient reveals that he is unresponsive and is not breathing. What should you do next? A) Transport at once. B) Assess for a pulse. C) Ventilate the patient. D) Intubate his trachea.

B) Assess for a pulse.

Which of the following statements regarding prehospital vascular access and fluid therapy in the severely burned patient is correct? A) Most burn patients will require at least 4 L of IV fluid immediately. B) At least one large-bore IV should be started while en route to the hospital. C) Intraosseous cannulation is absolutely contraindicated in severely burned patients. D) An IV line in a lower extremity is preferable to one in a burned upper extremity.

B) At least one large-bore IV should be started while en route to the hospital.

40. Which of the following vascular access devices is "tunneled" under the skin and placed into the superior vena cava? A) Midline catheter B) Broviac catheter C) Dialysis catheter D) Double lumen catheter

B) Broviac catheter

8. Which of the following injury patterns is MOST suggestive of child abuse? A) Burns with splash marks B) Bruises on the abdomen C) Bruises on the lower leg D) Laceration to the chin

B) Bruises on the abdomen

Which of the following burn injuries would MOST likely require transport to a burn specialty center? A) Superficial burns to more than 40% of the body B) Burns that involve the hands, feet, or genitalia C) Partial-thickness burns to more than 5% of the body D) Any burn that occurs in a child under 5 years of age

B) Burns that involve the hands, feet, or genitalia

4. Which of the following statements regarding a child's chest wall is correct? A) Lung sounds are difficult to hear because of the thick intercostal muscles. B) Children are belly breathers because they rely heavily on their diaphragms. C) A child's chest wall has proportionately more subcutaneous fat on the chest. D) Retractions are less obvious in children owing to their noncompliant rib cages.

B) Children are belly breathers because they rely heavily on their diaphragms.

3. Which of the following patient populations is at GREATEST risk for abuse or neglect? A) Females between 6 and 12 years of age B) Children with chronic medical conditions C) Adolescents who rebel against their parents D) Males between 2 and 14 years of age

B) Children with chronic medical conditions

5. Which of the following bones is MOST vulnerable to injury? A) Sternum B) Clavicle C) Scapula D) Humerus

B) Clavicle

2. What part of the female genitalia is homologous to the glans penis of the male? A) Hymen B) Clitoris C) Urethra D) Vestibule

B) Clitoris

50. There is evidence that a 21-year-old woman was given Rohypnol before being raped. She presents with alcohol on her breath, drowsiness, and memory loss. Her blood pressure is 98/68 mm Hg, pulse is 58 beats/min and weak, and respirations are decreased and shallow. You attempt to administer high-flow oxygen, but she resists. What should you do now? A) Administer a sedative medication to facilitate her acceptance of the oxygen. B) Consider administering naloxone in case she was also given a narcotic drug. C) Start an IV line and administer 1 to 2 liters of an isotonic crystalloid solution. D) Recognize that because the patient is of legal age, she can refuse EMS treatment.

B) Consider administering naloxone in case she was also given a narcotic drug.

5. Which of the following will be of MOST help to the paramedic when formulating a field diagnosis? A) Receiving input from family members B) Considering or ruling out various conditions C) Identifying and treating immediate life threats D) Trending the vital signs over a period of time

B) Considering or ruling out various conditions

13. Which of the following is NOT a sight of potential injury when the aorta is subjected to shearing forces during rapid deceleration? A) Anulus B) Coronary sinus C) Aortic hiatus D) Ligamentum arteriosum

B) Coronary sinus

_________ forms the principal mass of the tooth and is much denser and stronger than bone. A) Pulp B) Dentin C) Enamel D) Gingiva

B) Dentin

107. Common medications used to treat pediatric seizures in the prehospital setting include all of the following, EXCEPT: A) Ativan. B) Dilantin. C) Diazepam. D) Midazolam.

B) Dilantin.

When treating a patient with an ocular injury, what should you do to avoid an increase in intraocular pressure? A) Apply light pressure to both eyes. B) Discourage the patient from coughing. C) Administer prophylactic atropine sulfate. D) Ensure that the patient remains supine.

B) Discourage the patient from coughing.

13. Which of the following conditions poses the LEAST risk for shock due to vaginal bleeding? A) Metrorrhagia B) Dysmenorrhea C) Polymenorrhea D) Hypermenorrhea

B) Dysmenorrhea

18. Which of the following is a potential complication of pelvic inflammatory disease? A) Uterine rupture B) Ectopic pregnancy C) Respiratory failure D) Urinary tract infection

B) Ectopic pregnancy

119. What forms of child maltreatment are often difficult to identify and may go unreported? A) Sexual and emotional abuse B) Emotional abuse and neglect C) Neglect and physical abuse D) Physical and emotional abuse

B) Emotional abuse and neglect

58. Which of the following statements regarding epiglottitis is correct? A) Unlike croup, epiglottitis most commonly occurs in the middle of the night, when the outside temperature is cool. B) Epiglottitis has become relatively rare in children due to vaccinations against the Haemophilus influenzae type b bacterium. C) Most cases of epiglottitis are progressive in their onset and result in severe swelling of the larynx, trachea, and bronchi. D) Characteristic signs of epiglottitis include a low-grade fever, a seal-like barking cough, and varying degrees of respiratory distress.

B) Epiglottitis has become relatively rare in children due to vaccinations against the Haemophilus influenzae type b bacterium.

What two major nerves provide sensory and motor control to the face? A) Facial and maxillary B) Facial and trigeminal C) Ophthalmic and vagus D) Maxillary and mandibular

B) Facial and trigeminal

16. Which of the following is NOT a function of the placenta? A) Synthesis of glycogen and cholesterol B) Fetal protection against all harmful substances C) Antibody production that protects the fetus D) Excretion of wastes in the maternal circulation

B) Fetal protection against all harmful substances

67. By which of the following mechanisms do patients with septic shock become hypovolemic? A) Sustained systemic vascular dilation B) Fluid leakage out of the vascular space C) Spontaneous destruction of red blood cells D) Frequent and severe vomiting and diarrhea

B) Fluid leakage out of the vascular space

9. Which of the following medications would MOST likely be used in the prehospital setting to treat some women with severe premenstrual syndrome? A) Haldol and Tylenol B) Glucose and fentanyl C) Valium and ibuprofen D) Insulin and acetaminophen

B) Glucose and fentanyl

24. Which of the following components represent the perfusion triangle? A) Red blood cells and oxygen B) Heart, blood, blood vessels C) Oxygen, lungs, red blood cells D) Organs, oxygen, carbon dioxide

B) Heart, blood, blood vessels

26. Stretch receptors in the lungs are responsible for the ____________ reflex, which causes you to cough if you take too deep a breath. A) Cushing B) Hering-Breuer C) pneumotaxic D) yawning

B) Hering-Breuer

Which of the following is a potentially life-threatening complication of missing one or more dialysis treatments? A) Uremic frost B) Hyperkalemia C) Hypocalcemia D) Peripheral edema

B) Hyperkalemia

47. Law enforcement request that you respond to a local apartment complex for a young woman who was sexually assaulted. When you arrive at the scene, you find the patient sitting on her couch, clearly upset. You see a small amount of blood on her shorts, near the groin area. What is your initial priority in the care of this patient? A) Asking her if she recognized the perpetrator B) Identifying and treating immediate life threats C) Not allowing her to shower or use the restroom D) Quickly assessing her blood pressure and pulse

B) Identifying and treating immediate life threats

28. Which of the following statements regarding a pericardial tamponade is correct? A) Most pericardial tamponades are caused by blunt chest trauma during an automobile crash. B) In a pericardial tamponade, blood collects between the visceral and parietal pericardium. C) Pericardial tamponade is characterized by a marked increase in preload and flat jugular veins. D) The parietal pericardium stretches easily, so significant blood accumulation is required before signs appear.

B) In a pericardial tamponade, blood collects between the visceral and parietal pericardium.

Which of the following statements regarding peritoneal dialysis is correct? A) Because of the high risk of peritonitis, peritoneal dialysis can only be performed in a hospital or specialized dialysis center. B) In peritoneal dialysis, large amounts of specially formulated dialysis fluid are infused into the abdominal cavity and left for 1 to 2 hours. C) Peritoneal dialysis involves the surgical placement of an arteriovenous shunt in the vasculature of the abdominal cavity. D) Peritoneal dialysis is the least-preferred method of dialyzing a patient and is only used in extreme emergency situations.

B) In peritoneal dialysis, large amounts of specially formulated dialysis fluid are infused into the abdominal cavity and left for 1 to 2 hours.

96. You are assisting in the delivery of a baby. As the infant's head begins to emerge from the vagina, you should support the head as it turns and then: A) carefully dry its face. B) assess for a nuchal cord. C) thoroughly suction its airway. D) gently guide the head upward.

B) assess for a nuchal cord.

40. Which of the following occurs during compensated shock? A) Cerebral hypoperfusion causes pupillary constriction B) Increased rate of breathing causes respiratory alkalosis C) Urine output decreases to less than 20 mL/hour D) Vasomotor failure leads to a drop in systolic blood pressure

B) Increased rate of breathing causes respiratory alkalosis

Which of the following is the LEAST common cause of death from fires? A) Pulmonary injury B) Integument burns C) Upper airway compromise D) Inhalation of toxic gases

B) Integument burns

55. Which of the following represents the correct drug, dose, and delivery route for an 18-kg child experiencing severe respiratory distress due to bronchospasm? A) Albuterol, 1 mg nebulized B) Ipratropium, 0.5 mg nebulized C) Albuterol, 0.25 mg nebulized D) Epinephrine, 0.1 mg/kg IM

B) Ipratropium, 0.5 mg nebulized

54. Which of the following solutions is preferred for fluid resuscitation of a patient in shock in the prehospital setting? A) Dextran or Plasmanate B) Isotonic crystalloid C) Hypertonic saline D) Dextrose in water

B) Isotonic crystalloid

17. What is the function of estrogen? A) It stimulates the release of progesterone from the pituitary gland. B) It signals the anterior pituitary gland to secrete luteinizing hormone. C) It releases androgens that are responsible for pubic and armpit hair. D) It signals the posterior pituitary gland to secrete gonadotropic hormones.

B) It signals the anterior pituitary gland to secrete luteinizing hormone.

48. Which of the following clinical signs may not be present in a patient with a tension pneumothorax and associated internal bleeding? A) Tachycardia B) Jugular vein distention C) Mediastinal shift D) Contralateral tracheal deviation

B) Jugular vein distention

60. Which of the following assessment findings should increase your index of suspicion for obstructive shock? A) Low blood pressure B) Jugular venous distention C) Increased lung compliance D) Generalized edema

B) Jugular venous distention

18. What structure helps maintain the stability of a joint and determines the degree of joint motion? A) Tendon B) Ligament C) Cartilage D) Joint capsule

B) Ligament

23. Which of the following factors complicates airway management in an obese patient? A) Larger upper airway B) Limited neck mobility C) Smaller patient head size D) Proportionately small tongue

B) Limited neck mobility

48. Which of the following hemodynamic parameters decreases, regardless of the etiology of the shock? A) Pulse rate B) Mean arterial pressure C) Central venous pressure D) Peripheral vascular resistance

B) Mean arterial pressure

27. Which of the following changes occurs in the respiratory system of a pregnant woman? A) Progesterone increases the threshold of the medullary respiratory center to carbon dioxide. B) Minute ventilation increases by as much as 50%, which causes the PaCO2 to drop by about 5 mm Hg. C) Oxygen consumption decreases by approximately 20%, causing a 40% decrease in tidal volume. D) An increase in blood bicarbonate levels causes a slight decrease in the pH level of the blood.

B) Minute ventilation increases by as much as 50%, which causes the PaCO2 to drop by about 5 mm Hg.

53. Which of the following factors is associated with the HIGHEST risk of newborn hypoglycemia? A) Neonatal polycythemia B) Morbid obesity in the mother C) The larger of discordant twins D) 5-minute Apgar score of less than 7

B) Morbid obesity in the mother

39. Which of the following thoracic injuries would you LEAST likely discover in the primary assessment? A) Flail chest B) Myocardial contusion C) Bronchial disruption D) Open pneumothorax

B) Myocardial contusion

21. Which of the following statements regarding gas exchange in the lungs is correct? A) There are more carbon dioxide molecules in the blood than in inhaled air. B) Oxygen molecules move from the alveoli into the blood by diffusion. C) There are more oxygen molecules in the blood than in the alveoli. D) Carbon dioxide molecules move from the alveoli into the blood by diffusion.

B) Oxygen molecules move from the alveoli into the blood by diffusion.

68. A 68-year-old woman presents with an acute onset of dyspnea and sharp chest pain. Her medical history is significant for a hip replacement 2 weeks ago. The patient is conscious and alert, with a blood pressure of 112/58 mm Hg, pulse rate of 90 beats/min and irregular, and respirations of 22 breaths/min and labored. Which of the following treatment interventions is MOST appropriate for this patient? A) Pharmacologically assisted intubation and IV therapy B) Oxygenation and ventilation support and rapid transport C) 324 mg of aspirin and ventilation support with a bag-mask D) Supplemental oxygen and elevation of the lower extremities

B) Oxygenation and ventilation support and rapid transport

27. Which of the following statements regarding toxic shock syndrome (TSS) is correct? A) TSS is a condition that is exclusive to females. B) Patients with TSS may show signs of liver failure. C) Most cases of TSS occur in the absence of a fever. D) Group B Streptococcus is a causative agent in TSS.

B) Patients with TSS may show signs of liver failure.

12. ___________ is a dense, fibrous membrane that is important for bone repair and maintenance. A) Cartilage B) Periosteum C) Synovium D) Endosteum

B) Periosteum

17. An oropharyngeal airway would MOST likely be indicated for a newborn with: A) gasping respirations. B) Pierre Robin sequence. C) a diaphragmatic hernia. D) prolonged periods of apnea.

B) Pierre Robin sequence.

73. Which of the following events occur during the third stage of labor? A) Crowning B) Placental delivery C) Delivery of the baby D) Mucous plug expulsion

B) Placental delivery

1. Which of the following is NOT an antepartum risk factor that increases the potential that a newborn may require resuscitation? A) Preeclampsia B) Prolapsed cord C) Polyhydramnios D) Multiple gestations

B) Prolapsed cord

63. Which of the following assessment parameters is MOST difficult to interpret when a pregnant woman is injured? A) Pupils B) Pulse rate C) Mental status D) Blood glucose

B) Pulse rate

11. When a person jumps from a height and lands on his or her feet, direct trauma occurs to the: A) tarsals. B) malleolus. C) calcaneus. D) metatarsals.

C) calcaneus.

95. A 66-year-old man with chronic bronchitis presents with severe respiratory distress. The patient's wife tells you that he takes medications for high blood pressure and bronchitis, is on home oxygen therapy, and has recently been taking an over-the-counter antitussive. She further tells you that he has not been compliant with his oxygen therapy. Auscultation of his lungs reveals diffuse rhonchi. What is the MOST likely cause of this patient's respiratory distress? A) Oxygen noncompliance B) Recent antitussive use C) An underlying infection D) Acute right heart failure

B) Recent antitussive use

Which of the following statements regarding scald burns is correct? A) Once hot liquids come in contact with clothing, heat is rapidly dissipated. B) Scald burns often cover large surface areas because liquids spread quickly. C) Scald burns caused by grease or oil are typically limited to the epidermis. D) Scald burns are less commonly seen in pediatric patients than adult patients.

B) Scald burns often cover large surface areas because liquids spread quickly.

What portion of the eye may become icteric in patients with hepatitis? A) Iris B) Sclera C) Cornea D) Conjunctiva

B) Sclera

26. Which of the following is the BEST example of an indirect injury? A) Patellar fracture after the knee strikes an automobile's dashboard B) Shoulder dislocation secondary to falling on an outstretched hand C) Fractured ankle after stepping in a hole and twisting the lower leg D) Dislocated olecranon process following direct trauma to the elbow

B) Shoulder dislocation secondary to falling on an outstretched hand

70. Which of the following statements regarding autism is correct? A) The vast majority of patients with autism are mute and have an IQ of less than 20. B) Some patients with autism are able to easily solve complex mathematical problems. C) Common causes of autism include traumatic brain injury and severe emotional trauma. D) You should expect that a patient with autism will respond favorably to physical contact.

B) Some patients with autism are able to easily solve complex mathematical problems.

1. Which of the following statements regarding the thorax is correct? A) The thoracic cavity extends to the ninth or tenth rib posteriorly. B) The diaphragm inserts into the anterior thoracic cage below the fifth rib. C) The dimensions of the thorax are defined inferiorly by the thoracic inlet. D) The dimensions of the thorax are defined anteriorly by the thoracic vertebrae.

B) The diaphragm inserts into the anterior thoracic cage below the fifth rib.

Which of the following statements regarding the renal system is correct? A) Urinary tract infections are more prevalent in men. B) The kidneys are located in the retroperitoneal space. C) Urinary tract infections are the most common renal disease. D) The urethra transports urine from the kidneys to the bladder.

B) The kidneys are located in the retroperitoneal space.

6. Which of the following does NOT occur after the ovum is not fertilized? A) The endometrium breaks down and is shed as menstrual flow. B) The secretory phase increases the size of the endometrial glands. C) The ovum dies and degenerates 36 to 48 hours after being released. D) The corpus luteum degenerates 10 to 12 days after lack of fertilization.

B) The secretory phase increases the size of the endometrial glands.

105. Which of the following statements regarding simple febrile seizures is correct? A) The overall prognosis for a child with simple febrile seizures worsens with each seizure episode. B) There is no relationship between simple febrile seizures and future developmental or learning disabilities. C) More than one simple febrile seizure in a child is highly suggestive of an underlying neurologic problem. D) Any child who experiences a simple febrile seizure is at significant risk for developing epilepsy.

B) There is no relationship between simple febrile seizures and future developmental or learning disabilities.

61. Which of the following clinical presentations is MOST consistent with a malfunctioning cerebrospinal fluid shunt? A) Tachycardia, tinnitus, and a narrowing pulse pressure B) Visual disturbances, headache, and altered mental status C) Hypotension, sudden loss of hearing, and severe nausea D) Tachypnea, fluid drainage from the ears, and restlessness

B) Visual disturbances, headache, and altered mental status

Which of the following statements regarding vitreous humor is correct? A) Vitreous humor is a jelly-like substance that is replenished if lost. B) Vitreous humor fills the posterior chamber and maintains the shape of the globe. C) Vitreous humor is a clear watery fluid that fills the anterior chamber. D) The most significant risk caused by a loss of vitreous humor is infection

B) Vitreous humor fills the posterior chamber and maintains the shape of the globe.

1. What function do the nasal turbinates serve? A) Separating the left and right nostrils B) Warming and humidifying inhaled air C) Decreasing the surface area of the nasopharynx D) Secreting mucus that traps viruses and bacteria

B) Warming and humidifying inhaled air

124. When assessing an otherwise healthy child who is injured, you notice that his general appearance is abnormal. This should make you MOST suspicious for: A) child abuse. B) a head injury. C) hypoglycemia. D) internal bleeding.

B) a head injury.

If you are unable to orotracheally intubate a patient due to massive maxillofacial trauma and severe oropharyngeal and nasopharyngeal bleeding, you would MOST likely have to perform: A) nasotracheal intubation. B) a needle or surgical cricothyrotomy. C) pharmacologically assisted intubation. D) digital (tactile) intubation.

B) a needle or surgical cricothyrotomy.

30. Reiter syndrome is: A) an acute life-threatening condition that has been linked directly to the use of high-absorbency tampons. B) a rare complication associated with untreated chlamydia and is characterized by arthritis and skin lesions. C) a far more common and serious complication of untreated chlamydia than pelvic inflammatory disease. D) a relatively common complication of untreated gonorrhea, and is characterized by low-grade fever and myalgia.

B) a rare complication associated with untreated chlamydia and is characterized by arthritis and skin lesions.

46. You are dispatched to a residence for a 34-year-old woman who is "sick." Upon your arrival, the patient greets you at the door. She tells you that she began experiencing a dull aching pain in the left lower quadrant of her abdomen. She further states that she recently had her menstrual period, which was accompanied by more pain than usual. Her vital signs are stable, she is conscious and alert, and she denies vaginal bleeding or fever. This patient MOST likely has: A) an ectopic pregnancy. B) a ruptured ovarian cyst. C) a tubo-ovarian abscess. D) pelvic inflammatory disease.

B) a ruptured ovarian cyst.

51. Following delivery of a stillborn baby at 16 weeks, the mother presents with fever, abdominal tenderness, and a foul-smelling vaginal discharge. This clinical presentation is consistent with: A) trichomoniasis. B) a septic abortion. C) toxoplasmosis. D) bacterial vaginosis.

B) a septic abortion.

38. An otherwise healthy adult whose normal hemoglobin level is 12 to 14 g/dL typically will begin to exhibit cyanosis when: A) hemoglobin levels fall below 12 g/dL. B) about 5 g/dL of hemoglobin is desaturated. C) his or her oxygen saturation falls below 50%. D) 10% of his or her hemoglobin is desaturated.

B) about 5 g/dL of hemoglobin is desaturated.

77. A patient presents with a sudden onset of unilateral eye pain and blurred vision. You should suspect: A) acute optic nerve hypoplasia. B) acute angle-closure glaucoma. C) central retinal arterial occlusion. D) spontaneous retinal detachment.

B) acute angle-closure glaucoma.

25. Diabetic patients would MOST likely present with atypical signs and symptoms of: A) bacterial pneumonia. B) acute coronary syndrome. C) an acute ischemic stroke. D) viral or fungal meningitis.

B) acute coronary syndrome.

Immediately following a dialysis treatment, a middle-aged woman complains of generalized weakness and nausea. Her blood pressure is 80/50 mm Hg, pulse rate is 40 beats/min and weak, and respirations are 22 breaths/min and regular. She is receiving supplemental oxygen, and an IV line has been established in the extremity opposite the shunt. The ECG reveals sinus bradycardia in lead II. Your next action should be to: A) obtain a 12-lead ECG tracing. B) administer 0.5 mg of atropine sulfate. C) administer a 20-mL/kg fluid bolus. D) administer calcium and bicarbonate.

B) administer 0.5 mg of atropine sulfate.

54. You receive a call to the county jail for a male inmate who is unresponsive. According to the jailor, the patient was arrested for being "drunk." Your assessment reveals that the patient is profusely diaphoretic, and his respirations are rapid and shallow. His blood glucose level reads 30 mg/dL. As your partner assists the patient's ventilations, you start an IV and administer 50% dextrose. Reassessment reveals that the patient is responsive to pain only and his blood glucose level is 46 mg/dL. You should: A) intubate his trachea to prevent aspiration and transport him immediately. B) administer a second dose of dextrose and prepare for immediate transport. C) give him 1 mg of glucagon IM and reassess his blood glucose. D) conclude that he will require immediate definitive care and begin transport.

B) administer a second dose of dextrose and prepare for immediate transport.

91. You are caring for an elderly woman with terminal sarcoma. She is conscious, extremely weak, and in severe pain. Her son tells you that she has a living will and an out-of-hospital do not resuscitate order, and produces the appropriate documentation. The patient is on home oxygen at 2 L/min via nasal cannula. You should: A) recognize that the patient's death is imminent and remain at the scene to provide any needed emotional support. B) administer an appropriate dose of morphine or fentanyl to the patient and provide emotional support to the son. C) apply high-flow oxygen via nonrebreathing mask, establish vascular access, and transport her to the hospital. D) leave the patient on her nasal oxygen, keep her warm, and transport her safely to the hospital for further palliative care.

B) administer an appropriate dose of morphine or fentanyl to the patient and provide emotional support to the son.

167. Following significant blunt trauma to the abdomen, a 9-year-old boy presents with diaphoresis and pallor. He is conscious and alert, with a blood pressure of 90/58 mm Hg, a heart rate of 130 beats/min, and a respiratory rate of 28 breaths/min with adequate depth. With an estimated ground transport time of 30 minutes, you should: A) assist his ventilations to increase tidal volume, cover him with a blanket, establish at least one large-bore IV line, administer a 20-mL/kg normal saline bolus, and transport to a trauma center. B) administer high-flow oxygen, apply spinal precautions if indicated, provide warmth, begin transport, establish vascular access en route, and administer enough crystalloid solution to maintain adequate perfusion. C) apply supplemental oxygen, start two large-bore IV lines with normal saline, administer several crystalloid boluses of 20 mL/kg, apply spinal precautions if indicated, and transport to an appropriate medical facility. D) apply warm blankets, elevate his lower extremities 12 inches, administer high-flow oxygen, insert an IO catheter, administer a 250-mL normal saline bolus, and transport expeditiously to an appropriate trauma center.

B) administer high-flow oxygen, apply spinal precautions if indicated, provide warmth, begin transport, establish vascular access en route, and administer enough crystalloid solution to maintain adequate perfusion.

64. During your rapid assessment of a newborn's cardiopulmonary status, you note that its respirations are adequate, you feel 8 pulsations in a 6-second time frame, and the newborn is centrally pink but peripherally cyanotic. The MOST appropriate next action should be to: A) provide 30 seconds of tactile stimulation. B) administer positive-pressure ventilations. C) assess the newborn's blood glucose level. D) give free-flow oxygen by mask at 5 L/min.

B) administer positive-pressure ventilations.

63. A 60-year-old woman slipped and fell on an icy sidewalk and landed on her outstretched hand. Your assessment reveals that she has an obvious Colles fracture. The patient denies any other injuries and is conscious and alert. Her vital signs are stable and she describes her pain as a 2 on a scale of 0 to 10. Given this patient's current status, the MOST appropriate way to treat her injury involves: A) gently straightening the fracture site and then applying a splint. B) administering analgesia and then properly splinting her injury. C) giving her a sedative for pain relief and then applying an air splint. D) manually stabilizing her wrist as your partner applies a vacuum splint.

B) administering analgesia and then properly splinting her injury.

19. Hypoventilating patients: A) eliminate too much carbon dioxide. B) become hypercapneic and acidotic. C) experience an increase in blood pH. D) typically do not have an open airway.

B) become hypercapneic and acidotic

Appropriate management for a patient with severe epistaxis, tachycardia, and diaphoresis following an injury to the face includes: A) positioning the patient supine and elevating his or her legs 12 to 18 inches. B) administering enough IV crystalloid fluids to maintain adequate perfusion. C) pinching the patient's nares together and instructing him or her to lean back. D) applying direct pressure to the bridge of the nose and keeping the patient calm.

B) administering enough IV crystalloid fluids to maintain adequate perfusion.

14. Your assessment of an elderly man gives you reason to suspect that he has been abused. The patient does not appear to have any life-threatening injuries. His son, who is the only family member present at the scene, asks you if he can accompany his father in the back of the ambulance during transport. You should: A) ask the son why he wants to accompany his father in the ambulance. B) allow the son to accompany his father in the back of the ambulance. C) advise the son to follow the ambulance in his own personal vehicle. D) remain at the scene until law enforcement arrives and questions the son.

B) allow the son to accompany his father in the back of the ambulance.

46. In addition to IV fluids, treatment for a patient in neurogenic shock may include: A) atropine. B) amiodarone. C) epinephrine 1:1,000. D) low-dose dopamine.

B) amiodarone.

The glomerular filtration rate is defined as the: A) percentage of filtrate that remains in the kidneys. B) amount of filtrate produced by the kidneys per minute. C) volume of blood that flows through the kidneys per minute. D) pressure in the glomerulus that forces filtrate from the blood.

B) amount of filtrate produced by the kidneys per minute.

70. A characteristic sign of neurogenic shock caused by a spinal injury is: A) pink, warm, dry skin above the level of the injury. B) an absence of sweating below the level of the injury. C) a reflex tachycardia secondary to vascular dilation. D) irregular breathing secondary to brainstem insult.

B) an absence of sweating below the level of the injury.

53. The fistula used for hemodialysis is a surgical connection between: A) two large veins. B) an artery and a vein. C) two large arteries. D) a vein and the peritoneum.

B) an artery and a vein.

Following blunt trauma to the face, a 30-year-old man presents with epistaxis, double vision, and an inability to look upward. You should be MOST suspicious of: A) traumatic conjunctivitis. B) an orbital blowout fracture. C) traumatic retinal detachment. D) fracture of the cribriform plate.

B) an orbital blowout fracture.

88. Unlike sinus tachycardia, reentry supraventricular tachycardia in infants is characterized by: A) a presence of P waves. B) an unvarying pulse rate. C) a history of fever or dehydration. D) a pulse rate greater than 180 beats/min.

B) an unvarying pulse rate.

8. The posterior lobe of the pituitary gland secretes: A) thyroxine and growth hormone. B) antidiuretic hormone and oxytocin. C) three gonadotropic hormones and oxytocin. D) growth hormone and thyroid-stimulating hormone.

B) antidiuretic hormone and oxytocin.

When the solute concentration of the blood increases: A) water is excreted from the body by the kidneys. B) antidiuretic hormone is released into the bloodstream. C) the release of angiotensin II causes the blood pressure to fall. D) the distal convoluted tubule becomes less permeable to water.

B) antidiuretic hormone is released into the bloodstream.

60. A 26-year-old unrestrained woman struck her chest on the steering wheel when her car collided with another vehicle. After performing your primary assessment and administering high-flow oxygen, you assess her chest and note a segment of obviously fractured ribs that bulges outward during exhalation. You should: A) increase intrathoracic pressure with a demand valve. B) apply a bulky dressing to the segment of fractured ribs. C) position her on her injured side and monitor her breathing. D) apply pressure to the segment of ribs as the patient inhales.

B) apply a bulky dressing to the segment of fractured ribs.

70. A 17-year-old man jumped from a second-story balcony and landed on his feet. He complains of pain to both of his heels and knees. Your assessment reveals swelling and ecchymosis to both of his feet. His vital signs are stable and he is breathing without difficulty. In addition to caring for his lower-extremity injuries, it is MOST important that you: A) administer high-flow supplemental oxygen. B) apply spinal motion restriction precautions. C) try to determine why he jumped from the balcony. D) start a large-bore IV line of an isotonic crystalloid.

B) apply spinal motion restriction precautions.

A 52-year-old man sustained superficial and partial-thickness burns to his left arm approximately 15 minutes ago when he opened the radiator cap on his car. He is conscious, alert, and in severe pain. His BP is 138/76 mm Hg, pulse is 110 beats/min and strong, respirations are 22 breaths/min and regular, and oxygen saturation is 99% on room air. He denies any other injuries. Initial management for this patient involves: A) applying ice to the burn to provide immediate pain relief. B) applying cool, wet dressings to the burn and elevating his arm. C) starting an IV of normal saline and administering 2 mg of morphine. D) administering oxygen and applying an anesthetic cream to the burn.

B) applying cool, wet dressings to the burn and elevating his arm.

58. Nearly half of all birth-related injuries: A) result in varying degrees of central nervous system dysfunction. B) are potentially avoidable with anticipation of obstetric risk factors. C) cause death of the newborn secondary to prolonged cerebral anoxia. D) are associated with negligence on the part of the health care provider.

B) are potentially avoidable with anticipation of obstetric risk factors

41. Diffuse rales, rhonchi, and wheezing in an infant: A) can usually be heard without a stethoscope. B) are typical signs of lower airway inflammation. C) suggest swelling of the supraglottic structures. D) are signs of acute asthma until proven otherwise.

B) are typical signs of lower airway inflammation.

Patients requiring chronic dialysis: A) are typically not able to ambulate. B) are usually dialyzed every 2 or 3 days. C) are almost always inpatients in a hospital. D) stay on the dialysis machine for 6 to 8 hours.

B) are usually dialyzed every 2 or 3 days.

30. Afterload is increased following alpha-1 stimulation because of: A) decreased preload. B) arteriolar constriction. C) increased contractility. D) widespread vasodilation..

B) arteriolar constriction.

During your primary assessment of a 21-year-old man with a suspected inhalation injury, you note that he is combative and his respirations are profoundly labored and stridorous. The closest appropriate medical facility is approximately 25 miles by ground, and the local air transport service is unavailable. You should: A) provide supplemental oxygen via nonrebreathing mask, insert an intraosseous catheter, and administer a sedative medication. B) assist ventilations with a bag-mask device, start an IV, administer a sedative and a neuromuscular blocker, and intubate his trachea. C) administer humidified oxygen, start at least one large-bore IV, and visualize his upper airway to assess the severity of soft-tissue swelling. D) insert an oropharyngeal airway, ventilate him with a bag-mask device at 20 breaths/min, and prepare to nasotracheally intubate him.

B) assist ventilations with a bag-mask device, start an IV, administer a sedative and a neuromuscular blocker, and intubate his trachea.

43. The primary role of cortisol is to: A) maintain an adequate blood pressure. B) assist with the body's response to stress. C) regulate the metabolism of carbohydrates. D) decrease the body's inflammatory response.

B) assist with the body's response to stress.

34. Tracheobronchial injuries have a high mortality due to: A) massive internal hemorrhage. B) associated airway obstruction. C) perforation of the esophagus. D) concomitant spinal cord injury.

B) associated airway obstruction.

67. If a woman who is 35 weeks pregnant is placed in a supine position: A) a paradoxical bradycardia will occur. B) atrial preload may be reduced markedly. C) aortic compression will cause hypotension. D) uterine circulation will increase exponentially.

B) atrial preload may be reduced markedly.

94. When attempting resuscitation of a child with pulseless electrical activity, you should: A) administer epinephrine via the ET tube if possible. B) attempt to identify an underlying cause of the arrest. C) perform synchronized cardioversion if the rate is fast. D) give atropine if the heart rate is less than 60 beats/min.

B) attempt to identify an underlying cause of the arrest.

You receive a call to a residence for a 60-year-old man who is bleeding from his dialysis shunt. When you arrive, the patient's wife, who has been properly trained on the use of the dialysis machine, tells you that she panicked and called EMS. The dialysis cannula has loosened from the needle, which is still in the shunt. Your initial action should be to: A) immediately clamp off the cannula and apply direct pressure. B) attempt to tighten the connection between the needle and cannula. C) remove the dialysis needle from the shunt and apply direct pressure. D) apply direct pressure over the shunt and carefully remove the needle.

B) attempt to tighten the connection between the needle and cannula.

A 74-year-old man experienced partial- and full-thickness burns to his arms and chest resulting from a fire that started after he fell asleep while smoking his cigar. The patient's son, who arrived at the scene shortly after you, states that his father has congestive heart failure, rheumatoid arthritis, and atrial fibrillation. In addition to administering supplemental oxygen, it is MOST important for you to: A) avoid narcotic analgesics because of his medical history. B) auscultate his breath sounds before administering IV fluids. C) obtain a 12-lead ECG to assess for signs of cardiac ischemia. D) apply cold, moist dressings to his burns to provide pain relief.

B) auscultate his breath sounds before administering IV fluids.

148. A 10-month-old infant presents with an acute onset of increased work of breathing. According to the infant's mother, the child was crawling around in the living room prior to the event and was fine 10 minutes earlier. Your assessment reveals that the infant appears alert to his surroundings, has loud inspiratory stridor, and pink skin. You should: A) look inside the infant's mouth using a tongue blade and penlight. B) avoid agitating the infant, offer supplemental oxygen, and transport. C) deliver five sharp back slaps between the infant's shoulder blades. D) apply a pediatric nonrebreathing mask and transport expeditiously.

B) avoid agitating the infant, offer supplemental oxygen, and transport.

43. When assessing a multisystems trauma patient, it is MOST important to: A) administer an analgesic promptly to minimize pain. B) avoid being distracted by visually impressive injuries. C) splint swollen, painful extremities as soon as possible. D) only splint fractures involving long bones at the scene.

B) avoid being distracted by visually impressive injuries.

130. When mechanically securing an injured child's head and neck to a backboard, you should: A) place padding underneath the occiput. B) avoid placing a strap or tape over the chin. C) use towel rolls instead of a cervical collar. D) manually stabilize the child's torso first.

B) avoid placing a strap or tape over the chin.

136. If a child with a functioning central venous line requires emergency drug therapy, you should: A) administer the drug through the central line, but only give half the usual dose of the drug. B) avoid using the central line if possible and attempt to establish peripheral IV access elsewhere. C) carefully cleanse the injection port on the central line and administer the drug in the usual fashion. D) flush the central line with at least 30 mL of normal saline first and then administer the emergency drug.

B) avoid using the central line if possible and attempt to establish peripheral IV access elsewhere.

153. A 12-year-old boy presents with marked respiratory distress; hot, moist skin; and anxiety. He is sitting with his chin thrust forward and has inspiratory stridor. According to the child's grandmother, his symptoms began suddenly about 30 minutes ago. You should be MOST suspicious for: A) acute viral croup. B) bacterial epiglottitis. C) subglottic narrowing. D) laryngotracheobronchitis.

B) bacterial epiglottitis.

15. In a person who is not bedridden, most pulmonary infections occur in the: A) middle lobes of the lungs. B) bases of the lungs. C) apices of the lungs. D) upper portion of the lungs.

B) bases of the lungs.

45. Abnormal breath sounds associated with pneumonia and congestive heart failure are MOST often heard in the: A) right middle lobe. B) bases of the lungs. C) apices of the lungs. D) midaxillary line.

B) bases of the lungs.

When assessing a burn patient, it is MOST important to: A) accurately calculate the extent of body surface area burned. B) be alert for occult trauma that could affect patient outcome. C) apprise medical control of the situation as soon as possible. D) rapidly determine if the patient will require an escharotomy.

B) be alert for occult trauma that could affect patient outcome.

65. After performing the initial steps of resuscitation, you assess a newborn and note that its respirations are poor and its pulse rate is 50 beats/min. You should: A) immediately begin positive-pressure ventilations and chest compressions and then reassess the newborn's pulse rate in 30 seconds. B) begin chest compressions if the heart rate remains below 60 beats/min after 30 seconds of effective positive-pressure ventilation. C) begin chest compressions, insert an endotracheal tube, and administer 0.1 to 0.3 mL/kg of epinephrine 1:10,000 down the endotracheal tube. D) perform tactile stimulation for 30 seconds, reassess the infant's respirations and pulse rate, and begin positive-pressure ventilations if there is no improvement.

B) begin chest compressions if the heart rate remains below 60 beats/min after 30 seconds of effective positive-pressure ventilation.

55. Most patients with an aortic injury will complain of pain: A) while taking a shallow breath. B) behind the sternum or in the scapula. C) in the region of the posterior pharynx. D) that radiates from the chest to the flank.

B) behind the sternum or in the scapula.

27. Signs of a diaphragmatic hernia include all of the following, EXCEPT: A) a scaphoid or concave abdomen. B) bilaterally absent breath sounds. C) noted increased work of breathing. D) audible bowel sounds in the chest.

B) bilaterally absent breath sounds

Hyphema is defined as: A) severe ecchymosis to the orbital region. B) blood in the anterior chamber of the eye. C) marked swelling of the globe of the eye. D) double vision following blunt eye trauma.

B) blood in the anterior chamber of the eye.

Loss of function of the lower arms and hands following trauma to the anterior neck is indicative of damage to the: A) carotid artery. B) brachial plexus. C) vagus nerves. D) parathyroid glands.

B) brachial plexus.

47. Inspiratory and expiratory__________ sounds are both loud, but the inspiratory sounds are shorter than the expiratory sounds. A) tracheal B) bronchial C) vesicular D) bronchovesicular

B) bronchial

10. The mainstem bronchus ends at the level of the: A) lobar bronchi. B) bronchioles. C) segmental bronchi. D) subsegmental bronchi.

B) bronchioles.

10. Secretion of the parathyroid hormone is regulated by blood levels of: A) sodium. B) calcium. C) potassium. D) phosphorus.

B) calcium.

113. Sorbitol is not recommended for use in young children because it: A) induces vomiting, which increases the risk for pulmonary aspiration. B) can cause severe diarrhea and life-threatening electrolyte abnormalities. C) has been linked to sudden cardiac death due to ventricular dysrhythmias. D) prolongs the QT interval and is associated with ventricular fibrillation.

B) can cause severe diarrhea and life-threatening electrolyte abnormalities.

23. Implantation of a fertilized egg within a fallopian tube: A) produces atypical signs of pregnancy. B) can cause severe intra-abdominal hemorrhage. C) represents only 3% of all ectopic pregnancies. D) is usually detected after the 20th week of pregnancy.

B) can cause severe intra-abdominal hemorrhage.

62. Intrinsic causes of cardiogenic shock include: A) pleural effusion. B) cardiomyopathy. C) pulmonary embolus. D) tension pneumothorax.

B) cardiomyopathy.

45. When an adult patient with hemorrhagic shock loses more than 40% of his or her blood volume: A) blood flow is diverted away from the skin to the liver, kidneys, and lungs. B) cardiovascular deterioration cannot be reversed by compensatory mechanisms. C) vital organ damage can be repaired if a blood transfusion is started promptly. D) immediate IV fluid replacement can rapidly restore adequate tissue perfusion.

B) cardiovascular deterioration cannot be reversed by compensatory mechanisms.

13. Tissue perfusion is primarily a function of the: A) endocrine system. B) cardiovascular system. C) respiratory system. D) peripheral nervous system.

B) cardiovascular system.

59. Hydrocephalus may occur if: A) too much cerebrospinal fluid is forced into the spinal canal. B) cerebrospinal fluid absorption into the bloodstream is reduced. C) bleeding in the brain causes an increase in intracranial pressure. D) the brain produces less than 500 mL of cerebrospinal fluid per day.

B) cerebrospinal fluid absorption into the bloodstream is reduced.

60. COPD is characterized by: A) narrowing of the smaller airways that is often reversible with prompt treatment. B) changes in pulmonary structure and function that are progressive and irreversible. C) small airway spasms during the inhalation phase, resulting in progressive hypoxia. D) widespread alveolar collapse due to increased pressure during the exhalation phase.

B) changes in pulmonary structure and function that are progressive and irreversible.

A superficial burn is: A) usually painless because the nerve endings are not exposed. B) characterized by reddened skin with varying degrees of pain. C) painful, but will heal spontaneously, often with scar formation. D) a second-degree burn that is characterized by blister formation

B) characterized by reddened skin with varying degrees of pain.

82. While transporting a cancer patient who is receiving chemotherapy, you should recall that: A) it is likely that the patient's condition is terminal. B) chemotherapy weakens a patient's immune system. C) prophylactic antiemetic medications are often needed. D) chemotherapy is only used to treat metastatic cancer.

B) chemotherapy weakens a patient's immune system.

67. Emphysema is caused by: A) an abundance of pulmonary surfactant. B) chronic destruction of the alveolar walls. C) excessive mucus production in the bronchi. D) progressive weakening of the lung parenchyma.

B) chronic destruction of the alveolar walls.

43. Digital clubbing is MOST indicative of: A) acute hypoxemia. B) chronic hypoxia. C) right heart failure. D) peripheral vascular disease.

B) chronic hypoxia.

36. The BEST way to detect deformity or any other abnormality in an injured extremity is to: A) gently palpate the entire length of the extremity. B) compare it to the extremity on the opposite side. C) manipulate the extremity to assess for false motion. D) reduce any swelling with ice before performing the exam.

B) compare it to the extremity on the opposite side.

47. When assessing distal pulses in a patient with a lower extremity injury, it is MOST important to: A) suspect severe shock if a unilateral pulse deficit is present. B) compare the strength of the pulses in both lower extremities. C) remember that most people do not have a palpable pedal pulse. D) count the pedal pulse rate and compare it to the radial pulse rate.

B) compare the strength of the pulses in both lower extremities.

14. The endocrine component of the pancreas: A) comprises the pancreatic duct. B) comprises the islets of Langerhans. C) releases epinephrine and norepinephrine. D) secretes digestive enzymes into the duodenum.

B) comprises the islets of Langerhans.

80. Failure of a child's SpO2 to increase despite high-flow oxygen is MOST indicative of: A) relative hypovolemia. B) congenital heart disease. C) right-sided heart failure. D) decreased vascular tone.

B) congenital heart disease.

7. The outer layer of the fallopian tube: A) is made of smooth muscle that contracts and relaxes. B) consists of a serous membrane that protects the tube. C) is where fertilization of the ovum typically occurs. D) contains secretory cells and cilia that help move the ovum.

B) consists of a serous membrane that protects the tube.

68. Premonitory signs of labor include all of the following, EXCEPT: A) vaginal expulsion of the mucous plug. B) contractions at 5- to 15-minute intervals. C) a relief of pressure in the upper abdomen. D) an increase in pressure in the pelvic area

B) contractions at 5- to 15-minute intervals.

Patients suspected of having burns to the upper airway will benefit MOST from: A) unhumidified oxygen. B) cool, humidified oxygen. C) an inhaled beta-2 agonist. D) anticholinergic bronchodilators.

B) cool, humidified oxygen.

A burn caused by a sodium metal should be treated by: A) administering calcium chloride. B) covering the burn wound with oil. C) applying a moist, sterile dressing. D) irrigating the wound with water.

B) covering the burn wound with oil.

During an explosion, a 42-year-old construction worker sustained a large laceration to the lateral aspect of his neck when he was struck by a piece of flying debris. The patient is conscious, but complains of difficulty hearing. In addition to protecting his spine, you should be MOST concerned with: A) administering high-flow oxygen via nonrebreathing mask as soon as possible. B) covering the laceration with an occlusive dressing and controlling the bleeding. C) carefully examining his ear to determine if his tympanic membrane is ruptured. D) applying a bulky dressing to the laceration and securing it firmly with a bandage.

B) covering the laceration with an occlusive dressing and controlling the bleeding.

79. When auscultating the lungs of a patient with early pulmonary edema, you will MOST likely hear: A) inspiratory rhonchi to the bilateral apices of the lungs. B) crackles in the bases of the lungs at the end of inspiration. C) faint rhonchi to all lung fields on inspiration and expiration. D) coarse crackles to the bases of the lungs during inspiration.

B) crackles in the bases of the lungs at the end of inspiration.

17. Some patients with a terminal illness will continue aggressive medical treatment, hoping for a statistically improbable recovery or attempting to prolong life as much as possible. This is called: A) hospice care. B) curative care. C) palliative care. D) investigational care.

B) curative care.

29. Dysrhythmias following a myocardial contusion are usually secondary to: A) excess tachycardia that accompanies the injury. B) damage to myocardial tissue at the cellular level. C) aneurysm formation caused by vascular damage. D) direct damage to the vasculature of the epicardium.

B) damage to myocardial tissue at the cellular level.

162. Your primary assessment of an unresponsive 5-year-old, 40-pound child reveals that he is apneic and pulseless. CPR is initiated and the cardiac monitor is applied, which reveals ventricular fibrillation. You should: A) continue high-quality CPR and reassess in 2 minutes. B) defibrillate with 40 joules and immediately resume CPR. C) start an IV and administer 0.2 mg of epinephrine 1:10,000. D) charge the defibrillator to 80 joules while CPR is ongoing.

B) defibrillate with 40 joules and immediately resume CPR.

32. Several attempts to clear a plugged tracheostomy tube with suction have failed. The patient, who is on a mechanical ventilator, has a pulse rate of 150 beats/min and is making exaggerated attempts to breathe. You should: A) remove the tracheostomy tube, place a mask over the stoma, and ventilate with a bag-mask device. B) deflate the cuff of the tracheostomy tube and ventilate the patient in the usual fashion with a bag-mask device. C) administer high-flow oxygen via nonrebreathing mask as you prepare to replace the tracheostomy tube. D) provide free-flow oxygen as you remove the tracheostomy tube and replace it with a similarly sized endotracheal tube.

B) deflate the cuff of the tracheostomy tube and ventilate the patient in the usual fashion with a bag-mask device.

37. A patient with diabetic ketoacidosis experiences polydipsia as a result of: A) hyperglycemia. B) dehydration. C) metabolic acidosis. D) inefficient nutrient utilization.

B) dehydration.

1. When determining the optimal method to communicate with, assess, treat, and transport a patient with a special health care challenge, the paramedic should: A) routinely call medical control before talking to a caregiver. B) demonstrate confidence and enlist the expertise of the patient. C) recall that caregivers are often upset and therefore unreliable. D) remain professional and obtain initial information from a caregiver.

B) demonstrate confidence and enlist the expertise of the patient.

140. You are dispatched to a residence for a 17-year-old woman with acute abdominal pain. When you arrive and begin your assessment, it is clear that the patient is uncomfortable with the presence of her parents because she is reluctant to answer your questions. You should: A) recognize that the parents are an invaluable resource for information. B) diplomatically ask the parents if their daughter can have some privacy. C) tell the patient that her parents must legally be present during the exam. D) reassure the patient and tell her that her candor is vital to your treatment.

B) diplomatically ask the parents if their daughter can have some privacy.

50. Unlike the insertion of an indwelling urinary catheter, insertion of an intermittent (straight) urinary catheter: A) is necessary when the patient is bedridden for prolonged periods. B) does not involve inflation of a balloon to hold the catheter in place. C) is associated with a greater risk for damage to the urinary sphincter. D) generally does not require catheter lubrication with a water-soluble gel.

B) does not involve inflation of a balloon to hold the catheter in place.

26. Common symptoms of type 2 diabetes include all of the following, EXCEPT: A) thirst. B) dysuria. C) fatigue. D) blurred vision.

B) dysuria.

65. The transducer or drainage system of an intracranial pressure monitor is typically aligned at the same height as the patient's: A) forehead. B) ear canal. C) eyebrow. D) temporal bone.

B) ear canal.

53. The Trendelenburg position is accomplished by: A) keeping the legs below the level of the heart. B) elevating the foot end of the backboard 6 to 12 inches. C) elevating the patient's torso to a 45-degree angle. D) placing the patient on his or her side and elevating the legs.

B) elevating the foot end of the backboard 6 to 12 inches.

41. Within a few days after having a therapeutic abortion, a 33-year-old woman presents with malaise, fever, constipation, and pelvic pain. Her blood pressure is 124/84 mm Hg, pulse rate is 104 beats/min and strong, and respirations are 22 breaths/min and regular. You should be MOST suspicious for: A) acute cystitis. B) endometritis. C) a ruptured ovarian cyst. D) pelvic inflammatory disease.

B) endometritis.

31. Prior to administering 50% dextrose (D50) via IV push, it is MOST important to: A) protect the airway with an endotracheal tube. B) ensure that the IV line is patent and freely flowing. C) confirm a blood glucose reading of less than 40 mg/dL. D) draw blood for later analysis in the emergency department.

B) ensure that the IV line is patent and freely flowing.

83. The preferred initial pharmacologic agent for pediatric bradycardia is: A) atropine. B) epinephrine. C) dobutamine. D) amiodarone.

B) epinephrine.

93. Treatment for pediatric asystole includes: A) atropine. B) epinephrine. C) cardiac pacing. D) hyperventilation.

B) epinephrine.

159. A 6-year-old girl who has been running a fever for the past 2 days presents with lethargy and tachycardia. Her heart rate is 170 beats/min and varies with activity. Her skin is cool and clammy, and her capillary refill time is 4 seconds. The cardiac monitor reveals a narrow complex tachycardia with a rate that varies between 150 and 170 beats/min. After applying high-flow oxygen, you should: A) apply chemical ice packs to the child's face to try to slow her heart rate. B) establish vascular access and administer a 20-mL/kg normal saline bolus. C) start an IV line and give adenosine while monitoring her cardiac rhythm. D) transport immediately and establish vascular access en route to the hospital.

B) establish vascular access and administer a 20-mL/kg normal saline bolus.

12. During the proliferative phase: A) fertilization of the egg occurs in the distal third of the fallopian tube within 24 hours following ovulation. B) estrogen stimulates the endometrium to increase in thickness in preparation for the reception and future growth of a fertilized egg. C) the stratus functionalis of the endometrium is shed during menstruation when implantation of a fertilized egg does not occur. D) the middle, muscular layer of the fallopian tube propels a fertilized egg toward the uterus where it implants on the uterine wall.

B) estrogen stimulates the endometrium to increase in thickness in preparation for the reception and future growth of a fertilized egg.

The majority of victims struck by lightning: A) die within the first 24 hours following the injury. B) experience confusion and some degree of amnesia. C) require only short periods of ventilatory support. D) are permanently paralyzed due to a spinal cord injury

B) experience confusion and some degree of amnesia.

56. A 51-year-old man with type 2 diabetes presents with confusion, blurred vision, and signs of significant dehydration. According to the man's wife, he has had a fever and flu-like symptoms for the past few days. She further tells you that he has "stuck to his diet" as advised by his physician. His blood pressure is 90/50 mm Hg, pulse is 120 beats/min and weak, and respirations are rapid and shallow. You assess his blood glucose level, which reads "high." This patient is MOST likely: A) producing ketones due to fat metabolism. B) experiencing hyperosmolar nonketotic coma. C) significantly acidotic and requires bicarbonate. D) hyperglycemic secondary to being dehydrated.

B) experiencing hyperosmolar nonketotic coma.

52. The FIRST step in splinting a musculoskeletal injury involves: A) thoroughly assessing range of motion. B) exposing and assessing the injury site. C) assessing distal neurovascular functions. D) straightening the injury if it is angulated.

B) exposing and assessing the injury site.

If intubation of a burn patient becomes necessary, you should avoid cutting the ET tube down to make it shorter because: A) doing so increases the risk of intubating the right mainstem bronchus. B) facial edema may cause tube dislodgement 2 to 3 days after the burn. C) drugs given via the ET tube will not adequately disperse in the lungs. D) it may result in excessive volumes of air being delivered to the patient.

B) facial edema may cause tube dislodgement 2 to 3 days after the burn.

7. The carpals, especially the scaphoid, are MOST vulnerable to fracture when a person: A) experiences a crush injury. B) falls on an outstretched hand. C) also dislocates his or her elbow. D) excessively supinates the hand

B) falls on an outstretched hand.

75. By placing one hand on your forehead and your other hand over your abdomen, you are asking a hearing-impaired person if he or she: A) is hurt. B) feels sick. C) needs help. D) is in pain.

B) feels sick.

58. When assessing the abdomen of a woman who is 36 weeks pregnant, you should expect it to be: A) rigid and distended. B) firm and nontender. C) enlarged and tender. D) distended and guarded.

B) firm and nontender.

The appropriate treatment for MOST chemical burns is: A) application of a dry, sterile dressing. B) flushing with copious amounts of water. C) neutralization with an alkaline substance. D) application of a moist, sterile dressing.

B) flushing with copious amounts of water.

154. A 13-year-old, 40-pound girl is experiencing an acute asthma attack that has been unresponsive to 3 puffs of her albuterol inhaler. She is conscious and alert, but is notably dyspneic and has diffuse wheezing. In addition to administering supplemental oxygen, you should: A) give 0.35 mg of epinephrine 1:1,000 SQ. B) give 0.5 mg of nebulized ipratropium. C) administer another 2.5-mg dose of albuterol. D) assist her ventilations with a bag-mask device.

B) give 0.5 mg of nebulized ipratropium.

19. When the pancreas does not produce enough insulin or the cells do not respond to the effects of the insulin that is produced: A) the cells will metabolize oxygen and function normally. B) glucose levels in the blood and urine will be elevated. C) serum glucose levels will fall and brain damage may occur. D) the body will stop making glucose as a protective mechanism.

B) glucose levels in the blood and urine will be elevated.

26. The MOST common underlying cause of a tubo-ovarian abscess is: A) vaginitis. B) gonorrhea. C) an ectopic pregnancy. D) a ruptured ovarian cyst.

B) gonorrhea.

27. The inner cannula of a tracheostomy tube: A) should be equipped with a cuff if used in pediatric patients. B) has a 15-mm port that can be attached to a ventilator circuit. C) is the larger tube that passes directly into the patient's trachea. D) should only be removed during tracheostomy tube replacement.

B) has a 15-mm port that can be attached to a ventilator circuit.

54. A nulliparous woman: A) has never been pregnant. B) has never delivered a baby. C) has never had a miscarriage. D) has had at least one miscarriage.

B) has never delivered a baby.

49. Women who experience hyperemesis gravidarum: A) have been pregnant at least two times and are commonly underweight. B) have persistent nausea and vomiting throughout pregnancy that often causes dehydration and malnutrition. C) vomit once or twice a day, usually during the morning hours, and often develop hyperglycemia as a result. D) develop the condition secondary to increased progesterone and decreased human chorionic gonadotropin levels.

B) have persistent nausea and vomiting throughout pregnancy that often causes dehydration and malnutrition.

1. In contrast to adults, children: A) land on their feet when they fall. B) have proportionately larger heads. C) experience head injury less frequently. D) lose most body heat through the chest.

B) have proportionately larger heads

14. Establishing good rapport with the caregiver of a sick or injured child at the scene is vital because: A) caregivers often take their anger out on prehospital professionals. B) he or she will be a source of important information and assistance. C) doing so will quickly deescalate any hostility that he or she may have. D) the caregiver generally will not accompany the child in the ambulance.

B) he or she will be a source of important information and assistance.

52. Due to intrapulmonary hemorrhage, patients with a pulmonary contusion may present with: A) hypocarbia. B) hemoptysis. C) hematemesis. D) hematochezia.

B) hemoptysis.

30. The MOST significant immediate risk associated with an open fracture is: A) infection. B) hemorrhage. C) nerve damage. D) bone marrow loss.

B) hemorrhage.

89. The MOST appropriate vagal maneuver for an infant involves: A) blowing into an occluded straw. B) holding ice packs firmly to the face. C) firmly massaging the carotid artery. D) applying a heat stimulus to the body.

B) holding ice packs firmly to the face.

42. You would NOT expect a patient with a flail chest to present with: A) cyanosis. B) hyperpnea. C) shallow breathing. D) decreased breath sounds.

B) hyperpnea.

20. Microvascular complications of diabetes include all of the following, EXCEPT: A) retinopathy. B) hypertension. C) nephropathy. D) neuropathy.

B) hypertension.

42. A decrease in adrenal hormone production will result in all of the following, EXCEPT: A) weakness. B) hypertension. C) dehydration. D) inability to respond to stress.

B) hypertension.

49. A patient with Cushing syndrome would MOST likely present with: A) ketoacidosis. B) hypoglycemia. C) decreased urination. D) acute hyperactivity.

B) hypoglycemia.

Bradycardia and hypotension following an overaggressive dialysis treatment are MOST indicative of: A) hypovolemia. B) hypokalemia. C) hyperkalemia. D) air embolism.

B) hypokalemia.

46. The single MOST common cause of seizures in both term and preterm infants is: A) intracranial hemorrhaging. B) hypoxic ischemic encephalopathy. C) congenital or developmental defects. D) a severe derangement in electrolytes

B) hypoxic ischemic encephalopathy.

The secondary assessment of a severely burned patient is intended to: A) provide for a rapid means of assessing the patient for occult injuries. B) identify other injuries that may have a higher priority for treatment. C) focus on areas of the body that have sustained the most serious burns. D) locate and treat minor injuries after all serious injuries have been treated.

B) identify other injuries that may have a higher priority for treatment.

64. A young man has an isolated injury to his left lower leg. Your assessment reveals obvious deformity and ecchymosis. Distal circulation as well as motor and sensory functions are grossly intact, and the patient is hemodynamically stable. In addition to stabilizing the suspected fracture site, you should: A) carefully palpate to elicit crepitus. B) immobilize the knee and the ankle. C) apply a traction splint for pain relief. D) place a chemical heat pack over the injury.

B) immobilize the knee and the ankle.

23. The MOST common reasons for ineffective bag-mask ventilations in the newborn are: A) equipment malfunction and a ventilation rate that is too rapid. B) inadequate mask-to-face seal and incorrect head position. C) hyperflexion of the newborn's head and thick mucous plugs. D) pneumothorax and a face mask that is too large for the infant.

B) inadequate mask-to-face seal and incorrect head position.

29. Gestational diabetes is caused by: A) a decreased production of insulin secondary to excess estrogen levels. B) increased insulin production and decreased cellular sensitivity to insulin. C) decreased cellular utilization of insulin secondary to increased estrogen. D) ketones that are excreted in the urine during the course of the pregnancy.

B) increased insulin production and decreased cellular sensitivity to insulin.

98. During your visual exam of a woman in active labor, you see a loop of the umbilical cord protruding from her vagina. She is in the middle of a strong contraction, and you can see the baby's head just beyond the cord. You should immediately: A) place her on 100% supplemental oxygen. B) instruct her to pant during the contraction. C) push the baby's head back into the vagina. D) apply sterile, moist dressings over the cord.

B) instruct her to pant during the contraction.

38. Respiratory distress in children: A) represents the end result of prolonged hypoxia and indicates impending cardiopulmonary failure. B) is a compensated state in which increased work of breathing results in adequate pulmonary gas exchange. C) is associated with a decreased level of consciousness, abnormally slow respirations, and weak muscle retractions. D) is characterized by prominent use of the sternocleidomastoid muscles in infants and children younger than 2 years of age.

B) is a compensated state in which increased work of breathing results in adequate pulmonary gas exchange.

46. Jugular venous distention during a tension pneumothorax: A) indicates a significant increase in atrial preload. B) is caused by blood accumulation in the vena cava. C) occurs before a unilateral absence of breath sounds. D) manifests early as air accumulates in the pleural space.

B) is caused by blood accumulation in the vena cava.

30. Thirty minutes of hypoglycemia in a patient: A) will generally not result in permanent neurologic damage. B) is more dangerous than an equivalent period of hyperglycemia. C) is only life threatening if the blood glucose is less than 40 mg/dL. D) is less dangerous than an equivalent period of hyperglycemia.

B) is more dangerous than an equivalent period of hyperglycemia.

86. Intubation of a patient with severe asthma: A) is clearly indicated if the patient's condition does not resolve following field corticosteroid therapy. B) is often a last resort because asthmatics are difficult to ventilate and are prone to pneumothoraces. C) should only be performed after hyperventilating the patient with a bag-mask device for 2 to 3 minutes. D) is generally contraindicated because weaning the patient off of a ventilator can take several days.

B) is often a last resort because asthmatics are difficult to ventilate and are prone to pneumothoraces.

Because significant force is required to fracture the mandible: A) most mandibular fractures are associated with a spinal fracture. B) it is often fractured in more than one place and is unstable to palpation. C) patients with a possible mandibular fracture should be intubated routinely. D) a mandibular fracture can be ruled out in cases of minor blunt facial trauma.

B) it is often fractured in more than one place and is unstable to palpation.

28. When evaluating a child's oxygen saturation level with a pulse oximeter: A) you should recall that peripheral vasodilation from a warm environment will typically yield a false reading. B) it should be evaluated in the context of the pediatric assessment triangle and remainder of the primary assessment. C) you should provide ventilatory assistance with a bag-mask device if the reading is below 94% and not increasing rapidly. D) a reading of less than 96% on room air indicates respiratory distress and necessitates the administration of supplemental oxygen.

B) it should be evaluated in the context of the pediatric assessment triangle and remainder of the primary assessment.

The skin of a patient with chronic renal failure is MOST often: A) flushed. B) jaundiced. C) cyanotic. D) cool and dry.

B) jaundiced.

51. The female urinary meatus is located: A) superior to the clitoris. B) just above the vaginal opening. C) inferior to the vaginal opening. D) between the vagina and perineum.

B) just above the vaginal opening.

You are transporting a conscious middle-aged man with anterior neck trauma. He is on high-flow oxygen, has spinal precautions in place, and has a large-bore IV line of normal saline in place. When you reassess his vital signs, you note that his blood pressure is 90/64 mm Hg, his pulse rate is 120 beats/min, and his respirations are 22 breaths/min with adequate depth. You should: A) begin assisting his ventilations with a bag-mask device, rapidly infuse 2 L of IV fluid, and reassess. B) keep the patient warm and infuse enough isotonic crystalloid solution to maintain adequate perfusion. C) elevate his legs, apply a blanket, and administer IV fluid boluses until his heart rate is within a normal range. D) start a second IV line and administer crystalloid IV fluids until his systolic blood pressure is at least 100 mm Hg.

B) keep the patient warm and infuse enough isotonic crystalloid solution to maintain adequate perfusion.

80. When caring for a prolapsed umbilical cord, you should: A) position the mother in a left lateral recumbent position with her knees flexed into her abdomen. B) keep the presenting part of the baby off the umbilical cord during rapid transport to the hospital. C) have your partner cover the exposed portion of the umbilical cord with dry, sterile dressings. D) instruct the mother to push during each contraction to facilitate passage of the baby past the cord.

B) keep the presenting part of the baby off the umbilical cord during rapid transport to the hospital.

53. When present at low levels, oxygen binds easily to hemoglobin molecules, resulting in: A) small changes in oxygen saturation when large changes in PaO2 occur. B) large changes in oxygen saturation when small changes in PaO2 occur. C) small changes in oxygen saturation when small changes in PaO2 occur. D) large changes in oxygen saturation when large changes in PaO2 occur.

B) large changes in oxygen saturation when small changes in PaO2 occur.

54. Crackles or rales in the lungs following a myocardial contusion would MOST likely result from: A) intrapulmonary hemorrhage. B) left ventricular dysfunction. C) decreased right atrial preload. D) pulmonary vein disruption.

B) left ventricular dysfunction.

78. Uncontrollable coughing and hemoptysis in a cigarette smoker are clinical findings MOST consistent with: A) emphysema. B) lung cancer. C) pleural effusion. D) acute bronchitis.

B) lung cancer.

65. The use of a straight blade during pediatric intubation: A) is generally reserved for neonates only. B) makes it easier to manipulate the epiglottis. C) is associated with a higher risk of bradycardia. D) facilitates laryngoscopy by lifting the vallecula.

B) makes it easier to manipulate the epiglottis.

Unlike chemical burns, radiation burns: A) generally extend into the dermal layer. B) may appear hours or days after exposure. C) are typically confined to the epidermis. D) are immediately apparent after exposure.

B) may appear hours or days after exposure.

61. Proficiency in ventilating apneic infants or children with a bag-mask device: A) cannot be achieved by practicing on a manikin. B) may avert the need for endotracheal intubation. C) is more important for paramedics than EMTs. D) is difficult because their faces are much smaller.

B) may avert the need for endotracheal intubation.

33. A patient with respiratory distress who is willing to lie flat: A) should be intubated at once. B) may be acutely deteriorating. C) has minimal fluid in the lungs. D) likely has basilar pneumonia.

B) may be acutely deteriorating.

19. The self-splinting effect observed in patients with chest wall trauma: A) allows the body to compensate for the injury. B) may cause atelectasis, hypoxemia, or pneumonia. C) is often accompanied by subcutaneous emphysema. D) is characterized by a markedly increased tidal volume.

B) may cause atelectasis, hypoxemia, or pneumonia.

35. Eliciting for crepitus during your exam of a deformed extremity: A) should only be performed if the patient is not in significant pain. B) may cause further injury to the bone and surrounding soft tissues. C) should be performed to help confirm the presence of a fracture. D) is generally not performed, but will likely not cause further injury.

B) may cause further injury to the bone and surrounding soft tissues.

Full-thickness circumferential burns to the chest: A) require the paramedic to incise the burn to decompress it. B) may cause significant restriction of respiratory excursion. C) are generally not significant unless the skin is unyielding. D) necessitate immediate intubation and ventilatory support.

B) may cause significant restriction of respiratory excursion.

72. Patients with decompensated asthma or COPD who require positive-pressure ventilation: A) should be ventilated routinely at a rate that is slightly faster than the rate for a patient without an underlying pulmonary disease. B) may develop a pneumothorax or experience a decrease in venous return to the heart if they are ventilated too rapidly. C) should be intubated promptly and ventilated at a rate of 20 to 24 breaths/min to eliminate excess carbon dioxide. D) should be given forceful positive-pressure breaths because their primary problem is difficulty with inhalation.

B) may develop a pneumothorax or experience a decrease in venous return to the heart if they are ventilated too rapidly.

26. Counting an infant's respiratory rate for 15 seconds and then quadrupling that number: A) is recommended because it is the quickest way to determine if the infant's baseline respiratory rate is abnormally slow or abnormally fast. B) may yield a falsely low respiratory rate because infants may have periodic breathing or variable respiratory rates with short periods of apnea. C) is impractical because the inherent respiratory rate of an infant is usually rapid and counting for such a short period of time leaves room for error. D) is appropriate only if you are auscultating the child's respirations with a stethoscope while simultaneously listening to lung sounds.

B) may yield a falsely low respiratory rate because infants may have periodic breathing or variable respiratory rates with short periods of apnea.

24. Endotracheal intubation is clearly indicated in the newborn if: A) its heart rate is improving, but only because of adequate ventilations and chest compressions. B) meconium is present in the amniotic fluid and the newborn is limp and has a heart rate of 70 beats/min. C) central cyanosis is persistent despite the administration of free-flow oxygen for 30 to 45 seconds. D) a small, 27-gauge IV line is present and epinephrine is required to treat refractory bradycardia.

B) meconium is present in the amniotic fluid and the newborn is limp and has a heart rate of 70 beats/min

The renal _______ is the middle layer of the kidney and includes the renal pyramids. A) cortex B) medulla C) calyces D) capsule

B) medulla

7. The onset of menses is called ___________, and typically occurs between the ages of ___________. A) menorrhea, 10 and 12 years B) menarche, 11 and 14 years C) menopause, 12 and 15 years D) the climacteric, 14 and 16 years

B) menarche, 11 and 14 years

58. The return of myoglobin to the systemic circulation following a crush injury would result in all of the following conditions, EXCEPT: A) a decreased pH. B) metabolic alkalosis. C) hyperkalemia. D) renal dysfunction.

B) metabolic alkalosis.

99. Common signs and symptoms of meningitis in young children include all of the following, EXCEPT: A) poor feeding. B) nuchal rigidity. C) bulging fontanelle. D) irritability and fever.

B) nuchal rigidity.

Most urinary tract infections: A) are asymptomatic and are diagnosed when a urinalysis is performed during a routine physical exam. B) occur in women due to the relatively short urethra and its close proximity to the vagina and rectum. C) involve the lower urinary tract in males because the urethra's large surface area can house more bacteria. D) are the result of viruses or fungi entering the external urethral opening secondary to poor personal hygiene.

B) occur in women due to the relatively short urethra and its close proximity to the vagina and rectum.

Many of the physiologic changes caused by acute radiation syndrome: A) can be reversed if chemotherapy is administered within 24 hours. B) occur over time and will not be apparent in the prehospital setting. C) are a direct result of beta particles and are usually life threatening. D) manifest with lethal cardiac dysrhythmias and sudden cardiac arrest.

B) occur over time and will not be apparent in the prehospital setting.

41. A spontaneous abortion: A) affects one out of every three pregnancies and is typically idiopathic. B) occurs naturally and may or may not have an identifiable cause. C) is generally performed by a physician to prevent maternal death. D) is most often the result of a congenital abnormality of the placenta.

B) occurs naturally and may or may not have an identifiable cause.

Damage to the kidneys following an electrical injury: A) is caused by excess serum potassium levels. B) occurs when damaged muscle produces myoglobin. C) can be prevented with boluses of lactated Ringer's. D) is the result of electricity passing through the kidneys.

B) occurs when damaged muscle produces myoglobin.

13. After 35 years of age, age-related decrease in bone density is accelerated in females because: A) females have an inherently weak bone structure. B) of decreased estrogen levels secondary to menopause. C) excess estrogen levels destroy the bone's periosteum. D) of decreased bone marrow levels in the medullary canal.

B) of decreased estrogen levels secondary to menopause.

9. When communicating an intuitive decision and obtaining feedback to your team, you should FIRST advise your team: A) of what you think should be done next. B) of what you think the patient's problem is. C) of what led you to a particular diagnosis. D) to ask any questions or voice any concerns.

B) of what you think the patient's problem is.

Victims standing near an object that is struck by lightning: A) most commonly experience blast-type injuries. B) often have burns characterized by a feathering pattern. C) typically experience intractable ventricular fibrillation. D) experience full-thickness burns that require debridement.

B) often have burns characterized by a feathering pattern.

A marked decrease in urine output is called: A) anuria. B) oliguria. C) dysuria. D) polyuria.

B) oliguria.

9. The pressure within the right ventricle is: A) less than the pressure within the right atrium. B) one fourth of the pressure within the left ventricle. C) nearly equal to the pressure within the left ventricle. D) three times greater than the pressure in the left ventricle.

B) one fourth of the pressure within the left ventricle.

The __________ is a cone-shaped fossa that encloses and protects the eye. A) zygoma B) orbit C) condyle D) crista galli

B) orbit

59. The risk of a pulmonary embolism following musculoskeletal trauma is HIGHEST in patients with: A) upper extremity fractures that result in lengthy hospital admissions. B) pelvic and lower extremity injuries that lead to prolonged immobilization. C) numerous rib fractures who are taking anticoagulant medications. D) any proximal long bone fracture that occurred greater than 72 hours prior to medical intervention.

B) pelvic and lower extremity injuries that lead to prolonged immobilization.

45. If you have reason to believe that an unresponsive child has a foreign body airway obstruction, you should: A) assess for a pulse and then begin chest compressions. B) perform 30 chest compressions and then look in the mouth. C) administer abdominal thrusts until the object is expelled. D) try to remove it by performing a finger sweep of the mouth

B) perform 30 chest compressions and then look in the mouth.

When caring for a patient with a seemingly isolated ear injury, you should: A) carefully assess the external ear canal and inner ear for blood or CSF. B) perform a careful assessment to detect or rule out more serious injuries. C) recall that the pinna of the ear is highly vascular and bleeds profusely. D) consider direct transport of the patient to an audiologist for evaluation.

B) perform a careful assessment to detect or rule out more serious injuries.

66. Following blunt trauma to the anterior chest, a 44-year-old man presents with restlessness, respiratory distress, perioral cyanosis, and tachycardia. Further assessment reveals a midline trachea, engorged jugular veins, and absent breath sounds on the right side of his chest. You should: A) ventilate the patient with a bag-mask device and transport immediately. B) perform an immediate needle thoracentesis to the right side of the chest. C) give 100% oxygen and start a large-bore IV line en route to the hospital. D) transport at once and decompress the chest if tracheal deviation is observed.

B) perform an immediate needle thoracentesis to the right side of the chest.

46. If a 2-year-old child with a foreign body airway obstruction becomes unresponsive, you should position him or her supine and then: A) visualize the upper airway. B) perform chest compressions. C) assess for a carotid pulse. D) perform abdominal thrusts.

B) perform chest compressions.

22. Pneumothoraces create a ventilation-perfusion mismatch when: A) concomitant myocardial injury prevents adequate pulmonary perfusion and the lung collapses. B) perfusion of the involved lung continues while the pneumothorax prevents adequate ventilation. C) the vasculature of the affected lung is not intact and intrapulmonary gas exchange is impaired. D) the affected lung continues to expand adequately despite a decrease in pulmonary perfusion.

B) perfusion of the involved lung continues while the pneumothorax prevents adequate ventilation.

23. When assessing a child's circulation by looking at his or her skin, pallor is MOST indicative of: A) vasomotor instability and decompensated shock. B) peripheral vasoconstriction and compensated shock. C) poor oxygenation and a state of circulatory collapse. D) systemic vasodilation with resulting low blood pressure.

B) peripheral vasoconstriction and compensated shock.

6. On either side of the glottis, tissue forms a pocket called the: A) vallecula. B) piriform fossae. C) cuneiform cartilage. D) hypopharyngeal space.

B) piriform fossae.

131. To ensure that an infant's head is in a neutral position during spinal immobilization, you should: A) provide slight extension of his or her head. B) place padding under the infant's shoulders. C) place a towel roll behind the infant's neck. D) use towel rolls for lateral head stabilization.

B) place padding under the infant's shoulders.

94. You are dispatched to a residence for a 59-year-old man with difficulty breathing. The patient, who has a history of COPD, is conscious and alert. During your assessment, he tells you that he developed chills, fever, and a productive cough 2 days ago. Auscultation of his lungs reveals rhonchi to the left lower lobe. This patient is MOST likely experiencing: A) bronchitis. B) pneumonia. C) end-stage COPD. D) COPD exacerbation.

B) pneumonia.

24. The anterior and posterior tibial arteries are direct subdivisions of the: A) femoral artery. B) popliteal artery. C) descending aorta. D) external iliac artery.

B) popliteal artery.

35. The capillary "washout" phase occurs when: A) anaerobic metabolism causes the postcapillary sphincters to remain constricted, causing capillary fluid engorgement. B) postcapillary sphincters relax, releasing accumulated hydrogen, potassium, carbon dioxide, and thrombosed red blood cells. C) cellular ischemia causes the postcapillary sphincters to remain constricted, causing the capillaries to become engorged with fluid. D) precapillary sphincters constrict in response to the buildup of lactic acid, vasomotor failure, and increased carbon dioxide.

B) postcapillary sphincters relax, releasing accumulated hydrogen, potassium, carbon dioxide, and thrombosed red blood cells.

3. Premorbid conditions are those that: A) are a direct cause of death. B) precede the onset of a disease. C) are caused by an acute illness. D) have no effect on patient outcome.

B) precede the onset of a disease.

47. Abruptio placenta is MOST accurately defined as: A) separation of the placenta secondary to blunt maternal abdominal trauma. B) premature separation of a normally implanted placenta from the uterine wall. C) a condition in which the placenta progressively detaches from the uterine wall. D) a placenta that implants low in the uterus and partially or fully covers the cervix.

B) premature separation of a normally implanted placenta from the uterine wall.

14. Human chorionic gonadotropin hormone sends signals to the corpus luteum, which in turn: A) produces cells that clump together and ultimately form the main tissues of the placenta. B) produces hormones designed to support the pregnancy until the placenta has developed. C) facilitates transition of the blastocyst into an embryo and causes further maturation of the placenta. D) causes the developing placenta to produce projections that tap into the external tissue of the blastocyst.

B) produces hormones designed to support the pregnancy until the placenta has developed.

12. The initial steps of newborn resuscitation include: A) free-flow oxygen. B) proper positioning. C) assessment of pulse rate. D) positive-pressure ventilation.

B) proper positioning.

When assessing a patient with maxillofacial trauma, it is MOST important to: A) gently palpate the maxilla, mandible, and zygoma to elicit crepitus. B) protect the cervical spine and monitor the patient's neurologic status. C) apply a cervical collar and determine if the patient has visual disturbances. D) have the patient open his or her mouth and assess for dental malocclusion.

B) protect the cervical spine and monitor the patient's neurologic status.

146. A 10-year-old child fell approximately 15 feet from a balcony, landing on a sidewalk. He is conscious and alert, and complains of pain to the right side of his body. After completing your primary assessment, you should: A) apply spinal precautions, begin transport, and perform a rapid assessment while en route to the hospital. B) provide any immediately needed care, perform a rapid assessment, apply spinal precautions, and transport. C) perform a focused physical exam, obtain baseline vital signs, apply spinal precautions, and transport. D) correct immediate life threats, perform a detailed head-to-toe exam, apply spinal precautions, and transport.

B) provide any immediately needed care, perform a rapid assessment, apply spinal precautions, and transport.

20. The purpose of hospice care is to: A) render quality care to patients with a debilitating but temporary disease. B) provide quality end-of-life care through pain and symptom management. C) restore a person to his or her maximum physical and emotional potential. D) render around-the-clock intensive care to prevent cardiopulmonary arrest.

B) provide quality end-of-life care through pain and symptom management.

1. The external female genitalia are collectively referred to as the: A) vagina. B) pudendum. C) external labia. D) mons veneris.

B) pudendum.

18. Clinical indicators of sympathetic nervous system discharge include: A) diffuse wheezing. B) pupillary dilation.. C) hypoventilation. D) increased salivation.

B) pupillary dilation..

33. Incomplete glucose breakdown leads to an accumulation of: A) lactic acid. B) pyruvic acid. C) bicarbonate. D) carbonic acid.

B) pyruvic acid.

75. A child in decompensated shock with hypotension should: A) be intubated to protect his or her airway. B) receive initial fluid resuscitation at the scene. C) be given 25% dextrose to prevent hypoglycemia. D) receive volume expansion with 5% dextrose in water.

B) receive initial fluid resuscitation at the scene.

138. When caring for a child with a ventricular shunt or gastrostomy tube, it is important to: A) provide supportive care only and then rapidly transport the child to the most appropriate medical facility. B) recognize that the caregiver is a key resource and that his or her expertise should be utilized to assist in the care of the child. C) assure the caregiver that you can care for the child effectively and recommend that he or she follow the ambulance in his or her own vehicle. D) obtain a complete medical history from the caregiver and then develop a treatment plan based on your knowledge of special health care devices.

B) recognize that the caregiver is a key resource and that his or her expertise should be utilized to assist in the care of the child.

165. You are dispatched to a residence at 6:15 AM for an unconscious 3-month-old infant who is not breathing. Upon arrival at the scene, you find the father performing CPR on the infant. The infant's mother is sitting on the couch, crying. Your assessment reveals that the child is apneic and pulseless. His skin is pale and cold, and there is gross lividity to his chest. You should: A) continue CPR and assess the infant's cardiac rhythm to confirm asystole. B) recognize that the infant has been deceased for an extended period of time. C) pronounce the infant dead if he does not respond to 5 minutes of full ACLS. D) tell the parents that the child likely suffocated because he slept on his stomach.

B) recognize that the infant has been deceased for an extended period of time.

72. Hypotension during anaphylactic shock is caused primarily by: A) vascular damage. B) relative hypovolemia. C) cardiac muscle injury.. D) capillary leakage.

B) relative hypovolemia.

3. Unlike exocrine glands, endocrine glands: A) do not affect the rate of cellular metabolism. B) release chemicals directly into the bloodstream. C) have ducts that carry their secretions into a body cavity. D) produce chemicals that work faster than the nervous system

B) release chemicals directly into the bloodstream.

102. You are transporting a middle-aged man on a CPAP unit for severe pulmonary edema. An IV line of normal saline is in place. Prior to applying the CPAP device, the patient was tachypneic and had an oxygen saturation of 90%. When you reassess him, you note that his respirations have increased and his oxygen saturation has dropped to 84%. You should: A) continue the CPAP treatment and administer a diuretic to remove fluids from his lungs quickly. B) remove the CPAP unit, assist his ventilations with a bag-mask device, and prepare to intubate him. C) suspect that he has developed a pneumothorax and prepare to perform a needle chest decompression. D) decrease the amount of positive-end expiratory pressure that you are delivering and reassess.

B) remove the CPAP unit, assist his ventilations with a bag-mask device, and prepare to intubate him.

It is important for the paramedic to consult with medical control prior to administering analgesia to a patient with severe flank pain and suspected acute renal failure because: A) patients with renal failure often require high doses of analgesia. B) renal failure may cause analgesics to accumulate to toxic levels. C) most patients who are given analgesia will experience hypotension. D) analgesia will mask the patient's pain and skew further examination.

B) renal failure may cause analgesics to accumulate to toxic levels.

11. If a newborn does not respond to the initial steps of resuscitation, the need for further intervention is based upon: A) pulse rate, activity, and appearance. B) respiratory effort, pulse rate, and color. C) appearance, skin color, and muscle tone. D) respirations, appearance, and muscle tone.

B) respiratory effort, pulse rate, and color.

Aldosterone produces all of the following physiologic effects, EXCEPT: A) increased water reabsorption by the kidneys. B) retention of potassium ions in the body. C) increased sodium and chloride reabsorption. D) decreased reabsorption of potassium ions.

B) retention of potassium ions in the body.

42. Hepatomegaly and jugular venous distention are MOST suggestive of: A) left heart failure. B) right heart failure. C) pulmonary edema. D) severe pneumonia.

B) right heart failure.

8. The anterior-most portion of the heart is the: A) left ventricle. B) right ventricle. C) right atrium. D) left atrium.

B) right ventricle.

A conscious but combative patient with severe facial trauma is fully immobilized on a backboard. During your assessment, the patient begins coughing up large amounts of blood. You suction his oropharynx, but his mouth quickly refills with blood. You should: A) preoxygenate the patient with a bag-mask for 2 minutes and then perform nasotracheal intubation. B) roll the backboard on its side, suction his oropharynx, and prepare to perform pharmacologically assisted intubation. C) alternate suctioning of his oropharynx for 15 seconds and assisting ventilations for 2 minutes until his airway is clear of blood. D) continually suction his oropharynx until it is clear of blood, apply oxygen via nonrebreathing mask, and administer a sedative drug.

B) roll the backboard on its side, suction his oropharynx, and prepare to perform pharmacologically assisted intubation.

114. Any child with unexplained hyperpnea should be suspected of having _________ toxicity. A) opiate B) salicylate C) beta blocker D) organophosphate

B) salicylate

Proper treatment for an open wound to the neck includes: A) administering 2 L of IV crystalloid solution. B) sealing the wound with an occlusive dressing. C) prompt transportation to a hyperbaric chamber. D) applying a circumferential pressure dressing.

B) sealing the wound with an occlusive dressing.

19. The MOST common device used to provide positive-pressure ventilation to a newborn in the prehospital setting is a: A) T-piece resuscitator. B) self-inflating bag-mask device. C) flow-inflating bag-mask device. D) manually triggered ventilator.

B) self-inflating bag-mask device

9. Difficulty walking or sitting, nightmares, and bedwetting are MOST suggestive of: A) neglect. B) sexual abuse. C) physical abuse. D) emotional abuse.

B) sexual abuse.

17. Pelvic inflammatory disease MOST commonly affects: A) the external genitalia. B) sexually active women. C) postmenopausal women. D) women over 30 years of age.

B) sexually active women.

23. Apneustic breathing is characterized by: A) a sustained pattern of tachypnea and increased tidal volume. B) short, brisk inhalations with a long pause before exhalation. C) regular respirations with a normal rate and adequate tidal volume. D) a crescendo-decrescendo pattern of breathing with apneic periods.

B) short, brisk inhalations with a long pause before exhalation.

19. By maintaining a state of partial contraction, __________ muscle allows the body to maintain its posture. A) smooth B) skeletal C) involuntary D) non-striated muscle

B) skeletal

53. The MOST practical method of splinting multiple fractures in a critically injured patient is to: A) apply vacuum splints en route to the hospital. B) splint the axial skeleton using a long backboard. C) use air splints so you can visualize the injuries. D) sedate the patient before applying any splints.

B) splint the axial skeleton using a long backboard.

39. The sudden twisting of a joint beyond its normal range of motion, causing a temporary subluxation, is called a: A) strain. B) sprain. C) dislocation. D) fracture-dislocation.

B) sprain.

12. A 15-year-old child can be difficult to treat for all of the following reasons, EXCEPT: A) peer pressure. B) stranger anxiety. C) independence issues. D) cognizance of body image.

B) stranger anxiety

42. Respiratory distress in a premature infant is MOST often the result of: A) a pneumothorax. B) surfactant deficiency. C) pneumonia at birth. D) intracranial hemorrhage

B) surfactant deficiency

100. An elderly woman with COPD presents with peripheral edema. The patient is conscious but agitated. She is breathing with slight difficulty but has adequate tidal volume. During your assessment, you note that her jugular veins engorge when you apply pressure to her right upper abdominal quadrant. She tells you that she takes a "water pill" and Vasotec for high blood pressure. You should: A) expect to hear crackles when you auscultate her lungs. B) suspect acute right heart failure and administer oxygen. C) start an IV of normal saline and give her a 250-mL bolus. D) conclude that she has been noncompliant with her diuretic.

B) suspect acute right heart failure and administer oxygen.

36. In the late stage of syphilis: A) most patients become acutely ill and complain of symptoms such as blurred vision, a headache, and nausea. B) the patient is usually asymptomatic, but damage to the brain, heart, blood vessels, and liver is occurring. C) the patient experiences weight loss, muscle aches, and headaches that will not resolve without treatment. D) the disease has damaged the central nervous system permanently and is considered untreatable at this point.

B) the patient is usually asymptomatic, but damage to the brain, heart, blood vessels, and liver is occurring.

66. Treatment for cyanide poisoning may include all of the following, EXCEPT: A) amyl nitrite. B) methylene blue. C) calcium gluconate. D) hydroxocobalamin.

C

92. You are dispatched to the parking lot of a shopping mall for a pregnant woman with severe vomiting. When you arrive and assess the patient, she tells you that she is 26 weeks pregnant and that this is her first child. She further tells you that her normal morning sickness nausea became worse 3 days ago and that she has vomited numerous times since then; she also noticed small amounts of blood in her vomitus. Her blood pressure is 94/60 mm Hg, pulse rate is 124 beats/min and weak, and respirations are 22 breaths/min and regular. After loading the woman in the ambulance and placing her on her left side, you should: A) conclude that increased hormone levels are causing a transient worsening of her morning sickness, start an IV of normal saline and set it to keep the vein open, administer 12.5 mg of promethazine to control the vomiting, and transport her. B) suspect hyperemesis gravidarum, administer high-flow oxygen, start a large-bore IV and give a 250-mL normal saline bolus, assess her blood sugar and cardiac rhythm, consider administering diphenhydramine if protocols allow, and transport promptly. C) suspect upper gastrointestinal bleeding, administer oxygen via nonrebreathing mask, start two large-bore IV lines and give 2 L of normal saline, apply the cardiac monitor, insert a nasogastric tube to evacuate blood from her stomach, and transport at once. D) be suspicious that she has acute gastritis, apply oxygen via nasal cannula at 4 L/min, start an IV line and give a 20-mL/kg normal saline bolus, administer an antiemetic if she continues to vomit, obtain a 12-lead ECG tracing, and transport her promptly.

B) suspect hyperemesis gravidarum, administer high-flow oxygen, start a large-bore IV and give a 250-mL normal saline bolus, assess her blood sugar and cardiac rhythm, consider administering diphenhydramine if protocols allow, and transport promptly.

Movement of both of the eyes in unison is called: A) dysconjugate gaze. B) sympathetic eye movement. C) extraocular movement. D) physiologic anisocoria.

B) sympathetic eye movement.

4. Common effects of gag reflex stimulation include all of the following, EXCEPT: A) vomiting. B) tachycardia. C) bradycardia. D) increased intracranial pressure.

B) tachycardia.

70. In contrast to a child with pulmonary edema secondary to congestive heart failure, the respirations of a hypercarbic child without pulmonary edema would MOST likely be: A) rapid with audible rhonchi. B) tachypneic and without retractions. C) slow with increased work of breathing. D) bradypneic with periods of marked apnea.

B) tachypneic and without retractions.

52. A 29-year-old man presents with bizarre behavior and profuse sweating. His wife tells you that he has type 1 diabetes and that he took his insulin today. During your assessment, you will MOST likely find that the patient is: A) dehydrated. B) tachypneic. C) hyperglycemic. D) breathing deeply.

B) tachypneic.

64. Unlike bronchodilator therapy, corticosteroid therapy: A) causes immediate improvement in breathing. B) takes a few hours to reduce bronchial edema. C) is administered exclusively in a hospital setting. D) is the primary treatment for acute bronchospasm.

B) takes a few hours to reduce bronchial edema.

59. Caput succedaneum is defined as: A) bilateral temporal bone fractures caused by a delivery that includes the use of forceps. B) temporary swelling of the soft tissue of the baby's scalp secondary to pressure from the dilating cervix. C) an area of bleeding between the parietal bone and its covering periosteum that resolves in 1 to 2 months. D) permanent cranial disfigurement caused by vaginal delivery in a woman with cephalopelvic disproportion.

B) temporary swelling of the soft tissue of the baby's scalp secondary to pressure from the dilating cervix.

3. As the preovulatory phase of the menstrual cycle progresses: A) the maturation of an oocyte occurs when follicle cells respond to follicle-stimulating hormone. B) the anterior pituitary gland releases luteinizing hormone, which stimulates the release of an ovum. C) gonadotropin-releasing factor stimulates the hypothalamus to release follicle-stimulating hormone. D) follicle-stimulating hormone is released by the hypothalamus, which stimulates the process of ovulation.

B) the anterior pituitary gland releases luteinizing hormone, which stimulates the release of an ovum.

The skin sheds excess heat from the body through: A) retention of salt and water. B) the evaporation of sweat. C) cutaneous vasoconstriction. D) decreased sebum production

B) the evaporation of sweat.

54. With regard to pulse oximetry, the more hypoxic a patient becomes: A) the slower his or her PaO2 will fall. B) the faster he or she will desaturate. C) the slower he or she will desaturate. D) the less reliable the pulse oximeter is.

B) the faster he or she will desaturate.

88. Spina bifida occurs when: A) hydrocephalus causes a significant increase in pressure within the spinal canal, resulting in chronic compression of the spinal cord. B) the fetus's spinal column does not close properly or completely and vertebrae do not develop, leaving a portion of the spinal cord exposed. C) trauma during birth causes distracting injuries to the cervical and thoracic vertebrae, resulting in partial or complete paralysis below the injury. D) growth of the fetus's spinal column stops at the thoracic vertebrae, which leaves the lumbar portion of the spinal cord completely unprotected.

B) the fetus's spinal column does not close properly or completely and vertebrae do not develop, leaving a portion of the spinal cord exposed.

75. Total anomalous pulmonary venous return is a rare congenital defect in which: A) pressure in the lungs causes pulmonary hypertension. B) the four pulmonary veins connect to the right atrium. C) venous blood mixes with arterial blood in the heart. D) blood returns to the lungs after being reoxygenated.

B) the four pulmonary veins connect to the right atrium.

74. During true labor: A) analgesics often abolish the pain from contractions. B) the interval between contractions gradually shortens. C) the intensity of uterine contractions remains constant. D) contractions are irregularly spaced from one to the next.

B) the interval between contractions gradually shortens.

End-stage renal disease occurs when: A) the initial signs of azotemia are present. B) the kidneys have lost all ability to function. C) urine output falls to less than 500 mL/day. D) the glomerular filtration rate is increased.

B) the kidneys have lost all ability to function.

34. Pneumonia can be especially virulent during pregnancy because: A) pregnancy causes a reduction in tidal volume. B) the mother's immune system is already depressed. C) pregnancy causes an increased risk of bronchospasm. D) the mother's cough reflex markedly diminishes.

B) the mother's immune system is already depressed.

86. Uterine inversion occurs when: A) the musculature of the uterine fundus is inherently weak, causing the uterus to prolapse. B) the placenta fails to detach properly and adheres to the uterine wall when it is expelled. C) postpartum bleeding is improperly managed with inadequate massage of the uterine fundus. D) excessive postpartum hemorrhage causes uterine ischemia and subsequent expulsion from the vagina.

B) the placenta fails to detach properly and adheres to the uterine wall when it is expelled.

88. A 19-year-old diabetic woman who is 22 weeks pregnant presents with an occipital headache, blurred vision, and edema to her hands and feet. Her blood pressure is 152/94 mm Hg, pulse rate is 108 beats/min, and respirations are 20 breaths/min. The patient is conscious and alert, and states that she is "retaining a lot of water." Your MOST immediate concern should be: A) the risk of fetal compromise. B) the possibility of a seizure. C) lowering her blood pressure. D) her current blood sugar level.

B) the possibility of a seizure.

5. If there is a physiologic level of antidiuretic hormone in the bloodstream, then: A) blood pressure decreases secondary to dilation of the vessels. B) the renal tubules are stimulated to reabsorb sodium and water. C) potassium, phosphorus, and magnesium are lost through diuresis. D) the kidneys excrete excessive sodium and water from the body.

B) the renal tubules are stimulated to reabsorb sodium and water.

52. The quickest way to prevent newborn hypothermia involves: A) administering warmed IV fluids. B) thoroughly drying the newborn after birth. C) applying a hot water bottle to the groin area. D) administering warmed, humidified oxygen.

B) thoroughly drying the newborn after birth

36. You respond to a residence where an infant's apnea monitor has alarmed several times within the last 30 minutes. When you assess the infant, you note that she is alert, she has strong peripheral pulses, and her skin is pink, warm, and dry. You should: A) expect that her oxygen saturation will be below 90%. B) transport the infant and monitor her breathing en route. C) administer blow-by oxygen to the infant and reassess her. D) advise the parents to contact the apnea monitor manufacturer.

B) transport the infant and monitor her breathing en route.

A common cause of intrarenal acute renal failure is: A) hypovolemia. B) type 1 diabetes. C) a renal calculus. D) myocardial infarction.

B) type 1 diabetes.

109. In contrast to toxic ingestions in toddlers, toxic ingestions in adolescents: A) are usually unintentional. B) typically involve multiple agents. C) are associated with lower mortality. D) involve small quantities of a single agent.

B) typically involve multiple agents.

33. To evaluate function of an infant's or child's cerebral cortex, you should: A) assess pupil reaction. B) use the AVPU scale. C) assess for posturing. D) evaluate motor activity.

B) use the AVPU scale.

22. In addition to meeting the metabolic needs of the developing fetus, the mother's blood supply increases significantly in order to perfuse her own organs, especially the: A) heart and lungs. B) uterus and kidneys. C) liver and pancreas. D) central nervous system.

B) uterus and kidneys.

85. Patients with obvious respiratory failure require immediate: A) intubation. B) ventilation support. C) passive oxygenation. D) bronchodilator therapy.

B) ventilation support.

20. The uterine fundus is measured: A) horizontally from one side of the uterus to the other side. B) vertically from the top of the pubic bone to the top of the fundus. C) from just below the umbilicus to the inferior part of the sternum. D) vertically from the top of the fundus to just below the umbilicus.

B) vertically from the top of the pubic bone to the top of the fundus.

66. A patient with a history of asthma is at GREATEST risk for respiratory arrest if he or she: A) takes a bronchodilator and a corticosteroid. B) was previously intubated for his or her condition. C) was recently evaluated in an emergency department. D) has used his or her inhaler twice in the previous week.

B) was previously intubated for his or her condition.

53. A child who is experiencing a moderate asthma attack would MOST likely present with: A) a markedly prolonged expiratory phase. B) wheezing during inspiration and expiration. C) an inability to speak in complete sentences. D) an oxygen saturation between 80% and 90%

B) wheezing during inspiration and expiration.

37. Naloxone is NOT recommended for use in newborns: A) who weigh less than 5.5 lbs. B) who are born to narcotic-addicted mothers. C) unless the umbilical vein has been cannulated. D) with shallow breathing and persistent bradycardia.

B) who are born to narcotic-addicted mothers.

If an acutely burned patient is in shock in the prehospital setting: A) it is likely that he or she is experiencing burn shock. B) you should look for another injury as the source of shock. C) administer a 250-mL bolus of normal saline and reassess. D) avoid IV fluids unless the systolic BP is below 80 mm Hg.

B) you should look for another injury as the source of shock.

Which of the following general statements regarding trauma is correct? A) Bullet impact is less if the energy in the bullet is applied to a small area. B) The position of the patient at the time of the event is considered to be an internal factor. C) Blunt trauma is difficult to diagnose by paramedics in the field and is often more lethal than penetrating trauma. D) Rapidly applied amounts of energy are better tolerated than a similar amount of energy applied over a longer period.

Blunt trauma is difficult to diagnose by paramedics in the field and is often more lethal than penetrating trauma.

. The ______________ consists of eight bones that encase and protect the brain. A) skull B) cerebrum C) cranial vault D) cribriform plate

C

10. The parietal lobe of the brain: A) is where the optic nerve originates and processes vision. B) is important for voluntary motor action and personality traits. C) controls somatic functions for the contralateral side of the body. D) controls functions such as long-term memory

C

103. You are dispatched to an apartment complex for a suicide attempt. While you are en route, an on-scene law enforcement officer advises you that the patient, who is unresponsive, ingested an unknown quantity of an unknown drug. Upon arriving at the scene, you should: A) identify what the patient took before providing treatment. B) gain rapid access to the patient and begin your assessment. C) safely gain access to the patient while looking for an egress route. D) ask the police officer to bring the patient to the ambulance.

C

105. Hyperacute pain to touch is called: A) parasthesia. B) anesthesia. C) hyperesthesia. D) akathisia.

C

109. You are transporting a chronic heroin abuser to whom you have just administered naloxone. The patient is responsive to verbal stimuli, and his respirations, blood pressure, and pulse rate have improved following your treatment. With an estimated time of arrival at the hospital of 20 minutes, which of the following should concern you the MOST? A) There is a high potential that the patient will suddenly become violent. B) The patient will require immediate intubation if his respirations decrease. C) The patient may deteriorate and require further naloxone administration. D) Low doses of naloxone often precipitate seizures in chronic heroin abusers.

C

70. Beta receptor stimulation results in all of the following effects, EXCEPT: A) positive cardiac inotropy. B) positive cardiac chronotropy. C) vascular smooth muscle contraction. D) relaxation of bronchiole smooth muscle

C

115. You respond to a local motel for a young woman who was sexually assaulted. Upon your arrival, you find the patient sitting on the bed talking to a police officer. The last thing she remembers is meeting "some guy" at a nightclub the evening before and then having a few drinks with him. She is conscious, but sleepy. Her respirations are 12 breaths/min and regular, pulse rate is 56 beats/min and strong, and blood pressure is 102/58 mm Hg. The cardiac monitor reveals sinus bradycardia at 50 to 60 beats/min. You should: A) assist her ventilations with a bag-mask device, start an IV line, administer 0.5 mg of atropine, and transport. B) give her supplemental oxygen, conduct a secondary assessment at the scene to collect evidence, and transport her. C) administer high-flow oxygen, monitor her oxygen saturation, begin transport, and start an IV line en route to the hospital. D) conclude that she was unknowingly administered a narcotic analgesic, start an IV line, and give her 2 mg of naloxone.

C

125. A skier wiped out while skiing down a large hill. He is conscious and alert and complains of being very cold; he also complains of neck stiffness and numbness and tingling in all of his extremities. A quick assessment reveals that his airway is patent and his breathing is adequate. You should: A) perform a detailed neurologic exam and carefully palpate his neck. B) apply a cervical collar and start an IV line with warm normal saline. C) immobilize his spine and quickly move him to a warmer environment. D) administer oxygen and perform a detailed secondary assessment.

C

130. A 39-year-old man crashed his vehicle into a wooded area and was not found for approximately 8 hours. When you arrive at the scene and assess him, you note that he is conscious but anxious. He is unable to feel or move below his mid-thoracic area and complains of a severe headache. His blood pressure is 210/130 mm Hg, heart rate is 44 beats/min, and respirations are 22 breaths/min. This patient's clinical presentation is MOST consistent with: A) neurogenic shock. B) intracranial pressure. C) autonomic dysreflexia. D) symptomatic bradycardia.

C

14. You would expect a person to be hypertensive and tachycardic following exposure to all of the following, EXCEPT: A) cocaine. B) parathion. C) phenobarbital. D) pseudoephedrine.

C

16. An acetone breath odor is common following exposure to all of the following toxins, EXCEPT: A) aspirin. B) isopropyl alcohol. C) camphor. D) methyl alcohol.

C

22. Alcohol potentiates Valium. This means that: A) Valium makes alcohol a toxic substance. B) alcohol antagonizes the effects of Valium. C) alcohol enhances the effects of Valium. D) the use of alcohol negates the use of Valium.

C

24. A scalp laceration that occurs in conjunction with a nondisplaced skull fracture: A) typically causes significant hypertension. B) should be treated with firm direct pressure. C) is considered to be an open skull fracture. D) often causes an infection that progresses rapidly.

C

26. Toxic effects of alcohol on the liver include all of the following, EXCEPT: A) coagulopathy. B) hypoglycemia. C) hyperglycemia. D) gastrointestinal bleeding.

C

28. Symptoms of delirium tremens usually begin within ___ to ___ hours after the last alcohol intake. A) 12, 24 B) 24, 48 C) 48, 72 D) 72, 96

C

30. Following a traumatic brain injury, initial swelling of the brain occurs due to: A) severe ischemia. B) acute hypertension. C) cerebral vasodilation. D) an increase in cerebral water.

C

33. Your FIRST priority when dealing with a patient who may have overdosed is to: A) ascertain what the patient took. B) enter the scene carefully. C) request law enforcement. D) assess the patient's airway.

C

35. Decerebrate posturing is characterized by: A) flexion of the arms and extension of the legs. B) inward flexion of the wrists and flexed knees. C) extension of the arms and extension of the legs. D) pulling in of the arms toward the core of the body.

C

38. Which of the following types of medications does NOT contain amphetamine? A) Diet pills B) Nasal decongestants C) Drugs used to treat insomnia D) Drugs used to treat attention deficit disorder

C

4. Management for an ingested poison focuses mainly on: A) the prompt induction of vomiting. B) administering a counteracting agent. C) neutralizing the poison in the stomach. D) treating the systemic effects that result.

C

40. An epidural hematoma typically causes rapid deterioration in the head-injured patient's condition because: A) numerous axons are severely damaged. B) the meningeal veins are often disrupted. C) it is associated with brisk arterial bleeding. D) concomitant spinal cord injury is often present

C

44. Priority care for an unresponsive patient who has overdosed on phenobarbital includes: A) administering oxygen and giving naloxone. B) administering diazepam to prevent seizures. C) securing the airway and preventing aspiration. D) observing the ECG closely for lethal dysrhythmias.

C

50. When heroin passes through the liver: A) the liver rapidly metabolizes the drug, removes the toxic ingredients, and excretes it via the renal system. B) it rapidly becomes hepatotoxic, continues to exert narcotic effects, and causes liver failure within 24 to 36 hours. C) it is metabolized into acetyl-morphine, which continues to exert narcotic effects that may outlast the effects of naloxone. D) absorption through the rest of the body decreases markedly, allowing its narcotic effects to be reversed easily with naloxone.

C

51. Cardiac arrest following a narcotic overdose is usually the result of: A) seizures. B) renal failure. C) respiratory arrest. D) a cardiac dysrhythmia.

C

55. Organophosphates exert their effect by: A) destroying the body's acetylcholine. B) agonizing the sympathetic nervous system. C) stimulating the cholinergic nervous system. D) blocking the parasympathetic nervous system.

C

60. Pulse oximetry will not provide a true assessment of arterial oxygenation in patients with carbon monoxide toxicity because: A) the device falsely interprets oxyhemoglobin as carboxyhemoglobin. B) carbon monoxide damages the computer chip inside the pulse oximeter. C) the device cannot determine whether carbon monoxide or oxygen is bound to the hemoglobin. D) carbon monoxide turns the blood dark red, which indicates low oxygen content.

C

71. Which of the following statements regarding methyl alcohol is correct? A) Also referred to as methanol, methyl alcohol is colorless and odorless and requires large amounts to cause toxicity. B) It is not recognized as a poison, although it has many properties of a poison when consumed in sufficient quantities. C) Methyl alcohol is also known as wood alcohol, and is present in paints, paint removers, windshield washer fluid, and varnishes. D) The signs and symptoms of methyl alcohol poisoning typically appear within 15 to 20 minutes following ingestion of as little as 5 to 10 mL.

C

72. The hyperpnea and tachypnea associated with methyl alcohol intoxication is secondary to: A) hypoxemia. B) heart failure. C) metabolic acidosis. D) an elevated blood pH.

C

75. Which of the following statements regarding the hangman's fracture is correct? A) Hangman's fractures occur when the patient's skull rapidly accelerates. B) Most hangman's fractures occur during a fall from greater than 10 feet. C) It is a fracture of C2 that is secondary to significant distraction of the neck. D) Severe hyperflexion of the neck commonly results in a hangman's fracture.

C

78. Common signs and symptoms of a tricyclic antidepressant overdose include: A) excessive salivation and diarrhea. B) tachypnea and severe hypertension. C) altered mental status and tachycardia. D) constricted pupils and AV heart block.

C

8. Which of the following toxins causes the MOST serious consequences when absorbed through the skin? A) Dry lime B) Poison oak C) Pesticides D) Sulfuric acid

C

80. A complete spinal cord injury to the upper cervical spine: A) results in quadriplegia but the patient usually retains his or her ability to breathe spontaneously. B) is not compatible with life and results in immediate death due to cardiopulmonary failure. C) will result in permanent loss of all cord-mediated functions below the level of the injury. D) results in neurologic dysfunction that is considered to be permanent if it lasts longer than 24 hours.

C

82. Which of the following antidepressant medications has the HIGHEST safety margin? A) Doxepin B) Imipramine C) Paroxetine D) Nortriptyline

C

89. Death from acetaminophen overdose is MOST often caused by: A) metabolic alkalosis. B) acute splenic rupture. C) progressive liver failure. D) gastrointestinal bleeding.

C

94. A metallic taste in the mouth, explosive diarrhea, and a skin rash are MOST indicative of: A) lead poisoning. B) cyanide poisoning. C) arsenic poisoning. D) mercury poisoning.

C

95. Any motor or sensory deficits noted during the neurologic examination of a patient with a possible spinal cord injury: A) indicate a complete spinal cord injury. B) require you to repeat the primary assessment. C) should be documented and monitored. D) must be reported to the hospital at once.

C

95. The dieffenbachia plant is also referred to as "dumb cane" because: A) ingestion typically results in death in less than 5 minutes. B) its leaves are highly toxic to the ears and cause deafness. C) ingestion can result in the patient being unable to speak. D) its toxic effects cause stupor, ataxia, and bizarre behavior.

C

A 19-year-old woman fell from a second story window and landed on her head. She is unconscious with a blood pressure of 148/94 mm Hg, heart rate of 58 beats/min, and irregular respirations of 8 breaths/min. Further assessment reveals blood draining from her nose and bilaterally dilated pupils that are slow to react. In addition to employing full spinal precautions, the MOST appropriate treatment for this patient involves: A) hyperventilating her with a bag-mask device at a rate of 20 breaths/min, starting two large-bore IV lines, applying a cardiac monitor, administering 5 mg of Valium to prevent seizures, and transporting to a trauma center. B) preoxygenating her with a bag-mask device and 100% oxygen for 2 to 3 minutes, performing nasotracheal intubation, transporting at once, starting at least one large-bore IV line en route, and obtaining her Glasgow Coma Scale score. C) intubating her trachea after preoxygenating her for 2 to 3 minutes with a bag-mask device, transporting immediately, starting at least one large-bore IV en route, applying a cardiac monitor, and performing frequent neurologic assessments. D) applying oxygen via nonrebreathing mask, covering her with blankets, starting an IV of normal saline set to keep the vein open, applying a cardiac monitor, initiating transport, and monitoring her pupils while en route to the hospital.

C

A 20-year-old woman complains of difficulty breathing and the feeling of a lump in her throat approximately 10 minutes after being stung by a wasp. Your assessment reveals that she is anxious, is in obvious respiratory distress, and has hives covering her arms and legs. Further assessment reveals diffuse wheezing, a blood pressure of 80/50 mm Hg, and a heart rate of 120 beats/min. You should: A) immediately sedate and intubate her to protect her airway, administer 25 mg of Benadryl IM, and consider an epinephrine infusion. B) administer albuterol via an inline nebulizer attached to a bag-mask device, assist ventilations, and consider administering epinephrine. C) apply oxygen via nonrebreathing mask, start an IV with normal saline, and administer 1 mg of epinephrine 1:10,000 via IV push. D) administer high-flow oxygen, start a large-bore IV, infuse up to 2 liters of normal saline, and administer 0.3 mg of epinephrine 1:1,000 SQ.

C

A 41-year-old woman attempted to kill herself by cutting her wrists. Law enforcement personnel are at the scene. You have controlled the bleeding with direct pressure and a pressure bandage. The patient's vital signs are stable and she is conscious and alert; however, she refuses to go to the hospital. Despite pleas from her family to go to the hospital, she still refuses. You should: A) respect her wishes because she is coherent, but leave her with a trusted family member. B) use reasonable force to physically restrain the patient, administer a sedative, and transport. C) remain with the patient, contact medical control, and request law enforcement intervention. D) contact medical control and advise him or her that you will be transporting the patient against her will.

C

A chemistry student accidentally splashed a strong acid chemical into his eyes. He wears rigid gas-permeable contact lenses and is experiencing intense pain. You should: A) leave his contact lenses in place and irrigate his eyes throughout transport. B) carefully remove his contact lenses and cover both eyes with dry dressings. C) carefully remove his contact lenses and flush his eyes for at least 20 minutes. D) leave his contact lenses in place and neutralize the acid with a strong alkali.

C

A man who smiles pleasantly when he tells you of the recent death of his wife: A) is thinking circumstantially. B) has a disorder of perception. C) has an inappropriate affect. D) has delusions of persecution.

C

A middle-aged male who received a kidney transplant called EMS because he was not feeling well. Which of the following assessment findings is MOST suggestive of organ rejection? A) Excessive urine output, shortness of breath, and a diffuse rash B) Blood in the urine, diffuse abdominal pain, and hypothermia C) Fever with swelling and tenderness over the implanted kidney D) Bilateral flank pain that radiates to both shoulders and scapulae

C

A motorcycle or football helmet should be removed if: A) the patient complains of severe neck pain and the helmet fits snugly. B) you are going to transport the patient to a medical treatment facility. C) the patient is breathing shallowly and access to the airway is difficult. D) you are properly trained in the technique, even if you are by yourself.

C

A patient with a dysconjugate gaze following an ocular injury: A) most likely has a concomitant basilar skull fracture. B) should have ice applied to the eyes to prevent blindness. C) has discoordination between the movements of both eyes. D) should be treated by irrigating both eyes for 20 minutes.

C

A person who is neologistic: A) repeats what he or she hears. B) talks with grossly slurred speech. C) invents words. D) has accelerated or pressured speech.

C

A person who is taking antibiotic therapy following a positive TB skin test and chest radiograph should not consume alcohol because: A) this increases the risk of active TB. B) alcohol can cause a violent reaction. C) the antibiotics are toxic to the liver. D) alcohol causes immunocompromise.

C

A psychiatric emergency: A) generally occurs following the use of an illicit drug or excessive consumption of alcohol. B) is caused by a chemical imbalance in the brain but does not interfere with activities of daily living. C) exists when a person's abnormal behavior threatens the safety and well-being of himself or herself or others. D) is generally the result of a short-term emotional crisis that is treated effectively with behavior modification.

C

Abnormal behavior that stems from a psychosocial etiology would MOST likely occur in a patient who: A) is under severe stress caused by work. B) resorts to alcohol to mitigate daily stress. C) experienced emotional trauma as a child. D) recently experienced the death of a loved one.

C

Adults in anaphylactic shock should receive the ________ concentration of epinephrine via the ____ route in a dose of ____ mg. A) 1:1,000, IM, 1 B) 1:1,000, IV, 0.5 C) 1:10,000, IV, 0.1 D) 1:10,000, IM, 0.5

C

Adverse drug reactions in elderly people are the result of: a. partial digestion secondary to delayed gastric emptying b. changes in body composition and an increase in body water c. changes in drug metabolism due to diminished hepatic function d. increased drug elimination due to decreased antidiuretic hormone

C

After delivering a patient with a high fever and dry cough to the emergency department, you are later informed that the patient was diagnosed with severe acute respiratory syndrome. Reflecting back on the care you provided to the patient, you recall being in close proximity to her because she was hearing impaired, but you do not recall wearing a protective mask. What will MOST likely happen? A) There will be no special precautions taken because you did not receive a significant exposure. B) You will be tested for HIV and hepatitis B, and will be placed on a 7-day trial of antibiotics. C) You may be quarantined for up to 10 days and will be asked to check your temperature daily. D) You will receive an immune globulin injection and will not be allowed to return to work for a week.

C

Alkali or strong acid burns to the eye should be irrigated for at least ____ minutes. A) 10 B) 15 C) 20 D) 30

C

Allergic rhinitis and asthma are MOST often the result of exposure to an allergen via the ___________ route. A) injection B) ingestion C) inhalation D) absorption

C

An individual's ability to fight off infection is called: A) virulence. B) immunity. C) host resistance. D) communicability.

C

Because of its weight-bearing capacity, the ________ spine is especially susceptible to injury. A) cervical B) thoracic C) lumbar D) coccygeal

C

Central vision is processed by the ________, which is the ________. A) retina, central portion of the macula B) pupil, circular opening within the iris C) macula, central portion of the retina D) iris, circular opening within the pupil

C

Cerebrospinal fluid is manufactured in the __________ of the brain and serves to ______________. A) subdural space, prevent infection B) cortex, protect the brain from injury C) ventricles, cushion and protect the brain D) subarachnoid space, oxygenate the brain

C

Common signs and symptoms of a panic attack include all of the following, EXCEPT: A) trembling and parasthesias. B) a sensation of being smothered. C) a prolonged loss of consciousness. D) nausea and abdominal discomfort.

C

Common signs and symptoms of meningitis include: A) irritability, back pain, headache, and hypertension. B) slow-onset fever, tinnitus, and an occipital headache. C) mental status changes, fever, stiff neck, and headache. D) a dark red rash, combativeness, and a low-grade fever.

C

Common symptoms of diabetic retinopathy include all of the following, EXCEPT: A) floaters. B) blurred vision. C) yellow vision. D) blind spots.

C

Commonly used antidepressants include all of the following, EXCEPT: A) Paxil. B) Celexa. C) Xanax. D) Zoloft.

C

Defibrillation of a patient who is in asystole is detrimental to the patient because it: A) makes ruling out underlying causes impossible. B) stops the cardiac cells from spontaneously depolarizing. C) causes an unnecessary interruption in chest compressions. D) has been shown to render epinephrine ineffective.

C

Disorganization and disorientation: A) are clinical presentations that are most common in younger patients. B) reinforce the fact that the patient's problem is psychiatric in nature. C) are ways that certain psychiatric conditions may present themselves. D) are presentations that represent a relatively small number of EMS calls.

C

During the attempted resuscitation of a patient in cardiac arrest, you look at the cardiac monitor and note that the patient's end-tidal CO2 has abruptly increased from 17 mm Hg to 40 mm Hg. You should: A) increase the rate of your ventilations. B) suspect inadvertent tube dislodgement. C) assess for a pulse for up to 10 seconds. D) decrease the rate of your ventilations.

C

During the secondary assessment of a patient experiencing a severe allergic reaction, you should: A) focus exclusively on the patient's blood pressure. B) expect the patient to vomit, often without nausea. C) apply the cardiac monitor to detect dysrhythmias. D) recall that capnography will be a less reliable tool.

C

Early signs and symptoms of hepatitis B infection include all of the following, EXCEPT: A) fatigue. B) anorexia. C) jaundice. D) low-grade fever.

C

Elderly people are more susceptible to intracranial bleeding b/c of a. a marked increase in brain size b. depletion of cerebrospinal fluid c. enlargement of the subdural space d. constriction of the cerebral vessels

C

Fever, hoarseness, and swollen lymph nodes in the neck are common clinical signs of: A) tracheitis. B) oral candidiasis. C) laryngitis. D) Ludwig angina.

C

Following the primary response to a foreign substance, the body: A) recognizes the substance as foreign, but does not produce antibodies until subsequent exposure. B) utilizes macrophages to immediately destroy the substance and eliminate it from the body. C) develops sensitivity and is able to recognize the substance following subsequent exposure. D) releases massive amounts of antigen-specific antibodies, which produce a severe allergic reaction.

C

Glaucoma is a condition caused by: A) a loss of peripheral vision. B) retinal artery occlusion. C) increased intraocular pressure. D) decreased vitreous humor.

C

Gonorrhea in females: A) is usually more acute and pronounced than it is in males. B) typically manifests as dysuria within 24 hours of infection. C) often goes unnoticed until signs of acute abdomen appear. D) remains communicable for up to 48 hours after treatment.

C

Hepatitis A is often described as a benign disease because: A) people with hepatitis A are typically asymptomatic. B) there is an effective vaccination to prevent infection. C) lifelong immunity occurs once the disease is acquired. D) no known method of transmission has been identified.

C

Histamine release causes all of the following effects, EXCEPT: A) vasodilation, which results in flushed skin and hypotension. B) contraction of the smooth muscles of the respiratory system. C) increased cardiac contractility, which results in hypertension. D) increased vascular permeability, which results in tissue edema.

C

How does CPR change after an advanced airway device is inserted? A) One breath should be delivered every 10 to 12 seconds. B) Ventilations should be faster after the device is inserted. C) Ventilations should be asynchronous with chest compressions. D) Cycles of 30 compressions and 2 breaths should be delivered.

C

Hypotension secondary to histamine release is due to: A) profound bradycardia and vascular dilation. B) decreased cardiac filling because of tachycardia. C) vasodilation and decreased cardiac contractility. D) capillary leakage and increased cardiac afterload.

C

If a cardiac arrest patient's airway is maintained with an oral airway and ventilation with a bag-mask device is producing adequate chest rise, then: A) a King LT or Combitube is preferred over tracheal intubation. B) the paramedic should deliver one breath every 5 to 6 seconds. C) insertion of an advanced airway device is not a high priority. D) there should be no pause in chest compressions to deliver a breath.

C

In addition to the supporting muscles and ligaments found in the vertebral column, the thoracic spine is further stabilized by the: A) diaphragm. B) upper sternum. C) rib attachments. D) large spinous processes.

C

In contrast to body substance isolation precautions, standard precautions: A) is a term used to describe infection control practices that reduce the risk of exposure to blood. B) describe a universal approach in which all blood and bodily fluids are assumed to be infectious. C) emphasize protection from moist body substances that may transmit bacterial or viral infections. D) specify that sweat is an effective carrier of infectious diseases, even if the sweat makes contact with intact skin.

C

In which of the following situations would endotracheal intubation MOST likely be indicated? A) The patient has an end-tidal CO2 reading of 36 mm Hg with a King LT airway in place. B) The patient is experiencing a ventricular fibrillation that is refractory to defibrillation and epinephrine. C) Following return of spontaneous circulation, the patient remains comatose. D) Bag-mask ventilation is adequate, but the patient has undergone more than 2 minutes of cardiac arrest.

C

Inability to feel or move below the level of the nipple line indicates injury to which spinal nerve root? A) C7 B) T1 C) T4 D) L1

C

Leukoplakia: A) is caused by the fungus Candida albicans. B) presents with red swollen gums that bleed easily during brushing. C) is a smoker's disease that causes excess cell growth in the mouth. D) presents with dark yellow or brown patches in the oral cavity.

C

Metal filings are particularly inflammatory because they cause a "rust ring" of the ________ after 24 hours. A) iris B) sclera C) cornea D) retina

C

Most infectious disease exposures in health care providers occur due to: A) indirect contact. B) blood splatter. C) sharps injuries. D) inhaled droplets.

C

Most psychotropic medications exert their therapeutic effects by: A) inhibiting endocrine function. B) suppressing cerebellar function. C) targeting the autonomic nervous system. D) enhancing the effects of the thyroid gland.

C

Nocturnal itching and the presence of a rash involving the hands are indicative of: A) lice. B) herpes. C) scabies. D) shingles.

C

Paralysis of cranial nerve ____ can cause unilateral facial and gag reflex paralysis. A) V B) VI C) VII D) X

C

Paralysis of the extremities would MOST likely result from injury to the: A) cerebellum. B) diencephalon. C) cerebral cortex. D) hypothalamus.

C

Regardless of the method of spinal immobilization used, you must: A) always secure the head before the torso. B) secure the patient in the position found. C) keep the head, neck, and trunk in alignment. D) stabilize the head using slight manual traction.

C

Resting tremor of an extremity and bradykinesia are associated with: a. vascular dementia b. focal motor seizures c. parkinson disease d. Alzheimer disease

C

Signs of neurogenic shock include all of the following, EXCEPT: A) bradycardia. B) flushed skin. C) diaphoresis. D) hypothermia

C

Signs or symptoms of otitis media include: A) hypersensitivity to sound. B) bilateral chronic tinnitus. C) bulging tympanic membrane. D) vertigo and violent vomiting.

C

Spinal cord injuries that cause neurogenic shock generally produce: A) cool, clammy skin distal to the site of the spinal cord injury. B) reflex tachycardia due to sympathetic nervous system stimulation. C) flaccid paralysis and complete loss of sensation distal to the injury. D) signs and symptoms that are identical to those of hypovolemic shock.

C

The FIRST step in any neurologic assessment involves: A) obtaining an initial Glasgow Coma Scale score. B) asking the patient if he or she can feel or move. C) determining the patient's level of consciousness. D) assessing the pupils for size, equality, and reactivity.

C

The LEAST common cause of death in spinal cord injury patients who are discharged from the hospital is: A) pneumonia. B) septicemia. C) muscular atrophy. D) pulmonary embolism

C

The MOST effective ways to protect your patients from nosocomial infections include: A) placing gloves on all patients with drug-resistant infections and placing a mask on yourself. B) routinely placing masks on all patients you treat and ensuring that your vaccinations are current. C) not reporting to work when you are sick and keeping the interior of the ambulance clean and disinfected. D) receiving the hepatitis B vaccine, having a titer drawn every 2 years, and wearing gloves on every EMS call.

C

The ______________ decrease(s) the weight of the skull and provide(s) resonance for the voice. A) turbinates B) nasal septum C) paranasal sinuses D) nasal mucosa

C

The afferent and efferent nerves: A) are regulated by the sympathetic nervous system. B) are receptors stimulated by the sympathetic nervous system. C) are responsible for the somatic functions of the spinal cord. D) originate from the hypothalamus and regulate temperature.

C

The innermost meningeal layer that rests directly on the brain and spinal cord is the: A) cortex. B) arachnoid. C) pia mater. D) dura mater.

C

The large cartilaginous external portion of the ear is called the: A) ossicle. B) cochlea. C) auricle. D) oval window.

C

The medical assessment of an elderly patient can be complex because: a. he or she often makes up symptoms that do not exist. b. caregivers typically prefer that you speak with them. c. he or she often has more than one medical condition. d. most elderly patients are hearing or visually impaired.

C

The middle ear consists of the: A) cochlea and semicircular canals. B) organ of Corti and the external auditory canal. C) inner portion of the tympanic membrane and the ossicles. D) pinna and the exterior portion of the tympanic membrane.

C

The occipital condyles are the: A) key anatomic structures to which the meninges are attached. B) passageways for the olfactory nerves to exit the nasal cavity. C) points of articulation between the skull and vertebral column. D) horizontal bones of the cribriform plate that contain foramina.

C

The primary infection with syphilis produces: A) numerous small pustules on the genitalia. B) low-grade fever and pain in the genital area. C) an ulcerative chancre at the site of infection. D) dysuria and a purulent discharge in the urine.

C

The term anaphylaxis is MOST accurately defined as: A) overprotection. B) atopic disease. C) without protection. D) immune suppression.

C

Treatment for a patient with neurogenic shock may include all of the following, EXCEPT: A) a vagolytic medication. B) a vasopressor medication. C) prevention of hyperthermia. D) fluid volume to maintain perfusion.

C

When disinfecting the ambulance after transporting a patient with rubella, you should: A) use a 1:1 bleach and water mixture. B) clean all surfaces with boiling water. C) use standard disinfection procedures. D) remove all equipment from the ambulance.

C

Which of the following is a complication associated with dysfunction of cranial nerves VI, VII, IX, and XII? A) Hearing impairment B) Acute ischemic stroke C) Aspiration pneumonia D) Upper airway obstruction

C

Unlike a person with bulimia nervosa, a person with anorexia nervosa: A) is less likely to experience problems related to electrolyte imbalance. B) commonly describes his or her eating disorder as "out of control." C) experiences weight loss that may cause poor health or even death. D) compensates for binge eating by using various purging methods.

C

Untreated depression in people over 65 years of age a. causes homicidal behavior in as much as 50% of this age group b. is often recognized by the p/t, who subsequently asks for help c. is associated with a higher suicide rate than in any other rage group d. usually goes unrecognized b/c it is often mistaken for delirium

C

Virulence is defined as the: A) severity of infection once an organism enters the body. B) degree of difficulty that it takes to destroy an organism. C) ability of an organism to invade and create disease in a host. D) amount of time that it takes for an organism to infect the host.

C

What physiologic effect of anaphylactic shock is seen MOST commonly in patients with neurogenic shock? A) Profound tachycardia B) Decreased cardiac contractility C) Widespread vasodilation D) Fluid leakage into the tissues

C

When a person has a personality disorder: A) the disorder most often stems from fear. B) the paramedic can influence the patient easily. C) another psychiatric illness typically is present. D) medication therapy generally is not indicated.

C

When assessing the severity of a traumatic brain injury, the MOST important assessment parameter is the patient's: A) initial Glasgow Coma Scale score. B) blood pressure. C) level of consciousness. D) response to verbal stimuli.

C

When caring for a patient experiencing a psychotic episode, you should: A) involve people the patient trusts. B) be calm, direct, and straightforward. C) first assess the situation for danger. D) clearly identify yourself to the patient.

C

When caring for a patient who is experiencing a panic attack: A) beta blockers are typically used to control tachycardia. B) you should move the patient to the ambulance quickly. C) your manner must convey that everything is under control. D) a benzodiazepine should be administered as soon as possible.

C

When evaluating a patient with a behavioral emergency, virtually all of the diagnostic information you obtain must come from: A) the medical history. B) the patient's family members. C) your conversation with the patient. D) the patient's general mood and affect.

C

When immobilizing a sitting patient with a vest-type extrication device or short backboard, you should manually stabilize his or her head and then: A) apply an appropriately sized cervical collar. B) perform a rapid assessment to detect life threats. C) assess distal pulse and sensory and motor functions. D) carefully place the vest device behind the patient.

C

When interviewing an elderly patient with a medical complaint, you should avoid: a. detailed explanations of what you are doing to him or her. b. looking directly at the patient as this may upset him or her. c. speaking to the family member first rather than the patient. d. talking about the patient in the absence of family members.

C

Which cranial nerve innervates the muscles that cause motion of the eyeballs and upper eyelids? A) Optic B) Trigeminal C) Oculomotor D) Hypoglossal

C

Which of the following conditions is the older p/t at greatest risk for a. chronic hypotension b. rheumatoid arthritis c. peripheral vascular disease d. acute myocardial infarction

C

Which of the following factors would be the LEAST likely to result in secondary spinal cord injury? A) Inflammation B) Hypothermia C) Hyperglycemia D) Hypoxemia

C

Which of the following has the greatest potential for causing an airway problem? A) Oral candidiasis B) Leukoplakia C) Ludwig angina D) A dental abscess

C

Which of the following is NOT a factor in determining a person's risk of contracting an infectious disease following exposure? A) The organism's mode of entry B) The virulence of the organism C) The age and sex of the patient D) Host resistance of the individual

C

Which of the following is NOT an identified risk factor for suicide? A) Financial setback or loss of a job B) A sudden improvement in depression C) Hispanic woman older than 55 years of age D) Recent loss of a significant relationship

C

Which of the following medications causes the MOST IgE-mediated drug reactions in the United States? A) Sulfa B) Aspirin C) Penicillin D) Erythromycin

C

Which of the following risk factors is associated with the highest rate of mortality following a burn injury in an elderly person a. age over 55 yrs b. musculoskeletal injury c. preexisting medical condition d. proportional increase in adipose tissue

C

Which of the following statements regarding schizophrenia is correct? A) Although schizophrenia is a complex disorder, it is easily treated. B) Social influences have not been shown to contribute to schizophrenia. C) The typical onset of schizophrenia occurs during early adulthood. D) In schizophrenia, dysfunctional symptoms become less prominent over time.

C

Which of the following statements regarding the EpiPen Jr is correct? A) It is used for children who weigh less than 44 lb (20 kg). B) It is contraindicated for children with a history of asthma. C) It contains 0.15 mg of a 1:2,000 solution and is given IM. D) Benadryl should be given before the EpiPen in children.

C

Which of the following statements regarding the hepatitis D virus (HDV) is MOST correct? A) The typical incubation period for HDV infection ranges from 180 to 360 days. B) The most common route of transmission of HDV is through sexual contact. C) Infection with HDV requires the host to be infected with the hepatitis B virus. D) If a documented exposure occurs, testing begins with the person who was exposed.

C

Which of the following statements regarding the mumps is correct? A) Mumps can cause sterility in males past the age of puberty. B) Postexposure vaccination against the mumps is recommended. C) Mumps presents with fever and swelling of the parotid glands. D) A variety of bacteria have been identified as causing the mumps.

C

Which of the following statements regarding vitreous humor is correct? A) Vitreous humor can be replenished, but it takes many years. B) Vitreous humor is contained within the posterior chamber of the eye. C) Vitreous humor is a jelly-like substance that maintains the shape of the globe. D) Vitreous humor is a clear fluid that is responsible for tear production.

C

Work restriction guidelines enforced by OSHA require an employee to use sick time for an illness, unless: A) the employee opted to take the hepatitis B vaccine. B) a physician deems that the illness was unavoidable. C) the illness is the result of an occupational exposure. D) the illness occurred after the post-hire probation period.

C

You and your team are attempting to resuscitate a 45-year-old man who is in cardiac arrest. After 2 minutes, you assess his cardiac rhythm and determine that he is in asystole. After instructing your team to resume CPR, you should: A) insert an advanced airway device and then resume cycles of CPR. B) establish IV or IO access and administer 1 mg of atropine sulfate rapidly. C) administer 1 mg of epinephrine 1:10,000 after obtaining vascular access. D) perform endotracheal intubation and ventilate at a rate of 15 breaths/min.

C

17. Most ingested poisons will cause: A) headache and seizures. B) tremors and weakness. C) salivation and coma. D) nausea and vomiting.

D

You are assessing an elderly man who has asthma and uses an inhaled corticosteroid on a regular basis. He has white patches on his tongue and the inside of his mouth. He is conscious and alert and his airway is patent. You should: A) suspect that he has leukoplakia. B) irrigate his mouth with sterile water. C) provide supportive care and transport. D) carefully scrape the lesions from his tongue.

C

You are dispatched to an apartment complex for a 20-year-old woman who is sick. When you arrive at the scene and begin assessing the patient, she tells you that she has been experiencing a purulent vaginal discharge, but denies vaginal bleeding or a fever. Her blood pressure is 104/64 mm Hg, pulse rate is 88 beats/min and strong, and respirations are 14 breaths/min and regular. This patient MOST likely has: A) syphilis. B) chlamydia. C) gonorrhea. D) pelvic inflammatory disease.

C

You are dispatched to an assisted living center for an 80-year old woman who is vomiting bright red blood. Upon your arrival, you find the patient sitting in a chair. She is conscious and alert, but is markedly pale and diaphoretic. Her medical history is significant for hypertension, congestive heart failure, type 2 diabetes, hypothyroidism, and osteoarthritis. Her medications include Toprol, lisinopril, Glucophage, Synthroid, and ibuprofen. Her blood pressure 76/56 mm Hg, pulse is 76 beats/min and weak, and respirations are 24 breaths/min and shallow. Which of the following statements regarding this scenario is MOST correct? a. you should suspect shock due to lower gastrointestinal bleeding, start two larger-bore IV lines, and rapidly infuse normal saline until her systolic blood pressure is greater than 90 mm Hg. b. gross hematemesis suggests gastrointestinal bleeding; however, although she is in shock, you should avoid infusing isotonic crystalloid solutions due to her congestive heart failure. c. this p/t who is in shock probably has a bleeding peptic ulcer secondary to ibuprofen use and the lack of compensatory tachycardia is likely the result of the beta blocker she is taking d. this p/t vital signs are clearly the result of the medications she is taking and you should focus on the likelihood that her pallor and diaphoresis are most likely the result of hypoglycemia

C

You have been attempting resuscitation of an 80-year-old woman for approximately 15 minutes. An advanced airway has been placed, ventilations have been delivered at the appropriate rate, high-quality CPR was performed with minimal interruptions, and rhythms-specific medications were administered. Despite your efforts, the patient remains in asystole. Which of the following statements regarding this scenario is correct? A) Transcutaneous cardiac pacing may restore a perfusing rhythm and should therefore be attempted immediately. B) It would be advisable to attempt a single defibrillation in case the patient was actually in fine ventricular fibrillation. C) You have clearly performed all the appropriate interventions and termination of resuscitative efforts should be strongly considered. D) Since older people are prone to hypothermia, and hypothermia protects the hypoxic brain, you should continue your efforts for 5 more minutes.

C

29. If a patient's hemoglobin level is only 10 g/dL, ___ % would have to be desaturated before he or she would appear cyanotic. A) 10 B) 25 C) 30 D) 50

D) 50

You receive a call to an apartment complex for a patient with an unknown behavioral problem. The scene has been secured by law enforcement prior to your arrival. The patient, a 39-year-old man, appears very anxious and worried. He tells you that he has obsessive-compulsive disorder and stopped taking his prescribed medication a week ago because he did not like its side effects. He tells you that he has had fleeting thoughts of suicide, but denies homicidal thoughts. You should: A) tell him that stopping his medications abruptly could result in death. B) transport him to the hospital and ask a police officer to accompany you. C) safely transport him to the hospital and monitor his behavior en route. D) make contact with his psychiatrist to apprise him or her of the situation.

C

aging kidneys have a decreased glomerular filtration rate, which predisposes an older person to all of the following conditions, except: a. hyperkalemia b. hypernatremia c. profound dehydration d. acute volume overload

C

an increase in blood pressure that commonly occurs with aging is physiologically exacerbated by: a. a compensatory increase in preload and cardiac ejection fraction b. a decrease in stroke volume due to age-induced cardiomyopathy c. an overproduction of collagen and decreased quantities of elastin d. the patient's noncompliance with his or her antihypertensive drugs

C

kyphosis is characterized by: a. cartilage degeneration b. vertebral disc compression c. a hunchback appearance d. lateral curvature of the spine

C

the heart hypertrophies with age, most likely in response to: a. a progressive increase in preload and chronic myocardial stretching b. left-sided heart failure that results in chronic pulmonary hypertension c. a chronically increased after load caused by arteriosclerotic blood vessels d. a significant decline in cardiac output due to a reduction stroke volume.

C

typical signs and symptoms of Meniere disease include all of following Except: a. vertigo b. tinnitus c. otorrhea d. hearing loss

C

when assessing an older person you should remember that it is common for him or her to a. slur his or her words b. present with hemiparesis c. respond slowly to questions d. not understand your questions

C

which of the following is the most common respiratory infection that causes death in the elderly population? a. pneumocystis carinii b. respiratory syncytial virus c. pneumococcus bacteria d. mycobacterium tuberculosis

C

you respond to the residence of a 68-year old man with terminal cancer. as you begin your assessment the p/t's wife tells you that he has an out-of-hospital do not resuscitate order and hands you a document that appears to be valid. the p/t who is clearly emaciated, is conscious but is in severe pain. your most appropriate action should be to: a. advise the wife that the DNR order prohibits you from rendering any prehospital care b. recall that DNR are invalid in the prehospital setting and be prepared to provide full ACLS c. provide supportive care, administer narcotic analgesia as indicated and transport the patient to the hospital d. accept the stipulations of the DNR order, obtain a signed refusal form the wife and return to service.

C

25. Which of the following substances or elements reinforces red blood cells, creating the final step in the formation of a blood clot? A) Fibrin B) Plasminogen C) Fibrinogen D) Platelets

C) Fibrinogen

39. Which of the following questions is the MOST important to ask when obtaining a patient's gynecologic history? A) "Do you have any abdominal pain?" B) "When was your last sexual encounter?" C) "When was your last menstrual period?" D) "Is there a chance that you are pregnant?"

C) "When was your last menstrual period?"

66. The typical upper limit of the fetal heart rate is: A) 120 beats/min. B) 140 beats/min. C) 160 beats/min. D) 180 beats/min.

C) 160 beats/min.

Alkali or strong acid burns to the eye should be irrigated continuously for at least ___ minutes. A) 10 B) 15 C) 20 D) 30

C) 20

51. The recommended approach for fluid resuscitation of a patient in shock is ___ mL/kg until ______. A) 20, normotension is achieved B) 10, mental status improves C) 20, radial pulses become stronger D) 10, the skin becomes pink and dry

C) 20, radial pulses become stronger

11. Which of the following scenarios would MOST likely cause a bias to decision making? A) 72-year-old female with fever, chills, and severe flank pain B) 19-year-old male with acute dyspnea and pleuritic chest pain C) 23-year-old male with crushing substernal chest pain and nausea D) 56-year-old female with acute onset confusion and slurred speech

C) 23-year-old male with crushing substernal chest pain and nausea

4. In the average female, the menstrual cycle lasts ___ days. A) 21 B) 24 C) 28 D) 35

C) 28

90. If an initial cardioversion attempt is unsuccessful in a 33-pound child, you should repeat the procedure using ___ joules: A) 10 B) 15 C) 30 D) 50

C) 30

You are caring for a 41-year-old man who was trapped in his burning house before being rescued by fire fighters. He has full-thickness burns to his head and anterior trunk, and mixed partial- and full-thickness burns to both anterior upper extremities. What percentage of his total body surface area has been burned? A) 18% B) 27% C) 36% D) 45%

C) 36%

57. Why are children more prone to croup when they acquire a viral infection than adults infected with the same virus? A) Children's immune systems are not as developed as adults', so they are more prone to infection. B) Adults were vaccinated against the virus that causes croup, whereas most children were not. C) A child's airway is narrower than an adult's, and even minor swelling can result in obstruction. D) The virus that causes croup replicates far more aggressively in children than it does in adults.

C) A child's airway is narrower than an adult's, and even minor swelling can result in obstruction.

91. You are assessing a 36-year-old woman who is in the 33rd week of her pregnancy. The patient complains of bright red vaginal bleeding, but denies abdominal pain or cramping. She tells you that she last felt her baby move about 5 or 10 minutes ago. Fetal heart tones are audible at a rate of 130 beats/min. Gentle palpation of her abdomen reveals that it is soft and nontender. Which of your assessment findings is MOST suggestive of placenta previa? A) The age of the patient B) Bright red vaginal bleeding C) Absence of abdominal pain D) Audible fetal heart tones

C) Absence of abdominal pain

20. Which of the following is an element of the Fick principle? A) Adequate production of pyruvic acid B) A relatively constant end-tidal CO2 C) An adequate number of red blood cells D) Consistent white blood cell production

C) An adequate number of red blood cells

33. Which of the following statements regarding diaphragmatic injury is correct? A) During the latent phase of a diaphragmatic injury, abdominal contents herniate through the defect, cutting off their blood supply. B) The spleen provides significant protection to the diaphragm on the left side, resulting in a higher incidence of right-sided diaphragmatic injuries. C) Because the diaphragm is protected by the liver on the right side, most diaphragmatic injuries caused by blunt force trauma occur on the left side. D) Once the diaphragm has been injured, the healing process is facilitated by the natural pressure differences between the abdominal and thoracic cavities.

C) Because the diaphragm is protected by the liver on the right side, most diaphragmatic injuries caused by blunt force trauma occur on the left side.

101. Which of the following is the MOST easily correctable problem in a child with an altered mental status? A) Ingestion of aspirin 2 hours ago B) High fever with a widespread rash C) Blood glucose reading of 40 mg/dL D) Dehydration associated with hypokalemia

C) Blood glucose reading of 40 mg/dL

59. Which of the following vital signs values are MOST consistent with neurogenic shock? A) Blood pressure, 120/70; pulse, 70; respirations, 14 B) Blood pressure, 160/100; pulse, 40; respirations, 8 C) Blood pressure, 80/60; pulse, 50; respirations, 24 D) Blood pressure, 70/40; pulse, 120; respirations, 26

C) Blood pressure, 80/60; pulse, 50; respirations, 24

118. Which of the following is LEAST characteristic of an apparent life-threatening event in an infant? A) Pallor or cyanosis B) A period of apnea C) Brief loss of a pulse D) Loss of muscle tone

C) Brief loss of a pulse

23. Which of the following lists, in the correct order, the smallest to largest units of life? A) Tissues, organs, cells, organism, organ systems B) Organs, organ systems, tissues, cells, organism C) Cells, tissues, organs, organ systems, organism D) Organism, organs, organ systems, tissues, cells

C) Cells, tissues, organs, organ systems, organism

125. Which of the following statements regarding chest trauma in children is correct? A) The pliability of children's rib cages predisposes them to sternal fractures. B) Signs of a pneumothorax are often more obvious in children than in adults. C) Children are more prone to intrathoracic trauma due to compression forces. D) Most cases of fatal chest trauma occur in children who fall more than 10 feet.

C) Children are more prone to intrathoracic trauma due to compression forces.

A 50-year-old man presents with a painful penile erection that has persisted for the past several hours. He is conscious, but restless, and his vital signs are stable. Which of the following conditions could cause his clinical presentation? A) Renal failure B) Head trauma C) Cocaine abuse D) Nitroglycerin use

C) Cocaine abuse

5. Which of the following types of maltreatment involves isolating the victim from others, resulting in a substantial change in the victim's behavior or cognitive function? A) Neglect B) Abandonment C) Emotional abuse D) Sexual exploitation

C) Emotional abuse

7. While preparing equipment for newborn resuscitation, which of the following items is/are NOT considered optional? A) Pulse oximeter B) Cardiac monitor C) Endotracheal tubes D) Laryngeal mask airway

C) Endotracheal tubes

69. Which of the following clinical findings is MOST suggestive of pneumonia in a patient with COPD? A) Nonproductive cough B) White sputum and rales C) Fever and localized crackles D) Dyspnea and diffuse wheezing

C) Fever and localized crackles

49. Which of the following statements regarding fever in the newborn is correct? A) Fever in newborns is defined as a rectal temperature greater than 99.0°F. B) The ability of the newborn to dissipate heat through sweating is prominent. C) Fever may not always be a presenting feature in newborns with an infection. D) Because of their active immune systems, newborns commonly experience fever.

C) Fever may not always be a presenting feature in newborns with an infection

34. Which of the following statements regarding the human papillomavirus (HPV) is correct? A) HPV infection is characterized by fever and a genital chancroid. B) Genital warts caused by HPV are far more common in females. C) HPV has been identified as a causative agent in cervical cancer. D) Only one type of HPV is spread via unprotected sexual contact.

C) HPV has been identified as a causative agent in cervical cancer.

64. You are dispatched to a residence for an injured person. The scene has been secured by law. The patient, a young female, tells you that her boyfriend kicked her in the chest yesterday during an argument. Your assessment reveals that the patient is in significant pain, is dyspneic, has a strong heart rate of 98 beats/min, and has an area of ecchymosis over her left lower rib cage. Auscultation to the left side of her chest reveals coarse crackles. Which of the following treatment interventions is likely NOT indicated for this patient? A) Cardiac monitoring B) Titrated IV analgesics C) IV fluid boluses D) End-tidal CO2 monitoring

C) IV fluid boluses

67. A 45-year-old man was cutting down a large tree when it collapsed, pinning him at the thighs. The patient's wife found him approximately 4 hours after the incident. The patient is conscious and in severe pain. His blood pressure is 128/68 mm Hg, pulse is 120 beats/min and regular, and respirations are 22 breaths/min with adequate depth. In addition to supplemental oxygen and cardiac monitoring, which of the following treatments should you provide BEFORE the tree is removed from his legs? A) Two large-bore IV lines with a 3- to 4-L crystalloid bolus and 1 mEq/kg of sodium bicarbonate B) A large-bore IV line set to keep the vein open and 10 mL of a 10% solution of calcium chloride C) IV therapy with a crystalloid fluid bolus and albuterol via nebulizer or mucosal atomizer device D) An IV of normal saline, fluid restriction, and amiodarone if the ECG shows a widened QRS complex

C) IV therapy with a crystalloid fluid bolus and albuterol via nebulizer or mucosal atomizer device

55. Which of the following statements regarding peritoneal dialysis is correct? A) A patient undergoing peritoneal dialysis will have an AV shunt. B) Peritoneal dialysis can only be performed in a hospital setting. C) In peritoneal dialysis, fluid is instilled into the abdominal cavity. D) Peritoneal dialysis involves filtering the blood through a machine.

C) In peritoneal dialysis, fluid is instilled into the abdominal cavity.

1. Which of the following would MOST likely allow you to reach the status of entry-level competence? A) Functioning for at least 2 years in the field B) Passing the state or national paramedic exam C) Interning with an experienced preceptor D) Studying didactic material at least 2 hours per week

C) Interning with an experienced preceptor

78. You and your team are treating an unresponsive trauma patient. Which of the following questions would be the MOST relevant to ask your team when evaluating for the presence of a tension pneumothorax? A) What is the patient's systolic blood pressure? B) Are the patient's heart tones becoming muffled? C) Is the patient becoming difficult to ventilate? D) Does the patient's pulse weaken during inhalation?

C) Is the patient becoming difficult to ventilate?

47. Which of the following anticonvulsant medications would MOST likely be administered to a newborn with seizures in the prehospital setting? A) Dilantin B) Depakote C) Lorazepam D) Phenobarbital

C) Lorazepam

28. What type of fracture occurs at an angle across the bone and is typically caused by direct or twisting force? A) Spiral B) Linear C) Oblique D) Transverse

C) Oblique

Which of the following cranial nerves innervates the muscles that cause motion of the eyeballs and upper eyelids? A) Optic B) Ophthalmic C) Oculomotor D) Trigeminal

C) Oculomotor

89. Which of the following statements regarding poliomyelitis (polio) is correct? A) Any warm-blooded animal can serve as a host for the polio virus. B) Patients with nonparalytic polio usually have permanent neurologic injury. C) Polio initially presents with a headache, sore throat, fever, and vomiting D) Subclinical polio is characterized by symptoms that last less than a month.

C) Polio initially presents with a headache, sore throat, fever, and vomiting

69. Which of the following statements regarding nasogastric (NG) and orogastric (OG) insertion in children is correct? A) Gastric decompression with an NG or OG tube is only appropriate for children older than 10 years of age. B) The correct size NG or OG tube for a child should be half the ET tube size that he or she would need. C) Prior to inserting an NG or OG tube in an unresponsive child without a gag reflex, you should intubate his or her trachea. D) Insertion of an orogastric tube is contraindicated in children with severe head trauma or injury to the midface.

C) Prior to inserting an NG or OG tube in an unresponsive child without a gag reflex, you should intubate his or her trachea.

Which of the following statements regarding pyelonephritis is correct? A) Untreated pyelonephritis typically heals spontaneously. B) Most cases of pyelonephritis occur in the lower urinary tract. C) Pyelonephritis is an inflammation of the linings of the kidney. D) In pyelonephritis, the ureters become inflamed and infected.

C) Pyelonephritis is an inflammation of the linings of the kidney.

55. Which of the following disorders or conditions is related to decreased glycogen stores in the newborn? A) Large for gestational age B) Hypoxia or hypothermia C) Small for gestational age D) Maternal diabetes mellitus

C) Small for gestational age

161. You are assessing a 10-year-old child with apparent ventricular tachycardia, but cannot decide whether electrical or pharmacologic therapy is the most appropriate initial treatment approach. Which of the following interventions would pose the GREATEST potential for harm? A) Administering high-flow oxygen and obtaining a 12-lead ECG tracing B) Establishing IO access, administering a sedative, and cardioverting at 15 joules C) Starting an IV line and administering amiodarone followed by procainamide D) Establishing vascular access and rapidly administering 3 mg of adenosine

C) Starting an IV line and administering amiodarone followed by procainamide

81. Rheumatoid arthritis: A) is more common in the elderly. B) does not respond to analgesia. C) can affect other body systems. D) occurs secondary to an infection.

C) can affect other body systems.

56. When treating a dialysis patient, it is especially important to: A) keep the patient in a supine position. B) elevate the arm with the AV fistula. C) carefully titrate any IV fluids given. D) treat acidosis with sodium bicarbonate.

C) carefully titrate any IV fluids given.

A 4-year-old boy pulled a pot of boiling water off of the stove and experienced partial-thickness splash burns to his neck, anterior trunk, and both anterior arms. During your assessment, you note that the child is conscious but is not crying. He is tachypneic and tachycardic, and his skin is cool and moist. Other than the burns, there are no other gross injuries. Which of the following statements regarding this scenario is correct? A) You should assist the child's ventilations and prepare to intubate his trachea. B) An IV should be established and you should administer a 20-mL/kg bolus of D5W. C) The child may be hypoglycemic and requires assessment of his blood glucose level. D) It is likely that this child's burn was intentionally inflicted and you should report it.

C) The child may be hypoglycemic and requires assessment of his blood glucose level.

17. Which of the following statements regarding the umbilical cord is correct? A) The placenta receives arteriovenous blood from the umbilical arteries. B) The umbilical cord is composed of two large veins and one small artery. C) The umbilical vein carries oxygenated blood from the placenta to the fetus. D) The fetus receives the majority of its oxygen supply from the umbilical arteries.

C) The umbilical vein carries oxygenated blood from the placenta to the fetus.

57. Which of the following is a disadvantage of using a crystalloid solution when treating a patient with hemorrhagic shock? A) They cause platelets to clump together. B) They do not expand the circulating volume. C) They do not have oxygen-carrying capacity.. D) They increase the viscosity of the blood.

C) They do not have oxygen-carrying capacity..

85. Which of the following is the first-line treatment for a hemodynamically unstable child with bradycardia? A) Epinephrine IV or IO B) Chest compressions C) Ventilatory support D) Transcutaneous pacing

C) Ventilatory support

18. Diabetes is MOST accurately defined as: A) inadequate insulin secretion, which leads to increased circulating blood glucose levels. B) a decrease in circulating insulin levels, which results in a drop in serum blood sugar levels. C) a metabolic disorder in which the body's ability to metabolize simple carbohydrates is impaired. D) an endocrine disorder in which the liver is unable to produce and store adequate amounts of glycogen.

C) a metabolic disorder in which the body's ability to metabolize simple carbohydrates is impaired.

80. A person who experiences sharp chest pain followed by increasing dyspnea after he or she coughs MOST likely has: A) pleurisy. B) a pleural effusion. C) a pneumothorax. D) acute pneumonia.

C) a pneumothorax.

123. An infant or small child who falls from a significant height would MOST likely experience: A) lumbar spine fractures. B) lateral thoracic trauma. C) a traumatic brain injury. D) bilateral femur fractures.

C) a traumatic brain injury.

66. Down syndrome is a condition in which: A) chromosome 23 fails to separate. B) the ovum contains 21 chromosomes. C) a triplication of chromosome 21 occurs. D) the human somatic cell contains 24 chromosomes.

C) a triplication of chromosome 21 occurs.

6. A man leaves his 4-year-old child home alone while he plays golf with his friends. This is an example of: A) assault. B) neglect. C) abandonment. D) emotional abuse.

C) abandonment.

46. The MOST common cause of Addisonian crisis is: A) overwhelming stress. B) a severe acute infectious process. C) abrupt termination of corticosteroid use. D) acute failure of one or both of the adrenal glands.

C) abrupt termination of corticosteroid use.

90. A 30-year-old woman presents with bright red vaginal bleeding and severe abdominal pain. She tells you that she is 35 weeks pregnant and that this episode began suddenly about 30 minutes ago. She further tells you that she has not felt the baby move in over an hour. As your partner is treating the patient for shock, you obtain her medical history. The patient tells you that she has high blood pressure and admits to using cocaine throughout her pregnancy. This patient is MOST likely experiencing: A) placenta previa. B) a ruptured uterus. C) abruptio placenta. D) a threatened abortion.

C) abruptio placenta.

104. You are dispatched to a residence for a young woman with difficulty breathing. When you arrive, you find the patient sitting in a tripod position, noticeably dyspneic and tachypneic. She tells you that she experienced a sudden sharp pain to the left side of her chest and then started having trouble breathing. She denies any past medical history and states that she only takes birth control pills. Based on this patient's clinical presentation, you should be MOST suspicious for: A) a pleural effusion. B) spontaneous pneumothorax. C) acute pulmonary embolism D) hyperventilation syndrome.

C) acute pulmonary embolism

151. A 9-year-old, 55-pound girl presents with generalized hives, marked facial swelling, and loud inspiratory stridor. She is conscious but appears sleepy. You can MOST rapidly improve this child's condition by: A) starting an infusion of epinephrine at 5 µg/min. B) administering up to 50 mg of diphendydramine. C) administering 0.25 mg of epinephrine IM. D) intubating her and administering albuterol via the ET tube.

C) administering 0.25 mg of epinephrine IM.

98. A 21-year-old man experienced an acute onset of pleuritic chest pain and dyspnea while playing softball. He is noticeably dyspneic, has an oxygen saturation of 93% on room air, and has diminished breath sounds to the upper right lobe. The MOST appropriate treatment for this patient involves: A) performing a needle decompression to the right side of his chest. B) assisting his ventilations in order to increase his oxygen saturation. C) administering high-flow supplemental oxygen and transporting at once. D) applying a CPAP unit and starting an IV line en route to the hospital.

C) administering high-flow supplemental oxygen and transporting at once.

51. Unlike hypothyroidism, hyperthyroidism: A) results in a decreased cardiac output. B) causes a decrease in the metabolic rate. C) causes an increase in oxygen demand. D) often results in acute myxedema coma.

C) causes an increase in oxygen demand.

51. Unlike a tension pneumothorax, a massive hemothorax would MOST likely present with: A) signs of shock. B) tracheal deviation. C) collapsed jugular veins. D) severe respiratory distress.

C) collapsed jugular veins.

56. A sudden increase in end-tidal CO2 may be the earliest indicator of: A) ineffective chest compressions. B) inadvertent ET tube dislodgment. C) return of spontaneous circulation. D) developing respiratory alkalosis.

C) return of spontaneous circulation.

156. You receive a call for a "sick child." When you arrive at the scene, the child's mother tells you that her 5-year-old son has had vomiting and diarrhea for the past day and will not eat or drink anything. On exam, the child's level of consciousness appears consistent with his age. His skin is cool and pale, he is tachypneic, his capillary refill time is 4 seconds, and his heart rate is 150 beats/min. The MOST appropriate treatment for this child involves: A) applying high-flow oxygen via pediatric nonrebreathing mask, assessing his blood glucose level, elevating his legs 12 inches, and transporting at once. B) establishing IV access and administering a 20-mL/kg normal saline bolus, applying high-flow oxygen, administering 25% dextrose, and transporting. C) administering supplemental oxygen, keeping the child warm, assessing his blood glucose level, transporting, and establishing vascular access en route. D) administering supplemental oxygen, starting an IV line, assessing his blood glucose level, delivering at least two 20-mL/kg normal saline boluses, and transporting.

C) administering supplemental oxygen, keeping the child warm, assessing his blood glucose level, transporting, and establishing vascular access en route.

41. Adrenal insufficiency is characterized by decreased function of the ______________ and consequent underproduction of ______________. A) adrenal glands, catecholamines B) adrenal medulla, norepinephrine C) adrenal cortex, cortisol and aldosterone D) adrenal glands, epinephrine and norepinephrine

C) adrenal cortex, cortisol and aldosterone

15. The force or resistance against which the heart pumps is called: A) preload. B) blood pressure. C) afterload. D) systemic vascular resistance.

C) afterload.

59. A 50-year-old man was working on his car when the jacks collapsed and the car landed on his chest. Your assessment reveals profound cyanosis and swelling to his chest and face, agonal respirations, and a weak carotid pulse. This patient will benefit MOST from: A) bilateral needle thoracenteses and 100% oxygen. B) rapid IV fluid boluses en route to a trauma center. C) aggressive airway management and rapid transport. D) prompt endotracheal intubation and hyperventilation.

C) aggressive airway management and rapid transport.

31. The barrel-chest appearance classically seen in emphysemic patients is secondary to: A) widespread atelectasis. B) chest wall hypertrophy. C) air trapping in the lungs. D) carbon dioxide retention.

C) air trapping in the lungs.

29. The clinical presentation of a hypoglycemic patient would MOST likely resemble that of a patient with: A) classic heatstroke. B) an opiate overdose. C) alcohol intoxication. D) a hemorrhagic stroke.

C) alcohol intoxication.

12. If the body experiences a drop in volume or blood pressure: A) aldosterone stimulates the sweat glands, resulting in diaphoretic skin. B) adrenocorticotropic hormone causes a reduction in the secretion of cortisol. C) aldosterone secretion stimulates the kidneys to reabsorb sodium from the urine. D) catecholamine release inhibits the conversion of glycogen to glucose in the liver.

C) aldosterone secretion stimulates the kidneys to reabsorb sodium from the urine.

28. A fenestrated tracheostomy tube: A) should be used for patients who require constant support from a ventilator. B) is not equipped with an inflatable cuff and is secured in place with sutures. C) allows the patient to speak, breathe, or clear secretions from the upper airway. D) requires a special adapter in order to be compatible with a ventilation device.

C) allows the patient to speak, breathe, or clear secretions from the upper airway.

6. Infection, hypertensive crisis, and medication noncompliance are differential diagnoses that should be considered when treating a patient who presents with: A) acute chest pain. B) unilateral weakness. C) altered mental status. D) vomiting and diarrhea.

C) altered mental status.

27. A pulmonary contusion following blunt chest trauma results in: A) blood leakage from injured lung tissue into the pleural space. B) decreased pulmonary shunting with rupture of the alveolar sacs. C) alveolar and capillary damage with intraparenchymal lung hemorrhage. D) pulmonary vasodilation as the body attempts to shunt blood to the injury

C) alveolar and capillary damage with intraparenchymal lung hemorrhage.

44. A typical finding for a foreign body aspiration is: A) a child with recent flu-like symptoms who presents with acute stridor. B) an otherwise healthy child with a progressive increase in work of breathing. C) an afebrile child with a sudden onset of coughing or gagging while playing. D) a temperature less than 102°F with sudden drooling, crowing, and dyspnea.

C) an afebrile child with a sudden onset of coughing or gagging while playing.

74. A patient who is coughing up thick pulmonary secretions should NOT take: A) a diuretic. B) antihistamines. C) an antitussive. D) bronchodilators.

C) an antitussive.

5. When a child experiences a low cardiac output state, he or she relies MOST on: A) increased tidal volume. B) central vasoconstriction. C) an increase in heart rate. D) increased stroke volume.

C) an increase in heart rate

49. A patient who is coughing up purulent sputum is MOST likely experiencing: A) emphysema. B) dehydration. C) an infection. D) pulmonary edema.

C) an infection.

Thermal burns are MOST commonly caused by exposure to: A) hot liquids. B) hot solid objects. C) an open flame. D) superheated steam.

C) an open flame

The MOST important therapy the paramedic can administer to a patient with an isolated renal calculus is: A) oxygen. B) rehydration. C) analgesia. D) an antiemetic.

C) analgesia.

10. Most of the heart is protected: A) by the anterior rib cage. B) by the tough pericardium. C) anteriorly by the sternum. D) by the sternal manubrium.

C) anteriorly by the sternum.

87. Use of a spacer device in conjunction with a metered-dose inhaler: A) is only indicated in children under 6 years of age, who are generally not able to use the inhaler effectively. B) may be required when assisting a patient who is breathing inadequately, but generally results in less medication delivery to the lungs. C) collects medication as it is released from the canister, allowing more to be delivered to the lungs and less to be lost to the environment. D) requires the patient to time his or her inhalation to coincide with the discharge of the metered-dose inhaler.

C) collects medication as it is released from the canister, allowing more to be delivered to the lungs and less to be lost to the environment.

43. You are dispatched to a grocery store for a woman with severe abdominal pain. When you arrive, you find the patient lying on her side in the manager's office. She is confused, is diaphoretic, and appears to be bleeding from her vagina. Her blood pressure is low, and her pulse and respiratory rates are elevated. You should: A) administer 100% oxygen, elevate her legs 12 to 18 inches, keep her warm, establish vascular access and give a 500-mL fluid bolus, reassess her blood pressure, and transport at once with continuous monitoring en route to the hospital. B) immediately place her in a supine position, firmly massage her uterine fundus to control the bleeding, establish two large-bore IV lines and run them wide open, transport at once, and apply high-flow oxygen en route to the hospital. C) apply high-flow oxygen, visually inspect her vagina and cover it with sterile dressings, keep her warm, begin transport, establish at least one large-bore IV en route, and administer enough crystalloid fluid to maintain radial pulses. D) assist her ventilations with a bag-mask device, carefully place a trauma dressing inside her vagina to control the bleeding, begin rapid transport, establish two large-bore IV lines en route, and administer 20-mL/kg fluid boluses as needed.

C) apply high-flow oxygen, visually inspect her vagina and cover it with sterile dressings, keep her warm, begin transport, establish at least one large-bore IV en route, and administer enough crystalloid fluid to maintain radial pulses.

139. Most injuries in pediatric patients: A) can be totally eliminated with training. B) involve trauma to the chest and spine. C) are predictable and preventable events. D) occur due to gross caregiver negligence.

C) are predictable and preventable events.

16. Diarthroses: A) are also referred to as fused joints. B) is the process of bone deterioration. C) are the most mobile joints of the body. D) secrete synovial fluid into the joint cavity.

C) are the most mobile joints of the body.

12. During your assessment of a 4-year-old female, you find injury patterns that are highly suggestive of abuse. The child's mother is present; she is apprehensive and is hesitant to allow you to transport her daughter. Your MOST appropriate action should be to: A) ask the mother to leave the room so that you can ask the girl if the injuries that she has were intentionally inflicted by someone else. B) advise the mother that the child's injuries are inconsistent with accidental trauma and let her know that the child needs to be transported. C) ask your partner to retrieve an extra set of latex-free gloves from the ambulance, as this will enable him or her to discreetly call the police. D) act as though you are not suspicious, but call the child's father and inquire about the relationship between the child and her mother.

C) ask your partner to retrieve an extra set of latex-free gloves from the ambulance, as this will enable him or her to discreetly call the police.

When caring for a patient with fractured or avulsed teeth following an assault, you should: A) handle any avulsed teeth by the root only, not the crown. B) flush the patient's mouth with sterile water for 20 minutes. C) assess the knuckles of the person who assaulted the patient. D) remove any partially avulsed teeth and immerse them in water.

C) assess the knuckles of the person who assaulted the patient.

The large cartilaginous external portion of the ear is called the: A) ossicle. B) cochlea. C) auricle. D) oval window.

C) auricle.

20. The work-of-breathing component of the pediatric assessment triangle includes all of the following, EXCEPT: A) listening for grunting or audible wheezing. B) noting the child's position during breathing. C) auscultating the lungs for adventitious sounds. D) looking for substernal or intercostal retractions.

C) auscultating the lungs for adventitious sounds.

38. Any patient with a presumptive diagnosis of a pneumothorax should: A) receive a prophylactic needle thoracentesis. B) be intubated and ventilated at a rate of 15 breaths/min. C) be considered unstable and reassessed every 5 minutes. D) be transported to a trauma center via air medical transport.

C) be considered unstable and reassessed every 5 minutes.

99. You are transporting a patient with a long history of emphysema. The patient called 9-1-1 because his shortness of breath has worsened progressively over the past few days. He is on high-flow oxygen via nonrebreathing mask and has an IV of normal saline in place. The cardiac monitor shows sinus tachycardia and the pulse oximeter reads 89%. When you reassess the patient, you note that his respiratory rate and depth have decreased. You should: A) remove the nonrebreathing mask and apply a nasal cannula. B) administer a sedative and a paralytic and then intubate his trachea. C) begin assisting his ventilations with a bag-mask and 100% oxygen. D) insert a nasal airway, apply a CPAP unit, and notify medical control.

C) begin assisting his ventilations with a bag-mask and 100% oxygen.

21. Morbid obesity is defined as a body mass index that is: A) greater than 30 kg/m2. B) between 30 and 39.9 kg/m2. C) between 40 and 49.9 kg/m2. D) greater than 50 kg/m2.

C) between 40 and 49.9 kg/m2.

4. Causes of delayed fetal transition include all of the following, EXCEPT: A) acidosis. B) hypothermia. C) birth at 41 weeks. D) meconium aspiration.

C) birth at 41 weeks.

32. It is important to remember that blood pressure is only one component in the overall assessment of a child because: A) it is an unreliable measurement of perfusion in all children. B) hypotension is seen much earlier in children than in adults. C) blood pressure may remain adequate in compensated shock. D) it generally yields a falsely low reading in agitated children

C) blood pressure may remain adequate in compensated shock.

16. Choanal atresia is defined as a: A) small chin that causes a posteriorly positioned tongue. B) condition in which high-flow oxygen causes blindness. C) bony or membranous obstruction of the back of the nose. D) condition in which the occipital skull is abnormally large.

C) bony or membranous obstruction of the nose

38. During multiple-organ dysfunction syndrome, the release of __________, a potent vasodilator, leads to tissue hypoperfusion and may contribute to hypotension. A) renin B) epinephrine C) bradykinin D) angiotensin

C) bradykinin

25. An unresponsive patient who overdosed on a central nervous system depressant drug would be expected to have __________ respirations. A) eupneic B) hyperpneic C) bradypneic D) Kussmaul

C) bradypneic

63. The primary treatment of bronchospasm is: A) assisted ventilation. B) humidified oxygen. C) bronchodilator therapy. D) corticosteroid therapy.

C) bronchodilator therapy.

72. Infants and children in shock: A) typically become hypotensive sooner than adults because of a relative decrease in total blood volume. B) generally remain alert for longer periods than adults despite a significant decrease in cerebral perfusion. C) compensate more efficiently than adults by increasing heart rate and peripheral vascular resistance. D) maintain end-organ perfusion longer than adults, making capillary refill time a less reliable perfusion indicator.

C) compensate more efficiently than adults by increasing heart rate and peripheral vascular resistance.

32. If a patient's initial presentation makes you suspicious about a particular respiratory condition, you must: A) begin immediate treatment based on your suspicion. B) make your field impression based on the presentation. C) confirm your suspicions with a thorough assessment. D) immediately perform a focused physical examination.

C) confirm your suspicions with a thorough assessment.

49. When urine becomes evident in the tubing during insertion of an indwelling urinary catheter in a male, you should: A) connect the indwelling catheter to the urine drainage system, unclamp the tubing, and allow urine to drain. B) inflate the balloon with the prefilled syringe and gently pull back on the catheter until you feel resistance. C) continue inserting the catheter until the Y between the drainage port and the balloon port is at the tip of the penis. D) insert the catheter approximately 1 inch farther, inflate the balloon, and then pull back on the catheter until you feel resistance.

C) continue inserting the catheter until the Y between the drainage port and the balloon port is at the tip of the penis.

18. When a patient is receiving palliative care, medical care: A) ceases, and the disease process is allowed to continue until the point of death. B) ceases, and efforts focus on relieving pain until the point at which the patient dies. C) continues, although aggressive, invasive, and uncomfortable interventions cease. D) continues, and only minimally invasive procedures are performed to prolong life.

C) continues, although aggressive, invasive, and uncomfortable interventions cease.

45. Immediate treatment for an open pneumothorax involves: A) assisting ventilations with a bag-mask device. B) covering the open wound with a porous dressing. C) converting the pneumothorax to a closed injury. D) administering oxygen via nonrebreathing mask.

C) converting the pneumothorax to a closed injury.

3. The eighth, ninth, and tenth ribs are indirectly attached to the sternum by the: A) manubrium. B) angle of Louis. C) costal cartilage. D) suprasternal notch.

C) costal cartilage.

General care for an eye injury involves: A) applying direct pressure to the globe. B) irrigating the eye with sterile saline solution. C) covering both eyes to minimize further injury. D) applying a cold compress to the eyeball.

C) covering both eyes to minimize further injury.

You are dispatched to a high school where a 16-year-old male was stabbed in the eye with a pencil. The patient is conscious and in severe pain. A classmate removed the pencil prior to your arrival. The MOST appropriate care for this patient's injury includes: A) irrigating the injured eye with sterile saline, covering both eyes with a protective eye shield, and transporting immediately. B) applying an icepack to the affected eye, administering 1 µg/kg of fentanyl IM, elevating the patient's legs, and transporting. C) covering the affected eye with a sterile dressing and protective eye shield, covering the unaffected eye, and transporting promptly. D) covering the affected eye with a moist, sterile dressing, applying gently pressure to reduce intraocular pressure, and transporting at once.

C) covering the affected eye with a sterile dressing and protective eye shield, covering the unaffected eye, and transporting promptly.

81. One of the hallmarks of a pulmonary embolism is: A) the disappearance of radial pulses during inhalation. B) pleuritic chest pain that occurs after a strong cough. C) cyanosis that does not resolve with oxygen therapy. D) jugular venous distention while in a supine position.

C) cyanosis that does not resolve with oxygen therapy.

95. Prior to administering pharmacologic therapy to an infant or child with pulseless ventricular tachycardia, the paramedic should perform: A) intubation. B) cardioversion. C) defibrillation. D) CPR for 5 minutes.

C) defibrillation.

41. Signs of bursitis include all of the following, EXCEPT: A) warmth. B) swelling. C) deformity. D) erythema.

C) deformity.

31. Assessment of a child in a cold environment would MOST likely yield: A) a rapid, weak pulse. B) flushing of the skin. C) delayed capillary refill. D) a slow, irregular pulse.

C) delayed capillary refill.

72. When an atrial septal defect is present: A) blood flow to the lungs is significantly decreased, which leads to severe hypoxemia. B) the heart must push harder to force blood flow past a narrowed aorta, resulting in increased afterload. C) deoxygenated blood is able to shift from one atrium to the other and mix with oxygen-rich blood. D) blood is allowed to bypass the right ventricle and lungs due to the fetus's lungs being filled with fluid.

C) deoxygenated blood is able to shift from one atrium to the other and mix with oxygen-rich blood

Unlike partial-thickness burns, full-thickness burns: A) extend completely through the epidermis and produce severe pain. B) are characterized by reddened, moist skin and large fluid-filled blisters. C) destroy the base membrane of the dermis that produces new skin cells. D) often heal spontaneously over a long period of time with massive scarring.

C) destroy the base membrane of the dermis that produces new skin cells.

49. The goal of prehospital pain control in a patient with a musculoskeletal injury should be to: A) use cryotherapy instead of narcotic analgesics. B) sedate the patient with diazepam or lorazepam. C) diminish the patient's pain to a tolerable level. D) give enough analgesia to make the patient pain-free.

C) diminish the patient's pain to a tolerable level.

18. A patent airway: A) is evidenced by visible chest rise. B) should be prophylactically suctioned. C) does not equate to adequate ventilation. D) is characterized by adequate tidal volume

C) does not equate to adequate ventilation.

After an adult victim is struck by lightning and experiences cardiac arrest: A) 5 minutes of CPR generally restores a pulse. B) perform a compression to ventilation ratio of 15:2. C) his or her heart may resume beating spontaneously. D) the ECG usually shows an organized cardiac rhythm.

C) his or her heart may resume beating spontaneously.

76. If your EMS system protocols permit you to administer oxytocin to the mother following birth to control severe postpartum bleeding, it is MOST important to: A) administer a 500-mL normal saline bolus. B) first massage the uterus for at least 5 minutes. C) ensure that she is not delivering a second baby. D) obtain at least two blood pressure readings first.

C) ensure that she is not delivering a second baby.

Upon initial contact with a severely burned patient, you must: A) assess airway and breathing adequacy. B) cover the patient to prevent hypothermia. C) ensure that the patient is not still burning. D) quickly establish the extent of the burns.

C) ensure that the patient is not still burning.

111. The management for any potentially toxic exposure in children begins by: A) identifying the toxin. B) providing an antidote. C) ensuring a patent airway. D) assessing respiratory effort.

C) ensuring a patent airway.

64. The MOST immediate treatment for the patient with a tension pneumothorax is to: A) establish a large-bore IV line. B) ventilate with a bag-mask device. C) evacuate air from the pleural space. D) apply high-flow supplemental oxygen.

C) evacuate air from the pleural space.

71. The majority of acquired hearing loss in children and adults is the result of: A) a tumor on the acoustic nerve. B) frequent middle ear infections. C) excessive exposure to loud noise. D) long-term use of salicylates.

C) excessive exposure to loud noise.

The hyoid bone: A) articulates with the temporal bone at the temporomandibular joint. B) is anatomically attached to the skull and supports the vocal cords. C) floats in the superior aspect of the neck just below the mandible. D) is a hollowed section of bone that provides resonance for the voice

C) floats in the superior aspect of the neck just below the mandible.

Burn shock is caused by: A) a massive infection that occurs when microorganisms breach burned skin. B) renal failure secondary to excess myoglobin production from burned muscle. C) fluid loss across damaged skin and volume shifts within the rest of the body. D) acute dehydration, and it commonly manifests within 30 minutes after the burn.

C) fluid loss across damaged skin and volume shifts within the rest of the body.

82. When delivering multiple babies, you should clamp and cut the umbilical cord: A) after the placenta has delivered. B) only after the first baby delivers. C) following delivery of each baby. D) after all the babies have delivered.

C) following delivery of each baby.

15. The diaphragm: A) is an accessory muscle used during respiratory distress. B) works in conjunction with the sternum during inspiration. C) forms a barrier between the thoracic and abdominal cavities. D) creates positive intrathoracic pressure when it increases in size.

C) forms a barrier between the thoracic and abdominal cavities.

15. A newborn with central cyanosis, adequate respirations, and a heart rate of 120 beats/min should initially be treated with: A) continued observation only. B) high-flow oxygen via mask. C) free-flow oxygen at 5 L/min. D) positive-pressure ventilation.

C) free-flow oxygen at 5 L/min.

9. The dome-shaped top of the uterus is called the: A) os. B) body. C) fundus. D) cervix.

C) fundus.

2. A function of the musculoskeletal system is hematopoiesis, which is defined as the process of: A) filtering the blood. B) destroying platelets. C) generating blood cells. D) producing bone marrow.

C) generating blood cells.

21. The energy for muscular contraction is derived from the metabolism of _______ and results in the production of _______. A) oxygen, glucose B) pyruvic acid, oxygen C) glucose, lactic acid D) lactate, carbon dioxide

C) glucose, lactic acid

37. The leading cause of life-threatening infections in newborns is: A) candidiasis. B) bacterial vaginosis. C) group B streptococcus. D) human immunodeficiency virus.

C) group B streptococcus.

A patient with a dysconjugate gaze following an ocular injury: A) most likely has a concomitant basilar skull fracture. B) should have ice applied to the eyes to prevent blindness. C) has discoordination between the movements of both eyes. D) should be treated by irrigating both eyes for 20 minutes.

C) has discoordination between the movements of both eyes.

47. Patients with a colostomy: A) require a collection bag to collect solid fecal material after a section of large intestine is surgically removed. B) have a stoma that directs the small intestine to the outside of the abdomen where a collection bag is attached. C) have had a surgical procedure that directs the large intestine out through a stoma in the anterior abdominal wall. D) have a temporary surgical opening in the abdomen that collects waste material and allows the bowel to rest and heal.

C) have had a surgical procedure that directs the large intestine out through a stoma in the anterior abdominal wall.

39. When an infant or child is in respiratory failure: A) tachypnea is usually present despite a marked decrease in heart rate. B) decreased cerebral perfusion leads to restlessness and a weak, rapid pulse. C) he or she can no longer compensate, which causes hypoxia and hypercarbia. D) oxygen via nonrebreathing mask should be given if tidal volume is reduced.

C) he or she can no longer compensate, which causes hypoxia and hypercarbia.

129. In young children, air bags pose a particular threat for injuries to the: A) thoracic organs. B) abdominal organs. C) head and neck. D) soft tissues of the face.

C) head and neck.

16. The pediatric assessment triangle was designed to: A) formulate a working field diagnosis upon first sight of an ill child. B) identify immediate life threats through a rapid hands-on assessment. C) help EMS providers form a hands-off general impression of an ill child. D) provide a means for performing a rapid head-to-toe physical assessment.

C) help EMS providers form a hands-off general impression of an ill child.

18. A sick or injured child's general appearance is MOST reflective of: A) the etiology of the problem. B) his or her cardiovascular status. C) his or her central nervous system function. D) his or her ability to be consoled.

C) his or her central nervous system function.

6. The headache commonly experienced by women during their menstrual cycle is caused by: A) acute stress. B) vasoconstriction. C) hormonal release. D) transient cerebral edema.

C) hormonal release.

26. Adult patients who have a tracheostomy tube in place and are ventilator-dependent should receive: A) cool, dry air through the ventilator circuit. B) deep tracheal suctioning every other day. C) humidification and heating of inspired air. D) ventilation at a rate of 20 to 24 breaths/min.

C) humidification and heating of inspired air.

35. Severe dehydration that commonly accompanies diabetic ketoacidosis is caused by: A) prolonged compensatory hyperventilation. B) decreased fluid intake secondary to hyperglycemia. C) hyperglycemia-induced osmotic diuresis and vomiting. D) the loss of key electrolytes such as sodium and potassium.

C) hyperglycemia-induced osmotic diuresis and vomiting.

24. An increase in maternal cardiac output can cause a benign systolic flow murmur, which results from: A) an increase in afterload and slight mitral valve regurgitation. B) enlargement of the aortic valve due to increased blood volume. C) hypertrophy of the heart and dilation across the tricuspid valve. D) pulmonic valve prolapse due to transient pulmonary hypertension

C) hypertrophy of the heart and dilation across the tricuspid valve.

58. You are dispatched to a residence for an elderly woman who is "sick." When you arrive and assess her, you note that she is responsive to pain only and has hot, moist skin and rapid, shallow respirations. You find prednisone, Paxil, and multivitamins on her nightstand. Further assessment of this patient will MOST likely reveal: A) a normal blood glucose level and a "moon face" appearance. B) severe hypertension, flattened T waves, and asymmetric pupils. C) hypoglycemia, hypotension, and ECG evidence of hyperkalemia. D) a blood glucose reading above 400 mg/dL and a fruity breath odor.

C) hypoglycemia, hypotension, and ECG evidence of hyperkalemia.

3. The oropharynx and nasopharynx meet in the back of the throat at the: A) larynx. B) hyoid bone. C) hypopharynx. D) glottic opening.

C) hypopharynx.

A patient with prerenal acute renal failure would MOST likely present with: A) confusion and hypertension. B) joint pain and bladder distention. C) hypotension and tachycardia. D) peripheral edema and hematuria.

C) hypotension and tachycardia.

13. The adrenal medulla secretes norepinephrine following stimulation from the: A) brainstem. B) diencephalon. C) hypothalamus. D) pituitary gland.

C) hypothalamus.

6. The primary anatomic link between the endocrine system and the nervous system is/are the: A) pancreas. B) adrenal glands. C) hypothalamus. D) adrenal cortex.

C) hypothalamus.

The MOST acute complication associated with large body surface area burns is: A) infection. B) hypovolemia. C) hypothermia. D) myoglobinemia.

C) hypothermia.

29. Systemic effects of epinephrine include: A) bronchoconstriction. B) decreased preload. C) increased afterload. D) diffuse vasodilation.

C) increased afterload.

Azotemia is defined as: A) electrolyte disturbances in the blood. B) an accumulation of uric acid in the blood. C) increased nitrogenous wastes in the blood. D) excess potassium excretion by the kidneys.

C) increased nitrogenous wastes in the blood.

79. Unlike other types of shock, a child in cardiogenic shock would MOST likely present with: A) an enlarged spleen. B) unlabored tachypnea. C) increased work of breathing. D) a primary cardiac dysrhythmia.

C) increased work of breathing.

64. An intra-aortic balloon pump functions by: A) deflating a balloon during systole, which pushes blood into the central circulation. B) deflating a balloon during diastole, which creates a vacuum and decreases afterload. C) inflating a balloon during diastole, which pushes blood into the systemic circulation. D) inflating a balloon during systole, which creates a vacuum and decreases afterload.

C) inflating a balloon during diastole, which pushes blood into the systemic circulation.

If a burn patient presents with a hoarse voice and states, "I'm cold," your MOST immediate concern should be: A) hypothermia. B) burn shock. C) inhalation injury. D) cyanide toxicity.

C) inhalation injury.

34. Accumulating acids and other waste products in the blood: A) act as potent vasoconstrictors, which worsens ischemia. B) cause a left shift in the oxyhemoglobin dissociation curve. C) inhibit hemoglobin from binding with and carrying oxygen. D) cause a marked increase in the production of red blood cells.

C) inhibit hemoglobin from binding with and carrying oxygen.

Furosemide (Lasix) causes diuresis by: A) increasing circulating blood glucose. B) converting angiotensin I to angiotensin II. C) inhibiting sodium reabsorption in the kidneys. D) inhibiting the production of antidiuretic hormone.

C) inhibiting sodium reabsorption in the kidneys.

30. If you cannot palpate the femoral pulse in an unresponsive infant, you should: A) apply an AED at once. B) palpate the brachial pulse. C) initiate CPR immediately. D) assess for adequate breathing.

C) initiate CPR immediately.

The middle ear consists of the: A) cochlea and semicircular canals. B) organ of Corti and the external auditory canal. C) inner portion of the tympanic membrane and the ossicles. D) pinna and the exterior portion of the tympanic membrane.

C) inner portion of the tympanic membrane and the ossicles.

58. You are assessing a 39-year-old man who experienced blunt chest trauma. He is semiconscious and has poor respiratory effort with stridor. You should: A) perform laryngoscopy to visualize his airway for an obstruction. B) administer oxygen via nonrebreathing mask and assess circulation. C) insert a nasal airway and assist ventilations with a bag-mask device. D) suction his airway and prepare for immediate orotracheal intubation.

C) insert a nasal airway and assist ventilations with a bag-mask device.

67. You have been providing bag-mask ventilation to a newborn with a sustained heart rate of 75 beats/min for approximately 5 minutes. The infant's abdomen is markedly distended. Although you are properly trained, your protocols do not allow you to intubate newborns. The MOST appropriate intervention involves: A) intubating immediately. B) suctioning the oropharynx. C) inserting an orogastric tube. D) manual gastric decompression.

C) inserting an orogastric tube.

5. The space between the second and third ribs is called the: A) third intercostal space. B) intrathoracic margin. C) second intercostal space. D) second subcostal margin

C) second intercostal space.

Unlike peritoneal dialysis, hemodialysis: A) is associated with a higher risk of peritonitis. B) filters nitrogenous waste products from the blood. C) involves the circulation of blood through a machine. D) is only used for patients experiencing acute renal failure.

C) involves the circulation of blood through a machine.

42. In contrast to osteoarthritis, rheumatoid arthritis: A) is a disease of the joints that occurs as they age and begin to wear. B) generally does not respond well to anti-inflammatory medication therapy. C) is a systemic inflammatory disease that affects joints and other body systems. D) causes severe pain secondary to crystallized uric acid accumulation in a joint.

C) is a systemic inflammatory disease that affects joints and other body systems.

10. An 8-year-old child: A) is analytic but is not capable of abstract thought. B) should not be the initial historian regarding an illness. C) is anatomically and physiologically similar to an adult. D) generally requires little reassurance and encouragement.

C) is anatomically and physiologically similar to an adult.

35. You would MOST likely encounter the presence of a ventricular assist device in a patient who: A) has severe emphysema. B) has pulmonary hypertension. C) is awaiting a heart transplant. D) has transient right heart failure.

C) is awaiting a heart transplant.

10. The fibula: A) can be felt just beneath the skin. B) forms the distal medial malleolus. C) is not a component of the knee joint. D) is more easily fractured than the tibia.

C) is not a component of the knee joint.

50. Epiglottitis in children: A) presents with a sudden onset of low-grade fever and dyspnea. B) should be suspected if the child presents with diffuse wheezing. C) is rare now that children are vaccinated against Haemophilus influenza type B. D) should be confirmed by visualizing the larynx and epiglottis with a laryngoscope.

C) is rare now that children are vaccinated against Haemophilus influenza type B.

8. The cricothyroid membrane: A) is a cartilaginous structure located between the thyroid and cricoid cartilages. B) serves as the primary entry point to the airway when a patient is not breathing. C) is relatively avascular and is covered by skin and minimal subcutaneous tissue. D) is a thin membrane that can be palpated easily just above the thyroid cartilage.

C) is relatively avascular and is covered by skin and minimal subcutaneous tissue.

25. The length-based resuscitation tape: A) is only reliable in children who weigh less than 20 kg. B) should not be relied upon for determining pediatric drug doses. C) is used to estimate a child's weight based on his or her height. D) is generally more accurate than the weight given by a caregiver.

C) is used to estimate a child's weight based on his or her height.

40. In contrast to primary apnea, secondary apnea: A) is characterized by profound tachycardia. B) commonly follows a brief period of hypoxia. C) is usually unresponsive to stimulation alone. D) necessitates immediate endotracheal intubation.

C) is usually unresponsive to stimulation alone.

34. A patient standing with his or her head cocked toward a knocked-down left shoulder MOST likely has a fracture of the: A) right clavicle. B) left shoulder. C) left clavicle. D) right shoulder.

C) left clavicle.

You are dispatched to a residence for an elderly man with an altered mental status. As you are assessing the patient, his wife tells you that he goes to dialysis several times a week, but has missed his last three treatments because their car broke down. The patient's skin is yellow, his blood pressure is 98/60 mm Hg, and his pulse rate is 118 beats/min. The ECG reveals sinus tachycardia with peaked T waves. You should be MOST concerned with the potential for: A) severe hypovolemia. B) acute bradycardia or heart block. C) lethal ventricular dysrhythmias. D) hypokalemia-induced cardiac arrest.

C) lethal ventricular dysrhythmias.

After the cells lining the proximal convoluted tubule remove all organic nutrients, plasma proteins, and ions from the filtrate, additional reabsorption of water and electrolytes occurs in the: A) glomerulus. B) macula densa. C) loop of Henle. D) distal convoluted tubule.

C) loop of Henle.

66. You have applied board splints to a suspected lower leg fracture in a young woman and have given her fentanyl for pain. En route to the hospital, the patient states that the pain is excruciating. Further narcotics fail to relieve the pain. Reassessment of the injured area reveals that the overlying skin is taut and the pedal pulse is weak. You should: A) start a second IV line and administer 1 mEq/kg of sodium bicarbonate. B) remove the splint to prevent excessive swelling of the extremity. C) loosen the splint, elevate the leg, apply ice, and notify the hospital. D) remove the board splints, apply an air splint, and then reassess her.

C) loosen the splint, elevate the leg, apply ice, and notify the hospital.

49. While caring for an unresponsive young woman who was apparently sexually assaulted, you note that her respirations are slow and shallow, her pulse is slow and weak, and her blood pressure is low. There is a small amount of dried blood on her thigh, but no grossly active bleeding. Immediate care for this patient involves: A) establishing an IV line and administering 0.5 mg of atropine. B) visualizing her vaginal area to assess for external bleeding. C) maintaining her airway and providing ventilatory assistance. D) elevating her lower extremities to improve her blood pressure.

C) maintaining her airway and providing ventilatory assistance.

61. You arrive at the scene of a motorcycle crash and find the rider lying supine approximately 20 feet from his bike; he is still wearing his helmet. As you approach him, you note that he has bilaterally deformed femurs. You should: A) immediately stabilize both lower extremities. B) remove his helmet and apply a cervical collar. C) manually stabilize his head and assess his airway. D) carefully straighten his legs and assess distal pulses.

C) manually stabilize his head and assess his airway.

The skin of a patient with end-stage renal disease: A) is hot and dry. B) is thickened. C) may appear bruised. D) is usually flushed.

C) may appear bruised.

31. Gestational hypertension: A) often requires an antihypertensive. B) is common in underweight women. C) may be an early sign of preeclampsia. D) is benign in the vast majority of cases.

C) may be an early sign of preeclampsia.

63. If the left ventricle fills with 85 mL of blood and ejects 60 mL during a contraction, the ejection fraction is approximately ___%. A) 55 B) 60 C) 65 D) 70

D) 70

21. Compared to subsequent breaths, the first few positive-pressure breaths delivered to a distressed newborn: A) should provide a volume equal to 40 to 45 mm Hg. B) should make the chest rise significantly. C) may necessitate manual disabling of the pop-off valve. D) generally require a significantly lower volume of air.

C) may necessitate manual disabling of the pop-off valve.

A person who is exposed to cement: A) typically only experiences burns to the epidermal layer because calcium oxide is a weak chemical. B) often does not experience a burn unless he or she is exposed to the cement for longer than 2 hours. C) may not notice a skin burn for hours because cement penetrates through clothing and reacts with sweat. D) experiences immediate pain and inflammation to the area because of the calcium oxide in the cement.

C) may not notice a skin burn for hours because cement penetrates through clothing and reacts with sweat.

Cardiac arrest following an electrical shock: A) typically presents as pulseless atrial fibrillation or atrial flutter. B) is most often caused by an electrical current stronger than 1 ampere. C) may occur secondarily from hypoxia or as a direct result of the shock. D) is typically of short duration and is often reversed with 2 minutes of CPR.

C) may occur secondarily from hypoxia or as a direct result of the shock.

78. A child in anaphylactic shock: A) should receive 0.1 mg/kg of epinephrine IM. B) is treated primarily with saline fluid boluses. C) may require a low-dose epinephrine infusion. D) should receive epinephrine 1:1,000 via the IV route.

C) may require a low-dose epinephrine infusion.

41. Hepatojugular reflux occurs when: A) left-sided heart failure causes blood to accumulate in the patient's liver. B) the jugular veins collapse in response to palpation of the right upper quadrant. C) mild pressure placed on the patient's liver further engorges the jugular veins. D) a patient's jugular veins are markedly engorged when lying in a supine position.

C) mild pressure placed on the patient's liver further engorges the jugular veins.

A patient with full-thickness burns surrounded by areas of superficial and partial-thickness burns should be treated with all of the following, EXCEPT: A) analgesia. B) high-flow oxygen. C) moist dressings. D) sterile burn pads.

C) moist dressings.

26. When suctioning the newborn's oropharynx to clear secretions prior to intubation, it is MOST important to: A) limit suctioning to 15 seconds. B) use a flexible suction catheter. C) monitor the newborn's heart rate. D) assess pulse oximetry and capnography.

C) monitor the newborn's heart rate.

11. The ____________of the uterus is composed of three layers of muscle fibers that contract and help expel the fetus during childbirth. A) cervix B) endometrium C) myometrium D) perimetrium

C) myometrium

157. You are assessing a 7-month-old infant who presents with listlessness, pallor, and increased work of breathing. The infant's mother tells you that the child was born 2 months premature and was in the neonatal intensive care unit for 3 weeks. She denies any recent vomiting, diarrhea, or fever. The infant's oxygen saturation is 89% and does not improve with supplemental oxygen. Her heart rate is rapid and weak and does not vary with activity. When you apply the cardiac monitor, you will MOST likely encounter a: A) wide QRS complex rhythm with occasional P waves and a rate greater than 150 beats/min. B) rhythm with QRS complexes greater than 0.08 seconds in duration and a heart rate greater than 180 beats/min. C) narrow QRS complex rhythm with absent P waves and a heart rate greater than 220 beats/min. D) rhythm with QRS complexes less than 0.08 seconds in duration and a heart rate less than 220 beats/min.

C) narrow QRS complex rhythm with absent P waves and a heart rate greater than 220 beats/min.

106. The MOST appropriate airway management for an actively seizing child whose airway is not maintainable with positioning involves: A) immediate endotracheal intubation. B) insertion of an oropharyngeal airway adjunct. C) nasal airway insertion and suctioning as needed. D) 100% oxygen and a left lateral recumbent position.

C) nasal airway insertion and suctioning as needed.

21. An open pneumothorax causes ventilatory inadequacy when: A) positive pressure created by expiration forces air into the pleural space. B) the heart stops perfusing the lung on the side of the open chest injury. C) negative pressure created by inspiration draws air into the pleural space. D) the glottic opening is much larger than the open wound on the chest wall.

C) negative pressure created by inspiration draws air into the pleural space.

1. An immature egg is called a(n): A) ovum. B) zygote. C) oocyte. D) follicle.

C) oocyte.

166. A 2-year-old girl fell approximately 12 feet from a second-story window, landing on her head. Your primary assessment reveals that she is unresponsive; has slow, irregular respirations; and has blood draining from her mouth and nose. A rapid scan of her body does not reveal any gross injuries or bleeding. You should: A) manually stabilize her head and neck in a neutral position, insert a nasal airway, and hyperventilate her at a rate of 35 breaths/min. B) suction her mouth and nose for no longer than 15 seconds, insert an oral airway, and apply high-flow oxygen with a pediatric nonrebreathing mask. C) open her airway with the jaw-thrust maneuver, suction her mouth and nose, insert an oral airway, and assist her ventilations with a bag-mask device. D) insert an oral airway, apply a cervical collar, preoxygenate her with a bag-mask device and 100% oxygen for 30 seconds, and intubate her trachea.

C) open her airway with the jaw-thrust maneuver, suction her mouth and nose, insert an oral airway, and assist her ventilations with a bag-mask device.

If a patient is unable to follow your finger above the midline following blunt trauma to the face, you should be MOST suspicious for a(n): A) Le Fort II fracture. B) nasal bone fracture. C) orbital skull fracture. D) basilar skull fracture.

C) orbital skull fracture.

4. Luteinizing hormone stimulates the ovarian cells to release all of the following hormones, EXCEPT: A) relaxin. B) estrogen. C) oxytocin. D) progesterone.

C) oxytocin.

76. A person with amblyopia has: A) difficulty visualizing object from afar. B) an area missing from his or her visual field. C) partial or complete vision loss in one eye. D) difficulty visualizing objects that are close.

C) partial or complete vision loss in one eye.

37. In contrast to decreased PO2 levels, increased PCO2 levels typically manifest as: A) anxiety. B) combativeness. C) sedation or sleepiness. D) restlessness or confusion.

C) sedation or sleepiness.

4. The appendicular skeleton is composed of the: A) bones of the spinal column, scapulae, and clavicles. B) bones of the upper extremities and the structures of the torso. C) pectoral girdle, pelvic girdle, and bones of the extremities. D) thoracic ribs, cervical vertebrae, and bones of the cranium.

C) pectoral girdle, pelvic girdle, and bones of the extremities.

60. The outflow catheter of a cerebrospinal fluid shunt is MOST commonly placed into the patient's: A) right atrium. B) pulmonary cavity. C) peritoneal cavity. D) left ventricle.

C) peritoneal cavity.

34. Signs of hypovolemia in the newborn include all of the following, EXCEPT: A) persistent pallor. B) weak central pulses. C) persistent acrocyanosis. D) persistent bradycardia.

C) persistent acrocyanosis

46. Following removal of an ostomy device and cleansing the stoma, you should: A) apply the new device over the stoma. B) flush the inside of the stoma with saline. C) place a clean gauze pad over the stoma. D) cover the stoma with the provided wafer

C) place a clean gauze pad over the stoma.

57. To maintain a neutral airway position in an unresponsive infant, you should: A) slightly extend the infant's head. B) pad underneath the infant's occiput. C) place a towel roll under the shoulders. D) insert an appropriate-sized oral airway.

C) place a towel roll under the shoulders.

93. A 76-year-old woman with emphysema presents with respiratory distress that has worsened progressively over the past 2 days. She is breathing through pursed lips and has a prolonged expiratory phase and an oxygen saturation of 76%. She is on home oxygen at 2 L/min. Your initial action should be to: A) increase her oxygen flow rate to 6 L/min. B) administer a beta-2 agonist via nebulizer. C) place her in a position that facilitates breathing. D) auscultate her lungs for adventitious breath sounds.

C) place her in a position that facilitates breathing

97. You are delivering a baby who was in a breech presentation. The baby's body has delivered, and you are attempting to deliver its head by lifting its body upward. After about 3 minutes, the baby's head has not delivered. You should: A) elevate the mother's hips with pillows, administer high-flow oxygen, and transport immediately. B) support the baby's body, carefully turn the mother on her left side, and transport expeditiously. C) place your gloved hand in the vagina and gently lift the baby's face away from the vaginal wall. D) elevate the mother's hips and apply gentle traction to the baby's body until the head has delivered.

C) place your gloved hand in the vagina and gently lift the baby's face away from the vaginal wall.

30. Common signs of gestational diabetes include: A) confusion. B) diaphoresis. C) polydipsia. D) tachycardia.

C) polydipsia.

12. The MOST common cause of amenorrhea is: A) stress. B) exercise. C) pregnancy. D) anorexia nervosa.

C) pregnancy.

65. A robbery suspect was shot once in the left anterior chest by law enforcement personnel when he pulled a gun on them. The patient is exhibiting obvious signs of shock, is in significant respiratory distress, and is coughing up blood. Further assessment reveals collapsed jugular veins and absent breath sounds over the left hemithorax. After covering the gunshot wound with the appropriate dressing, you should: A) perform a needle thoracentesis to the left side of the chest, initiate rapid transport, and administer 20-mL/kg fluid boluses en route. B) administer 100% oxygen, administer 1 to 2 L of normal saline, and transport to a trauma center for an emergency pericardiocentesis. C) provide oxygenation and ventilation support, transport at once, and maintain adequate perfusion with IV fluids while en route to a trauma center. D) ventilate the patient with a demand valve, transport to a trauma center, and run two large-bore IV lines wide open while en route to the hospital.

C) provide oxygenation and ventilation support, transport at once, and maintain adequate perfusion with IV fluids while en route to a trauma center.

145. A 9-year-old who fell off his bike has an isolated deformity to his forearm and is in significant pain. The child is conscious and alert, his vital signs are stable, and his mother is present. Your initial effort to relieve this child's pain should involve: A) encouraging the child to breathe high-flow oxygen. B) not allowing the child to visualize his deformed arm. C) providing calm reassurance to both mother and child. D) administering morphine or fentanyl via slow IV push.

C) providing calm reassurance to both mother and child.

73. A young woman experiences a sudden nervous system reaction that produces temporary, generalized vasodilation and causes her to faint. This is MOST descriptive of _____________ shock A) neurogenic B) obstructive C) psychogenic D) hypovolemic

C) psychogenic

Vascular injury following trauma to the anterior neck would MOST likely present with: A) hemoptysis. B) parasthesia. C) pulse deficits. D) dysphagia.

C) pulse deficits.

63. When ventilating an apneic child with a bag-mask device, it is important for the paramedic to remember that: A) each ventilation should be delivered over a period of 2 to 3 seconds. B) the presence of chest rise is an unreliable indicator of proper ventilation. C) regurgitation and aspiration may occur, even with proper ventilation technique. D) posterior cricoid pressure will virtually eliminate the risk of pulmonary aspiration.

C) regurgitation and aspiration may occur, even with proper ventilation technique.

13. Wheezing is resolved with medications that: A) dry up secretions in the lower airway. B) reduce soft tissue swelling in the larynx. C) relax the smooth muscle of the bronchioles. D) cause bronchoconstriction and improved airflow.

C) relax the smooth muscle of the bronchioles.

58. An oral or nasal airway in an unresponsive infant or child may serve all of the following purposes, EXCEPT: A) facilitating oral suctioning. B) averting the need for intubation. C) replacing manual head positioning. D) helping to maintain an open airway.

C) replacing manual head positioning.

41. Mortality and morbidity are high among infants who are delivered at 24 weeks' gestation, usually because of: A) congenital heart defects. B) infection and hypothermia. C) respiratory and neurologic problems. D) metabolic and immune deficiencies.

C) respiratory and neurologic problems

37. In contrast to adults, cardiac arrest in children is usually caused by: A) a dysrhythmia. B) a toxic ingestion. C) respiratory failure. D) congenital anomalies.

C) respiratory failure.

43. Signs of a threatened abortion include all of the following, EXCEPT: A) sporadic contractions. B) menstrual-like cramps. C) severe abdominal pain. D) bleeding in the first trimester.

C) severe abdominal pain.

29. If a woman who is being treated with the antibiotic metronidazole for bacterial vaginosis consumes alcohol during treatment: A) she is at high risk for sudden cardiac death. B) severe exacerbation of the vaginosis occurs. C) she may develop severe nausea and vomiting. D) the antibiotic may become a toxic substance.

C) she may develop severe nausea and vomiting.

Dry powder chemicals: A) will react violently with water and should not be irrigated. B) cause coagulation necrosis if they are absorbed by the body. C) should be brushed off the skin before irrigation with water. D) are effectively neutralized on the skin with isopropyl alcohol.

C) should be brushed off the skin before irrigation with water.

86. The presence of tachycardia in children: A) commonly reflects an underlying cardiac pathology that requires emergent intervention. B) often causes hypotension and is usually associated with a QRS complex greater than 0.08 seconds. C) should be interpreted in the context of the pediatric assessment triangle and the primary assessment. D) necessitates a 20-mL/kg bolus of an isotonic crystalloid solution until the cardiac rhythm is assessed.

C) should be interpreted in the context of the pediatric assessment triangle and the primary assessment.

22. Skeletal muscle is innervated by: A) sympathetic nerve fibers. B) involuntary nerves. C) somatic motor neurons. D) the autonomic nervous system.

C) somatic motor neurons.

55. The likelihood of experiencing systemic complications from a musculoskeletal injury is related to all of the following factors, EXCEPT the: A) patient's overall health. B) anatomic location of the injury. C) splinting method used in the field. D) degree of force that caused the injury.

C) splinting method used in the field.

Immediate care for a burn patient involves: A) applying sterile burn sheets. B) establishing a patent airway. C) stopping the burning process. D) maintaining body temperature.

C) stopping the burning process.

23. The upper extremity's blood supply originates from the: A) radial artery. B) brachial artery. C) subclavian artery. D) axillary artery.

C) subclavian artery.

61. You are assisting in the delivery of a baby. After the baby's head emerges from the vagina, you should quickly assess for the presence of a nuchal cord and then: A) assess for facial cyanosis. B) administer free-flow oxygen. C) suction its mouth and nose. D) dry its face to stimulate breathing.

C) suction its mouth and nose.

Open soft-tissue facial trauma following a significant mechanism of injury: A) often requires removal of foreign bodies that are impaled in the face. B) is of most concern due to the possibility of permanent disfigurement. C) suggests that the patient may have a closed head injury or spinal injury. D) dictates the need for immediate intubation to protect the patient's airway.

C) suggests that the patient may have a closed head injury or spinal injury.

42. A 66-year-old man with severe left heart failure is receiving an inotropic medication via an infusion pump. He presents with an altered mental status, increased breathing difficulty, and hypotension. You should: A) administer oxygen and discontinue the medication infusion by turning the pump off. B) assist his breathing, slowly increase the dose of his inotropic medication, and transport. C) support his breathing, continue his medication infusion, and contact medical control. D) administer oxygen, discontinue the medication infusion, and establish a peripheral IV.

C) support his breathing, continue his medication infusion, and contact medical control.

34. Diabetic ketoacidosis occurs when: A) blood glucose levels rise above 250 mg/dL. B) the renal system begins to excrete ketones. C) the cells metabolize fat and produce ketones. D) insulin production exceeds glucagon production.

C) the cells metabolize fat and produce ketones.

61. There is a higher incidence of abdominal injuries in association with chest trauma in pregnant women because: A) seat belts are usually not worn. B) the peritoneum is maximally stretched. C) the diaphragm is elevated nearly 2 inches. D) the abdomen is large and protuberant.

C) the diaphragm is elevated nearly 2 inches.

32. Epinephrine is indicated during newborn resuscitation if: A) the heart rate does not increase above 80 beats/min after 30 to 60 seconds of effective positive-pressure ventilation. B) the newborn is bradycardic and thick secretions are hindering your ability to provide effective positive-pressure ventilations. C) the heart rate remains below 60 beats/min after 30 seconds of effective ventilation and an additional 30 seconds of chest compressions. D) profound central cyanosis persists despite 30 seconds of effective positive-pressure ventilation with 100% supplemental oxygen.

C) the heart rate remains below 60 beats/min after 30 seconds of effective ventilation and an additional 30 seconds of chest compressions.

40. In general, patients suspected of having a partial tracheal tear should be managed with: A) immediate endotracheal intubation. B) an oxygen-powered ventilation device. C) the least invasive airway techniques possible. D) placement of a laryngeal mask airway device.

C) the least invasive airway techniques possible.

29. A stress fracture would MOST likely occur when: A) a person with a relatively weak bone structure does not engage regularly in strenuous activities. B) a person with large musculature regularly engages in strenuous activity using the upper extremities. C) the muscle develops faster than the bone and places exaggerated stress on the bone. D) an underlying medical condition causes progressive weakening of the bones, making them prone to fracture.

C) the muscle develops faster than the bone and places exaggerated stress on the bone.

24. Insulin resistance occurs when: A) autoantibodies break down insulin before it can be utilized by the body. B) the release of epinephrine and norepinephrine renders insulin less effective. C) the pancreas produces enough insulin, but the body cannot utilize it effectively. D) the body produces excessive insulin, which causes a profound drop in blood glucose.

C) the pancreas produces enough insulin, but the body cannot utilize it effectively.

ECG monitoring of a patient with a suspected urologic emergency is especially important because of: A) myocardial ischemia that is often caused by urologic dysfunction. B) the medications that many patients with urologic problems are taking. C) the potential for electrolyte imbalances and their effect on the heart. D) frequent fluctuations in the patient's heart rate caused by severe pain.

C) the potential for electrolyte imbalances and their effect on the heart.

71. Crowning is MOST accurately defined as: A) bulging of the baby's head from the vaginal opening. B) passage of the baby through the fully dilated cervix. C) the presenting of the baby at the vaginal opening. D) the thinning and shortening of the cervix during labor.

C) the presenting of the baby at the vaginal opening.

135. You would MOST likely encounter a child with a tracheostomy tube breathing spontaneously on room air if: A) he or she has a brainstem abnormality that affects the respiratory drive. B) the tracheostomy tube was placed because of a congenital airway anomaly. C) the purpose of the tube is to bypass a mechanical upper airway obstruction. D) a self-limiting condition necessitated placement of the tracheostomy tube.

C) the purpose of the tube is to bypass a mechanical upper airway obstruction.

69. Relative bradycardia during neurogenic shock occurs because: A) systemic venous pooling of the blood overstimulates the vagus nerve. B) the parasympathetic nervous system does not release acetylcholine. C) the sympathetic nervous system is not stimulated to release catecholamines. D) the brainstem does not receive messages to increase the heart rate.

C) the sympathetic nervous system is not stimulated to release catecholamines.

48. The presence of diffuse rhonchi (low-pitched crackles) in the lungs indicates: A) right-sided congestive heart failure. B) isolated consolidation of secretions. C) thick secretions in the large airways. D) air being forced through narrowed airways.

C) thick secretions in the large airways.

43. A patient with severe dehydration is found to be hypotensive during your assessment. The MOST important intervention in this case is: A) applying blankets to keep the patient warm. B) immediate fluid resuscitation at the scene.. C) transport with fluid resuscitation en route. D) high-flow oxygen via nonrebreathing mask.

C) transport with fluid resuscitation en route.

39. Signs and symptoms of multiple-organ dysfunction syndrome may include: A) severe polyuria. B) marked hyperglycemia. C) uncontrollable bleeding. D) warm, flushed skin.

C) uncontrollable bleeding

16. Pelvic inflammatory disease commonly affects all of the following organs, EXCEPT the: A) uterus. B) ovaries. C) urinary bladder. D) fallopian tubes.

C) urinary bladder.

As the filtrate passes through the rest of the nephron, tubular reabsorption and tubular secretion convert it to: A) salt. B) water. C) urine. D) glucose.

C) urine.

21. The Candida albicans fungus is the causative agent in: A) bacterial vaginosis. B) gardnerella vaginitis. C) vaginal yeast infections. D) gonorrhea and chlamydia.

C) vaginal yeast infections.

49. The MOST important initial treatment for a child in respiratory failure due to suspected croup is: A) prompt intubation before the airway closes. B) a 2.25% concentration of racemic epinephrine. C) ventilatory assistance with a bag-mask device. D) continuous administration of a beta-2 agonist.

C) ventilatory assistance with a bag-mask device.

Disequilibrium syndrome is a condition in which: A) large amounts of water move by osmosis into the brain, resulting in an acute subdural hematoma. B) dialysis patients miss one or two of their treatments, resulting in hyperkalemia, azotemia, and hypotension. C) water initially shifts from the bloodstream into the cerebrospinal fluid, causing an increase in intracranial pressure. D) the concentration of urea in the bloodstream is lowered slowly, while the solute concentration of the cerebrospinal fluid remains high.

C) water initially shifts from the bloodstream into the cerebrospinal fluid, causing an increase in intracranial pressure.

The degree of absorption of a corrosive chemical determines: A) the type of liquid used to irrigate the burn. B) whether the burn should be flushed. C) whether toxicity is local or systemic. D) the antidote required to reverse the effects

C) whether toxicity is local or systemic

57. In contrast to gravida 3 mothers, gravida 1 mothers: A) have delivered at least one baby. B) are at a lower risk for complications. C) will typically take more time to deliver. D) often experience a precipitous delivery.

C) will typically take more time to deliver.

37. A patient with a long-term vascular access device is in cardiac arrest and requires epinephrine. In order to remove any heparin from the device, the paramedic should: A) follow the epinephrine with a 10-mL saline flush. B) briskly flush the device with 20 mL of saline. C) withdraw up to 10 mL of blood and discard it. D) slowly inject 3 to 5 mL of sodium bicarbonate.

C) withdraw up to 10 mL of blood and discard it.

In which of the following patients should the impaled object be removed? A) Apneic patient with a shard of glass impaled in the center of the chest B) Pulseless and apneic patient with a knife impaled in the lower abdomen C) Cardiac arrest patient with an ice pick impaled in the center of the back D) Semiconscious patient with a screwdriver impaled in the side of the head

Cardiac arrest patient with an ice pick impaled in the center of the back

Which of the following statements regarding compartment syndrome is correct? A) Compartment syndrome is more likely to occur with closed injuries. B) Compartment syndrome is caused by increased pressure within the bone. C) Definitive treatment almost always includes amputation of the affected limb. D) Local tissue death occurs after 2 hours of persistent compartment syndrome

Compartment syndrome is more likely to occur with closed injuries.

Which of the following statements regarding crush injury is correct? A) A crush injury can occur if the PASG is left in place for greater than 1 hour. B) Gangrene often sets in if a body part is entrapped for longer than 30 minutes. C) Crush syndrome can occur if the body part is entrapped for more than 4 hours. D) In a crush injury, the external appearance is a good predictor of internal damage.

Crush syndrome can occur if the body part is entrapped for more than 4 hours

1. The galea aponeurotica is: A) the dense fibrous membrane covering the surface of the cranium. B) a loose connective tissue that is easily damaged during "scalping" injuries. C) the subcutaneous layer of the scalp that contains easily damaged major veins. D) a tendon expansion that connects the frontal and occipital muscles of the cranium.

D

100. A young man is found unresponsive by his girlfriend. Your assessment reveals marked respiratory depression; a slow, weak pulse; and pinpoint pupils. There are numerous medication bottles found in his home. Of these, he has MOST likely ingested: A) Valium. B) Sudafed. C) Benadryl. D) Percodan.

D

104. You arrive at the scene of an unknown drug-related emergency. Law enforcement is present and has ensured scene security. The patient, a young male, is found sitting at the kitchen table. He is laughing uncontrollably and tells you, "Life sure is good!" Your partner finds a basin of water and an empty box of baking soda on the counter. You should be MOST suspicious that this patient: A) is speedballing. B) has injected heroin. C) was snorting cocaine. D) has smoked crack cocaine.

D

108. An unresponsive middle-aged man ingested a half-full bottle of Valium approximately 20 minutes ago. His respirations are slow and shallow, his pulse is slow and weak, and his blood pressure is significantly low. The cardiac monitor reveals sinus bradycardia. You should: A) insert a Combitube, establish vascular access, administer up to 4 liters of normal saline, and give him 0.1 mg/kg of naloxone. B) administer oxygen via nonrebreathing mask, start an IV line, and give 150 mg of amiodarone to prevent lethal ventricular dysrhythmias. C) immediately intubate his trachea, hyperventilate him to minimize acidosis, establish vascular access, and administer up to 10 mg of flumazenil. D) assist his ventilations, administer flumazenil via slow IV push if allowed by protocol, and consider that he likely ingested another type of CNS depressant.

D

11. Exposure to sarin or tabun would result in: A) hyperthermia. B) pupillary dilation. C) severe tachycardia. D) excessive lacrimation.

D

113. A 69-year-old man presents with confusion, a headache, dyspnea, and palpitations after he rescued his two grandchildren from their burning house. During your assessment, you note that he has an odd odor on his breath; however, he denies being diabetic. You should: A) start an IV line of normal saline and administer 10 mL of a 10% solution of calcium chloride. B) administer 1 to 2 g of pralidoxime infused with normal saline solution over a 5- to 10-minute period. C) start an IV line, sedate and chemically paralyze the patient, and then perform endotracheal intubation. D) have him inhale amyl nitrate for 20 seconds and then 100% oxygen for 40 seconds out of each minute.

D

120. Corticosteroid use for a spinal cord injury may lead to significant complications, particularly if: A) the patient is younger than 45 years of age. B) it is initiated less than 3 hours after the injury. C) the patient is also taking beta-blocker medications. D) the injury is accompanied by penetrating trauma.

D

19. The emotional state of craving a drug to maintain a feeling of well-being is called: A) addiction. B) habituation. C) physical dependence. D) psychological dependence.

D

2. A therapeutic dose of Valium simply relaxes one person, but causes severe central nervous system depression in another patient. This is an example of: A) synergism. B) potentiation. C) hypersensitivity. D) an idiosyncrasy.

D

24. Which of the following is atypical of an alcoholic? A) Drinking early in the day B) Green tongue syndrome C) Memory loss or blackouts D) Chronically pale face and palms

D

30. When caring for an unresponsive patient with a toxicologic emergency, you should: A) intubate at once, obtain baseline vital signs, transport immediately, and perform all other interventions en route to the hospital. B) administer high-flow oxygen, perform a detailed secondary assessment, obtain vital signs, and transport to the closest hospital. C) try to neutralize any ingested toxins, secure a definitive airway, obtain baseline vital signs, start an IV line, and transport as soon as possible. D) protect the airway, perform a rapid assessment, obtain vital signs, try to gather a medical history from the family, and transport promptly.

D

31. Which of the following interventions is influenced strongly by the amount of time that has elapsed since a patient ingested a toxic substance? A) Transport B) IV therapy C) Intubation D) Gastric lavage

D

36. Which of the following ECG abnormalities is MOST suggestive of cocaine toxicity? A) Narrowing of the PR interval B) Marked flattening of the T wave C) Narrowing of the QRS complex D) Prolongation of the QT interval

D

41. Signs and symptoms of marijuana use include all of the following, EXCEPT: A) euphoria. B) drowsiness. C) bloodshot eyes. D) decreased appetite.

D

45. Signs of meningeal irritation, such as nuchal rigidity, are MOST commonly seen in patients with a(n): A) subdural hematoma. B) epidural hematoma. C) intracerebral hematoma. D) subarachnoid hemorrhage.

D

46. Which of the following interventions generally is preferred for a patient who overdosed on a barbiturate more than 1 hour ago? A) Gastric lavage B) Syrup of ipecac C) Urine alkalinization D) Activated charcoal

D

52. The recommended dose and method for administering naloxone to a patient who overdosed on a narcotic and is unresponsive and hypoventilating is: A) 0.1 mg/kg rapidly until the patient's respirations improve. B) 0.4 to 2 mg rapidly until the patient regains consciousness. C) 5 to 10 mg via the endotracheal tube until the pupils dilate. D) 2 mg injected slowly until the patient's respirations improve.

D

58. Carboxyhemoglobin: A) is a combination of oxygen and hemoglobin. B) effectively carries oxygen to the body's cells. C) is the chemical by-product of cyanide poisoning. D) is hemoglobin combined with carbon monoxide.

D

60. As the body ages, the intervertebral discs: A) calcify and become more rigid. B) enlarge and result in increased height. C) are not able to protect the spinal cord. D) lose water content and become thinner.

D

63. When chlorine gas comes in contact with the body's mucous membranes, it forms: A) boric acid. B) a strong alkali. C) sulfuric acid. D) hydrochloric acid.

D

65. Cyanide blocks the utilization of oxygen at the cellular level by: A) binding to monoamine oxidase. B) directly destroying red blood cells. C) binding to the hemoglobin molecule. D) combining with cytochrome oxidase.

D

7. Clinical signs and symptoms following exposure to a toxin will manifest MOST rapidly if the patient: A) is older than 70 years of age. B) ingests a large quantity of toxin. C) breathes in the toxic chemical. D) is exposed by the injection route.

D

70. Gamma-hydroxybutyrate is MOST commonly used to: A) induce euphoria. B) enhance the senses. C) treat chronic coughing. D) facilitate sexual assault.

D

73. What is a lethal dose of ethylene glycol in a 190-pound man? A) 50 mL B) 120 mL C) 150 mL D) 175 mL

D

81. Early signs of MAOI overdose include all of the following, EXCEPT: A) nystagmus. B) tachycardia. C) hyperactivity. D) hypoventilation.

D

84. Lithium is MOST commonly used to treat patients with: A) depression. B) schizophrenia. C) chronic anxiety. D) bipolar disorder.

D

87. Prehospital treatment for a patient who overdosed on aspirin may include: A) flumazenil. B) an antipyretic. C) calcium chloride. D) sodium bicarbonate.

D

93. You should be MOST suspicious that a patient has systemic iron toxicity if he or she presents with: A) bradypnea. B) hypertension. C) severe nausea. D) hematemesis.

D

98. The poisonous part of the apricot plant is/are the _______, which contain(s) _______. A) leaves, iron B) root, tyramine C) bulb, oxalic acid D) seed, cyanide

D

99. Poisoning with ________________ is MOST often the result of improper food storage or canning. A) Listeria B) Salmonella C) Toxoplasma D) Clostridium botulinum

D

According to the National Spinal Cord Injury Database, MOST spinal cord injuries are caused by: A) acts of violence. B) athletic activities. C) falls in the elderly. D) motor vehicle crashes.

D

A 34-year-old female states that she feels like she has a grain of sand in her eye. Assessment reveals that her eye and the surrounding area are red. Treatment for her should include: A) carefully assessing her eye for an object imbedded in the globe and removing it if one is present. B) instructing her to continue rapid eye blinking and administering analgesia as indicated. C) gently rubbing the surface of the eye with a cotton-tipped applicator to remove the foreign body. D) gently irrigating her eye and taping the affected eye closed to prevent it from drying out.

D

A 4-year-old child presents with a deep "croup-like" cough, difficulty breathing, and a high fever. You should suspect: A) epiglottitis. B) pharyngitis. C) laryngitis. D) tracheitis.

D

A 45-year-old unrestrained man was ejected from his small truck when it struck a tree. The patient is found approximately 20 feet from the wreckage. Your primary assessment reveals that he is unresponsive and has sonorous respirations and a rapid pulse. Your initial actions should include: A) applying a cervical collar and assisting his ventilations with a bag-mask device. B) rolling the patient onto his side as a unit and suctioning his mouth for 15 seconds. C) performing a tongue-jaw lift and looking in his mouth for any obvious obstructions. D) manually stabilizing his head and opening his airway with the jaw-thrust maneuver.

D

A 50-year-old woman called 9-1-1 after she was suddenly awakened in the middle of the night with the feeling that she was being smothered. Your assessment reveals that she is clearly anxious, is trembling, and complains of chest pain and numbness and tingling to her face and hands. Her blood pressure is 168/96 mm Hg, pulse rate is 140 beats/min, and respirations are 30 breaths/min. Her medications include Xanax, Lipitor, and Vasotec. The MOST appropriate treatment for this patient includes: A) applying a nonrebreathing mask with the flow rate set at 2 L/min, assessing her blood glucose level, administering diazepam for sedation, and transporting her without lights and siren. B) attempting to regulate her breathing by asking her to hold her breath, assessing her end-tidal carbon dioxide level, administering 1 µg/kg of fentanyl IM for sedation, and transporting. C) applying a cardiac monitor, establishing vascular access and administering adenosine, administering oxygen via nasal cannula, considering a beta adrenergic antagonist to lower her blood pressure, and transporting. D) coaching her to slow her breathing, monitoring her oxygen saturation and end-tidal carbon dioxide levels, administering supplemental oxygen, assessing her cardiac rhythm, establishing vascular access, and transporting.

D

A 56-year-old man is experiencing a severe allergic reaction following multiple ant bites. He tells you that he has a prescribed EpiPen, which is expired, and that he takes a beta blocker for hypertension. Which of the following medications would MOST likely improve his condition, given his medical history? A) Proventil and Benadryl B) Epinephrine and albuterol C) A norepinephrine infusion D) Glucagon and ipratropium

D

A compression or burst fracture of the cervical spine would MOST likely occur following: A) a direct blow to the occipital region of the skull. B) rapid acceleration following a motor vehicle crash. C) axial loading after a patient falls and lands feet first. D) a significant fall in which the patient lands head first.

D

A full-body exam for a trauma patient with a significant mechanism of injury and signs of a spinal cord injury: A) is performed at least every 5 to 10 minutes. B) should focus specifically on the spinal region. C) is usually not practical in the prehospital setting. D) should be performed en route to the hospital

D

A hallmark sign of mania is: A) speech retardation. B) persistent insomnia. C) memory distortion. D) marked hyperactivity.

D

A health care worker's fear of contracting a communicable disease is MOST often the result of: A) obsessive-compulsive disorder. B) statistics published by the media. C) a prior history of disease exposure. D) a lack of proper education and training.

D

A moderate diffuse axonal injury: A) is generally characterized by a brief loss of consciousness, with or without retrograde amnesia. B) causes a prolonged loss of consciousness and affects axons in both cerebral hemispheres. C) is the most common result of blunt head trauma and is associated with temporary neuronal dysfunction. D) produces an immediate loss of consciousness and residual neurologic deficits when the patient wakes up.

D

A patient who is confused: A) cannot recall the events that preceded his or her illness. B) should be given 50% dextrose to rule out hypoglycemia. C) can usually be redirected by providing emotional support. D) has an impaired understanding of his or her surroundings.

D

A patient with diaphragmatic breathing without intercostal muscle use has MOST likely experienced a spinal injury above the level of: A) C2. B) C5. C) C7. D) T2.

D

A person with TB infection: A) poses a significant health risk to others. B) has active TB and is highly contagious. C) usually has a negative chest radiograph. D) has tested positive for exposure to TB.

D

A positive Babinski reflex is observed when the: A) toes curve or move downward when the sole of the foot is touched. B) patient responds to pain by flexing the arms and extending the legs. C) patient's reflexes are hyperactive in response to an external stimulus. D) toes move upward in response to stimulation of the sole of the foot.

D

A red, tender lump in the eyelid or at the lid margin is called a: A) rust ring. B) hyphema. C) chalazion. D) hordeolum.

D

A ruptured tympanic membrane: A) commonly results in permanent hearing loss. B) is characterized by CSF leakage from the ears. C) commonly leads to an infection of the middle ear. D) is extremely painful but typically heals spontaneously.

D

Accelerated thinking in which the mind skips very rapidly from one thought to the next is called: A) confabulation. B) circumstantial thinking. C) perseveration. D) flight of ideas.

D

According to Part G of the Ryan White Comprehensive AIDS Resources Emergency Act, medical facilities are required to notify emergency responders of potentially infectious diseases involving patients they transported no longer than ___ hours from the time they have a suspect case. A) 6 B) 12 C) 24 D) 48

D

Acquired immunodeficiency syndrome: A) occurs in as many as 85% of HIV-infected individuals. B) exists when T-helper lymphocytes are dangerously high. C) most often occurs within 5 to 10 years of HIV infection. D) is characterized by the presence of opportunistic infections.

D

An abnormal immune response that the body develops when it is reexposed to an allergen is called: A) anaphylaxis. B) secondary response. C) hypersensitivity. D) an allergic reaction.

D

An acute dystonic reaction is characterized by: A) a sudden catatonic state that results from oversedation with drugs such as Zyprexa and Mellaril. B) intermittent explosive behavior after a person suddenly stops taking medications for schizophrenia. C) dry mouth, blurred vision, and cardiac dysrhythmias following treatment with a neuroleptic drug. D) muscle spasms of the neck, face, and back within a few days of starting treatment with an antipsychotic drug.

D

An alteration in one's perception of reality is MOST characteristic of: A) depersonalization. B) posttraumatic stress. C) a histrionic personality. D) a dissociative disorder.

D

Any patient who is experiencing a panic attack: A) should rebreathe his or her carbon dioxide from a paper bag. B) must be given high-flow oxygen with a nonrebreathing mask. C) should be presumed to have experienced a recent stressful event. D) should be evaluated to rule out an underlying medical condition.

D

At a minimum, you should be able to assess an emotionally disturbed patient's: A) ability to recall. B) blood pressure. C) blood sugar level. D) general appearance.

D

Cardiovascular effects of anaphylaxis include: A) diaphoresis, bradycardia, and edema. B) an irregular pulse, pallor, and pruritus. C) peripheral vasoconstriction and cool skin. D) tachycardia, flushed skin, and hypotension.

D

Cerebrospinal fluid drainage from the ears is MOST indicative of: A) a nasal fracture. B) intracerebral bleeding. C) an epidural hematoma. D) a skull fracture.

D

Common central nervous system manifestations of anaphylactic shock include all of the following, EXCEPT: A) anxiety. B) headache. C) confusion. D) combativeness.

D

Components of the environmental assessment of an elderly patient include all of the following, EXCEPT: a. assessing for the odor of urine or feces. b. looking for rotted floors or faulty wiring. c. ensuring that food is adequate and unspoiled. d. inquiring about any delays in obtaining food.

D

During your assessment of a 79 year old woman who was involved in a motor vehicle crash you find that she is responsive to pain only and is breathing with a marked reduction in tidal volume. Your rapid assessment reveals that she has ecchymosis and crepitus overseceral her ribs. you should: a. avoid any form of positive-pressure ventilation as this may cause barotrauma and a pneumothorax b. stabilize her fractured ribs with bulky dressing and administer high-flow oxygen via non-breathing mask c. immediately secure her airway with an endotracheal tube b/c she is at increased risk for aspiration d. assit her ventilations with a bag-mask device, delivering just enough volume to produce visible chest rise

D

During your assessment of an unresponsive 70-year-old man, the patient's wife tells you that he takes Luvox and BuSpar, among other medications. From this limited information, you should conclude that the patient suffers from: A) bipolar disorder. B) paranoid schizophrenia. C) chronic manic behavior. D) depression and anxiety.

D

Failure of the heart's primary pacemaker and the development of alternate pacemakers in the atria would most likely result in: a. junctional rhythms b. atrioventricular block c. ventricular ectopy d. atrial fib

D

Following a significant exposure, the source patient is routinely tested for all of the following, EXCEPT: A) HIV. B) HCV antibody. C) HBV antigen. D) HBV antibody.

D

10. An infant born with a pink body and blue extremities, a pulse rate of 90 beats/min, a strong cry, and active movement should be assigned an initial Apgar score of: A) 5. B) 6. C) 7. D) 8.

D) 8.

Following retirement, many older people: A. experience a rapid decline in their underlying health and become incapacitated within 12 months B. often experience an improvement in their overall health because the stress of working has been eliminated C. return to work within 6 months b/c their retirement pension does not support them adequately D. commonly experience decreased self-esteem b/c they no longer feel useful or productive in society

D

Following return of spontaneous circulation, a patient remains comatose. Which of the following interventions would MOST likely be performed? A) Dextrose infusion B) Field extubation C) Epinephrine bolus D) Induced hypothermia

D

Hepatitis B is also referred to as: A) CSF hepatitis. B) fecal hepatitis. C) enteral hepatitis. D) serum hepatitis.

D

Hyperventilation of a patient who is in cardiac arrest: A) increases preload and enhances cardiac output. B) causes a marked decrease in intrathoracic pressure. C) is indicated if the arrest interval exceeds 10 minutes. D) has been shown to reduce coronary artery perfusion.

D

If a psychotic person's level of consciousness is fluctuating, you should: A) administer 25 g of dextrose. B) immediately assess blood pressure. C) consider administering haloperidol. D) suspect an organic brain syndrome.

D

In anxiety disorders, the dominant moods are: A) anger and agitation. B) confusion and apathy. C) euphoria and elation. D) fear and apprehension.

D

In contrast to a 30 year old man if a 70 year old were to consume an excessive amount of alcohol a. he would experience a lower blood alcohol concentration secondary to a decrease in total body water b. the alcohol would be eliminated from his body much faster unless he is taking an antihypertensive drug c. it would take longer for him to develop alcohol toxicity secondary to decreased metabolism in the liver d. he would experience a higher blood alcohol concentration due to decreases in body mass and total body water

D

In contrast to acquired immunity, natural immunity occurs when: A) the body is vaccinated, allowing it to produce antibodies without having to experience the disease itself. B) groups of people are immunized against a substance, which protects vulnerable people in the group. C) the mother passes antibodies to the fetus via the placenta, thus protecting the fetus against a variety of diseases. D) the body encounters the antigen and experiences a full immune response with all the pathology of the disease.

D

In dark-skinned patients, cyanosis can be detected in the: A) sclera. B) iris. C) cornea. D) conjunctiva.

D

In which of the following situations would it be MOST appropriate to apply a vest-type extrication device or a short backboard to a patient who is seated in his or her crashed motor vehicle? A) Conscious with bilateral femur fractures B) Unconscious with obvious spinal deformity C) Confused with lower back pain and tachycardia D) Conscious with neck pain and stable vital signs

D

Which of the following locations would provide the BEST protection from a lightning strike? A) An open shed or lean-to B) A spot at least 5 miles away from the storm C) Curled up in a ball in an open area D) A car with the windows rolled up

D) A car with the windows rolled up

Occupationally acquired hepatitis C virus infection: A) is not possible because an effective one-series vaccine is available. B) is most commonly contracted via blood exposure to nonintact skin. C) occurs by ingestion of food that is contaminated with infected feces. D) is related to a contaminated needlestick with visible blood on the sharp.

D

Organic brain syndrome is a term used to describe a condition in which a person's abnormal behavior is: A) irreversible and cannot be linked to a physical abnormality or the influence of illicit substances. B) caused by premature brain cell deterioration that is progressive and cannot be reversed. C) directly related to an endocrine disturbance that results in the underproduction of key hormones. D) caused by a physical illness or the influence of a substance that interferes with normal cerebral function.

D

Patients with ______________ are at an increased risk for anaphylaxis. A) strep infection B) acute pharyngitis C) immunosuppression D) atopic dermatitis

D

Patients with evidence of trauma above the _________ should be considered at risk for an associated spine injury. A) diaphragm B) pelvis C) umbilicus D) clavicles

D

Proprioception is defined as: A) a person's ability to sense light touch. B) the loss of thermoregulatory function distal to an injury. C) a person's awareness of pain and the ability to localize it. D) the ability to perceive the position and movement of one's body.

D

Psychomotor abnormalities associated with depression include: A) a flat affect. B) distractibility. C) fatigue and insomnia. D) agitation and pacing.

D

Side effects of MAOI antidepressants include: A) hypersomnia. B) acute bradycardia. C) suicidal behavior. D) orthostatic hypotension.

D

Signs and symptoms of HIV infection may include all of the following, EXCEPT: A) acute febrile illness. B) swollen lymph glands. C) malaise and a headache. D) right upper quadrant pain.

D

Signs and symptoms of a ruptured tympanic membrane include: A) permanent deafness and a unilateral headache. B) CSF leakage from the ear and severe tinnitus. C) a low-grade fever and fullness in the ear. D) hearing loss and blood drainage from the ear.

D

Signs and symptoms of depression that last for at least 2 years: A) are typical of major depressive disorder. B) require long-term hospitalization to treat. C) are typically not associated with anxiety. D) are highly suggestive of dysthymic disorder.

D

The MOST effective way for the paramedic to minimize further injury in a patient with a spinal injury is: A) prompt transport of the patient to a trauma center. B) rapid administration of corticosteroid medications. C) aggressive administration of IV crystalloid solutions. D) spinal motion restriction and prevention of heat loss.

D

The ________________ secretes and drains tears from the eye. A) nasolacrimal duct B) conjunctiva C) aqueous humor D) nasolacrimal apparatus

D

The anterior chamber is the portion of the globe between the _____ and the _____, and is filled with _____ humor. A) iris, lens, vitreous B) cornea, iris, aqueous C) lens, iris, vitreous D) lens, cornea, aqueous

D

The bony sockets for the teeth that reside in the mandible and maxilla are called: A) cusps. B) pulp. C) dentin. D) alveoli.

D

The cervical spine: A) is the weakest part of the spinal column, yet is generally the least injured part of the spine. B) includes the first seven bones of the vertebral column that indirectly protect the spinal cord. C) consists of vertebrae that are slightly larger than those of the less powerful thoracic spine. D) supports the weight of the head and permits a high degree of mobility in multiple planes.

D

The chance for return of spontaneous circulation is BEST when: A) a vasopressor is administered every 3 to 5 minutes during CPR. B) an advanced airway device is inserted during a resuscitation attempt. C) an antidysrhythmic drug is used if ventricular fibrillation is present. D) timely chest compressions are performed with little or no interruption.

D

The leading cause of lower respiratory tract infections in infants, older people, and immunocompromised individuals is the: A) rotovirus. B) influenza virus. C) parainfluenza virus. D) respiratory syncytial virus.

D

The onset of fever commonly occurs as early as ___ days following exposure to the measles. A) 2 B) 3 C) 5 D) 7

D

The paramedic should consider extending the resuscitation effort of a patient with refractory cardiac arrest if: A) the cardiac arrest was caused by submersion in warm water. B) ventricular fibrillation occurred at any point during the arrest. C) the patient has no known significant past medical history. D) return of spontaneous circulation of any duration occurred.

D

Transport of a patient in anaphylactic shock may be delayed for all of the following reasons, EXCEPT: A) aggressive airway control. B) epinephrine administration. C) assessment of lung sounds. D) a secondary assessment.

D

Unlike a cerebral concussion, a cerebral contusion is: A) typically not associated with retrograde amnesia or focal neurologic deficits. B) a type of diffuse axonal injury that often leads to permanent neurologic damage. C) a diffuse brain injury that results in severe intracranial hemorrhage and pressure. D) associated with physical brain damage and more pronounced neurologic deficits.

D

What is the pathophysiology of Meniere disease? A) Irritation and swelling in the inner ear affect the nerves of the inner ear and produce dizziness, loss of balance, and tinnitus. B) Dysfunction of the eustachian tube leads to increased pressure within the middle ear, which causes damage to the malleus, incus, and stapes. C) The angle of the eustachian tube does not allow for proper drainage, which allows infective material to collect in the middle ear and cause infection. D) Endolymphatic rupture creates increased pressure in the cochlear duct, which then leads to damage to the organ of Corti and the semicircular canals.

D

What part of the eye is MOST commonly injured following a thermal burn? A) Globe B) Retina C) Cornea D) Eyelids

D

What portion of the brainstem is responsible for maintenance of consciousness, specifically one's level of arousal? A) Diencephalon B) Limbic system C) Basal ganglia D) Reticular activating system

D

What spinal nerve tract carries information regarding pain and temperature? A) Corticospinal B) Spinocerebellar C) Reticulospinal D) Lateral spinothalamic

D

When a disease infects large numbers of people and spreads all over the world, it is considered a(n): A) endemic. B) epidemic. C) outbreak. D) pandemic.

D

When administering an EpiPen to a 30-year-old man with a severe allergic reaction, you should recall that: A) a 1:2,000 solution is used because the patient is an adult. B) the SQ route is used in order to achieve a rapid effect. C) 0.15 mg is the usual dose delivered by the adult EpiPen. D) the drug cartridge contains 0.3 mg of a 1:1,000 solution.

D

When an unrestrained passenger's head strikes the windshield of a motor vehicle following rapid deceleration: A) the anterior portion of the brain sustains stretching or tearing injuries, and the posterior portion of the brain sustains compression injuries. B) the head falls back against the headrest or seat and the brain collides with the rear of the skull, resulting in direct injury to the occipital lobe. C) the brain initially strikes the rear of the skull, resulting in direct bruising, and then rebounds and strikes the front part of the skull. D) compression injuries occur to the anterior portion of the brain, and stretching or tearing injuries occur to the posterior portion of the brain.

D

When asking an emotionally disturbed patient a series of questions, you should: A) keep your questions as indirect as you possibly can. B) simplify the process by asking closed-ended questions. C) try to ask questions that can be answered with a yes or no. D) use words such as "how" and "what" whenever possible.

D

When performing the standing takedown technique to immobilize a patient's spine, the patient is secured to the long backboard with straps: A) while still in the standing position. B) after the board is placed on the stretcher. C) after a cervical collar has been applied. D) after he or she is lowered to the ground.

D

Which of the following central nervous system functions does not change as a person ages a postural stability b. cognitive speed c. memory retrieval d. intelligence level

D

Which of the following diseases is bacterial in nature, has an insidious onset, and is characterized by an irritating cough that may become paroxysmal in about 1 to 2 weeks? A) Tetanus B) Bronchitis C) Diphtheria D) Pertussis

D

Which of the following medical procedures would pose the LEAST risk of exposure to an infectious disease? A) Administering a subcutaneous injection B) Covering a wound that is bleeding minimally C) Delivering the baby of an HIV-negative mother D) Assessing a patient's temperature by the oral route

D

Which of the following medications has the SLOWEST onset of action when given to a patient with a severe allergic reaction? A) Albuterol B) Glucagon C) Diphenhydramine D) Methylprednisolone

D

Which of the following questions would be the MOST appropriate initial question to ask when broaching the subject of suicide with a depressed patient? A) "Did you ever feel that you'd be better off dead?" B) "Have you ever thought of causing harm to yourself?" C) "Do you have the means to carry out a suicide attempt?" D) "Have you ever thought that life wasn't worth living?"

D

Which of the following statements regarding West Nile virus (WNV) is correct? A) It is estimated that approximately 20% to 30% of mosquitoes carry WNV. B) The incubation period for WNV is 3 to 14 days following the bite of an infected tick. C) Up to 80% of people infected with WNV experience a severe headache, body rash, and fever. D) There is no period of communicability because WNV is not transmitted from person to person.

D

40. Immediate pain from the heel to the calf and a sudden inability for plantar flexion of the foot is MOST indicative of: A) acute tendonitis. B) ligament disruption. C) a dislocated ankle. D) Achilles tendon rupture.

D) Achilles tendon rupture.

Which of the following statements regarding genital herpes is MOST correct? A) The lesions of genital herpes remain infectious for 12 to 24 days. B) The incubation period for genital herpes often lasts up to 3 weeks. C) In females, genital herpes presents as a single vesicle on the vulva. D) Acyclovir is used to reduce a herpetic outbreak, but there is no cure.

D

Which of the following statements regarding the brainstem is correct? A) The brainstem provides protection to the cerebellum. B) The brainstem is responsible for muscle coordination. C) All but 4 of the 12 cranial nerves exit the brainstem. D) The brainstem connects the spinal cord to the brain.

D

Which of the following statements regarding the hypothalamus is correct? A) The hypothalamus works in conjunction with the subthalamus and regulates motor functions of the body. B) The hypothalamus is located in the inferior portion of the diencephalon and influences body movement in response to fear. C) The hypothalamus is located between the brainstem and cerebrum and is solely responsible for maintaining body temperature. D) The hypothalamus is the most inferior portion of the diencephalon and controls functions such as hunger, sleep, and vomiting.

D

Which of the following steps of caring for a patient with a psychiatric emergency would MOST likely give the patient the feeling that something is being done to help? A) Asking direct questions B) Encouraging the patient to talk C) Remaining calm and confident D) Developing a definite plan

D

Which of the following types of skull fracture would be the LEAST likely to present with gross physical signs? A) Open fracture B) Depressed fracture C) Basilar fracture D) Linear fracture

D

While applying the defibrillator pads, you note that the patient has a transdermal nitroglycerin patch on the right anterior chest. You should: A) place the pads at least 1 inch from the patch. B) place the second pad in between the scapulae. C) move the patch to the left anterior chest. D) remove the patch and wipe away any residue.

D

While assessing a 59-year-old man with an acute onset of crushing chest pain and diaphoresis, the patient tells you that he recently tested positive to a tuberculin skin test. His vital signs reveal hypertension and tachycardia. You should be MOST concerned with: A) applying a mask to the patient to reduce your chance of exposure. B) reporting this to your supervisor and receiving a tuberculin skin test. C) establishing vascular access and rapidly transporting to the hospital. D) the fact that he may be experiencing an acute myocardial infarction.

D

Within the cochlea, at the ______________, vibration stimulates hair movements that form nerve impulses that travel to the brain via the auditory nerve. A) oval window B) cochlear duct C) semicircular canals D) organ of Corti

D

You and your partner are performing CPR on a 60-year-old woman who was initially in asystole. After 2 minutes, you look at the cardiac monitor and determine that she is in ventricular fibrillation. Your next action should be to: A) quickly assess for a carotid or femoral pulse and then deliver a single shock with 200 biphasic joules. B) check her cardiac rhythm in two contiguous leads to confirm that she is in ventricular fibrillation. C) continue CPR, establish vascular access, and administer 300 mg of amiodarone or 1.5 mg/kg of lidocaine. D) deliver a single shock and instruct your partner to resume chest compressions while you resume ventilations.

D

You and your partner are transferring a 76-year-old woman from a local nursing facility to the emergency department for evaluation of an acute onset of fever. While reviewing the transfer record, you note that the patient was recently treated with daptomycin. This patient MOST likely has: A) vancomycin-resistant enterococci. B) hemorrhagic fever caused by hantavirus. C) an infection caused by the hepatitis A virus. D) methicillin-resistant staphylococcus aureus.

D

You are assessing an 80-year old man who complains of generalized weakness. He denies chest pain or shortness of breath and tells you that he has become progressively weaker over the past 3 days. His medical history is significant for two prior heart attacks, hypertension, chronic renal insufficiency, and atrial fibrillation. His blood pressure is 108/60mm Hg, pulse rate is 94 beats/min and irregular, and respirations are 20 breaths/min and unlabored. Auscultation of his lungs reveals scattered crackles, and the ECG reveals atrial fibrillation. In addition to administering supplemental oxygen, you should: a. start an IV with D5W, assess his blood glucose level, and administer half the usual dose of diltiazem to lower the rate of his renal fibrillation. b. obtain a 12-lead ECG tracing, start an IV line with normal saline and give a 250-mL fluid bolus, reassess his blood pressure, and transport him. c. establish vascular access, administer 2 mg of morphine to treat his apparent pulmonary edema, continue to monitor the ECG, and transport. d. obtain a 12-lead ECG tracing, assess his blood glucose level, establish vascular access and set the rate to keep the vein open, and transport to the hospital.

D

You are assessing an unresponsive patient and note the presence of a hyphema to his left eye. This is evidence of: A) a foreign body in the eye. B) a chemical burn to the eye. C) lacrimal gland dysfunction. D) blunt force trauma to the eye.

D

You are performing one-rescuer CPR on a 50-year-old woman in cardiac arrest. A bystander returns with an AED. You ask the bystander to attach the pads to the patient's chest as you continue CPR. After rhythm analysis, the AED states, "Shock advised." You should: A) confirm the absence of a pulse and deliver the shock. B) perform CPR for 2 more minutes and then defibrillate. C) defibrillate the patient and reassess for a carotid pulse. D) deliver a single shock and immediately resume CPR

D

You are the team leader in the attempted resuscitation of an adult man in ventricular fibrillation. An advanced airway device has been inserted and vascular access has been obtained. As you observe the actions of your team members, you should ensure that: A) the patient is defibrillated one time every 60 seconds as necessary. B) the person managing the airway delivers one breath every 3 to 5 seconds. C) no one person performs chest compressions for more than 5 minutes at a time. D) compressions are hard and fast, with full chest recoil between compressions.

D

You have just completed spinal immobilization of a hemodynamically stable patient with a possible spinal injury. Prior to moving the patient to the ambulance, it is MOST important to: A) start an IV of normal saline in case the patient deteriorates. B) apply a cardiac monitor and obtain a full set of vital signs. C) perform a detailed secondary assessment to detect other injuries. D) reassess pulse, motor, and sensory functions in all extremities.

D

You have just defibrillated an adult woman who is in pulseless ventricular tachycardia. After performing CPR for 2 minutes, you reassess her cardiac rhythm and determine that she is experiencing torsade de pointes; she also remains pulseless. It has been approximately 2 minutes since you administered the last dose of epinephrine. You should next: A) give 300 mg of amiodarone via rapid IV or IO push. B) give 40 units of vasopressin while CPR is ongoing and reassess in 2 minutes. C) push the synchronize button on the defibrillator and cardiovert with 200 joules. D) give 1 to 2 g of magnesium sulfate without interrupting chest compressions.

D

You would MOST likely encounter oral candidiasis in a patient: A) who has had numerous cavities. B) who takes oral corticosteroids. C) with a history of gingivitis. D) who is immunocompromised.

D

a common cause of nearly loss in the elderly is presbycusis, which is defined as: a. a progressive loss of hearing particularly in the low freq. which explains why their TV is turned up b. an acute hearing impairment caused by idiopathic rupture of the tympanic membrane in the middle part of the ear c. a degenerative condition in which the person experiences difficulty hearing both low and high freq. sounds d. loss of high-freq. sounds and the inability to differentiate a particular sound from background noise

D

after falls, _________ is/are the 2nd leading cause of accidental death among elderly people a. thermal burns b. drug interactions c. submersion injury d. motor vehicle accidents

D

an elderly person is at increased risk for aspiration, primarily from: a. atrophy of the epiglottis b. a decreased ability to swallow c. slowing of the ciliary mechanisms d. decreased cough and gag reflex

D

in contrast to delirium dementia is: a. often caused by conditions such as poisonings and infections b. an acute state of confusing that may last for up to 1 week c. often reversible if the underlying cause is identified rapidly d. a progressive disease that produces irreversible brain failure

D

in contrast to younger p/t, elderly p/t a. present with multiple imaginary complaints when they are ill b. typically overdramatize their medical problems to get attention c. are the population with the highest rate of hypochondriasis d. tend not to complain even when they have legitimate

D

the most common risk factor for the development of type 2 diabetes in people over 65 years of age is a. poor dietary habits and a secondary lifestyle b. freq infections that do not heal properly c. hypertension of longer than 5 years' duration d. the presence of more than one chronic disease

D

the relatively high use of non steroidal anti-inflammatory drugs by older p/ts predisposes them to: a. constipation b. cholelithiasis c. mesenteric ischemia d. peptic ulcer disease

D

which of the following cardiac dysrhythmias is associated with the highest risk of stroke a. av heart block b. sinus tachycardia c. junctional rhythm d. atrial fib

D

which of the following is an extrinsic cause of falls in the elderly a. postural hypotension b. dizziness or syncope c. a pathologic fracture d. an uneven sidewalk

D

you respond to a residence for an unknown emergency involving an 83 year old man. when you arrive you find the p/t who is conscious and alert sitting on his couch with his head held over bowl. he tells you that his nose started bleeding about 20 min ago and he cannot get it to stop. as your partner assists the p/t controlling the bleeding you inquire about this medical history.he tells you that he has chronic atrial fib his high blood pressure and is occasionally depressed. when you ask him about compliance with his medications he tells you I take my medication every day but the writing on the bottles is so small. The p/t blood pressure is 112/58 mm Hg, pulse rate is 88 beats/min and irregular, and respirations are 18 and unlabored. This p/t has most likely inadvertently. a. overdosed on his Xanax b. underused on his Toprol c. underused on his Digoxin d. overdosed on his warfarin

D

26. By the end of week ____ of gestation, a woman's diastolic blood pressure should return to its prepregnancy level. A) 30 B) 32 C) 34 D) 36

D) 36

22. The correct positive-pressure ventilation rate for an apneic newborn is: A) 12 to 20 breaths/min. B) 20 to 30 breaths/min. C) 30 to 40 breaths/min. D) 40 to 60 breaths/min.

D) 40 to 60 breaths/min

66. The MOST appropriate ET tube for a 6-year-old child is: A) 4.0 mm, cuffed. B) 4.5 mm, cuffed. C) 5.0 mm, uncuffed. D) 5.5 mm, uncuffed.

D) 5.5 mm, uncuffed.

33. The recommended IV dose and concentration of epinephrine for the newborn is: A) 0.01 mg/kg, 1:1,000. B) 0.3 to 1 mL/kg, 1:10,000. C) 0.01 to 0.03 mg/kg, 1:1,000. D) 0.1 to 0.3 mL/kg, 1:10,000.

D) 0.1 to 0.3 mL/kg, 1:10,000

31. In a closed femur fracture, blood loss may exceed _______ before enough pressure develops to tamponade the bleeding. A) 250 mL B) 500 mL C) 750 mL D) 1,000 mL

D) 1,000 mL

16. What is the mean arterial pressure of a person who has a blood pressure of 140/90 mm Hg? A) 82 mm Hg B) 97 mm Hg C) 101 mm Hg D) 107 mm Hg

D) 107 mm Hg

68. A(n) ___ orogastric or nasogastric tube would the MOST appropriate size for a 4-year-old child. A) 4F B) 6F C) 8F D) 10F

D) 10F

13. If you feel 13 pulsations in a 6-second time frame, the newborn's heart rate is approximately: A) 30 beats/min. B) 60 beats/min. C) 90 beats/min. D) 130 beats/min.

D) 130 beats/min.

112. Which of the following is the MOST appropriate dose of activated charcoal for a 45-pound child? A) 5 g B) 10 g C) 15 g D) 20 g

D) 20 g

44. Closed bilaterally fractured femurs can result in internal blood loss of up to: A) 1,000 mL. B) 1,500 mL. C) 2,000 mL. D) 3,000 mL.

D) 3,000 mL.

43. In which of the following situations would a newborn MOST likely experience a seizure? A) Hyperglycemia B) Post-term gestation C) Maternal aspirin use D) 33 weeks' gestation

D) 33 weeks' gestation

24. Which of the following statements regarding acrocyanosis is correct? A) Acrocyanosis is seen in the skin and mucous membranes and is a late finding if respiratory failure or shock is present. B) Acrocyanosis is only considered to be a normal finding in newborns and usually resolves within 12 hours following birth. C) Acrocyanosis is a bluish discoloration of the chest, abdomen, and face and is the most extreme visual indicator of poor perfusion. D) Acrocyanosis is cyanosis of the hands and feet, and is a normal finding in infants younger than 2 months of age who are cold.

D) Acrocyanosis is cyanosis of the hands and feet, and is a normal finding in infants younger than 2 months of age who are cold.

2. Which of the following is/are NOT an exocrine gland? A) Liver B) Sweat glands C) Salivary glands D) Adrenal glands

D) Adrenal glands

36. Which of the following clinical findings is MOST suggestive of inadequate oxygenation? A) Irregular tachycardia B) Blood pressure of 90/50 mm Hg C) Accessory muscle use D) Altered mental status

D) Altered mental status

12. What type of medication dries secretions in the airway and prevents the cilia from removing them effectively? A) Diuretic B) Antibiotic C) Antipyretic D) Antihistamine

D) Antihistamine

57. You are assessing a 70-year-old man with a blood glucose reading of 400 mg/dL and note the presence of sharply peaked T waves on the cardiac monitor. Which of the following medications would this patient MOST likely receive in the prehospital setting? A) Insulin B) Glucagon C) Potassium D) Bicarbonate

D) Bicarbonate

56. Which of the following statements regarding bronchiolitis is correct? A) Infants who were born past 42 weeks are at highest risk for respiratory failure and arrest secondary to bronchiolitis. B) The pathophysiology of bronchiolitis is acute bronchospasm secondary to a bacterium that enters the lower respiratory tract. C) Bronchiolitis is usually caused by the metapneumovirus and occurs with greatest frequency during late spring and early summer. D) Bronchiolitis is a viral infection of the lower airway that commonly affects infants and children younger than 2 years of age.

D) Bronchiolitis is a viral infection of the lower airway that commonly affects infants and children younger than 2 years of age.

Which of the following burn injuries or patterns should make you the MOST suspicious for abuse? A) Burns to the forearm B) Splash burns to a leg C) An arc burn to the hand D) Burns with formed shapes

D) Burns with formed shapes

76. What is formed when carbon monoxide binds to the hemoglobin molecule? A) Myoglobin B) Oxyhemoglobin C) Methemoglobin D) Carboxyhemoglobin

D) Carboxyhemoglobin

58. Which of the following injuries or conditions would cause obstructive shock? A) Severe burns B) Massive sepsis C) Pelvic fracture D) Cardiac tamponade

D) Cardiac tamponadev

48. What should you do prior to inserting an indwelling urinary catheter if a home health care patient has sensation in the penile area? A) Inject 10 mL of lidocaine into the urethra. B) Administer a mild sedative prior to catheterization. C) Minimize any pain by allowing the patient to sit up. D) Coat the end of the catheter with an anesthetic gel.

D) Coat the end of the catheter with an anesthetic gel.

48. Which of the following statements regarding croup is correct? A) Croup is also referred to as acute bacterial subglottic stenosis. B) Hallmark signs of croup include high fever and a sore throat. C) Most cases of croup result in severe hypoxia and hypercarbia. D) Croup is a viral upper airway infection that may cause stridor.

D) Croup is a viral upper airway infection that may cause stridor.

4. Which of the following premorbid conditions would MOST likely occur in an otherwise healthy adult? A) Renal failure B) Congestive heart failure C) Coronary artery disease D) Drug toxicity

D) Drug toxicity

35. Which of the following statements regarding Rh disease during pregnancy is correct? A) Isoimmunization occurs when an Rh-positive woman becomes pregnant by an Rh-negative man. B) Rh disease is a problem during the first pregnancy and occurs when the mother's blood is Rh positive. C) If the fetus inherits Rh-positive blood, it will create antibodies that can result in maternal hemolysis. D) During subsequent pregnancies, the Rh antibody will cross the placental barrier and attack the fetal red blood cells.

D) During subsequent pregnancies, the Rh antibody will cross the placental barrier and attack the fetal red blood cells.

Which of the following bones is part of the cranial vault AND the face? A) Vomer B) Palatine C) Lacrimal D) Ethmoid

D) Ethmoid

What part of the eye is MOST commonly injured following a thermal burn? A) Globe B) Retina C) Cornea D) Eyelid

D) Eyelid

52. What effect would the restoration of normotension have on a patient with internal bleeding and a blood pressure of 70/54 mm Hg? A) Increased hemostasis and improved cerebral perfusion B) Acute pulmonary edema secondary to volume overload C) Facilitation of the clotting mechanisms that stop bleeding D) Formed clot dislodgement and worsened internal bleeding

D) Formed clot dislodgement and worsened internal bleeding

11. ________ cells are found in the lining of the airways and produce a blanket of mucus that covers the entire lining of the conducting airways. A) Kupfer B) Alveolar C) Ciliary D) Goblet

D) Goblet

102. Which of the following clinical presentations is MOST consistent with cocaine ingestion in a child? A) Diaphoresis, miosis, tachycardia, and bronchospasm B) Miosis, bradycardia, hypoventilation, and hypotension C) Mydriasis, diarrhea, hypothermia, and hallucinations D) Hypertension, tachycardia, diaphoresis, and mydriasis

D) Hypertension, tachycardia, diaphoresis, and mydriasis

Which of the following abdominal segments is anterior and is the most inferior? A) Umbilical region B) Iliac region C) Hypochondrial region D) Hypogastric region

D) Hypogastric region

36. Which of the following is often not acquired during the SAMPLE history of an adult, but should be routinely acquired in an infant or child? A) Prescribed medications B) Nature of symptoms C) Preceding events D) Immunizations

D) Immunizations

3. Which of the following conditions, if it remains undetected until puberty, can result in acute pain, severe constipation, and low back pain at the onset of menses? A) Ovarian cyst B) Endometriosis C) Ectopic pregnancy D) Imperforate hymen

D) Imperforate hymen

22. Which of the following statements regarding ectopic pregnancy is MOST correct? A) Ectopic pregnancy occurs when a fertilized egg implants in a fallopian tube. B) Most ectopic pregnancies present with symptoms during the second trimester. C) Use of an intrauterine device is the most common cause of an ectopic pregnancy. D) In ectopic pregnancy, a fertilized egg implants somewhere other than the uterus.

D) In ectopic pregnancy, a fertilized egg implants somewhere other than the uterus.

16. Which of the following statements regarding insulin is correct? A) Insulin stimulates the conversion of glycogen to glucose. B) An increase in insulin levels causes an increase in blood glucose. C) Insulin is a pancreatic hormone that performs exocrine functions. D) Insulin is the only hormone that decreases blood glucose levels.

D) Insulin is the only hormone that decreases blood glucose levels.

89. Which of the following medications is a parasympathetic bronchodilator? A) Alupent B) Albuterol C) Bronkosol D) Ipratropium

D) Ipratropium

27. Which of the following musculoskeletal injuries would MOST likely occur together? A) Scaphoid and femur fractures B) Calcaneal and c-spine fractures C) Scapular and clavicular fractures D) Knee dislocation and tibial fracture

D) Knee dislocation and tibial fracture

41. Which of the following is the BEST indicator of tissue perfusion during compensated shock? A) Systolic blood pressure B) Pulse rate and quality C) Skin color and condition D) Level of responsiveness

D) Level of responsiveness

41. Which of the following thoracic injuries generally does NOT require immediate treatment? A) Flail chest B) Open pneumothorax C) Tension pneumothorax D) Myocardial contusion

D) Myocardial contusion

45. You are caring for a young woman with constant, diffuse abdominal pain, referred pain to both shoulders, and vaginal bleeding. She states that her last menstrual period was 2 months ago, but she adamantly denies being pregnant. Her blood pressure is 86/50 mm Hg, pulse rate is 120 beats/min and weak, and respirations are 24 breaths/min and regular. Which of the following interventions is NOT appropriate for this patient? A) IV fluid boluses B) Thermal management C) Cardiac monitoring D) Narcotic analgesia

D) Narcotic analgesia

7. Which of the following behaviors or actions should make you the MOST suspicious that a caregiver has abused his or her child? A) Asking other siblings to leave the room while the child is examined B) Demanding that you take the child to the hospital as soon as possible C) Difficulty recalling the last time the child was seen by a physician D) Offering unsolicited explanations for abnormal physical exam findings

D) Offering unsolicited explanations for abnormal physical exam findings

19. Which of the following are often the primary tasks for paramedics who are caring for a patient with a terminal illness? A) Airway care and thermal management B) ECG monitoring and antidysrhythmic therapy C) Antibiotic and antipyretic therapy D) Pain assessment and management

D) Pain assessment and management

46. Which of the following is NOT one of the 6 Ps of musculoskeletal injury assessment? A) Pallor B) Parasthesias C) Pulselessness D) Passive extension

D) Passive extension

Which of the following is the MOST significant complication associated with a fractured nasal bone? A) Facial swelling B) Lateral displacement C) Damage to the septum D) Posterior epistaxis

D) Posterior epistaxis

Which of the following statements regarding postrenal acute renal failure (ARF) is correct? A) Patients with postrenal ARF typically develop severe hypokalemia. B) Postrenal ARF involves damage to the renal parenchyma or tubules. C) Postrenal ARF typically results in decreased pressure on the nephrons. D) Postrenal ARF is caused by obstruction of urine flow from the kidneys.

D) Postrenal ARF is caused by obstruction of urine flow from the kidneys.

87. Which of the following components is NOT used to distinguish sinus tachycardia from reentry supraventricular tachycardia? A) Pulse rate B) P wave presence C) Systolic blood pressure D) QRS complex width

D) QRS complex width

5. What is the corpus luteum? A) The by-product of the release of progesterone B) The thickened inner lining of the uterine wall C) A hormone that is excreted throughout the ovarian cycle D) Remnants of the follicle after the egg has been released

D) Remnants of the follicle after the egg has been released

97. A morbidly obese man called 9-1-1 because of difficulty breathing. When you arrive, you find the 39-year-old patient lying supine in his bed. He is in marked respiratory distress and is only able to speak in two-word sentences. He has a history of hypertension, but denies any respiratory conditions. What should you do FIRST? A) Begin assisting his ventilations. B) Assess his oxygen saturation level. C) Administer a beta-2 agonist drug. D) Sit him up or place him on his side.

D) Sit him up or place him on his side.

57. A dialysis patient requires pharmacologically assisted intubation. Which of the following medications should be avoided? A) Midazolam B) Etomidate C) Vecuronium D) Succinylcholine

D) Succinylcholine

7. Which of the following is an example of endocrine regulation via a negative feedback mechanism? A) As blood glucose levels fall, glucagon is secreted and stimulates the liver to convert glycogen to glucose. B) A stress response stimulates the sympathetic nervous system to release epinephrine into the bloodstream. C) When circulating volume is decreased, the pituitary gland secretes antidiuretic hormone into the bloodstream. D) The neural regulating mechanism decreases its signals to the adrenal medulla, decreasing epinephrine release.

D) The neural regulating mechanism decreases its signals to the adrenal medulla, decreasing epinephrine release.

68. Which of the following clinical signs would differentiate septic shock from hypovolemic shock? A) Severe hypotension B) Altered mental status C) Weak, thready pulse D) Warm or hot skin

D) Warm or hot skin

20. Common signs and symptoms of gardnerella vaginitis include: A) high fever, polymenorrhea, dysuria, and pain during sex. B) a thick vaginal discharge, abdominal pain, and vaginal irritation. C) low-grade fever, itching, vaginal bleeding, and abdominal pain. D) a "fishy" vaginal odor, itching, irritation, and vaginal discharge.

D) a "fishy" vaginal odor, itching, irritation, and vaginal discharge.

25. Most hemothoraces occur when: A) the intercostal arteries are lacerated. B) a penetrating injury perforates the lung. C) severe barotrauma ruptures one of the lungs. D) a fractured rib injures the lung parenchyma.

D) a fractured rib injures the lung parenchyma.

74. A 40-year-old man had a syncopal episode after receiving news of the death of a loved one. He complains of a headache and is unable to walk without becoming dizzy. You should be the MOST suspicious for: A) hypoglycemia. B) a dysrhythmia. C) hypovolemia. D) a head injury.

D) a head injury.

42. A young child with marked respiratory distress who is agitated and thrashing about should receive oxygen via: A) nonrebreathing mask because agitation indicates cerebral ischemia. B) the blow-by technique while he or she sits on the lap of a caregiver. C) positive-pressure ventilation after he or she has been properly sedated. D) a method that minimizes metabolic demand and oxygen consumption.

D) a method that minimizes metabolic demand and oxygen consumption.

33. An illness or infection would MOST likely cause: A) diabetic ketoacidosis. B) acute hyperglycemia. C) blood glucose above 500 mg/dL. D) a slow onset of hyperglycemia.

D) a slow onset of hyperglycemia.

2. Bony structures of the thorax include all of the following, EXCEPT the: A) ribs. B) scapulae. C) clavicles. D) acromion

D) acromion

7. The visceral pericardial layer: A) comprises the pericardial sac itself. B) is attached directly to the diaphragm. C) is the outermost layer of the pericardium. D) adheres to the heart and forms the epicardium.

D) adheres to the heart and forms the epicardium.

A 33-year-old man was burned when the hot water heater he was working on exploded. The patient has superficial and partial-thickness burns to his face, neck, and arms. Your primary assessment reveals that he is restless and tachypneic. His BP is 80/54 mm Hg and his heart rate is 120 beats/min and weak. You should: A) conclude that he is experiencing burn shock, start two large-bore IV lines of normal saline, and administer fluids based on the Parkland formula. B) assist his ventilations with a bag-mask device, cover him with a blanket, and start a large-bore IV of normal saline set at a keep vein open rate. C) apply oxygen via nonrebreathing mask, cover his burns with cold moist dressings, start an IV with normal saline, and give up to 4 mg of morphine for pain. D) administer high-flow oxygen, keep him warm, start at least one large-bore IV of normal saline, and administer fluid boluses to maintain adequate perfusion.

D) administer high-flow oxygen, keep him warm, start at least one large-bore IV of normal saline, and administer fluid boluses to maintain adequate perfusion.

The primary risk associated with oral and dental injuries is: A) malocclusion. B) intraoral infection. C) permanent tooth loss. D) airway compromise.

D) airway compromise

40. Prehospital treatment for patients with hyperosmolar hyperglycemic nonketotic coma focuses on: A) intubation and insulin administration. B) correction of electrolyte abnormalities. C) high-flow oxygen and IV or IM glucagon. D) airway management and fluid rehydration.

D) airway management and fluid rehydration.

23. Excessive alcohol consumption can lead to low blood glucose levels because: A) alcohol destroys any insulin that is produced. B) alcohol antagonizes the pancreatic beta cells. C) alcohol blocks the pancreatic release of insulin. D) alcohol depletes glycogen stores in the liver.

D) alcohol depletes glycogen stores in the liver.

152. You receive a call at 11:50 PM for a 3-year-old boy with respiratory distress. As soon as you enter the child's residence, you can hear a loud, barking cough. You find the child sitting on his mother's lap. He is conscious and appears alert to his surroundings. According to the child's mother, he has been sick for the past few days with a low-grade fever, but then began experiencing a high-pitched cough. His skin is warm and dry, his heart rate is 120 beats/min, and his oxygen saturation is 99% on room air. There are no signs of increased work of breathing. You should: A) administer high-flow oxygen via pediatric nonrebreathing mask, keep him calm, and transport. B) establish vascular access, give an appropriate dose of methylprednisolone, and transport. C) administer 0.5 mL of racemic epinephrine via nebulizer, apply the cardiac monitor, and transport. D) allow the child to assume a position of comfort, avoid agitating him, and transport him to the hospital.

D) allow the child to assume a position of comfort, avoid agitating him, and transport him to the hospital.

34. Retractions of the sternum or ribs during inhalation: A) are common in patients with emphysema. B) are signs of acute respiratory failure in adults. C) occur when soft tissue is pulled in around the bones. D) are especially common in infants and small children.

D) are especially common in infants and small children.

8. Postmenopausal women: A) often experience hot flashes and bradycardia. B) tend to experience fewer urinary tract infections. C) are usually over 35 years of age and tend to be obese. D) are more susceptible to atherosclerosis and osteoporosis.

D) are more susceptible to atherosclerosis and osteoporosis.

Chemicals such as Lewisite and phosgene oxime: A) are strong alkalis that cause liquefaction necrosis. B) damage the body by extracting water from the tissues. C) are most commonly found in drain and oven cleaners. D) are vesicant agents that produce cutaneous blisters rapidly

D) are vesicant agents that produce cutaneous blisters rapidly

Patients with chronic renal failure may present with uremic frost, especially: A) in the urine. B) during emesis. C) to the flank area. D) around the face.

D) around the face.

12. As soon as the aorta exits the left ventricle, it: A) descends toward the abdomen. B) branches into the femoral arteries. C) becomes the brachiocephalic artery. D) ascends toward the right shoulder.

D) ascends toward the right shoulder.

108. The use of lorazepam for seizures in the prehospital setting is limited by its: A) short half-life. B) slow onset of action. C) long duration of action. D) refrigeration requirement.

D) refrigeration requirement.

38. If a 17-year-old woman with a gynecologic problem answers your question, "Is there any possibility that you are pregnant?" with a firm "No way!" you should: A) document the response and continue your assessment. B) assume that she is pregnant and document this thoroughly. C) ask her mother or father about the possibility of pregnancy. D) ask the patient why she is sure that she is not pregnant.

D) ask the patient why she is sure that she is not pregnant.

When managing the airway of an unresponsive patient with serious anterior neck trauma and shallow breathing, you should: A) apply a cervical collar and perform intubation immediately. B) ventilate the patient with an oxygen-powered ventilation device. C) give oxygen via nonrebreathing mask and apply a pulse oximeter. D) assist ventilations with a bag-mask device and prepare to intubate.

D) assist ventilations with a bag-mask device and prepare to intubate.

103. A known heroin abuser is found unconscious on a park bench. Your assessment reveals that his respirations are slow and shallow, and his pulse is slow and weak. You should: A) suction his oropharynx, perform intubation, and then administer naloxone via slow IV push. B) preoxygenate him with a bag-mask device for 2 to 3 minutes and then intubate his trachea. C) apply oxygen via nonrebreathing mask, administer naloxone, and be prepared to assist ventilations. D) assist ventilations with a bag-mask device, administer naloxone, and reassess his ventilatory status.

D) assist ventilations with a bag-mask device, administer naloxone, and reassess his ventilatory status.

57. A 16-year-old man collapsed after being struck in the center of the chest by a line drive during a high school baseball game. Your assessment reveals that he is pulseless and apneic. As your partner initiates one-rescuer CPR, your MOST important action should be to: A) perform intubation to secure the patient's airway. B) rapidly assess the chest for signs of a sternal fracture. C) start an IV line and administer an antiarrhythmic drug. D) attach the ECG leads and be prepared to defibrillate.

D) attach the ECG leads and be prepared to defibrillate.

60. When dispatched to a residence for an injury involving a pregnant woman, you should: A) contact medical control. B) quickly access the patient. C) request a backup ambulance. D) be alert for evidence of abuse.

D) be alert for evidence of abuse.

147. You are dispatched to a daycare center for a 5-year-old girl with trouble breathing. Upon arriving at the scene, you assess the child and note that she is responsive to pain only, has weak intercostal retractions, and is breathing at a slow rate with shallow depth. You should: A) apply oxygen via pediatric nonrebreathing mask and attach a pulse oximeter. B) deliver two effective rescue breaths and assess her pulse for at least 5 seconds. C) administer high-flow oxygen, assess her cardiac rhythm, and establish IO access. D) begin assisting her ventilations with a bag-mask device and assess her pulse rate.

D) begin assisting her ventilations with a bag-mask device and assess her pulse rate.

The pain associated with a kidney stone MOST often: A) is described by the patient as a dull ache. B) begins as a sharp pain in the right upper quadrant. C) radiates from the umbilicus to the pubic symphysis. D) begins in the flank region and radiates to the groin.

D) begins in the flank region and radiates to the groin.

71. The hypoxic drive is a phenomenon in which: A) a chronically hypoxic patient's primary respiratory drive is stimulated by increased levels of carbon dioxide in the arterial blood. B) a relatively large percentage of patients with COPD become acutely apneic after receiving high-flow oxygen. C) high levels of oxygen rapidly depress a COPD patient's respiratory rate and depth, leading to worsened hypoxia and severe acidosis. D) bicarbonate ions migrate into the cerebrospinal fluid of a chronically hypoventilating patient, making the brain think that acid and base are in balance.

D) bicarbonate ions migrate into the cerebrospinal fluid of a chronically hypoventilating patient, making the brain think that acid and base are in balance.

65. If a pregnant woman is injured and is bleeding severely: A) signs of shock will appear earlier than expected. B) her body will automatically shunt blood to the fetus. C) her blood pressure will fall after a 70% loss of blood. D) blood flow will be diverted away from the uterus.

D) blood flow will be diverted away from the uterus.

50. Signs and symptoms of hypothyroidism include: A) tachycardia and hypertension. B) hot, flushed skin and restlessness. C) diarrhea and emotional lability. D) bradycardia and sluggish reflexes.

D) bradycardia and sluggish reflexes.

92. Use of an automated transport ventilator is NOT appropriate for patients who are: A) in cardiac arrest. B) apneic with a pulse. C) chemically paralyzed. D) breathing spontaneously.

D) breathing spontaneously.

When considering analgesia for a burn patient who is in severe pain, you must remember that: A) due to the risk of causing hemodynamic compromise, analgesia should be avoided in the field. B) one half of the usual dose of narcotic analgesics should be given in order to avoid drug toxicity. C) benzodiazepines are preferred over narcotics because they are less likely to cause hypotension. D) burns increase the metabolic rate, which may necessitate higher than normal doses of analgesics.

D) burns increase the metabolic rate, which may necessitate higher than normal doses of analgesics.

Relative to the entrance wound caused by an electrical burn, the exit wound: A) is a predictor of internal injury. B) heals without surgical intervention. C) is often much smaller in diameter. D) can be quite extensive and deep.

D) can be quite extensive and deep

The application of ice to partial-thickness burns: A) often negates the need to administer a narcotic. B) is not necessary because such burns are painless. C) offers excellent pain relief and minimizes swelling. D) can exacerbate tissue injury and should be avoided.

D) can exacerbate tissue injury and should be avoided.

27. The by-product of cellular respiration is: A) oxygen. B) lactic acid. C) pyruvic acid. D) carbon dioxide.

D) carbon dioxide.

28. Initial signs and symptoms of toxic shock syndrome include all of the following, EXCEPT: A) myalgia. B) scleral injection. C) headache and fever. D) cardiac dysrhythmias.

D) cardiac dysrhythmias.

47. An adult patient presents with a blood pressure of 78/60 mm Hg, a pulse rate of 120 beats/min and irregular, and labored breathing. Further assessment reveals pale, cool, moist skin and diffuse crackles in all lung fields. You should suspect: A) septic shock. B) neurogenic shock. C) hypovolemic shock. D) cardiogenic shock.

D) cardiogenic shock.

84. Common complications associated with cerebral palsy include all of the following, EXCEPT: A) seizures. B) visual impairment. C) mental retardation. D) cardiovascular disease.

D) cardiovascular disease.

62. The majority of current hemodynamic monitoring involves placement of different types of catheters into areas within the: A) central nervous system. B) brain and spinal cord. C) pulmonary circulation. D) cardiovascular system.

D) cardiovascular system.

121. Once you suspect that a child may have been abused, you should: A) apprise the caregiver of your suspicions. B) transport the child to the hospital at once. C) question the child in front of the caregiver. D) carefully document what you see and hear.

D) carefully document what you see and hear.

84. Appropriate care for postpartum bleeding in the prehospital setting includes all of the following, EXCEPT: A) infusing oxytocin at a rate of 0.2-0.3 units/min. B) massaging the uterus in a clockwise fashion. C) administering IV fluids to maintain perfusion. D) carefully placing sanitary pads in the vagina.

D) carefully placing sanitary pads in the vagina.

15. The joints that connect the ribs to the sternum are examples of: A) fused joints. B) fibrous joints. C) synovial joints. D) cartilaginous joints.

D) cartilaginous joints.

73. A person is able to hear, but is unable to interpret speech when other background noises are present. This is consistent with: A) auditory neuropathy. B) conductive hearing loss. C) sensorineural hearing loss. D) central auditory processing disorder.

D) central auditory processing disorder.

In addition to massive bleeding, injury to a carotid or vertebral artery would MOST likely cause: A) hemiparalysis. B) an air embolism. C) severe bradycardia. D) cerebral hypoxia.

D) cerebral hypoxia.

59. Pneumonitis is especially common in older patients with: A) frequent infections. B) a history of a stroke. C) immunocompromise. D) chronic food aspiration.

D) chronic food aspiration.

127. If a child who is wearing a helmet strikes a fixed object on his or her bicycle and flies over the handlebars, you would MOST likely encounter: A) facial fractures with associated brain injury. B) stretching or tearing injuries to the kidneys. C) open or closed fractures of the lower extremities. D) compression injuries to the intra-abdominal organs.

D) compression injuries to the intra-abdominal organs.

10. A tendency to gather and rely on information that confirms your existing views and avoids or downplays information that does not confirm your preexisting hypothesis or field differential is called a(n) ___________ bias. A) personal B) anchoring C) differential D) confirmation

D) confirmation

50. Frothy sputum that has a pink tinge to it is MOST suggestive of: A) tuberculosis. B) antihistamine use. C) chronic bronchitis. D) congestive heart failure.

D) congestive heart failure.

68. You and your partner are caring for a 5-pound distressed newborn. After providing 30 seconds of effective bag-mask ventilations, the newborn's heart rate remains below 60 beats/min. You should: A) cannulate the umbilical vein and give 0.5 mL of epinephrine. B) try tactile stimulation as you continue bag-mask ventilations. C) start a peripheral IV line and give 4 mEq of sodium bicarbonate. D) continue bag-mask ventilations and initiate chest compressions.

D) continue bag-mask ventilations and initiate chest compressions

A 24-year-old woman was struck by lightning. Bystanders moved the patient to an area of safety but did not provide any other care before your arrival. Your primary assessment reveals that the patient is pulseless and apneic. You begin CPR and apply the cardiac monitor, which reveals asystole. After requesting a backup paramedic unit, the MOST appropriate treatment for this patient involves: A) instructing your partner to resume one-rescuer CPR, establishing an IV of normal saline, and reassessing her cardiac rhythm in 5 minutes. B) continuing CPR, providing full spinal precautions, intubating her trachea, and ventilating her at a rate of 20 to 24 breaths per minute. C) performing adequate BLS, following standard ACLS protocol, and considering terminating your efforts if asystole persists after 10 minutes. D) continuing CPR, protecting her spine while ventilating, reassessing her cardiac rhythm after 2 minutes of CPR, and defibrillating if necessary.

D) continuing CPR, protecting her spine while ventilating, reassessing her cardiac rhythm after 2 minutes of CPR, and defibrillating if necessary.

70. During the second stage of labor: A) amniotic fluid typically gushes out of the vagina. B) the baby's head begins to bulge through the cervix. C) delivery in a multiparous woman occurs in a few hours. D) contractions become more intense and more frequent.

D) contractions become more intense and more frequent.

11. The adrenal cortex produces hormones called _______________, which: A) catecholamines, increase the blood pressure. B) glucocorticoids, stimulate energy production. C) gonadotropin, regulate testosterone production. D) corticosteroids, regulate the body's metabolism.

D) corticosteroids, regulate the body's metabolism.

63. A convenience store clerk was stabbed during a robbery attempt. He is semiconscious with shallow breathing and weak radial pulses. During the rapid assessment, you find a single stab wound to his left anterior chest. His jugular veins are distended and his breath sounds are bilaterally diminished but equal. The MOST appropriate treatment for this patient involves: A) performing bilateral needle thoracenteses, intubating the patient and ventilating at 10 to 12 breaths/min, and transporting him to a trauma center. B) administering oxygen via nonrebreathing mask, transporting at once, and placing an occlusive dressing over the stab wound if his oxygen saturation is low. C) assisting his ventilations, initiating transport, starting a large-bore IV line en route, and administering fluids to maintain a systolic blood pressure of 100 mm Hg. D) covering the stab wound with an occlusive dressing, assisting ventilations, transporting at once, and establishing large-bore IV lines en route.

D) covering the stab wound with an occlusive dressing, assisting ventilations, transporting at once, and establishing large-bore IV lines en route.

54. In contrast to an AV fistula, an AV graft: A) is used for patients who are undergoing peritoneal dialysis. B) can be palpated as a pulsating bulge on an upper extremity. C) is typically found on the lower extremity of a dialysis patient. D) creates a raised area beneath the skin that resembles a large vessel.

D) creates a raised area beneath the skin that resembles a large vessel.

55. A 30-year-old woman presents with 3 days of generalized weakness, dizziness, and excessive urination. She is conscious but restless, and she tells you that she is extremely thirsty. Her blood pressure is 96/66 mm Hg, her pulse is 110 beats/min and full, and her respirations are increased and somewhat deep. On the basis of this patient's clinical presentation, she will MOST likely require oxygen and: A) 25 g of 50% dextrose. B) in-hospital antibiotics. C) 0.5 to 1 mg of glucagon. D) crystalloid fluid hydration.

D) crystalloid fluid hydration.

45. In contrast to primary adrenal insufficiency, secondary adrenal insufficiency is caused by: A) idiopathic atrophy of both of the adrenal glands that results in a deficiency of all the steroid hormones they secrete. B) adrenal gland destruction caused by tuberculosis; viral, bacterial, or fungal infections; or cancer of the adrenal gland. C) acute hypertension and overhydration due to excess sodium reabsorption. D) decreased cortisol secretion secondary to a lack of adrenocorticotropic hormone secretion from the pituitary gland.

D) decreased cortisol secretion secondary to a lack of adrenocorticotropic hormone secretion from the pituitary gland.

62. Appropriate bag-mask ventilation for an apneic 3-year-old child involves: A) ensuring a consistently delivered tidal volume of 400 mL. B) providing hyperventilation to ensure carbon dioxide elimination. C) hyperextending the head to ensure an adequate mask-to-face seal. D) delivering each breath over 1 second until the chest rises visibly.

D) delivering each breath over 1 second until the chest rises visibly.

28. The tissues of the central nervous system: A) can metabolize fat and proteins to make energy. B) are able to store glucose and use just what is needed. C) can only survive for about an hour without glucose. D) depend entirely on glucose as their source of energy.

D) depend entirely on glucose as their source of energy.

168. You are called to a residence for a ventilator-dependent child with respiratory distress. Upon your arrival, the child's mother tells you that the child was doing fine, but then suddenly began experiencing labored breathing. She further tells you that the child's home ventilator was recently replaced with a newer one. Assessment of the child reveals that she is in marked respiratory distress and has intercostal retractions. Your FIRST action should be to: A) suction the child's tracheostomy tube to rule out secretions as the problem. B) assess the patency of the tracheostomy tube to determine if it is dislodged. C) remove the tracheostomy tube and replace it with a similar-sized ET tube. D) disconnect the child from the ventilator and begin bag-mask ventilations.

D) disconnect the child from the ventilator and begin bag-mask ventilations.

A 51-year-old woman sustained a large laceration to her cheek when she was cut by a knife during a robbery attempt. The patient is conscious and alert and has severe oral bleeding. She denies any other trauma. Your FIRST action should be to: A) suction her oropharynx for up to 15 seconds. B) manually stabilize her head in a neutral position. C) control the intraoral bleeding with sterile gauze. D) ensure that she is sitting up and leaning forward.

D) ensure that she is sitting up and leaning forward.

69. During the delivery of a post-term baby, you note the presence of particulate meconium in the amniotic fluid. Your post-delivery assessment of the newborn reveals that it is active, has a strong cry, and has a heart rate of 110 beats/min. You should: A) deliver free-flow oxygen at 5 L/min while performing deep oropharyngeal suctioning with a bulb syringe aspirator. B) avoid any form of tactile stimulation, perform laryngoscopy, and suction meconium from the trachea with an ET tube. C) preoxygenate the newborn with bag-mask ventilations for 30 seconds and then perform endotracheal intubation. D) ensure that the infant is warm and dry, administer free-flow oxygen if needed, and provide continuous monitoring.

D) ensure that the infant is warm and dry, administer free-flow oxygen if needed, and provide continuous monitoring.

43. An unresponsive woman with diabetes is wearing an insulin pump. Her blood glucose level is 24 mg/dL. You should: A) give a 20-mL/kg crystalloid bolus. B) administer 0.1 mg of glucagon IM. C) give her half the usual dose of dextrose. D) ensure that the insulin pump is turned off.

D) ensure that the insulin pump is turned off.

70. Patients with COPD typically experience an acute exacerbation of their condition because of: A) a secondary condition such as congestive heart failure or a pneumothorax. B) progressively worsening pneumonia that results in a diminished cough reflex. C) chronic noncompliance with their prescribed medications and home oxygen. D) environmental changes such as weather or the inhalation of trigger substances.

D) environmental changes such as weather or the inhalation of trigger substances.

74. When caring for an infant or child who is in compensated shock, you should: A) intubate at the earliest sign of altered mentation. B) administer a 10-mL/kg normal saline fluid bolus. C) assist ventilations to improve tissue oxygenation. D) establish IV or IO access en route to the hospital.

D) establish IV or IO access en route to the hospital.

28. Respiratory alkalosis is the result of: A) carbon dioxide retention. B) slow and shallow respirations. C) increased hydrogen ion production. D) excess carbon dioxide elimination.

D) excess carbon dioxide elimination.

Blood supply to the face is provided primarily through the: A) temporal artery. B) vertebral artery. C) internal carotid artery. D) external carotid artery.

D) external carotid artery.

8. Fertilization of an egg usually occurs in the: A) ovary. B) uterus. C) infundibulum. D) fallopian tube.

D) fallopian tube.

An arteriovenous fistula is usually located in the: A) peritoneal cavity. B) patient's groin area. C) distal lower extremity. D) forearm or upper arm.

D) forearm or upper arm.

33. The onset of eclampsia is marked by the presence of: A) hypertension. B) protein in the urine. C) thrombocytopenia. D) generalized seizures.

D) generalized seizures.

22. A person with type 1 diabetes: A) is often an older person whose pancreas does not produce adequate insulin. B) can often control his or her diabetes with a proper diet and regular exercise. C) is not as likely to experience hypoglycemia as a person with type 2 diabetes. D) generally does not produce any insulin and requires daily insulin injections.

D) generally does not produce any insulin and requires daily insulin injections.

87. Muscular dystrophy can be defined as a(n): A) autoimmune disease in which the skeletal muscles are rapidly destroyed. B) nonprogressive neuromuscular disorder caused by fetal brain hypoxia. C) birth defect caused by improper development of the fetal neural tube. D) genetic disease that causes a slow, progressive degeneration of muscle fibers.

D) genetic disease that causes a slow, progressive degeneration of muscle fibers.

62. A 40-year-old woman has an unstable pelvis following a motor vehicle crash. She is conscious but confused. Her blood pressure is 80/50 mm Hg, pulse is 120 beats/min and weak at the radial arteries, and respirations are 24 breaths/min and shallow. After starting at least one large-bore IV line, you should: A) run it wide open until her systolic blood pressure is greater than 100 mm Hg. B) administer a 250-mL normal saline bolus and then reassess her blood pressure. C) set the IV flow rate to keep the vein open unless her systolic blood pressure falls below 70 mm Hg. D) give enough isotonic crystalloid fluids to improve her mental status and radial pulse quality.

D) give enough isotonic crystalloid fluids to improve her mental status and radial pulse quality.

6. The upper extremity joins the shoulder girdle at the: A) acromion. B) olecranon process. C) acromioclavicular joint. D) glenohumeral joint.

D) glenohumeral joint.

The main filter for blood in the kidney is the: A) hilus. B) calyces. C) renal pyramid. D) glomerulus.

D) glomerulus.

51. Severely hypothermic newborns may present with sclerema, which is defined as: A) a yellow or orange tint to the white portion of the eyes. B) spontaneous bleeding due to blood-clotting abnormalities. C) an inability to shiver due to an immature immune system. D) hardening of the skin associated with reddening and edema.

D) hardening of the skin associated with reddening and edema.

137. Ventricular shunts are typically placed in children who: A) are born with a congenital condition in which the ventricles of the brain produce excessive amounts of cerebrospinal fluid. B) have experienced a severe traumatic brain injury that results in chronic cerebral edema and increased intracranial pressure. C) are born with an abnormally small brain, which results in a relative increase in the amount of circulating cerebrospinal fluid. D) have impaired circulation and absorption of cerebrospinal fluid, leading to increased size of the ventricles of the brain and increased intracranial pressure.

D) have impaired circulation and absorption of cerebrospinal fluid, leading to increased size of the ventricles of the brain and increased intracranial pressure.

The purpose of estimating a patient's total body surface area burns in the prehospital setting is to: A) obtain an accurate calculation of how severe the patient's burns are. B) determine whether the patient should be transported via a helicopter. C) ascertain how much IV fluid the patient should receive during transport. D) help the paramedic determine the most appropriate destination hospital.

D) help the paramedic determine the most appropriate destination hospital.

51. Correctly splinting an injured extremity: A) typically provides complete pain relief without the need to administer narcotic analgesia. B) effectively reduces swelling and inflammation by shunting blood away from the injured area. C) eliminates the need to elevate the extremity because immobilization causes blood stasis. D) helps to control internal bleeding by allowing clots to form where vessels are damaged.

D) helps to control internal bleeding by allowing clots to form where vessels are damaged.

83. You would MOST likely observe a grossly low respiratory rate and volume in a patient who overdosed on: A) LSD. B) ibuprofen. C) Prozac. D) heroin.

D) heroin.

89. A 29-year-old woman complains of abdominal cramping and vaginal bleeding. The patient is 22 weeks pregnant and tells you that she passed several large clots of blood while using the toilet. Your partner confirms that she can clearly recognize a small fetus in the toilet. The patient is conscious and alert, but her skin is diaphoretic. Her blood pressure is 98/58 mm Hg, pulse rate is 108 beats/min and regular, and respirations are 22 breaths/min with adequate depth. In addition to bringing the fetus to the hospital for inspection, the MOST appropriate treatment for this patient involves: A) oxygen via nasal cannula at 2 L/min, two large-bore IV lines, a 20-mL/kg normal saline bolus, emotional support, and rapid transport to the hospital. B) supplemental oxygen, careful packing of the vagina to control the bleeding, an IV line set to keep the vein open, emotional support as needed, and transport. C) oxygen by face mask at 8 L/min, placing her in a left lateral recumbent position, a sanitary pad over her vagina, emotional support, and transport to the hospital. D) high-flow oxygen, a sanitary pad over her vagina, a large-bore IV, crystalloid boluses as needed to maintain adequate perfusion, emotional support, and prompt transport.

D) high-flow oxygen, a sanitary pad over her vagina, a large-bore IV, crystalloid boluses as needed to maintain adequate perfusion, emotional support, and prompt transport.

44. The diaphragm of the stethoscope is designed to auscultate: A) heart tones. B) low-pitched sounds. C) bowel sounds. D) high-pitched sounds.

D) high-pitched sounds.

132. In contrast to the SAMPLE history of a child with an illness, the SAMPLE history of an injured child should include a specific inquiry regarding: A) routine medication use. B) any known drug allergies. C) any prior hospitalizations. D) his or her last tetanus shot.

D) his or her last tetanus shot.

80. During your assessment of a patient with paraplegia, you touch the patient's leg and he screams in pain. This is an example of: A) parasthesias. B) hypoplasia. C) hyperplasia. D) hyperasthesia.

D) hyperasthesia.

122. Bruises that occur _________________ are rarely incurred accidentally. A) in a toddler B) to both shins C) to the forehead D) in a straight line

D) in a straight line

28. Anaerobic metabolism is the process in which: A) the cells produce carbon dioxide and water. B) adequate amounts of oxygen reach the cell level. C) the adrenal glands fail to release catecholamines. D) inefficient cellular metabolism produces lactic acid.

D) inefficient cellular metabolism produces lactic acid.

158. A 4-year-old boy is found unresponsive by his mother. When you begin your assessment, the child's mother tells you that her son apparently ingested some of her antihypertensive medication. The child has poor perfusion and is breathing poorly. As you are assisting the child's ventilations with high-flow oxygen, your partner informs you that the child's heart rate is 50 beats/min and weak and that the cardiac monitor reveals sinus bradycardia. You should: A) ask your partner to insert an IO catheter and administer epinephrine 1:10,000. B) attempt immediate transcutaneous pacing while continuing ventilation assistance. C) establish immediate vascular access and administer 0.02 mg/kg of atropine sulfate. D) initiate one-rescuer CPR while your partner attempts to establish vascular access.

D) initiate one-rescuer CPR while your partner attempts to establish vascular access.

8. The ischium, ilium, and pubis are fused together to form the: A) hip socket. B) acetabulum. C) sacroiliac joint. D) innominate bone.

D) innominate bone.

69. Mental retardation is MOST accurately defined as: A) the inability to provide adequate self-care because of intense emotional abuse during childhood. B) a substandard intelligence quotient secondary to a congenital infection or complications at birth. C) a genetic condition in which the patient is unable to interact normally and acts younger than his or her peers. D) insufficient development of the brain that results in the inability to learn and socially adapt at the usual rate.

D) insufficient development of the brain that results in the inability to learn and socially adapt at the usual rate.

52. In contrast to upper airway emergencies, lower airway emergencies: A) often present with more prominent retractions. B) are generally associated with high-grade fever. C) include laryngotracheobronchitis and diphtheria. D) involve restriction of airflow during exhalation.

D) involve restriction of airflow during exhalation.

A partial-thickness burn is considered to be critical if it: A) occurs in any patient over the age of 45 years. B) is located to the proximal aspect of an extremity. C) is rated as at least a 5 on a pain scale of 0 to 10. D) involves more than 30% of the body surface area.

D) involves more than 30% of the body surface area.

19. Unlike the prenatal period, the gestational period: A) begins at conception. B) only last about 2 weeks. C) usually takes 40 weeks. D) is 38 weeks in duration.

D) is 38 weeks in duration.

Angiotensin II: A) relaxes smooth muscle. B) decreases sodium reabsorption. C) decreases blood pressure. D) is a potent vasoconstrictor.

D) is a potent vasoconstrictor.

39. A newborn is at GREATEST risk for meconium aspiration if he or she: A) is large for his or her gestational age. B) requires positive-pressure ventilations. C) has respiratory depression at the time of birth. D) is born at more than 42 weeks' gestation.

D) is born at more than 42 weeks' gestation.

A ruptured tympanic membrane: A) commonly results in permanent hearing loss. B) is characterized by CSF leakage from the ears. C) commonly leads to an infection of the middle ear. D) is extremely painful but typically heals spontaneously.

D) is extremely painful but typically heals spontaneously.

92. Cardiopulmonary arrest in the pediatric patient: A) usually presents with pulseless electrical activity. B) requires high epinephrine doses. C) typically requires defibrillation. D) is most often a secondary event.

D) is most often a secondary event.

52. A primagravida woman: A) has had one abortion. B) has delivered one baby. C) will deliver her second baby. D) is pregnant for the first time.

D) is pregnant for the first time.

In contrast to the male urethra, the female urethra: A) is approximately 20 cm long. B) is divided into three regions. C) consists of a prostatic urethra. D) is significantly shorter in length.

D) is significantly shorter in length.

36. Unlike the patient with hypoglycemia, the patient with severe hyperglycemia: A) usually does not vomit. B) has a normal breath odor. C) rapidly improves with treatment. D) is tachypneic and hyperpneic.

D) is tachypneic and hyperpneic.

With regard to a thermal burn injury, the zone of coagulation: A) may undergo necrosis within 24 to 48 hours after the burn. B) surrounds the central part of the burn and is often inflamed. C) is the area least affected by the burn and will likely recover. D) is the central part of the burn and suffers the most damage.

D) is the central part of the burn and suffers the most damage.

13. The egg is referred to as a blastocyst when: A) it becomes fertilized with sperm in the lower part of the fallopian tube. B) the placenta has fully formed and has attached to the superior part of the uterus. C) the amniotic sac and placenta begin to differentiate into their specialized duties. D) it has been fertilized and enters the uterus and begins absorbing uterine fluid through the cell membrane.

D) it has been fertilized and enters the uterus and begins absorbing uterine fluid through the cell membrane.

164. You and your partner arrive at the scene shortly after a 2-year-old child experienced an apparent seizure. The child's father tells you that his son's entire body began shaking and that the episode lasted less than 5 minutes. Your assessment of the child reveals that he is conscious, is crying, and has hot, moist skin. His heart rate is 160 beats/min, and his respirations are 40 breaths/min. You should: A) advise the father to take his son to see a pediatrician the following day. B) cool the child with tepid water, administer high-flow oxygen, and transport. C) establish vascular access, give a 20-mL/kg saline bolus, and transport him. D) keep the child cool and transport him to the hospital for physician evaluation.

D) keep the child cool and transport him to the hospital for physician evaluation.

Definitive treatment for a patient with end-stage renal disease involves: A) renal dialysis. B) diuretic therapy. C) crystalloid fluids. D) kidney transplant.

D) kidney transplant.

28. After inserting an orogastric tube in a newborn, you should: A) leave the 20-mL syringe attached. B) perform intubation within 2 minutes. C) connect the tube to continuous suction. D) leave the tube open to allow air to vent.

D) leave the tube open to allow air to vent.

34. You are assessing a patient and discover that he has a ventricular assist device because of severe left heart failure. In this case, the device is MOST likely connected to the: A) left atrium. B) right atrium. C) right ventricle. D) left ventricle.

D) left ventricle.

The anterior chamber is the portion of the globe between the _____ and the _____, and is filled with _____ humor. A) iris, lens, vitreous B) cornea, iris, aqueous C) lens, iris, vitreous D) lens, cornea, aqueous

D) lens, cornea, aqueous

36. Seizures during pregnancy should be treated with: A) diazepam. B) valproic acid. C) phenobarbital. D) magnesium sulfate.

D) magnesium sulfate.

42. Herbal preparations would MOST likely cause an induced abortion by: A) precipitating premature labor and the delivery of a nonviable fetus. B) causing premature separation of the placenta from the uterine wall. C) causing a fetal intracranial hemorrhage due to severe vasoconstriction. D) making the uterus and bloodstream too toxic for the fetus to survive.

D) making the uterus and bloodstream too toxic for the fetus to survive.

142. While assessing the airway of a 3-year-old girl who is unresponsive, you hear a snoring sound during each of her slow, shallow breaths. You should: A) insert an oropharyngeal airway and apply high-flow oxygen. B) begin bag-mask ventilations to improve her low tidal volume. C) provide free-flow oxygen as you nasotracheally intubate her. D) manually maneuver her head and reassess her breathing status.

D) manually maneuver her head and reassess her breathing status.

A young man was assaulted and has extensive maxillofacial injuries. Your primary assessment reveals that he is semiconscious, has shallow breathing, and has blood draining from the corner of his mouth. Initial management for this patient involves: A) inserting an oropharyngeal airway, preoxygenating him with a bag-mask device for 2 minutes, and then intubating his trachea. B) applying a cervical collar, performing a blind finger sweep to clear his airway, and providing ventilatory assistance with a bag-mask device. C) fully immobilizing his spine, inserting a nasopharyngeal airway, and hyperventilating him with a bag-mask device at a rate of 20 breaths/min. D) manually stabilizing his head in a neutral position, suctioning his oropharynx, and assisting ventilations with a bag-mask device and 100% oxygen.

D) manually stabilizing his head in a neutral position, suctioning his oropharynx, and assisting ventilations with a bag-mask device and 100% oxygen.

A burn patient with a history of chronic obstructive pulmonary disease: A) is at a higher risk for infection than a patient without any medical problems. B) often requires prophylactic beta-2 agonist drugs to prevent respiratory arrest. C) should only be given high-flow oxygen if signs of hypoxia are grossly present. D) may be triaged as a critically burned patient, even if the burn injury is small.

D) may be triaged as a critically burned patient, even if the burn injury is small.

13. When assessing and caring for a 17-year-old gang member, it is MOST important to remember that he or she: A) must be separated from other gang members. B) generally desires the presence of a caregiver. C) typically boasts about the use of illicit drugs. D) may have a weapon and a reputation to earn.

D) may have a weapon and a reputation to earn.

7. Children between 1 and 3 years of age: A) are capable of basic reasoning. B) have a well-developed sense of cause and effect. C) generally explore the world exclusively by crawling. D) may have negative associations with health care providers.

D) may have negative associations with health care providers

11. Dysmenorrhea that occurs before, during, and after menstrual flow: A) affects about 80% of women. B) is called primary dysmenorrhea. C) is generally hormonal in nature. D) may signal an underlying illness.

D) may signal an underlying illness.

A 70-year-old female dialysis patient presents with a headache. She is conscious and alert, has a blood pressure of 190/100 mm Hg, has a pulse rate of 90 beats/min and regular, and has respirations of 14 breaths/min and regular. In addition to administering supplemental oxygen, you should: A) recognize that she probably received an overaggressive dialysis treatment. B) start an IV line with normal saline and infuse 200 mL of normal saline per hour. C) transport at once, start an IV line en route, and give nitroglycerin to lower her blood pressure. D) monitor her cardiac rhythm, transport, and start an IV line en route to the hospital.

D) monitor her cardiac rhythm, transport, and start an IV line en route to the hospital.

26. By definition, a massive hemothorax is characterized by: A) pulmonary injury with secondary myocardial injury. B) 10% of circulating blood volume within the pleural space. C) cardiac arrest secondary to severe intrapleural bleeding. D) more than 1,500 mL of blood within the pleural space.

D) more than 1,500 mL of blood within the pleural space.

48. The primary source of heat production in the newborn is: A) shivering. B) hyperventilation. C) peripheral vasoconstriction. D) nonshivering thermogenesis.

D) nonshivering thermogenesis

53. You should be MOST suspicious that your patient has a pericardial tamponade if he or she presents with hypotension, jugular vein distention, and: A) loud heart tones. B) respiratory distress. C) a bounding pulse. D) normal lung sounds.

D) normal lung sounds.

2. Compared to adults, the smaller diameter of a child's airway makes it more vulnerable to: A) laryngospasm. B) inhalation injury. C) oropharyngeal secretions. D) obstruction by the tongue.

D) obstruction by the tongue.

22. Difficulty with exhalation is MOST characteristic of: A) supraglottic swelling. B) upper airway obstruction. C) a mild asthma attack. D) obstructive lung disease.

D) obstructive lung disease.

A full-thickness burn is considered to be critical if it: A) is located on any part of the thorax or abdomen. B) covers more than 5% of the total body surface area. C) was irrigated with water prior to the arrival of EMS. D) occurs in a patient with a significant medical illness.

D) occurs in a patient with a significant medical illness.

1. Isolated musculoskeletal injuries: A) generally require high doses of analgesia. B) are difficult to identify during assessment. C) prove fatal in a significant number of cases. D) often result in short- or long-term disability.

D) often result in short- or long-term disability.

60. During transport of a newborn, timely intervention of acute deterioration is MOST effectively achieved by: A) ensuring placement of at least one IV line. B) reassessing vital signs every 5 to 10 minutes. C) cardiac monitoring and the use of capnography. D) ongoing observation and frequent reassessment

D) ongoing observation and frequent reassessment

63. You have just delivered a little girl who was born 4 weeks premature. There is no evidence of meconium in the amniotic fluid. After drying, warming, suctioning, positioning, and stimulating the infant, she remains acrocyanotic and is not crying. You should: A) determine the newborn's Apgar score. B) begin assisting her ventilations at once. C) resuction her mouth for up to 10 seconds. D) open her airway and assess respirations.

D) open her airway and assess respirations

Bradycardia that occurs shortly after you have dressed and bandaged an open neck wound is MOST likely the result of: A) decreased vagal tone secondary to direct injury to the vagus nerve. B) decreased venous return from the brain and an increase in intracranial pressure. C) an acute pulmonary embolism due to the entrainment of air into one of the jugular veins. D) parasympathetic nervous system stimulation due to excessive pressure on the carotid artery.

D) parasympathetic nervous system stimulation due to excessive pressure on the carotid artery.

36. Disseminated intravascular coagulation is defined as a(n): A) decrease in white blood cell count, which results in decreased resistance to infection. B) pathophysiologic reaction that occurs when cellular ischemia leads to anaerobic metabolism. C) abnormal process in which disease or injury causes hemostasis due to platelet aggregation.. D) pathological condition in which the proteins that normally control blood clotting become active.

D) pathological condition in which the proteins that normally control blood clotting become active.

32. With the exception of the aorta, great vessel injury is MOST likely to occur following: A) blunt trauma. B) shearing forces. C) rotational injury. D) penetrating trauma.

D) penetrating trauma.

81. You should be MOST suspicious for cardiogenic shock in an infant or child if: A) he or she appears listless or lethargic. B) his or her heart rate varies with activity. C) his or her heart rate is greater than 150 beats/min. D) perfusion decreases following a fluid bolus.

D) perfusion decreases following a fluid bolus.

65. A patient with status asthmaticus commonly presents with: A) compensatory respiratory alkalosis and stridor. B) accessory muscle use and inspiratory wheezing. C) audible expiratory wheezing and severe cyanosis. D) physical exhaustion and inaudible breath sounds.

D) physical exhaustion and inaudible breath sounds.

77. Patients with pneumonia often experience a coughing fit when they roll from one side to the other because: A) movement loosens pulmonary secretions and stimulates coughing. B) most cases of pneumonia occur in conjunction with bronchospasm. C) the secretions in their lungs suddenly disperse and impair breathing. D) pneumonia often occurs in the lung bases, typically on only one side.

D) pneumonia often occurs in the lung bases, typically on only one side.

6. A delay in clamping the umbilical cord and keeping the baby below the level of the placenta can result in fetal: A) anemia. B) hypovolemia. C) exsanguination. D) polycythemia.

D) polycythemia.

57. Compartment syndrome occurs when: A) metabolic waste products accumulate within a large hematoma that develops near a fracture site. B) yellow and red bone marrow seep from a fractured bone, resulting in excessive soft tissue swelling. C) blood accumulates in the medullary canal of a bone, resulting in decreased oxygenation of the bone tissue. D) pressure in the fascial compartment leads to impaired circulation, sensory changes, and progressive muscle death.

D) pressure in the fascial compartment leads to impaired circulation, sensory changes, and progressive muscle death.

39. The MOST clinically significant finding when questioning a patient with a chronic respiratory disease is: A) medication use prior to your arrival. B) a recent medication regimen change. C) a recent emergency department visit. D) prior intubation for the same problem.

D) prior intubation for the same problem.

57. In addition to an IV dextrose bolus, the MOST important treatment for newborn hypoglycemia is: A) IV fluid boluses. B) assisted ventilation. C) a 25% dextrose infusion. D) proper thermal management.

D) proper thermal management

A young woman presents with left-sided flank pain, dysuria, and fever. She tells you that she has been experiencing pain and difficulty with urination for the past week, but did not see her physician. She called 9-1-1 when the flank pain and fever began. You should suspect: A) renal failure. B) a renal calculus. C) kidney stones. D) pyelonephritis.

D) pyelonephritis.

79. The MOST important prehospital intervention for a footling breech or transverse presentation of the baby is: A) maternal vascular access. B) keeping the mother warm. C) delivery of high-flow oxygen. D) rapid transport to the hospital.

D) rapid transport to the hospital.

Most lightning-related injuries occur when the victim: A) experiences a direct hit while standing in a large open area. B) is talking on a phone and a utility pole is struck by lightning. C) is attempting to escape an oncoming thunderstorm by running. D) receives a "splash" effect after lightning strikes a nearby object.

D) receives a "splash" effect after lightning strikes a nearby object.

105. You respond to the residence of an elderly man with severe COPD. You recognize the address because you have responded there numerous times in the recent past. You find the patient, who is clearly emaciated, seated in his recliner. He is on oxygen via nasal cannula, is semiconscious, and is breathing inadequately. The patient's daughter tells you that her father has an out-of-hospital DNR order, for which she is frantically looking. You should: A) apply a nonrebreathing mask, assess his oxygen saturation level, and prepare for immediate transport. B) provide aggressive airway management unless the daughter can produce a valid DNR order. C) intubate him at once, begin transport, and advise the daughter to notify the hospital when she finds the DNR order. D) recognize that he is experiencing end-stage COPD, begin assisting his ventilations, and contact medical control as needed.

D) recognize that he is experiencing end-stage COPD, begin assisting his ventilations, and contact medical control as needed.

19. A child who is disinterested in your presence and has a blank stare and poor muscle tone: A) is likely hypoglycemic or in septic shock. B) should be ventilated with a bag-mask device. C) will most likely require pharmacologic support. D) requires immediate intervention and transport.

D) requires immediate intervention and transport.

18. A newborn with a pulse rate of 80 beats/min: A) requires ventilations and chest compressions. B) should be treated with 0.02 mg/kg of atropine. C) is likely under the influence of maternal opiates. D) requires immediate positive-pressure ventilation.

D) requires immediate positive-pressure ventilation

96. Ductal-dependent congenital heart defects typically present with __________ in the neonatal period. A) hypertension B) low-grade fever C) hyperirritability D) respiratory distress

D) respiratory distress

The ________ conducts signals to the brain via the optic nerve and interprets them as vision. A) iris B) lens C) pupil D) retina

D) retina

17. Cor pulmonale is defined as: A) increased preload caused by severe hypertension. B) left heart failure secondary to mitral valve damage. C) rupture of the alveoli due to increased surface tension. D) right heart failure secondary to chronic lung disease.

D) right heart failure secondary to chronic lung disease.

73. Tetralogy of Fallot is a combination of four heart defects, including: A) atrial septal defect. B) coarctation of the aorta. C) tricuspid atresia. D) right ventricular hypertrophy.

D) right ventricular hypertrophy

2. The risk of newborn complications is HIGHEST if the amniotic sac: A) encases the baby's face at birth. B) is still intact at the time of birth. C) contains thin, brown amniotic fluid. D) ruptured more than 18 hours before birth.

D) ruptured more than 18 hours before birth.

11. With respect to CPR and foreign body airway obstruction procedures, the child should be treated as an adult once: A) he or she reaches the age of 8 to 10 years. B) resting vital signs are consistent with an adult. C) his or her body weight is estimated at 55 pounds. D) secondary sexual characteristics have developed.

D) secondary sexual characteristics have developed

98. Etomidate should be avoided as an induction agent in pediatric intubation in the presence of: A) hypovolemia. B) tachycardia. C) hypotension. D) septic shock.

D) septic shock.

25. Assumptions by the paramedic based on stereotypes of a particular communicable disease: A) will maximize the paramedic's safety. B) are a violation of the patient's privacy. C) will expose the paramedic to legal action. D) serve only to undermine patient care efforts.

D) serve only to undermine patient care efforts.

53. If a woman is gravida 3 and para 2: A) she has been pregnant two times. B) she has delivered three babies. C) she has been pregnant five times. D) she has delivered two babies.

D) she has delivered two babies.

33. If a young female with a known history of gonorrhea presents with abdominal pain, nausea and vomiting, and bleeding between periods: A) you should suspect disseminated gonococcemia. B) one of her ovaries is probably grossly enlarged. C) it is likely that she has an ectopic pregnancy. D) she most likely has pelvic inflammatory disease.

D) she most likely has pelvic inflammatory disease.

14. Osteoporosis is MOST accurately defined as a(n): A) progressive loss of bone marrow. B) reduced range of motion in the joints. C) estrogen-related change in bone strength. D) significant decrease in bone density.

D) significant decrease in bone density.

116. Identified risk factors associated with sudden infant death syndrome include all of the following, EXCEPT: A) low birth weight. B) young maternal age. C) exposure to tobacco smoke. D) sleeping in a supine position.

D) sleeping in a supine position.

64. When preparing to intubate a small child, it is important to remember that: A) the small child's epiglottis is very rigid. B) prolonged attempts often cause tachycardia. C) you should hyperventilate before intubating. D) small children have a relatively large occiput.

D) small children have a relatively large occiput.

39. Triple lumen central catheters are usually placed in the: A) external jugular vein. B) internal carotid artery. C) antecubital vein in the arm. D) subclavian or femoral vein.

D) subclavian or femoral vein.

2. Approximately 70% of all child abuse or neglect cases involve: A) females over 8 years of age. B) an otherwise healthy child. C) males under 6 years of age. D) substance abuse by the perpetrator.

D) substance abuse by the perpetrator.

47. A blood pressure of 100/70 mm Hg in the presence of clinical signs of a tension pneumothorax: A) should be treated with crystalloid fluid boluses to prevent hypotension. B) indicates that prehospital needle decompression likely will not be required. C) is likely the result of systemic vasodilation in an attempt to reduce preload. D) suggests adequate cardiac compensation for the diminished venous return.

D) suggests adequate cardiac compensation for the diminished venous return.

69. A 71-year-old man slipped on wet grass and landed on his left side. He denies losing consciousness before or after the fall and is presently conscious and alert. He complains of pain to his left hip and in his neck. Your assessment reveals a hematoma to the left side of his head and an external rotation and shortening of his left leg. His vital signs are stable. The MOST appropriate treatment for this patient includes: A) spinal motion restriction precautions, stabilization of his hip with a traction splint, an IV of normal saline, 5 µg/kg of fentanyl, and transport. B) supplemental oxygen, application and inflation of the PASG to stabilize his hip, an IV line of lactated Ringer's, midazolam to relieve his pain, and transport. C) placing him on a scoop stretcher and stabilizing his hip with pillows, applying a cervical collar, starting an IV line set to keep the vein open, and transport. D) supplemental oxygen, spinal motion restriction precautions, an IV line of normal saline, stabilization of his hip with pillows, fentanyl if needed, and transport.

D) supplemental oxygen, spinal motion restriction precautions, an IV line of normal saline, stabilization of his hip with pillows, fentanyl if needed, and transport.

14. If the pleural space becomes filled with air or blood: A) increased surface tension ruptures the lung. B) surface tension forces the pleurae together. C) the lung expands and fills the thoracic space. D) surface tension is lost and the lung collapses.

D) surface tension is lost and the lung collapses.

13. When caring for a patient with suspected abuse or neglect, your FIRST priority should be to: A) summon law enforcement personnel to the scene. B) provide an immediate assessment of the patient. C) remove the patient from the abusive environment. D) take deliberate action to ensure your own safety.

D) take deliberate action to ensure your own safety.

43. If an infant or small child swallowed a rigid foreign body, he or she would MOST likely experience respiratory distress because: A) a foreign body in the esophagus would cause reflux and aspiration. B) when an infant or child is stressed, he or she tends to swallow a lot of air. C) the feeling of a foreign body in the throat would cause severe anxiety. D) the esophageal foreign body can compress the relatively pliable trachea.

D) the esophageal foreign body can compress the relatively pliable trachea.

31. According to the Frank-Starling mechanism: A) systemic venous pooling of blood results in a decrease in preload. B) an increase in systolic blood pressure causes a reflex bradycardia. C) coronary artery perfusion is directly proportional to cardiac output. D) the length of myocardial fibers determines force of cardiac contraction.

D) the length of myocardial fibers determines force of cardiac contraction.

The severity of a thermal burn correlates directly with: A) the body's ability to effectively dissipate significant heat energy and the patient's general state of health. B) the presence of any underlying medical problems, the duration of exposure, and the temperature of the heat source. C) the duration of exposure, the physical size of the patient, and the presence of concomitant traumatic injuries. D) the temperature of the heat source, the amount of heat energy possessed by the object or substance, and the duration of exposure

D) the temperature of the heat source, the amount of heat energy possessed by the object or substance, and the duration of exposure

81. Hydramnios is a condition in which: A) the amniotic fluid is infected. B) there is too little amniotic fluid. C) the amniotic sac has not ruptured. D) there is too much amniotic fluid.

D) there is too much amniotic fluid.

18. Ribs 4 through 9 are the most commonly fractured because: A) they are not anteriorly attached to any portion of the sternum. B) these particular ribs are inherently weak compared to other ribs. C) the person's height predisposes him or her to injury in this area. D) they are less protected by other bony and muscular structures.

D) they are less protected by other bony and muscular structures.

5. The most obvious external landmark of the larynx is the: A) cricoid cartilage. B) hyoid bone. C) arytenoid cartilage. D) thyroid cartilage.

D) thyroid cartilage.

100. After delivering a baby and clamping and cutting the umbilical cord, you note that the end of the umbilical cord attached to the baby is bleeding. You should: A) cannulate the baby's umbilical vein and infuse normal saline. B) cut the cord proximal to the first clamp and apply another clamp. C) pinch the distal end of the cord with your finger and assess the baby. D) tie or clamp the cord proximal to the first clamp and reexamine

D) tie or clamp the cord proximal to the first clamp and reexamine

95. You are caring for a 33-year-old woman who is 35 weeks pregnant and fell down a flight of stairs. Full spinal precautions have been taken, the patient is receiving high-flow oxygen, and a patent IV line is in place. During transport, you reassess her and note that she has become diaphoretic, tachycardic, and tachypneic. You should: A) reassess her blood pressure. B) administer a rapid fluid bolus. C) cover her with warm blankets. D) tilt the backboard to the left side.

D) tilt the backboard to the left side.

49. A 6-year-old male was struck in the abdomen. He is restless; his skin is cool, pale, and clammy; and his blood pressure is 94/60 mm Hg. After applying high-flow oxygen and keeping him warm, you should: A) defer vascular access unless his blood pressure begins to decrease. B) establish vascular access at the scene and then transport. C) position him on his side and transport without delay. D) transport promptly and establish vascular access en route.

D) transport promptly and establish vascular access en route.

40. The MOST important aspect in the care of a woman with severe vaginal bleeding is: A) controlling the vaginal bleeding. B) administering crystalloid fluid boluses. C) giving oxygen via nonrebreathing mask. D) treating for shock and transporting rapidly.

D) treating for shock and transporting rapidly.

8. In medicine, intuition would MOST likely be used to: A) rule out a particular diagnosis. B) justify not transporting a patient. C) downgrade the transport status. D) triage a patient to a higher category.

D) triage a patient to a higher category.

46. In contrast to an abruptio placenta, a placenta previa: A) typically presents with tearing abdominal pain. B) is usually caused by maternal abdominal trauma. C) is associated with an absence of fetal heart tones. D) usually presents with painless vaginal bleeding.

D) usually presents with painless vaginal bleeding.

75. A pregnant woman with a past history of a cesarean section is at GREATEST risk for: A) breech birth. B) preeclampsia. C) prolapsed cord. D) uterine rupture

D) uterine rupture

141. Upon arriving at the scene of a 4-year-old boy with respiratory distress, you enter the residence and see the child, who is conscious, sitting on his father's lap. The father is aware of your presence, but the child is not. Your initial action should be to: A) make physical contact with the child as soon as possible in order to identify any life threats. B) allow the father to carry his son to the ambulance, where you can perform an initial assessment. C) quickly build good rapport with the child by picking him up and asking him what his name is. D) visually assess the child from across the room for any signs of increased work of breathing.

D) visually assess the child from across the room for any signs of increased work of breathing.

The onset of ___________ soon after exposure to radiation is a predictor of poor outcomes. A) hair loss B) tachycardia C) confusion D) vomiting

D) vomiting

44. Prior to replacing an ostomy device in a patient, it is MOST important to: A) ensure that the patient is in a comfortable position. B) wash the area around the stoma with soap and water. C) use sterile technique when opening the new ostomy kit. D) wash your hands and apply personal protective equipment.

D) wash your hands and apply personal protective equipment.

22. Carbonic acid is formed by the combination of: A) water and bicarbonate. B) water and hemoglobin. C) lactate and pyruvic acid. D) water and carbon dioxide.

D) water and carbon dioxide.

97. Dilated cardiomyopathy is a condition in which the heart is: A) deprived of oxygen due to sudden coronary vasospasm. B) unusually thick and must pump harder to eject blood. C) temporarily impaired by an isolated bacterial infection. D) weakened and enlarged, making it a less efficient pump.

D) weakened and enlarged, making it a less efficient pump.

52. A pulse oximetry reading would be LEAST accurate in a patient: A) with chronic hypoxia. B) whose extremities are cool. C) with persistent tachycardia. D) with poor peripheral perfusion.

D) with poor peripheral perfusion.

Which of the following statements regarding exit wounds is correct? A) Exit wounds occur when the projectile's energy is not entirely dissipated along its trajectory through the body. B) Compared to entrance wounds, exit wounds are generally much smaller and typically have regular edges. C) Despite fragmentation of the projectile, there is usually only one exit wound for each round that is fired. D) With low-velocity gunshot wounds, the exit wound is always a mirror image of the entrance wound.

Exit wounds occur when the projectile's energy is not entirely dissipated along its trajectory through the body

Which of the following is the MOST common cause of necrotizing fasciitis? A) A fungal infection B) Clostridium tetani C) Hemolytic streptococci D) Clostridium perfringens

Hemolytic streptococci

Which of the following statements regarding primary blast injuries is correct? A) Primary blast injuries are typically the most obvious injuries. B) Primary blast injuries are the most easily overlooked. C) Primary blast injuries are the result of flying glass or shrapnel. D) Primary blast injuries are due entirely to intense heat.

Primary blast injuries are the most easily overlooked

Which of the following general statements regarding gunshot wounds is correct? A) The most important factor for the seriousness of a gunshot wound is the type of tissue through which the projectile passes. B) Injuries from a shotgun blast are most devastating when the distance between the gun and the target is less than 100 yards. C) Wounds to lower extremities that are not associated with a fracture or neurovascular compromise are always explored surgically. D) Tissue of high elasticity, such as muscle, is less able to tolerate temporary cavitation than tissue of low elasticity, such as the liver.

The most important factor for the seriousness of a gunshot wound is the type of tissue through which the projectile passes.

Which of the following statements regarding soft-tissue injuries is correct? A) Most soft-tissue injuries require immediate care to prevent blood loss. B) They are often the most obvious, but are seldom the most life threatening. C) Soft-tissue injuries should be covered immediately upon patient contact. D) Most soft-tissue injuries are hidden and require a systematic assessment

They are often the most obvious, but are seldom the most life threatening

Which of the following statements regarding lacerations is correct? A) Lacerations are linear cuts that tend to heal well due to their relatively even wound margins. B) The seriousness of a laceration depends on its depth and the structures that have been damaged. C) The first priority in treating a laceration is to cover it with a sterile dressing to prevent infection. D) A laceration must be sutured or otherwise closed within 8 to 10 hours following the injury.

The seriousness of a laceration depends on its depth and the structures that have been damaged.

In addition to bleeding and contamination, the principal danger associated with an avulsion is: A) undetectable internal damage. B) disfigurement due to severe scarring. C) invasion of the wound with Clostridium tetani. D) a loss of blood supply to the avulsed flap.

a loss of blood supply to the avulsed flap

What type of injuries will MOST likely occur if a passenger is wearing his or her lap belt above the pelvic bone during a frontal impact? A) Pelvic and lumbar spine B) Femur and thoracic spine C) Kidney and thoracic spine D) Abdominal and lumbar spine

abdominal and lumbar spine

When summoning an air transport service to transport a critically injured patient, it is MOST important to: A) determine the flight crew's credentials. B) ensure that the fire department is present. C) predetermine the destination facility. D) activate the service as soon as possible.

activate the service as soon as possible

When an adult pedestrian is struck by a motor vehicle, lateral and posterior injuries are most common because: A) the patient is thrust onto the hood of the vehicle. B) adults tend to turn to the side or away from the impact. C) the patient is thrown and lands on his or her side or back. D) the initial impact by the bumper spins the patient to the side.

adults tend to turn to the side or away from the impact

When a patient's leg is entrapped under a crushing object for a prolonged period of time, toxic metabolic waste products are released into the systemic circulation: A) and result in low serum potassium levels. B) after the patient's leg is freed from entrapment. C) after the leg has been entrapped for 2 hours. D) only if the renal system is functioning properly.

after the patient's leg is freed from entrapment

Renal failure, a key complication of crush syndrome, can be prevented by: A) administering calcium chloride. B) giving 40 mL/kg of lactated Ringer's. C) giving the patient 25 g of glucose. D) aggressively infusing normal saline

aggressively infusing normal saline

All of the following statements regarding front air bags are correct, EXCEPT: A) Small children riding in the front seat can be killed when the air bag deploys. B) Air bags will provide protection from both initial and secondary impacts. C) Without the use of a seat belt, front air bags are insufficient in preventing ejection. D) Front air bags will not activate in side impacts or impacts to the front quarter panel.

air bags will provide protection from both initial and secondary impacts

Your MAIN concern when caring for a patient with a soft-tissue injury to the face should be: A) airway compromise. B) hypovolemic shock. C) injuries to the eyes. D) preventing contamination

airway compromise

Lap belts that are worn alone and too high by a pregnant woman: A) usually do not injure the fetus because the uterus is a highly muscular organ. B) will provide adequate protection for the uterus if the air bag properly deploys. C) allow enough forward flexion and subsequent compression to rupture the uterus. D) will provide equal distribution of forces and prevent forward flexion of the mother.

allow enough forward flexion and subsequent compression to rupture the uterus

The subcutaneous tissue is: A) the layer of tissue above the dermis that mainly produces sweat. B) a thin layer of tissue from which blood vessels exclusively originate. C) also called the superficial fascia and consists mainly of adipose tissue. D) the deep fascial layer that ensheathes muscle and other internal structures

also called the superficial fascia and consists mainly of adipose tissue

The primary determinants of the extent of trauma a patient sustains are the: A) type of object that strikes a patient and the part of the body that sustains the most impact. B) amount of energy in the object and the mechanism by which the object is delivered to the body. C) size of the object that strikes the body and any secondary injuries that occur if the patient falls. D) physical size of the patient and the part of the body that sustains direct impact from an object.

amount of energy in the object and the mechanism by which the object is delivered to the body

The MOST common site of deceleration injury in the chest is the: A) heart. B) esophagus. C) aorta. D) vena cava

aorta

A 63-year-old diabetic woman presents with an open wound to her forearm that she experienced when she fell a week ago. She tells you that the wound has been draining purulent fluid, but has not been bleeding. The wound itself is red, inflamed, and warm to the touch. You should: A) carefully irrigate the wound with sterile water for 5 minutes. B) apply a moist, sterile dressing and transport to the hospital. C) apply a dry, sterile dressing and transport her to the hospital. D) apply a light coat of antibiotic ointment and cover the wound.

apply a dry, sterile dressing and transport her to the hospital.

A young woman attempted to commit suicide by cutting her wrist. Bright red blood is spurting from the injury site. Despite direct pressure and a pressure dressing, the wound continues to bleed heavily. You should: A) apply supplemental oxygen and keep her warm. B) elevate the extremity above the level of her heart. C) apply a tourniquet between her elbow and wrist. D) locate and apply digital pressure to the brachial artery.

apply a tourniquet between her elbow and wrist.

A 22-year-old man was struck in the forehead by a softball. He is conscious and alert, but complains of a severe headache. Your assessment reveals a large hematoma to his forehead. His vital signs are stable and his breathing is adequate. You should: A) apply firm manual pressure to the hematoma to reduce internal bleeding. B) place him in a sitting position and apply a chemical heat pack to his head. C) apply an icepack to the hematoma and monitor his level of consciousness. D) start an IV of normal saline and administer 2 mg of morphine for the pain

apply an icepack to the hematoma and monitor his level of consciousness.

Unlike deceleration injuries, crush and compression injuries occur: A) at the time of impact. B) before impact occurs. C) after the initial impact. D) from penetrating mechanisms.

at the time of impact

At a minimum, a Level II trauma center should: A) have an in-house neurosurgeon 24 hours a day. B) be able to initiate definitive care for all injured patients. C) have access to an emergency physician within 20 minutes. D) provide total care for every aspect of a patient's injuries.

be able to initiate definitive care for all injured patients

Unlike adults, children who are struck by a motor vehicle are MORE likely to: A) be run over by the vehicle as they are propelled to the ground. B) experience injuries to the lower extremities from the initial impact. C) be propelled onto the hood of the vehicle during the second impact. D) turn away from the oncoming vehicle, resulting in posterior trauma.

be run over by the vehicle as they are propelled to the ground

When the ambient temperature is high: A) the dermis produces less collagen, which temporarily decreases the skin's ability to retain warmth. B) blood vessels in the dermis dilate, which increases blood flow to the skin and allows heat to dissipate. C) sweat glands in the epidermis produce sweat, which is evaporated from the skin surface by the air. D) constriction of the vessels in the dermis brings warm blood to the surface of the skin, where it is eliminated

blood vessels in the dermis dilate, which increases blood flow to the skin and allows heat to dissipate

A vagus nerve-mediated form of cardiogenic shock without compensatory vasoconstriction that may be seen following a blast injury would MOST likely present with: A) syncope and hypertension. B) hypertension and bradycardia. C) hypotension and tachycardia. D) bradycardia and hypotension.

bradycardia and hypotension

A crushing or tearing amputation: A) is initially treated by applying a proximal tourniquet and retrieving any detached body parts. B) causes less blood loss than expected because the blood vessels retain their ability to constrict. C) cannot be surgically reattached due to the severe vascular and soft-tissue damage that accompanies it. D) can result in excessive blood loss due to hemorrhage if the paramedic does not intervene rapidly.

can result in excessive blood loss due to hemorrhage if the paramedic does not intervene rapidly.

You are dispatched to a residence for a man who cut his hand with a chainsaw. Upon arriving at the scene, your FIRST action should be to: A) immediately gain access to the patient. B) apply gloves, a gown, and facial protection. C) determine if air medical transport is available. D) carefully assess the scene for safety hazards.

carefully assess the scene for safety hazards.

When applying a dressing and bandage to a scalp wound, you should: A) carefully assess the skull for an underlying fracture. B) remove any foreign particles from the wound first. C) always use a loose dressing to soak up the blood. D) apply a cervical collar in case the c-spine is injured.

carefully assess the skull for an underlying fracture.

After a motorcyclist is ejected from his or her motorcycle, secondary collisions: A) most commonly involve a stationary object. B) cause an unpredictable combination of blunt injuries. C) typically cause bilateral fractures of the femurs and tibias. D) result in less severe injuries if the rider is wearing leather.

cause an unpredictable combination of blunt injuries

In contrast to a contusion, a hematoma is: A) accompanied by ecchymosis. B) caused by large vessel damage. C) rarely accompanied by a bruise. D) a less significant closed injury

caused by large vessel damage

According to the Waddell triad, the second impact from a motor vehicle occurs when the: A) head strikes the ground, resulting in skull and facial fractures. B) car's bumper strikes the pelvis and femurs instead of the knees. C) child is run over by the car as he or she is propelled to the ground. D) chest and abdomen strike the grille or low on the hood of the car.

chest and abdomen strike the grille or low on the hood of the car

_________ is a fibrous protein that gives the skin high resistance to breakage under mechanical stress. A) Fibrin B) Elastin C) Collagen D) Melanin

collagen

When a person falls from a significant height and lands on his or her feet, axial loading results in: A) compression or burst fractures to the lumbar spine. B) shearing injuries to the liver, spleen, and aorta. C) crushing injuries to the heels and hip dislocations. D) disc injuries to the lower cervical and upper thoracic spine

compression or burst fractures to the lumbar spine

The outermost layer of the epidermis: A) consists of nonliving cells that are continuously being shed. B) is a tough, highly elastic layer than contains melanin granules. C) contains numerous fibroblasts that secrete collagen and elastin. D) is comprised of living cells that give rise to the stratum corneum

consists of nonliving cells that are continuously being shed.

If a person survives the initial trauma from a shotgun wound at close range: A) nervous system damage is likely due to the internal dispersal of the pellets. B) contaminants that were driven into the wound can cause a severe infection. C) he or she typically dies within 24 hours secondary to liver or renal failure. D) it is likely that only soft tissue was injured and major organs were spared

contaminants that were driven into the wound can cause a severe infection

A patient taking _______________ would MOST likely experience a delay in the healing of a wound. A) antidepressants B) acetaminophen C) antihypertensives D) corticosteroids

corticosteroids

If your patient has an open wound in which there is a risk of air being drawn into the vasculature, you should: A) cover the wound with an occlusive dressing. B) cover the wound with a hemostatic agent. C) apply direct pressure with moist dressings. D) transport quickly to a hyperbaric chamber.

cover the wound with an occlusive dressing

Primary treatment in the prehospital setting for an abrasion involves: A) administering a narcotic analgesic. B) applying an antibiotic ointment or cream. C) covering it lightly with a sterile dressing. D) thoroughly cleaning it to prevent infection.

covering it lightly with a sterile dressing.

The third phase of a motor vehicle accident involves: A) crush injuries to the body. B) impact by another vehicle. C) deceleration of internal organs. D) injuries caused by flying debris.

deceleration of internal organs

Which of the following wounds usually requires substantial irrigation and debridement prior to closure? A) Degloving injuries B) Jagged lacerations C) Any wound to the face D) Wounds over tension lines

degloving injuries

Which of the following conditions or factors would MOST likely delay or impair healing of a wound? A) Diabetes B) Obesity C) Alcohol use D) Hypertension

diabetes

Sudden deceleration of a motor vehicle that is traveling at 60 mph: A) typically generates forces of up to 10 to 20 g. B) initially causes whiplash injuries to the patient's neck. C) dissipates tremendous forces and causes major injuries. D) causes the driver's body to stop moving at the same time.

dissipates tremendous forces and causes major injuries

When caring for an amputated body part: A) early notification of the hospital is important. B) the body part should be kept at room temperature. C) the part should be tightly wrapped in a dry dressing. D) applying ice to the body part will keep the cells viable.

early notification of the hospital is important.

Which of the following bandages is associated with the HIGHEST risk of blood flow compromise? A) Roller bandages B) Elastic bandages C) Triangular bandages D) Nonabsorbent bandages

elastic bandages

Which of the following interventions encourages drainage from the site of a closed wound and reduces swelling? A) Elevation B) Splinting C) Firm compression D) Application of ice

elevation

The law of conservation of energy states that: A) kinetic energy can be converted only to thermal or chemical energy. B) the force that an object can exert is the product of its mass multiplied by its acceleration. C) energy can be neither created nor destroyed; it can only change form. D) a body at rest will remain at rest unless acted upon by an outside force.

energy can be neither created nor destroyed; it can only change form.

The shock wave velocity from an explosion is slower and its duration is longer if a person is: A) closer to the explosion. B) standing behind a solid object. C) farther from the explosion. D) standing beside a solid object.

farther from the explosion

Following a head-on collision of a motorcycle and a truck, the motorcyclist would MOST likely experience initial injury to the: A) cervical spine. B) chest and abdomen. C) wrists and forearms. D) femurs and tibias

femurs and tibias

Systemic signs of infection secondary to a soft-tissue injury include: A) erythema. B) pus drainage. C) fever and chills. D) lymphangitis

fever and chills

Compared to a handgun, a rifle: A) is less accurate. B) fires a single projectile. C) fires at a higher velocity. D) has less powerful ammunition

fires at a higher velocity

When caring for a patient with an open chest wound, you should: A) routinely transport the patient in a left lateral recumbent position. B) place a porous dressing over the wound and secure it on three sides. C) secure a dressing in place by circumferentially wrapping the chest. D) frequently assess breath sounds for indications of a pneumothorax.

frequently assess breath sounds for indications of a pneumothorax

Necrosis of tissue caused by an anaerobic, toxin-producing bacterium is called: A) tetanus. B) gangrene. C) fasciitis. D) lymphedema.

gangrene

Following a rotational impact, the MOST severely injured patient(s. will likely be found at the point of: A) least deceleration. B) greatest deceleration. C) secondary impact. D) greatest acceleration

greatest deceleration

A specific attribute of a Level I trauma center is that it: A) is involved in an injury prevention program. B) can initiate definitive care for all injured patients. C) has 24-hour in-house coverage by general surgeons. D) has rapid access to an off-site anesthesiologist.

has 24-hour in-house coverage by general surgeons.

If the mechanism of injury does not appear to be significant, you should consider transporting an injured patient to a Level I trauma center if he or she: A) is older than 45 years of age. B) takes any kind of medication. C) is emotionally upset or angry. D) has a known bleeding disorder.

has a known bleeding disorder

Elevation of an extremity that has a venous laceration to it: A) is often the only intervention needed to control the bleeding effectively. B) helps control the bleeding when used in conjunction with direct pressure. C) should only be performed if pressure point control has proven ineffective. D) is of minimal to no benefit if the patient is bradycardic and hypotensive

helps control the bleeding when used in conjunction with direct pressure

The MOST significant immediate threat to a patient with a soft-tissue injury is: A) nerve damage. B) infection. C) disfigurement. D) hemorrhage

hemorrhage

During the inflammation phase of the healing process: A) white blood cells are forced away from the injury by vasoconstriction. B) histamine causes vasodilation and increased blood flow to the injury. C) damaged cell parts and microorganisms invade and infect the wound. D) the processes of epithelialization and collagen synthesis are impaired

histamine causes vasodilation and increased blood flow to the injury

Whether the contamination from an open wound produces infection depends MOSTLY on: A) how the wound is managed. B) the location of the wound. C) the patient's medical history. D) how large the open wound is

how the wound is managed

The bite from a ________ poses the greatest risk for serious infection. A) cat B) dog C) raccoon D) human

human

You are the first unit to arrive at the scene of a small building collapse. As you exit the ambulance, you can see a man pinned under a large metal beam. You should: A) free the patient first and then assess him. B) carefully access the patient and assess him. C) contact medical control for further guidance. D) immediately request a special rescue team.

immediately request a special rescue team.

An object increases its kinetic energy more by: A) decreasing its speed than by increasing its mass. B) increasing its velocity than by increasing its mass. C) decreasing its velocity than by decreasing its mass. D) increasing its mass than by increasing its velocity.

increasing its velocity than by increasing its mass

The skin is also referred to as the: A) melanin. B) integument. C) epithelium. D) collagen

integument

Unlike blunt trauma, penetrating trauma: A) is especially common during the primary blast injury following an explosion. B) often causes damage to a large body surface area, even from a single projectile. C) involves a disruption of the skin and underlying tissues in a small, focused area. D) is usually more fatal because of the severe external bleeding that accompanies it.

involves a disruption of the skin and underlying tissues in a small, focused area

After controlling the bleeding from a grossly contaminated open wound to the leg, you should next: A) irrigate with sterile water and apply a sterile dressing. B) apply a pressure bandage and elevate the extremity. C) elevate the extremity and administer 100% oxygen. D) gently pick out any foreign bodies with hemostats.

irrigate with sterile water and apply a sterile dressing.

Compared to the bleeding from an open wound, bleeding from a closed wound: A) is limited because the skin is unbroken. B) generally requires surgical intervention. C) is not significant enough to produce shock. D) can usually be controlled with direct pressure.

is limited because the skin is unbroken

Applying direct pressure to a bleeding wound stops the flow of blood because: A) pressure stimulates the release of fibrin. B) direct pressure facilitates vasoconstriction. C) it allows platelets to seal the vascular walls. D) pressure shunts blood away from the injury.

it allows platelets to seal the vascular walls.

A wound is at HIGHEST risk for infection if: A) the patient uses an antibacterial spray. B) it occurs to any part of the facial area. C) it is caused by a human or animal bite. D) the patient has poor peripheral circulation.

it is caused by a human or animal bite

The initial point of bodily impact when an unrestrained passenger takes the "down and under" pathway during a frontal collision is the: A) knees. B) pelvis. C) femurs. D) abdomen

knees

The swelling that occurs in conjunction with a contusion is caused by: A) inflammation of the injured blood vessels. B) rupture of large blood vessels in the dermis. C) aggregation of platelets to the injured site. D) leakage of fluid into spaces between the cells.

leakage of fluid into spaces between the cells

The peak magnitude of the pressure wave experienced by a person: A) causes secondary and tertiary injuries as a result of the explosion. B) will cause less severe trauma if the person is standing beside a solid object. C) lessens as the person is farther away from the center of the explosion. D) causes more severe trauma if the person is in an open area during the explosion.

lessens as the person is farther away from the center of the explosion

Which of the following organs is LEAST susceptible to pressure changes caused by an explosion? A) Liver B) Lungs C) Tympanic membrane D) Gastrointestinal tract

liver

Common air bag-related injuries include: A) spinal fractures. B) minor skin burns. C) hyperflexion injuries. D) abdominal abrasions.

minor skin burns

You should splint an open soft-tissue injury to an extremity because: A) most patients do not keep the extremity still when asked to do so. B) most open soft-tissue injuries are associated with a fracture. C) splinting is an excellent means of providing relief from pain. D) motion of the extremity may disrupt the blood-clotting process

motion of the extremity may disrupt the blood-clotting process.

During the neovascularization phase of the wound healing process: A) new blood vessels form as the body attempts to bring oxygen and nutrients to the injured tissue. B) histamine makes the capillaries more permeable, resulting in swelling in and around the injury site. C) collagen provides stability to the damaged tissue and joins wound borders, thereby closing the open tissue. D) microscopic vasculature damaged by the injury is digested by macrophages through a process called phagocytosis

new blood vessels form as the body attempts to bring oxygen and nutrients to the injured tissue

A properly worn motorcycle helmet will: A) not protect the cervical spine. B) eliminate the risk of head trauma. C) decrease the risk of a spinal injury. D) minimize the trauma caused by secondary impacts.

not protect the cervical spine

A laceration that lies perpendicular to the skin's tension lines: A) results in minimal external bleeding and typically heals spontaneously within 2 to 3 hours. B) generally remains closed and does not require suturing or other methods of wound closure. C) often remains open, heals more slowly, and is more likely to result in abnormal scar formation. D) does not disrupt the body's blood-clotting process and tends to heal without the formation of a scar

often remains open, heals more slowly, and is more likely to result in abnormal scar formation.

The second impact that occurs when an adult pedestrian is struck by a motor vehicle would MOST likely result in injuries to the: A) head and neck. B) hips and knees. C) pelvis and chest. D) lower extremities.

pelvis and chest

Which of the following medications would MOST likely interfere with hemostasis? A) Paxil B) Procrit C) Plavix D) Tylenol

plavix

The energy stored in an object, such as a bridge pillar, is called __________ energy, and the energy from motion is called __________ energy. A) kinetic, potential B) barometric, kinetic C) potential, kinetic D) chemical, potential

potential, kinetic

Knowledge of kinetics can help the paramedic: A) predict injury patterns found in a patient. B) determine which organs have been injured. C) quantify how much blood a patient has lost. D) differentiate between medical and trauma patients.

predict injury patterns found in a patient

Structural protection afforded to a motorcycle rider during a crash comes from: A) protective gear worn by the rider. B) the main frame of the motorcycle. C) side foot pedals and the handlebars. D) the handlebars and large engine block.

protective gear worn by the rider

All of the following are functions of the skin, EXCEPT: A) providing the immune response for the body. B) protecting the underlying tissue from injury. C) sensing changes in the external environment. D) assisting in the regulation of body temperature

providing the immune response for the body

You have dressed and bandaged a laceration to the arm of a 16-year-old woman and are transporting her to the hospital. En route, the patient complains that her fingers are tingling. You touch her hand and note that it is cool. You should: A) readjust the bandage if needed and reassess distal neurovascular function. B) conclude that the laceration has probably severed a major nerve in her arm. C) elevate her arm, apply an icepack over the bandage, and reassess her hand. D) contact the receiving facility and have them place a neurosurgeon on standby

readjust the bandage if needed and reassess distal neurovascular function

A patient with nerve compromise following an open injury to the hand: A) should be given analgesia for the pain. B) will likely lose all neurologic function. C) requires prompt transport to the hospital. D) will not be able to move his or her hand.

requires prompt transport to the hospital.

When muscles are crushed beyond repair, tissue necrosis develops and causes the release of harmful products. This process is called: A) rhabdomyolysis. B) myoglobinuria. C) hyperphosphatemia. D) necrotizing fasciitis

rhabdomyolysis

Which of the following substances is produced in the dermis and keeps the skin supple so that it doesn't crack? A) Sebum B) Elastin C) Collagen D) Ground substance

sebum

During abrupt deceleration: A) shearing or rupturing of internal organs can occur. B) the neck commonly sustains hyperextension injuries. C) the skull provides excellent protection for the brain. D) supporting structures of the aorta keep it attached.

shearing or rupturing of internal organs can occur

While hiking, a 24-year-old woman was pinned from the waist down under a rock that collapsed on her. Upon your arrival, the patient is conscious and alert, and states that she can't feel her legs. She further tells you that she thinks she has been pinned for about 5 hours. She is breathing adequately and has stable vital signs. In addition to administering supplemental oxygen, you should: A) start two large-bore IV lines of normal saline, apply a cardiac monitor, and contact medical control before removing the rock from her legs. B) quickly remove the rock from her legs to restore distal neurovascular function, and administer a 20-mL/kg bolus of lactated Ringer's solution. C) administer 2 mEq/kg of sodium bicarbonate followed by 25 g of 50% dextrose as you slowly and carefully remove the rock from her legs. D) start at least one large-bore IV line and administer 2 to 4 L of normal saline before attempting to remove the rock from her legs.

start two large-bore IV lines of normal saline, apply a cardiac monitor, and contact medical control before removing the rock from her legs.

A 41-year-old man was assaulted during a robbery attempt. Your primary assessment reveals that the patient is semiconscious. He has massive soft-tissue trauma to the face, inadequate breathing, and oropharyngeal bleeding. You should: A) apply direct pressure to his facial wounds and promptly intubate him. B) suction the blood from his mouth and assist ventilations with a bag-mask device. C) insert a nasal airway, apply oxygen via nonrebreathing mask, and transport. D) suction his oropharynx for 30 seconds and then perform endotracheal intubation

suction the blood from his mouth and assist ventilations with a bag-mask device

According to the American College of Surgeons, an injured patient should be transported to a Level I trauma center if his or her: A) heart rate is greater than 100 beats/min. B) systolic blood pressure is less than 90 mm Hg. C) respiratory rate is less than 14 breaths/min. D) Glasgow Coma Scale score is less than 15.

systolic blood pressure is less than 90 mm Hg.

Which of the following characteristics of an entry wound indicates that the weapon was fired at close range? A) Abrasions around the wound B) Indentation of cutaneous tissues C) Severe bleeding from the wound D) Tattoo marks from powder burns

tattoo marks from powder burns

During the process of wound healing, hemostasis: A) permanently stops the wound from bleeding and facilitates healing. B) is a physiologic process in which the body's platelets are destroyed. C) occurs when the bone marrow transiently produces more red blood cells. D) temporarily stops bleeding via vasoconstriction and platelet aggregation

temporarily stops bleeding via vasoconstriction and platelet aggregation

A 30-year-old man presents with jaw and neck stiffness and fever. During your assessment, he tells you that he cut his hand on a piece of metal about a week ago. You should be MOST suspicious that this patient has: A) tetanus. B) meningitis. C) a viral infection. D) a staph infection

tetanus

An infection characterized by painful muscle contractions is called: A) pertussis. B) polio. C) rabies. D) tetanus.

tetanus

When a patient takes the "up and over" pathway during a head-on collision: A) the head takes a higher trajectory, striking the windshield and causing stretching injuries to the neck. B) secondary injuries as the patient is ejected are often less severe than the primary injuries. C) the anterior part of the neck may strike the steering wheel, resulting in a fractured larynx. D) injuries to the parietal aspect of the skull are common as the head strikes the side window.

the anterior part of the neck may strike the steering wheel, resulting in a fractured larynx

If the windshield of a wrecked vehicle is cracked or broken: A) you should assume that the driver has a severe intracerebral hemorrhage. B) the front seat occupant has a cervical spine injury until proven otherwise. C) the rear seat passenger was likely thrust from the seat into the windshield. D) it is likely that the vehicle was traveling at least 55 mph at the time of impact.

the front seat occupant has a cervical spine injury until proven otherwise

Which of the following is NOT a factor when considering transport of a trauma patient via helicopter? A) The need for definitive airway management B) Distance from the scene to the landing zone C) Time it will take the aircraft to reach the scene D) Type of terrain on which the helicopter will land

the need for definitive airway management

During an explosion, secondary blast injuries occur when: A) hollow organs rupture due to the pressure wave. B) the patient is thrown against a stationary object. C) the patient sustains severe burns from the intense heat. D) the patient is struck by flying debris, such as shrapnel.

the patient is struck by flying debris, such as shrapnel

During a frontal collision, MOST pneumothoraces occur when: A) the patient takes a deep breath just before impact and the lungs rapidly decompress at the time of impact. B) the diaphragm rapidly ascends into the chest cavity during impact, causing an increase in intrathoracic pressure. C) the chest strikes the steering wheel, which fractures one or more ribs and causes a perforation injury to one of the lungs. D) the patient is ejected through the windshield, and his or her chest collides with a secondary object outside the vehicle.

the patient takes a deep breath just before impact and the lungs rapidly decompress at the time of impact.

The forces applied to the driver during a frontal vehicle collision will differ based on all of the following factors, EXCEPT: A) objects inside the vehicle. B) the physical size of the patient. C) the design of the motor vehicle. D) safety features of the motor vehicle.

the physical size of the patient

The severity of hyperextension injuries to the neck following a rear-end collision depends MOSTLY on: A) the proper use of seat belts. B) the position of the headrest. C) proper deployment of the air bags. D) whether the driver tenses up.

the position of the headrest

The MOST reliable indicator that significant energy was dissipated by braking before a motor vehicle collision is: A) deformity to the driver's brake pedal. B) severe damage to the front rims of the tires. C) a trail of debris leading to the site of impact. D) the presence of tire skid marks at the scene.

the presence of tire skid marks at the scene

Which of the following injury mechanisms would MOST likely result in blunt trauma? A) Small-caliber gunshot wound B) Explosion involving shards of glass C) Falling from a tree onto a fence D) The pressure wave caused by a blast

the pressure wave caused by a blast

The skin helps regulate body temperature through: A) peripheral vasodilation, which shunts cool blood to the core of the body. B) the production of sweat, which is evaporated from the surface of the skin. C) increased elastin production, which provides insulation to the epidermis. D) cutaneous vasoconstriction, which brings warm blood to the skin's surface

the production of sweat, which is evaporated from the surface of the skin

Which of the following factors is the LEAST influential in the severity of a stab wound? A) The size of the attacker B) The anatomic area involved C) The length of the knife blade D) The angle of penetration

the size of the attacker

The use of wet dressings in the field is limited because: A) their use may result in severe hypothermia. B) they are of no value in providing pain relief. C) their sterility cannot be maintained in the field. D) they provide a medium for pathogens to grow.

they provide a medium for pathogens to grow.

Penetrating trauma occurs when: A) internal organs are lacerated and bleed profusely. B) tissues are penetrated by single or multiple objects. C) blunt force trauma causes explosive open injuries. D) a fractured rib perforates the parenchyma of a lung.

tissues are penetrated by single or multiple objects

The acute physiologic and structural change that occurs in a patient's body when an external source of energy dissipates faster than the body's ability to sustain and dissipate it is called: A) injury. B) trauma. C) deceleration. D) kinematics.

trauma

During a lateral impact collision: A) the far-side occupant, even if properly restrained, experiences double the amount of force as the driver. B) the patient's head moves away from the object causing the impact, resulting in stretching injuries. C) properly worn seat belts protect the passenger from pelvic trauma at the time of impact. D) trauma to the upper extremities depends on the spatial orientation of the arms upon impact.

trauma to the upper extremities depends on the spatial orientation of the arms upon impact

Physical injury to the skin: A) causes a decrease in the production of macrophages and lymphocytes, thus increasing the risk of infection. B) commonly destroys the stratum corneum, the deep dermal layer of the skin, and causes nerve damage. C) promotes cutaneous vasoconstriction, which shunts blood away from the injury and manifests as pallor around the injury site. D) triggers mast cells to degranulate and synthesize special chemical mediators, which causes the injured area to become warm and red.

triggers mast cells to degranulate and synthesize special chemical mediators, which causes the injured area to become warm and red.

Which of the following injuries would MOST likely require transport to a Level I trauma center? A) Two or more proximal long bone fractures B) Superficial burns to an entire lower extremity C) Lateral neck pain following a motor vehicle crash D) Penetrating injury that is distal to the elbow or knee

two or more proximal long bone fractures

Which of the following statements regarding rollover motor vehicle crashes is correct? A) Injuries to the passengers are usually not serious if seat belts are worn properly. B) Ejection of the patient from the vehicle increases the chance of death by 5 times. C) Unrestrained passengers are struck with each change in direction the car makes. D) The restrained occupant's head and neck usually remain stationary during a rollover.

unrestrained passengers are struck with each change in direction the car makes

Compared to stab wounds to the posterior part of the body, stab wounds to the anterior part of the body are generally: A) downward. B) single. C) multiple. D) upward.

upward

If a Level I trauma center is 30 miles away, and a Level II trauma center is 10 miles away, it would be MOST appropriate to transport a patient with a severe traumatic brain injury: A) by ground to the Level I trauma center. B) to the closest hospital for stabilization. C) via air transport to the Level I trauma center. D) by ground to the Level II trauma center.

via air transport to the level I trauma center

The FIRST aspect to address in any patient with a soft-tissue injury is: A) bleeding. B) your safety. C) airway patency. D) decontamination

your safety


Ensembles d'études connexes

High School Health : 3. SOCIAL AND MENTAL HEALTH Quiz 1

View Set

Nurs 220 (Pharma) CoursePoint Chapter 3

View Set

Computer Networking Knowledge Checks 1-3

View Set

Chapter 12 Terms & Review Questions

View Set

Cybersecurity-Internet of Things Terms

View Set

Media Lab "Reading Gram Stained Smears From Cultures"

View Set

Preventing poor outcomes leadership case

View Set

Chapter 9: Climate Classification

View Set

Environmental Sustainability - What's the Answer?

View Set